Download 1 MULTIPLE CHOICE QUESTIONS Topic: Atomic Orbitals 1. In

Survey
yes no Was this document useful for you?
   Thank you for your participation!

* Your assessment is very important for improving the workof artificial intelligence, which forms the content of this project

Document related concepts
no text concepts found
Transcript
Chapter 1
MULTIPLE CHOICE QUESTIONS
Topic: Atomic Orbitals
1.
A)
B)
C)
D)
E)
In quantum mechanics a node (nodal surface or plane) is:
a place where Ψ is negative.
a place where Ψ is positive.
a place where Ψ = 0.
a place where Ψ2 is large.
a place where Ψ2 is negative.
Ans: C
Topic: Atomic Orbitals, Molecular Orbitals
2. When the 1s orbitals of two hydrogen atoms combine to form a hydrogen molecule,
how many molecular orbitals are formed?
A) 1
B) 2
C) 3
D) 4
E) 5
Ans: B
Topic: Atomic Orbitals
3. Which principle(s) or rule must be used to determine the correct electronic configuration
for carbon in its ground state?
A) Aufbau Principle
B) Hund's Rule
C) Pauli Exclusion Principle
D) (A) and (B) only
E) All three
Ans: E
1
Chapter 1
Topic: Atomic Orbitals, Bonding
4. What point on the potential energy diagram below represents the most stable state for
the hydrogen molecule?
A)
B)
C)
D)
E)
I
II
III
IV
V
Ans: C
Topic: Atomic Orbitals, Molecular Orbitals
5.
A)
B)
C)
D)
E)
According to molecular orbital theory, which molecule could not exist?
H2
He2
Li2
F2
N2
Ans: B
Topic: Atomic Orbitals, Molecular Orbitals
6. When the ls orbitals of two hydrogen atoms combine to form a hydrogen molecule,
which molecular orbitals are formed?
A) One bonding molecular orbital only
B) Two bonding molecular orbitals
C) One bonding molecular orbital and one antibonding molecular orbital
D) Two antibonding molecular orbitals
E) Three bonding molecular orbitals
Ans: C
2
Chapter 1
Topic: Atomic Orbitals, Hybridization
7. The following electron configuration represents _______.
A)
B)
C)
D)
E)
1s
2sp3 2sp3 2sp3 2sp3
the ground state of boron.
the sp3 hybridized state of carbon.
the sp3 hybridized state of nitrogen.
the ground state of carbon.
an excited state of carbon.
Ans: C
Topic: Atomic Orbitals, Molecular Orbitals
8. According to molecular orbital theory, in the case of a carbon-carbon double bond, the
carbon-carbon bonding electrons of higher energy occupy this molecular orbital.
A) σ bonding MO
B) π bonding MO
C) σ* antibonding MO
D) π* antibonding MO
E) π* bonding MO
Ans: B
Topic: Atomic Orbitals, Hybridization
9.
A)
B)
C)
D)
E)
Identify the atomic orbitals in the C-C sigma bond in ethyne.
(2sp2, 2sp2)
(2sp3, 2sp3)
(2sp, 2sp)
(2p, 2p)
(2sp, 1s)
Ans: C
3
Chapter 1
Topic: Lewis Structures, Formal Charges
10. Listed below are electron dot formulas for several simple molecules and ions. All
valence electrons are shown; however, electrical charges have been omitted deliberately.
A)
B)
C)
D)
E)
H : Be : H
H
..
H:B:H
..
H
H
..
H:N:H
..
H
..
H:N:H
..
H
H
..
H:O:H
..
I
II
III
IV
V
Which of the structures actually bear(s) a positive charge?
I
II
III
III & V
IV & V
Ans: E
Topic: Lewis Structures, Formal Charges
11. What is the formal charge on oxygen in the following structure?
..
H3C
O
CH3
CH3
A)
B)
C)
D)
E)
+2
+1
0
-1
-2
Ans: B
4
Chapter 1
Topic: Lewis Structures, Formal Charges
12. In which structure(s) below does the oxygen have a formal charge of +1?
..
H
O
H
H
..
..
O:
C
O
H
H
I
..
H
II
..
H
H3C
O
CH3
CH3
III
IV
s
A)
B)
C)
D)
E)
I only
II only
I and III
I and IV
I, III, and IV
Ans: E
Topic: Lewis Structures, Formal Charges
13. Which structure(s) contain(s) an oxygen that bears a formal charge of +1?
..
..
:O
:O
..
..
..
..
: O C O:
H C
H3C
O
H
H3C
O
CH3
C
:O
:O
..:
..: : O
..:
CH3
H
I
II
III
IV
V
A)
B)
C)
D)
E)
I and II
III and IV
V
II
I and V
Ans: B
5
Chapter 1
Topic: Lewis Structures, Formal Charges
14. Which of the following molecules or ions has a nitrogen with a formal charge of -1?
(Charges on ions have been omitted.)
..
A) : N H
H
B) H
C) H
..
N
.H.
N
H
D) H3C
H
CH3
..
N
H
E) H3C C
Ans: A
CH3
N:
Topic: Lewis Structures, Formal Charges
15. In which structure(s) below does nitrogen have a formal charge of +1?
H3C
N
CH2
H3C
..
N
..
H
..
H
N
H
..
.OH
.
..
H3C
NH2
H
I
A)
B)
C)
D)
E)
..
H3C
N
CH3
II
III
I
II and IV
III and V
I and V
V
Ans: A
6
IV
V
CH3
Chapter 1
Topic: Lewis Structures, Formal Charges
16. Which of the following is an ion with a single negative charge?
..
A) H3C O :
. . .. . .. .
B) : O N O :
..
..
C)
: O:
..
:O N
:O
..:
D) All of these
E) None of these
Ans: D
Topic: Lewis Structures, Formal Charges
17. Which of these is a correct electron-dot representation of the nitrite ion, NO2-?
..
.. .. ..
..
..
..
:
:O
:
N
:
:
O
O
:
:
N
:
:
:
O
:
:
N
:
O
:
..
.O.
..
..
..
..
I
A)
B)
C)
D)
E)
III
II
.. .. ..
:O:N:O:
..
..
.. .. ..
:O::N:O
.. ..
IV
V
I
II
III
IV
V
Ans: A
7
Chapter 1
Topic: Lewis Structures, Formal Charges
18. Which is NOT a correct Lewis structure?
..
A) H N F :
..
H
H
B)
H
C
C) H
.H.
O
..
D)
H
..
.O.
..
F:
..
..
O
H
. .. .
:O H
B
:O
.. H
E) None of these
Ans: A
Topic: Lewis Structures, Formal Charges
19. Listed below are electron dot formulas for several simple molecules and ions. All
valence electrons are shown; however, electrical charges have been omitted deliberately.
A)
B)
C)
D)
E)
H : Be : H
H
..
H:B:H
..
H
H
..
H:N:H
..
H
..
H:N:H
..
H
H
..
H:O:H
..
I
II
III
IV
V
Which of the structures is negatively charged?
I
II
III
IV
V
Ans: B
8
Chapter 1
Topic: Lewis Structures, Formal Charges
20. Which compound contains a nitrogen atom with a formal positive charge?
A)
B)
C)
D)
E)
N
H
N
N
I
II
III
I
II
III
More than one of the above
None of the above
Ans: D
Topic: Lewis Structures
21. Considering Lewis structures, which of these compounds possesses a single unpaired
electron?
A) N2
B) N2O
C) NO
D) N2O4
E) O2
Ans: C
Topic: Lewis Structures
..
22.
Z
Y
A)
B)
C)
D)
E)
..
Y is a generalized structural representation which can be used for all of the
following, except:
OF2
NH2–
H2S
BeBr2
There is no exception
Ans: D
9
Chapter 1
Topic: Lewis Structures, Formal Charges
23. In which of these cases does the central atom have a zero formal charge?
A) HFH
H
B)
C)
H3C O
F
F
B
CH3
F
F
H
D)
H3C
N
CH3
H
CH3
E)
H3C
C
CH3
CH3
Ans: E
Topic: Lewis Structures, Formal Charges
24. The formal charge on sulfur in sulfuric acid is:
O
H O
S
O H
O
A)
B)
C)
D)
E)
0
-1
+1
-2
+2
Ans: A
Topic: Lewis Structures, Formal Charges
25. ExpAns:ion of the valence shell to accommodate more than eight electrons is possible
with:
A) Fluorine
B) Nitrogen
C) Carbon
D) Sulfur
E) Beryllium
Ans: D
10
Chapter 1
Topic: Lewis Structures, Molecular Geometry
26.
A)
B)
C)
D)
E)
Based on VSEPR theory, which of the following would have a trigonal planar shape?
CH3)3N
HCN
NH4+
CH3−
CH3+
Ans: E
Topic: Lewis Structures, Molecular Geometry
27.
A)
B)
C)
D)
E)
VSEPR theory predicts an identical shape for all of the following, except:
NH3
H3O+
BH3
CH3All have the same geometry
Ans: C
Topic: Lewis Structures, Molecular Geometry
28.
A)
B)
C)
D)
E)
What shape does the methyl cation, CH3+, have?
Octahedral
Tetrahedral
Trigonal planar
Linear
Trigonal pyramidal
Ans: C
11
Chapter 1
Topic: Lewis Structures, Molecular Geometry
:
:
29. Which of the structures below would be trigonal planar (a planar triangle)? (Electrical
charges have been deliberately omitted.)
F
F
H
.. ..
..
.. ..
..
:F : N :
:
H:O:H
H:C:
B
..
..
.. ..
. . : .F.
H
F
F
H
..
..
..
..
..
:
..
..
:
..
I
A)
B)
C)
D)
E)
II
IV
III
I
II
III
IV
I and IV
Ans: A
Topic: Lewis Structures, Molecular Geometry
30. Which of the following would have a trigonal planar (or triangular) structure?
A)
B)
C)
D)
E)
: CH3
: CH3
: NH3
BH3
: OH3
I
II
III
IV
V
I, II, and IV
II and IV
IV
II, IV, and V
All of these
Ans: B
Topic: Molecular Geometry
31.
A)
B)
C)
D)
E)
What bond angle is associated with a tetrahedral molecule?
120°
109.5°
180°
90°
45°
Ans: B
12
Chapter 1
Topic: Molecular Geometry
32.
A)
B)
C)
D)
E)
What would be the spatial arrangement of the atoms of the methyl anion, :CH3-?
Octahedral
Tetrahedral
Trigonal planar
Linear
Trigonal pyramidal
Ans: E
Topic: Lewis Structures, Molecular Geometry
33.
A)
B)
C)
D)
E)
Which of these structures would be a perfectly regular tetrahedron?
CH3Br
CH2Br2
CHBr3
CBr4
More than one of these
Ans: D
Topic: Lewis Structures, Molecular Geometry
34. Which molecule would be linear? (In each case you should write a Lewis structure
before deciding.)
A) SO2
B) HCN
C) H2O2
D) H2S
E) OF2
Ans: B
Topic: Lewis Structures, Molecular Geometry
35.
A)
B)
C)
D)
E)
The bond angles in PH3 would be expected to be approximately:
60°
90°
105°
109°
120°
Ans: C
13
Chapter 1
Topic: Periodic Properties, Electronegativity
36.
A)
B)
C)
D)
E)
Select the most electronegative element.
H
O
N
B
C
Ans: N
Topic: Atomic Orbitals, Hybridization
37. Select the hybridized atomic orbital.
I
A)
B)
C)
D)
E)
II
III
IV
V
I
II
III
IV
V
Ans: C
Topic: Atomic Orbitals, Hybridization
38. How many 2p atomic orbitals from boron must be mixed with a 2s atomic orbital to
yield the bonding hybrid atomic orbitals in BF3?
A) 1
B) 2
C) 3
D) 4
E) 5
Ans: B
14
Chapter 1
Topic: Atomic Orbitals, Hybridization
39.
A)
B)
C)
D)
E)
In which molecule is the central atom sp3 hybridized?
CH4
NH3
H2O
All of these
None of these
Ans: D
Topic: Atomic Orbitals, Hybridization
40. In which of the following would you expect the central atom to be sp3 hybridized (or
approximately sp3 hybridized)?
A) BH4B) NH4+
C) CCl4
D) CH3:E) All of these
Ans: D
Topic: Atomic orbitals, hybridization
41.
A)
B)
C)
D)
E)
Which compound has the shortest carbon-carbon bond(s)?
CH3CH3
CH2=CH2
HC≡CH
CH3CH2CH3
All carbon-carbon bonds are the same length.
Ans: C
Topic: Atomic Orbitals, Hybridization
42.
A)
B)
C)
D)
E)
Which of the following contains an sp2-hybridized carbon?
CH4
CH3:−
CH3CH3
CH3+
HC≡CH
Ans: D
15
Chapter 1
Topic: Atomic Orbitals, Hybridization
43. Which is the shortest of the carbon-carbon single bonds indicated by arrows in the
following compounds?
A)
H3C
CH3
H3C
C
H3C
CH
B)
C)
CH
CH2
D)
HC
C
C
CH
E) H2C
HC
C CH
Ans: D
Topic: Atomic Orbitals, Hybridization
44.
A)
B)
C)
D)
E)
How many sigma (1s-2sp3) bonds are there in ethane?
7
6
5
3
1
Ans: B
Topic: Atomic Orbitals, Bonding
45.
A)
B)
C)
D)
E)
Which of these substances contains both covalent and ionic bonds?
NH4Cl
H2O2
CH4
HCN
H2S
Ans: A
16
Chapter 1
Topic: Atomic Orbitals, Periodic Trends, Electronegativity
46.
A)
B)
C)
D)
E)
The greatest degree of ionic character is anticipated for the bond between:
H and C
H and Cl
C and Cl
H and Br
Br and Cl
Ans: B
Topic: Atomic Orbitals, Hybridization
47.
A)
B)
C)
D)
Which molecule contains an sp-hybridized carbon?
HCN
CH2=CH2
CH3Cl
H
C O
H
E) CH3CH3
Ans: A
Topic: Atomic Orbitals, Lewis structures, Resonance
48. Which of the structures below is not expected to contribute to the CO2 resonance
hybrid?
A)
B)
O
C
O
O
C
C
O
O
C) O
D)
E)
O
O
C
C
O
O
Ans: D
17
Chapter 1
Topic: Atomic orbitals, Lewis structures, resonance
..
:
..
49. Which of the following could not be a resonance structure of CH3NO2?
A)
H
O
H
C
N
..
.O.
H
O
..
B)
:
H
H
C
C
N +2
O
O
H
..
:
..
H
:
O
..
..
H
..
..
D)
H
:
H
O
H
..
:
..
C)
N
C
N..
O
..
E) Both C and D
Ans: D
H
H
Topic: Atomic Orbitals, Lewis Structures, Resonance
O
:
:
:
C
:
:
:
:
:
50. Which of the following pairs are NOT resonance structures?
A)
and
H3C O N O :
H3C O N O :
:
..
..
O
..
and
O
:
C
..
..
O
:
:
B)
..
..
:
:
H3C
:
C)
O
N
O:
O
and
H3C
N
..
:
..
:
O
D) Each of these pairs represents resonance structures.
E) None of these pairs represents resonance structures.
Ans: C
18
Chapter 1
Topic: Atomic Orbitals, Lewis Structures, Resonance
51. How many resonance structures can be written for the NO3- ion in which the nitrogen
atom bears a formal charge of +1?
A) 1
B) 2
C) 3
D) 4
E) 5
Ans: C
Topic: Empirical and Molecular Formulas
52.
A)
B)
C)
D)
E)
What is the empirical formula for cyclohexane? (Its molecular formula is C6H12.)
CH
CH2
C2H4
C6H6
C2H2
Ans: B
Topic: Empirical and Molecular Formulas
53. A compound has the empirical formula, CCl. Its molecular weight is 285 +/- 5. What is
the molecular formula for the compound?
A) C2Cl2
B) C3Cl3
C) C4Cl4
D) C5Cl5
E) C6Cl6
Ans: E
Topic: Empirical and Molecular Formulas
54. A compound consists only of carbon, hydrogen and oxygen. Elemental analysis gave:
C, 70.5%, H, 13.8%. The molecular weight of the compound was found to be 103 +/- 3.
What is the molecular formula for the compound?
A) C6H12O
B) C5H12O2
C) C3H2O4
D) C3H6O3
E) C6H14O
Ans: E
19
Chapter 1
Topic: Isomerism
55. Which of the following is a set of constitutional isomers?
Br
Br
I
A)
B)
C)
D)
E)
Br
II
Br
III
IV
I and II
II and III
I, II, and III
II, III, and IV
I, III, and IV
Ans: E
Topic: General
56. Credit for the first synthesis of an organic compound from an inorganic precursor is
usually given to:
A) Berzelius
B) Arrhenius
C) Kekule
D) Wohler
E) Lewis
Ans: D
Topic: Isomerism
57.
A)
B)
C)
D)
E)
CH3CH2OCH2CH3 and CH3CH2CH2CH2OH are examples of what are now termed:
Structural isomers
Resonance structures
Functional isomers
Empirical isomers
Constitutional isomers
Ans: E
20
Chapter 1
Topic: Hybridization
58. What is the approximate hybridization state of the oxygen molecule in ethanol,
C2H5OH?
A) sp
B) sp2
C) sp3
D) p3
E) d2sp3
Ans: C
Topic: Hybridization
59. What is the approximate hybridization state of the oxygen molecule in trimethylamine,
(CH3)3N?
A) sp
B) sp2
C) sp3
D) p3
E) d2sp3
Ans: C
Topic: Lewis structures, Hybridization
60. Which molecule has a non-linear structure (i.e., for which molecule are the nuclei not in
a straight line)?
A) O=C=O
B) H–O–H
C) H–Cl
D) H–C≡N
E) H–C≡C–H
Ans: B
21
Chapter 1
Topic: Isomerism
61. Which of the following structures represent compounds that are constitutional isomers
of each other?
I
A)
B)
C)
D)
E)
II
III
IV
I and II
I and III
I, II, and III
I, II, III, and IV
II and III
Ans: C
Topic: Isomerism
62. Which compound is not an isomer of the others?
OH
O
O
OH
I
A)
B)
C)
D)
E)
II
III
I
II
III
IV
All of the above are isomers of each other.
Ans: A
22
IV
Chapter 1
Topic: Isomerism
63. Consider the following:
A)
B)
C)
D)
E)
CH3CH2CH=CHCH2CH3
I
CH3CH2CH2CH2CH=CH2
II
CH3CH=CHCH2CH2CH3
III
CH2=CHCH2CH2CH2CH3
IV
Which two structures represent the same compound?
I and II
II and III
I and III
II and IV
None of these
Ans: D
Topic: Isomerism
64. Consider the following:
A)
B)
C)
D)
E)
CH3CH2CH2CH=CHCH2CH2CH3
I
CH3CH2CH2CH2CH2CH2CH=CH2
II
CH3CH2CH=CHCH2CH2CH2CH3
III
CH2=CHCH2CH2CH2CH2CH2CH3
IV
Which structures can exist as cis-trAns: isomers?
I and II
I and III
I and IV
II and III
I alone
Ans: B
23
Chapter 1
Topic: Isomerism
65. Which of the following represent a pair of constitutional isomers?
A)
and
B) CH3CH=CH2 and CH2=CHCH3
Br
Br
C)
Br
H
H
Br
D)
H
and Br
H
Br
H
Br
H
and H
H
Br
E) More than one of these
Ans: E
Topic: Isomerism
66. Which of the following represent pairs of constitutional isomers?
OH
O
A)
O
and
Br
H
Br
CH3
H
and Br
H
CH2
C)
O
CH3
CH3
B)
H
Br
H
and H
CH3
D) More than one of these pairs
E) All of these pairs
Ans: D
Topic: Isomerism
67.
A)
B)
C)
D)
E)
Cis-trAns: isomerism is possible only in the case of:
CH2=CBr2
CH2=CHBr
BrCH=CHBr
Br2C=CHBr
Br2C=CBr2
Ans: C
24
Chapter 1
Topic: Atomic orbitals, molecular orbitals
68. When the 1s orbitals of two hydrogen atoms combine to form a hydrogen molecule,
how are the electrons distributed in the resulting molecular orbitals?
A) 2 electrons in the bonding molecular orbital
B) 1 electron in the bonding molecular orbital, 1 electron in the non-bonding molecular
orbital
C) 1 electron in the bonding molecular orbital, 1 electron in the antibonding molecular
orbital
D) 2 electrons in the non-bonding molecular orbital
E) 2 electrons in the antibonding molecular orbital
Ans: A
Topic: Atomic Orbitals, Electron Configuration, Hybridization
69. The following electron configuration represents:
A)
B)
C)
D)
E)
1s
2sp3 2sp3 2sp3 2sp3
The ground state of nitrogen
The ground state of oxygen
The sp3 hybridized state of carbon
The excited state of oxygen
None of the above correctly identifies the given electron configuration
Ans: E
Topic: Atomic Orbitals, Hybridization, Bonding
70. Identify the atomic orbitals involved in the C-2---C-3 sigma bond (indicated by an
arrow) in the following molecule:
1
4
2
6
7
A)
B)
C)
D)
E)
5
2
3
2
sp , sp
sp2, sp
sp2, sp3
sp3, sp2
sp, sp2
Ans: E
25
Chapter 1
Topic: Lewis Structures
71.
A)
B)
C)
D)
E)
In which of the following does the central atom have 2 pairs of non-bonding electrons?
O3
CO2
CO3 2−
NH4+
H2S
Ans: E
Topic: Molecular Geometry
72. Based on the VSEPR theory, which of the following would have a tetrahedral
arrangement of electrons around the central atom?
A) BH3
B) NO2 −
C) SiH4
D) CO3 2−
E) SO3
Ans: C
Topic: Molecular Geometry
73.
A)
B)
C)
D)
E)
What would be the spatial arrangement of the atoms of the ozone molecule (O3)?
Linear
Angular
Trigonal planar
Trigonal pyramidal
Tetrahedral
Ans: B
Topic: Periodic Trends, Electronegativity
74.
A)
B)
C)
D)
E)
Select the least electronegative element
P
N
Mg
Si
K
Ans: E
26
Chapter 1
Topic: Atomic Orbitals, Hybridization
75. In which molecule(s) can the molecular geometry be attributed to an sp2 hybridized
central atom?
A) PBr3
B) CH4
C) CHCl3
D) HNO2
E) None of the above has an sp2 hybridized central atom
Ans: D
Topic: Bonding, Atomic Orbitals, Hybridization
76. Which molecule has the shortest carbon-carbon single bond?
I
II
III
IV
A)
B)
C)
D)
E)
V
I
II
III
IV
V
Ans: E
Topic: Atomic Orbitals, Hybridization
77. How many s-sp2 bonds are there in the following substance?
A)
B)
C)
D)
E)
2
3
4
5
12
Ans: B
27
Chapter 1
Topic: Atomic Orbitals, Hybridization
78. How many s-sp3 bonds are there in the following substance?
A)
B)
C)
D)
E)
3
8
12
13
16
Ans: D
Topic: Resonance
79.
A)
B)
C)
D)
E)
Which of the following species exhibits resonance stabilization?
H2SO4
O3
CO2
CCl4
None of the above species exhibit resonance
Ans: B
SHORT ANS:WER QUESTIONS
Topic: General
80. Organic compounds were originally defined as compounds obtained from __________.
Ans: living sources/organisms
Topic: General
81. The modern definition of organic chemistry is _________________.
Ans: the study of carbon compounds
Topic: General
82. Different compounds with the same molecular formula are referred to as __________.
Ans: isomers
28
Chapter 1
Topic: General
83. Constitutional isomers differ in the _________________.
Ans: connectivity of their atoms
Topic: General
84. The bond that results when two atoms share a pair of electrons is called a
_________________.
Ans: covalent bond
Topic: Atomic orbitals
85. Define an orbital.
Ans: a region of space where the probability of finding an electron is high
Topic: Atomic orbitals
86. An orbital is defined as a region of space where the probability of _________________
is high.
Ans: finding an electron
Topic: Atomic orbitals
87. There are three fundamental rules that we use in writing electronic configurations for
atoms and molecules. The configuration shown below (for oxygen) violates one of these
rules. Which one?
1s
2s
2p
2p
Ans: Pauli exclusion principle
2p
Topic: Molecular orbitals
88. When atomic orbitals of opposite phase overlap a(n) _________________ molecular
orbital is formed.
Ans: antibonding
Topic: Molecular orbitals
89. When atomic orbitals of the same phase overlap a(n) _________________ molecular
orbital is formed.
Ans: bonding
29
Chapter 1
Topic: Lewis Structures
90. Draw the Lewis structure of the nitrite ion, NO2− , clearly indicating resonance
contributors as well as non-bonding pairs of electrons and formal charges, as relevant.
Ans:
O
. . ..
.O
.
.
.. .
..
N
..
. ..
..
. .O .
.
O
.
N
Topic: Isomers, Bond-Line Formulas
91. Draw all the isomers of C4H9Br, using bond-line formulas.
Br
Ans:
Br
Br
Br
Topic: Isomers, Bond-Line Formulas
92. Draw all the isomers of C4H10O, using bond-line formulas.
OH
Ans:
OH
OH
O
O
O
Topic: Isomers, Bond-Line Formulas
93. Draw all isomers of C4H8, using bond-line formulas.
Ans:
30
OH
Chapter 1
Topic: Lewis Structures
..
..
94. Draw the Lewis structure of acetic acid, CH3CO2H, clearly indicating all non-bonding
pairs of electrons.
O
Ans:
H
H
C
C
. .O. .
H
H
31
Chapter 2
MULTIPLE CHOICE QUESTIONS
Topic: Intermolecular forces
1. Which compound would you expect to have the lowest boiling point?
NH2
A)
B)
NH2
C)
H
N
D)
N
E)
NH2
Ans: D
:
Topic: Molecular geometry, dipole moment
2. Which molecule would you expect to have no dipole moment (i.e., μ = 0 D)?
A) CHF3
H
B) F
H
C) :NF3
D) F
H
F
F
H
E) CH2F2
Ans: B
32
Chapter 2
Topic: Intermolecular forces
3.
A)
B)
C)
D)
Which of these compounds would have the highest boiling point?
CH3OCH2CH2CH2OCH3
CH3CH2OCH2CH2OCH3
CH3CH2OCH2OCH2CH3
CH3OCH2CHOCH3
CH3
E) HOCH2CH2CH2CH2CH2OH
Ans: E
Topic: Intermolecular forces
4. Which of these would you expect to have the lowest boiling point?
A) CH3CH2CH2OH
B) CH3CHCH3
OH
C) CH3OCH2CH3
D) CH3CH2CH2CH2OH
E) CH3CH2OCH2CH3
Ans: C
Topic: Intermolecular forces
5.
A)
B)
C)
D)
E)
Which compound would have the highest boiling point?
CH3CH2CH2CH2CH2CH3
CH3CH2OCH2CH2CH3
CH3CH2CH2CH2CH2OH
CH3CH2OCH(CH3)2
CH3OCH2CH2CH2CH3
Ans: C
Topic: Intermolecular forces
6. Which of the following is not found in the following substance?
CH3CH2CH2CH2CH2OH
A) Ion-ion
B) van der Waals
C) Dipole-dipole
D) Resonance
E) Hydrogen bonding
Ans: D
33
Chapter 2
Topic: Intermolecular forces
7. Which compound would you expect to have the lowest boiling point?
O
A)
NH2
H
N
B)
O
O
C)
N
D)
NH2
O
E)
O
N
H
Ans: C
Topic: Intermolecular forces
8.
A)
B)
C)
D)
E)
Which compound would you expect to have the highest boiling point?
CH3OCH2CH2OCH3
CH3OCH2OCH2CH3
HOCH2CH2CH2CH2OH
CH3OCH2CH2CH2OH
(CH3O)2CHCH3
Ans: C
Topic: Molecular geometry, dipole moment
9. Which of the following would have no net dipole moment
(μ = 0 D)?
A) CBr4
B) cis-1,2-Dibromoethene
C) trAns:-1,2-Dibromoethene
D) 1,1-Dibromoethene
E) More than one of these
Ans: E
34
Chapter 2
Topic: Molecular geometry, dipole moment
10. Which molecule has dipole moment greater than zero?
F
A) F
H
B) F
F
C) F
H
H
H
H
H
F
D) More than one of these
E) None of these
Ans: D
Topic: Intermolecular forces
11.
A)
B)
C)
D)
E)
The strongest of attractive forces is which type?
van der Waals
Ion-dipole
Dipole-dipole
Cation-anion
Hydrogen bonds
Ans: D
Topic: Intermolecular forces
12.
A)
B)
C)
Of the following compounds, the one with the highest boiling point is:
CH3CH3
CH3CH2Cl
CH3C=O
H
D) CH3CH2OH
E) CH3CH2OCH2CH3
Ans: D
35
Chapter 2
Topic: Intermolecular forces
13. This alkane is predicted to have the highest melting point of those shown:
A) CH3CH2CH2CH3
B) CH3CHCH3
CH3
C) CH3CH2CH2CH2CH3
D) CH3CHCH2CH3
E)
CH3
CH3
CH3CCH3
CH3
Ans: E
Topic: Intermolecular forces
14. The solid alkane CH3(CH2)18CH3 is expected to exhibit the greatest solubility in which
of the following solvents?
A) CCl4
B) CH3OH
C) H2O
D) CH3NH2
E) HOCH2CH2OH
Ans: A
36
Chapter 2
Topic: Intermolecular forces
15. Which compound would have the lowest boiling point?
OH
OH
O
I
II
III
OH
OH
IV
A)
B)
C)
D)
E)
V
I
II
III
IV
V
Ans: A
Topic: Molecular geometry, Polarity
16. Which molecule(s) has dipole moment equal to zero?
Cl
A)
B) Cl
Cl
C)
Cl
Cl
D) Cl
Cl
Cl
E) None of these have dipole moment equal to zero
Ans: C
37
Chapter 2
Topic: Molecular geometry, polarity
17.
A)
B)
C)
D)
E)
Which molecule has a zero dipole moment?
SO2
CO2
CO
CHCl3
None of these
Ans: B
Topic: Molecular geometry, polarity
18.
A)
B)
C)
D)
E)
Which molecule has a zero dipole moment?
CH3Cl
CH2Cl2
CHCl3
CCl4
None of these
Ans: D
Topic: Molecular geometry, polarity
19.
A)
B)
C)
D)
E)
Which molecule would have a dipole moment greater than zero?
BeCl2
BCl3
CO2
H2O
CCl4
Ans: D
Topic: Molecular geometry, polarity
20. For a molecule to possess a dipole moment, the following condition is necessary but not
sufficient.
A) Three or more atoms in the molecule
B) Presence of one or more polar bonds
C) A non-linear structure
D) Presence of oxygen or fluorine
E) Absence of a carbon-carbon double or triple bond
Ans: B
38
Chapter 2
Topic: Molecular geometry, polarity
21.
A)
B)
C)
D)
E)
A non-zero dipole moment is exhibited by:
SO2
CO2
CCl4
BF3
Cl
Cl
Cl
Cl
Ans: A
Topic: Intermolecular forces
22.
A)
B)
C)
D)
E)
Which of these is the weakest of the intermolecular attractive forces?
Ion-ion
van der Waals
Dipole-dipole
Covalent bonding
Hydrogen bonding
Ans: B
Topic: Functional groups
23. Which compound listed below is a secondary alcohol?
A) CH3CHCH2CH3
OH
B) CH3CHCH2OH
C)
CH3
CH3
CH3COH
CH3
D) CH3CH2CH2CH2OH
E) CH3CH2CH2OCH3
Ans: A
39
Chapter 2
Topic: Functional groups
24. Which compound is a secondary amine?
A) CH3CH2CH2NH2
B) CH3CHCH3
NH2
C) CH3CH2NH
CH3
D) H3C N CH3
CH3
E) CH3CH2CHNH2
CH3
Ans: C
Topic: Functional groups
25. Which compound is an aldehyde?
O
O
O
NH
O
I
II
III
O
OH
A)
B)
C)
D)
E)
IV
I
II
III
IV
V
Ans: D
V
40
Chapter 2
Topic: Functional groups
26. Which compound is a ketone?
A) O
C OH
H
O
B)
CH3CCH2CH3
C) O
HCOCH3
D) O
C H
H
E) H3C
CH OH
H3C
Ans: B
Topic: Functional groups
27. Which compound is an ester?
O
O
O
O
NH
I
II
III
O
OH
IV
A)
B)
C)
D)
E)
V
I
II
III
IV
V
Ans: C
41
Chapter 2
Topic: Functional groups
28. The compound shown below is a synthetic estrogen. It is marketed as an oral
contraceptive under the name Enovid.
OH
O
A)
B)
C)
D)
E)
In addition to an alkane (actually cycloalkane) skeleton, the Enovid molecule also
contains the following functional groups:
Ether, alcohol, alkyne.
Aldehyde, alkene, alkyne, alcohol.
Alcohol, carboxylic acid, alkene, alkyne.
Ketone, alkene, alcohol, alkyne.
Amine, alkene, ether, alkyne.
Ans: D
Topic: Functional Groups
29. Which is a 3° alkyl halide?
F
Br
Cl
I
II
III
I
Br
IV
A)
B)
C)
D)
E)
V
I
II
III
IV
V
Ans: B
42
Chapter 2
Topic: Functional groups
30. Which is a 3° amine?
NH2
H2N
NH
I
II
III
NH2
O
N
IV
A)
B)
C)
D)
E)
V
I
II
III
IV
V
Ans: D
Topic: Functional groups
31. Which functional group is not contained in prostaglandin E1?
O
O
HO
H
A)
B)
C)
D)
E)
H
H
OH
H OH
Prostaglandin E1
Ketone
2° alcohol
3° alcohol
Carboxylic acid
Alkene
Ans: C
43
Chapter 2
Topic: Functional groups
32. The compound below is an adrenocortical hormone called cortisone. Which functional
group is not present in cortisone?
OH
O
O
OH
O
A)
B)
C)
D)
E)
1° alcohol
2° alcohol
3° alcohol
Ketone
Alkene
Ans: B
Topic: Functional groups
33. The compound shown below is a substance called Capsaicin, found in varying
concentrations in several varieties of hot peppers, and responsible for their respective
degrees of “heat”. Which functional groups are present in the molecule of capsaicin?
O
N
H
O
OH
A)
B)
C)
D)
E)
Capsaicin
Alkene, ketone, amine, alcohol, ester
Alkene, ketone, alcohol, ether
Alkene, amine, phenol, ether
Ether, phenol, alkene, amide
Ester, phenol, alkene, amide
Ans: D
44
Chapter 2
Topic: Functional groups
34. Drawn below is Atropine, found in Atropa belladonna, sometimes used in dilating
pupils during an eye-exam. Which of the following functional groups is NOT in
atropine?
N
O
O
OH
Atropine
A)
B)
C)
D)
E)
Amine
Ester
Alcohol
Benzene Ring
Ketone
Ans: E
Topic: Functional groups
35. The compound shown below is the male sex hormone, testosterone.
OH
O
O
A)
B)
C)
D)
E)
In addition to a cycloalkane skeleton, testosterone also contains the following functional
groups:
Alkene, ester, tertiary alcohol.
Alkene, ether, secondary alcohol.
Alkene, ketone, secondary alcohol.
Alkyne, ketone, secondary alcohol.
Alkene, ketone, tertiary alcohol.
Ans: C
45
Chapter 2
Topic: Functional groups
36. Which is a carboxylic acid?
OH
O
OH
O
O
I
O
II
III
O
O
O
OH
IV
A)
B)
C)
D)
E)
V
I
II
III
IV
V
Ans: E
Topic: Functional groups
37. Which compound is a tertiary alcohol?
CH3
H3CH2C O
OH
H
HO
CH3
I
II
III
O
IV
A)
B)
C)
D)
E)
OH
V
I
II
III
IV
V
Ans: E
46
Chapter 2
Topic: Functional groups
38. Which compound is a primary amine with the formula C5H13N?
H2N
NH
H2N
I
II
III
N
NH2
IV
A)
B)
C)
D)
E)
V
I
II
III
IV
V
Ans: C
Topic: Functional groups
39. Which compound can be classified as an ester as well as a ketone?
O
O
O
O
O
O
I
II
O
III
O
O
O
OH
OH
IV
A)
B)
C)
D)
E)
O
V
I
II
III
IV
V
Ans: D
47
Chapter 2
Topic: Functional groups
40.
A)
B)
C)
D)
E)
The C–O–C bond angle in diethyl ether is predicted to be approximately:
90º
105º
110º
120º
180º
Ans: C
Topic: Functional groups
41. Which compound(s) contain(s) tertiary carbon atom(s)?
F
I
II
III
Br
OH
OH
IV
A)
B)
C)
D)
E)
V
I, II, III
I
II, III
I, IV
V
Ans: D
Topic: Isomers
42. The number of unique open-chain structures corresponding to the molecular formula
C3H5Cl is:
A) 2
B) 3
C) 4
D) 5
E) 6
Ans: C
48
Chapter 2
Topic: General, Bonding
43. The C4-C5 carbon-carbon bond in the following molecule results from the overlap of
which orbitals ( in the order C4-C5) ?
o
7
5
A)
B)
C)
D)
E)
1
3
4
6
2
sp–sp2
sp–sp3
sp2–sp2
sp2–sp3
sp3–sp2
Ans: E
Topic: Functional groups
44. An example of a tertiary amine is:
H
N
NH2
I
II
NH2
IV
A)
B)
C)
D)
E)
H2N
III
N
V
I
II
III
IV
V
Ans: E
49
Chapter 2
Topic: Functional groups
45. Which functional groups are present in the following compound?
A)
B)
C)
D)
E)
Alkene, 1º alcohol, ketone
Alkene, 2º alcohol, aldehyde
Alkene, 2º alcohol, ketone
Alkyne, 1º alcohol, aldehyde
Alkyne, 2º alcohol, ketone
Ans: B
Topic: Functional groups, Isomerism
46.
A)
B)
C)
D)
E)
How many constitutional isomers are possible with the formula C4H10O?
3
4
5
6
7
Ans: E
Topic: Functional groups
47. A tertiary carbon atom is present in which of these compounds?
Cl
I
Cl
II
III
OH
HO
IV
A)
B)
C)
D)
E)
V
I
II, IV
III, V
IV
All of these
Ans: C
50
Chapter 2
Topic: Functional groups
48. Which of these compounds is a secondary alkyl chloride?
A) CH3CH2CH2CH2CH2Cl
CH3
B)
CH3CCH2CH3
Cl
C) CH3CHCH2CH2CH3
Cl
D) CH3CH2CHCl
CH2CH3
E) Two of these
Ans: E
Topic: Functional groups, Isomerism
49.
A)
B)
C)
D)
E)
How many 2º alkyl bromides, neglecting stereoisomers, exist with the formula C6H13Br?
4
5
6
7
8
Ans: C
51
Chapter 2
Topic: Functional Groups
50. Many organic compounds contain more than one functional group. Which of the
following is both an aldehyde and an ether?
O
O
O
O
O
I
O
II
O
III
O
O
OCH3
O
IV
A)
B)
C)
D)
E)
V
I,
II, IV
V
I, V
III
Ans: A
Topic: IR Spectroscopy
51. An oxygen-containing compound shows strong IR absorption at 1630-1780 cm-1 and
3200-3550 cm-1. What type of compound is it likely to be?
A) An alcohol
B) A carboxylic acid
C) An ether
D) A ketone
E) An aldehyde
Ans: B
Topic: IR Spectroscopy
52. The absorption band for the O-H stretch in the IR spectrum of an alcohol is sharp and
narrow in the case of:
A) a Nujol mull of the alcohol.
B) a concentrated solution of the alcohol.
C) a gas phase spectrum of the alcohol.
D) the spectrum of the neat liquid
E) none of these
Ans: C
52
Chapter 2
Topic: IR Spectroscopy
53. A split peak for the IR absorption due to bond stretching is observed for the carbonyl
group in which of these compounds?
O
A)
CH3CH2CH2COH
O
B)
CH3CH2CCl
O
C)
ca
CH3CH2CNH2
O
D)
CH3CH2COCH2CH3
O O
E)
CH3CH2COCCH2CH3
Ans: E
Topic: IR Spectroscopy
54.
A)
B)
C)
D)
E)
The IR stretching frequency occurs at the lowest frequency for which of these bonds?
C–H
C–O
C–Br
C–N
C–F
Ans: C
Topic: IR Spectroscopy
55. The IR stretching frequency can be expected to occur at the lowest frequency for which
of these bonds?
A) C–H
B) O–H
C) N–H
D) S–H
E) Difficult to predict
Ans: D
53
Chapter 2
Topic: IR Spectroscopy
56. The IR stretching frequency can be predicted to occur at the highest frequency for which
of these bonds?
A) C–H
B) C–F
C) C–Cl
D) C–Br
E) C–I
Ans: A
Topic: IR Spectroscopy
57.
A)
B)
C)
D)
E)
An anticipated IR absorption band may not be observed because:
it occurs outside the range of the instrument used.
no change occurs in the dipole moment during the vibration.
the absorption band is eclipsed by another.
the intensity is so weak that it cannot be differentiated from instrument noise.
All of these
Ans: E
Topic: IR Spectroscopy
58. IR evidence for the presence of the C=C would be most difficult to detect in the case of
which of these alkenes?
A)
B)
C)
D)
E)
Ans: D
54
Chapter 2
Topic: IR Spectroscopy
59. The IR absorption due to the stretching of which of these carbon-hydrogen bonds occurs
at the highest frequency?
H
H
H
I
II
III
H
IV
A)
B)
C)
D)
E)
H
V
I
II
III
IV
V
Ans: E
Topic: IR Spectroscopy
60. The IR spectrum of which type of compound will not show evidence of hydrogen
bonding?
A) Aldehyde
B) Alcohol
C) Carboxylic acid
D) Phenol
E) Primary amine
Ans: A
Topic: IR Spectroscopy
61. The IR spectrum of which type of compound generally exhibits evidence of hydrogen
bonding?
A) Aldehyde
B) Carboxylic acid
C) Alkene
D) Ester
E) Ketone
Ans: B
55
Chapter 2
Topic: Intermolecular forces
62. The following substance is expected to have low solubility in which of the following
solvent(s)?
O Na
O
A)
B)
C)
D)
E)
CCl4
C2H5OH
CHCl3
CH2OHCH2CH2CH2CH2CH2OH
The given substance is likely to be quite soluble in all of the solvents described
Ans: A
Topic: IR Spectroscopy
63.
A)
B)
C)
D)
E)
The IR stretching frequency occurs at the lowest frequency for which of these bonds?
B–H
O-H
N-H
S-H
Difficult to predict
Ans: D
Topic: IR Spectroscopy
64. An oxygen-containing compound which shows sharp IR absorption at 2200 cm-1 and
3300 cm-1 is likely to be what type of compound?
A) An ester
B) An alkene
C) An alkyne
D) An ether
E) An aldehyde
Ans: B
56
Chapter 2
Topic: IR Spectroscopy
65. The IR spectrum of which of the following substances is likely to show a small, but
sharp peak at 2200 cm-1?
OH
O
N
H
II
I
IV
A)
B)
C)
D)
E)
III
V
I
II
III
IV
V
Ans: E
SHORT ANSWER QUESTIONS
Topic: Functional Groups
66. Hydrocarbons containing carbon-carbon double bonds are referred to as ___________.
Ans: alkenes
Topic: Functional Groups, IR Spectroscopy
67. An IR spectrum has significant peaks at 2200 and 3300 cm-1. What functional group is
present in the molecule?
Ans: A terminal alkyne
Topic: Functional Groups
68. A group in which a carbon atom has a double bond to an oxygen atom is called a
__________.
Ans: carbonyl
57
Chapter 2
Topic: Functional Groups, IR Spectroscopy
69. An IR spectrum has significant peaks at 3080 and 1650 cm-1. What functional group is
present in the molecule?
Ans: an alkene
Topic: Functional Groups, IR Spectroscopy
70. Examine the following IR spectrum, for substance P (C8H22O). Which oxygen
containing functional group is present in P?
Ans: aldehyde
Topic: Functional Groups, IR Spectroscopy
58
Chapter 2
71. Examine the following IR spectrum, for substance P (C5H12O). Which oxygen
containing functional group is present in P?
Ans: alcohol
Topic: General
72. The six p-electrons in benzene are _____________ about the ring, which explains why
all of the C-C bonds are the same length.
Ans: delocalized
Topic: General
73. A polar covalent bond is one in which electrons are _____________.
Ans: not shared equally
Topic: Functional Groups
74. Organic compounds are classified into chemical families on the basis of similarities in
chemical properties; these similarities are primarily due to the presence of characteristic
arrangements of atoms known as ________________.
Ans: functional groups
Topic: Functional Groups
75. Unsaturated hydrocarbons may be distinguished from saturated hydrocarbons by the
presence of one or more _____________.
Ans: Pi bonds
Topic: Bonding, Solubility
76. Sodium chloride, which is quite soluble in water, is not very soluble in hexane. Why?
Ans: Sodium chloride, which is an ionic substance, is soluble in a polar solvent such as
water, but not in a non-polar solvent such as hexane.
Topic: Isomers, Functional Groups
77. Draw all tertiary amine isomers of C6H15N.
Ans:
N
N
N
N
N
N
59
Chapter 2
Topic: Isomers, Functional Groups
78. Draw all isomers of C6H14.
Ans:
Topic: Isomers, Functional Groups
79. Draw a structural formula for C8H18 , in which there are two quaternary carbons.
Ans:
Topic: Isomers, Functional Groups
80. Draw all isomers of C5H10O that are ketones.
O
O
O
Ans:
Topic: Isomers, Functional Groups
81. Draw all isomers of C3H8O and classify each according to functional group
OH
Ans:
C3H8O
OH
O
primary alcohol
secondary alcohol
ether
Topic: Isomers, Functional Groups
82. Draw all isomers of C6H12O that are aldehydes.
Ans:
O
O
O
O
O
O
O
Topic: Isomers, Functional Groups
83. Draw all isomers of C6H12O that are aldehydes and contain at least one tertiary carbon
Ans:
O
O
O
60
O
Chapter 2
Topic: IR Spectroscopy
84. The IR absorption frequencies of the C-H bond in alkanes, alkenes and alkynes are
measurably different. Briefly explain why.
Ans: IR absorption frequency depends on bond strength; the bond strength of C-H
bonds in alkanes, alkenes and alkynes is different because different atomic
orbitals (hybridized) of carbon are involved in the bond: the C-H bond in alkanes
is described as (sp3-s), that in alkenes is (sp2-s) and in alkynes, it is (sp-s). The
relative % s v. % p character of the hybrid orbitals of carbon would indicate
different bond lengths /bond strengths for alkanes, alkenes and alkynes, with the
bond length / bond strength being the longest/weakest respectively. This results
in different IR absorption frequencies.
Topic: Molecular Geometry, Dipole Moment
85. Carbon dioxide is non- polar, despite the fact that oxygen is much more electronegative
than carbon. Briefly explain why, using relevant diagrams as appropriate to illustrate
your Ans:wer.
Ans: The overall dipole moment of a polyatomic molecule depends on two factors: the
polarity of various bonds and molecular geometry, since dipole forces have both
magnitude and direction. In some molecules containing bonds of identical
polarity, the molecular geometry may result in a net cancellation of the overall
dipole forces. This is what happens in carbon dioxide: although there are two
polar C-O bonds, because of the linear geometry of the molecule, the net dipole is
zero.
..
..
: O C O:
Topic: Intermolecular Forces
86. Ethanol, C2H5OH, and propane, C3H8, have approximately the same molar mass, yet,
ethanol has a much higher boiling point. Briefly explain why.
Ans: Strong hydrogen bonding between molecules of ethanol leads to elevation in
boiling point. No hydrogen bonding is possible between molecules of propane,
resulting in a lower boiling point compared with ethanol.
Topic: Intermolecular Forces
87. Ethanol, C2H5OH, and dimethyl ether, CH3OCH3, have the same molar mass, yet,
ethanol has a much higher boiling point. Briefly explain why.
Ans: Strong hydrogen bonding between molecules of ethanol leads to elevation in
boiling point. No hydrogen bonding is possible between molecules of dimethyl
ether, resulting in a lower boiling point compared with ethanol.
61
Chapter 2
Topic: IR Spectroscopy
88. IR absorption signals of alcohols are typically broad. However, IR spectra of gaseous
samples show sharp peaks. Briefly explain why.
Ans: Broad signals of alcohols are due to hydrogen bonding associated with the O-H
group. In gaseous samples, no hydrogen bonding is possible, and the signal
becomes sharp.
62
Chapter 3
MULTIPLE CHOICE QUESTIONS
Topic: Acid-Base Definitions
1. According to the Lewis definition, a base is a(n):
A) Proton donor.
B) Electron pair donor.
C) Hydroxide ion donor.
D) Hydrogen ion donor.
E) Electron pair acceptor.
Ans: B
Topic: Acid-Base Definitions
2. Which of the following is not both a Bronsted-Lowry acid and a Bronsted-Lowry base?
A) HSO4−
B) H2PO4−
C) HCO3−
D) OH−
E) SH−
Ans: D
Topic: Acid-Base Definitions
3. Which of the following is not a conjugate acid - conjugate base pair (in that order)?
A) H3PO4, H2PO4−
B) HBF4, BF4−
C) CH3CH2OH, CH3CH2O−
D) H3O+, H2O
E) HPO4−, H2PO4−
Ans: E
Topic: Acid-Base Definitions
4. The conjugate base of sulfuric acid is:
A) H3SO4+
B) SO3
C) HSO4−
D) H2SO3
E) HSO3−
Ans: C
63
Chapter 3
Topic: Acid-Base Definitions
5. Consider the equilibrium
PO43− +
H2O ⇄
ΗPΟ42− +
ΟΗ−
Which are the Bronsted-Lowry bases?
A) PO43− and HPO42−
B) PO43− and OH−
C) PO43− and H2O
D) H2O and OH−
E) H2O and HPO42−
Ans: B
Topic: Acid-Base Definitions
6. Which of these is not a true statement?
A) All Lewis bases are also Bronsted-Lowry bases.
B) All Lewis acids contain hydrogen.
C) All Bronsted-Lowry acids contain hydrogen.
D) All Lewis acids are electron deficient.
E) According to the Bronsted-Lowry theory, water is both an acid and a base.
Ans: B
Topic: Acid-Base Definitions
7. For the equilibrium
CH3NH3+ + H2O ⇄ CH3NH3+ + H3O+
the two substances which both are acids are:
A) H2O and H3O+
B) CH3NH3+ and H2O
C) CH3NH3+ and CH3NH2
D) CH3NH3+ and H3O+
E) CH3NH2 and H2O
Ans: D
64
Chapter 3
Topic: Acid-Base Definitions
8. Which of the following is not a Lewis base?
A) NH3
B) H−
C) BF3
D) H2O
E) H3C−
Ans: C
Topic: Acid-Base Definitions
9. Which of the following is not a Bronsted-Lowry acid?
A) H2O
B) (CH3)3N
C) NH4+
D) CH3CO2H
E) HC≡CH
Ans: E
Topic: Acid-Base Definitions
10. The reaction between which combination of substances below cannot be classified as a
Bronsted-Lowry acid-base reaction?
A) CH3Li + C2H5OH
B) H2SO4 + CH3CO2Na
C) BF3 + NH3
D) H3O+ + CH3NHE) two of the above
Ans: C
Topic: Acid-Base Definitions
11.
A)
B)
C)
D)
E)
Which of these is not a Lewis acid?
AlCl3
H3O+
FeCl3
SO3
C4H10
Ans: E
65
Chapter 3
Topic: Acid-Base Definitions
12. This species is a carbon-based Lewis acid:
A) CH4
B) HCCl3
C) CH3+
D) :CH3−
E) ·CH3
Ans: C
Topic: Acid-Base Definitions
13. What is the conjugate base of ethanol?
A) CH3CH2O−
B) CH3CH2−
C) CH3CH2OH2+
D) CH3CH3
E) CH3OCH3
Ans: A
Topic: Acid-Base Strength
14. Which of the acids below would have the strongest conjugate base?
A) CH3CH2OH pKa = 18
B) CH3CO2H pKa = 4.75
C) ClCH2CO2H pKa = 2.81
D) Cl2CHCO2H pKa = 1.29
E) Cl3CCO2H pKa = 0.66
Ans: A
Topic: Acid-Base Definitions
15. Which of the following is a Lewis acid?
A) H3O+
B) BF3
C) NF3
D) OH−
E) N≡N
Ans: B
66
Chapter 3
Topic: Acid-Base Reactions
16. Adding sodium hydride to ethanol would produce:
D) CH3CH2Na + NaOH
A) CH3CH2OCH2CH3 + H2
B) CH3CH2OCH2CH3 + NaOH
E) CH3CH3 + NaOH
C) CH3CH2ONa + H2
D)CH3CH2Na + NaOH
E) CH3CH3 + NaOH
Ans: C
Topic: Acid-Base Reactions
17. Adding sodium amide (NaNH2) to 1-butyne (CH3CH2C≡CH) would produce:
A) CH3CH2C≡CNa + NH3
B) CH3CH2C≡C-C≡CH2CH3 + NaH + NH3
C) CH3CH2C≡CNH2 + NaH
D) NaCH2CH2C≡CH + NH3
E) CH3CH(Na)C≡CH + NH3
Ans: A
Topic: Acid-Base Reactions
18. Which acid-base reaction would not take place as written?
A) CH3Li + CH3CH2OH ⎯⎯⎯⎯→ CH4 + CH3CH2OLi
B) HC≡CH + NaOH ⎯⎯⎯⎯→ HC≡CNa + H2O
C) HC≡CNa + H2O ⎯⎯⎯⎯→ HC≡CH + NaOH
D) CH3OH + NaH ⎯⎯⎯⎯→ CH3ONa + H2
E) CH3CO2H + CH3ONa ⎯⎯⎯⎯→ CH3CO2Na + CH3OH
Ans: E
Topic: Acid-Base Reactions
19. Which acid-base reaction would not take place as written?
A) CH3Li + CH3CH2CH2CH2NH2 ⎯⎯⎯⎯→ CH4 + CH3CH2CH2CH2NHLi
B) CH3C≡CH + NaOCH3 ⎯⎯⎯⎯→ HC≡CNa + CH3OH
C) HC≡CNa + H2O ⎯⎯⎯⎯→ HC≡CH + NaOH
D) CH3OH + NaNH2 ⎯⎯⎯⎯→ CH3ONa + NH3
E) CH3CO2H + CH3ONa ⎯⎯⎯⎯→ CH3CO2Na + CH3OH
Ans: B
67
Chapter 3
Topic: Acid-Base Reactions
20. Which acid-base reaction would not take place as written?
A) CH3Li + CH3CH2OH ⎯⎯⎯⎯→ CH4 + CH3CH2OLi
B) H2C=CH2 + NaOH ⎯⎯⎯⎯→ H2C=CHNa + H2O
C) CH3C≡CNa + H2O ⎯⎯⎯⎯→ CH3C≡CH + NaOH
D) (CH3)2CHOH + NaH ⎯⎯⎯⎯→ (CH3)2CHONa + H2
E) CH3CO2H + CH3ONa ⎯⎯⎯⎯→ CH3CO2Na + CH3OH
Ans: B
Topic: Acid-Base Reactions
21. The amide ion, NH2−, is a base which can be used only in which of the solvents shown
below:
A) CH3OH
B) CH3CH2OH
C) H2O
D) D2O
E) Liquid NH3
Ans: E
Topic: Acid-Base Reactions
22. Acetic acid dissociates most completely in:
A) CCl4
B) Cl2C=CCl2
C) H2O
D) (CH3CH2)2O
E) the gas phase.
Ans: C
Topic: Acid-base definitions, Acid-Base Reactions
23. Which pair of species are both bases in the following reaction?
HCN and H2O ⇄
A)
B)
C)
D)
E)
H3O+ and CN−
H2O and CN−
H3O+ and H2O
HCN and H3O+
HCN and CN−
H3O+ and CN−
Ans: A
68
Chapter 3
Topic: Acid-Base Reactions
24. What compounds are produced when sodium nitrate is added to a mixture of water and
ethanol?
A) HNO3 + NaOH
B) HNO3 + CH3CH2ONa
C) NaOH + CH3CH2ONa
D) CH3CH2OCH2CH3 + NaOH
E) No reaction occurs.
Ans: E
Topic: Acid-Base Reactions
25. Which reaction of these potential acids and bases does not occur to any appreciable
degree due to an unfavorable equilibrium?
A) NaOH (aq) + CH3CH2CH2CO2H
B) CH3CH2ONa in ethanol + ethene
C) CH3Li in hexane + ethyne
D) NaNH2 in liq. NH3 + ethanol
E) NaC2H3O2 (aq) + HI
Ans: B
Topic: Acid-Base Reactions
26. Which reaction of these potential acids and bases does not occur to any appreciable
degree due to an unfavorable equilibrium?
A) NaNH2 + CH3CH2CH2CH2CH2CH3
B) CH3CH2CO2Na + HI
C) CH3Li in hexane + ethyne
D) NaH + methanol
E) Two of the above will not occur
Ans: A
Topic: Acid-Base Reactions
27. Which reaction of these potential acids and bases does not occur to any appreciable
degree due to an unfavorable equilibrium?
A) NaNH2 + CH3CH2CH2CH2CH2OH
B) CH3CH2CO2H + NaCl
C) NaHCO3 + CH3CH2CO2H
D) NaH + CH3CH2CH2CH2CH2NH2
E) all of the above will occur due to a favorable equilibrium
Ans: E
69
Chapter 3
Topic: Acid-Base Reactions
28. Which combination of reagents is the least effective in generating sodium ethoxide,
CH3CH2ONa?
A) CH3CH2OH + NaH
B) CH3CH2OH + NaNH2
C) CH3CH2OH + NaOH
D) CH3CH2OH + CH3Li
E) CH3CH2OH + HC≡CNa
Ans: C
Topic: Acid-Base Reactions
29. Which combination of reagents is effective in generating sodium ethoxide,
CH3CH2ONa?
A) CH3CH2OH + KH
B) CH3CH2OH + NaNH2
C) CH3CH2OH + Na
D) Two of the above
E) All of the above
Ans: E
Topic: Acid-Base Reactions
30. Which combination of reagents is effective in generating sodium propynide,
CH3C≡CNa?
A) CH3C≡CH + KH
B) CH3C≡CH + (CH3)2CHONa
C) CH3C≡CH + C2H5Li
D) Two of the above
E) All of the above
Ans: B
Topic: Acid-Base Reactions, base strength
31. In the reaction, Na+NH2− + CH3OH ⎯⎯⎯⎯→ CH3O−Na+ + NH3, the stronger
base is:
A) NaNH2
B) CH3OH
C) CH3ONa
D) NH3
E) This is not an acid-base reaction.
Ans: A
70
Chapter 3
Topic: Acid-Base Reactions, isotope labeling
32. Which sequence is the best one to use to prepare CH3C≡CD?
D2O
A) CH C CH NaH
3
B) CH C CH
3
C)
CH3C
CH
NaOH
D2O
CH3ONa
D2O
D) CH C CH DOH
3
E) None of these will be successful.
Ans: A
Topic: Acid-Base Reactions
33. Adding sodium hydride, NaH, to water produces:
A) H2 and NaOH(aq)
B) H-(aq) + Na+(aq)
C) H3O+(aq) + Na+(aq)
D) H3O−(aq) + Na+(aq)
E) Na2O + H2
Ans: A
Topic: Acid-Base Reactions
34. Adding methyllithium , CH3Li, to ethanol produces:
A) CH3CH2Li + CH3OH
B) CH3CH2OLi + CH4
C) CH3CH2OCH3 + LiH
D) All of the above
E) No reaction takes place
Ans: A
Topic: Acid-Base Reactions, isotope labeling
35. Which reaction will yield CH3CH2-D?
A) CH3CH3 + D2O
B) CH3CH2Li + D2O
C) CH3CH2OLi + D2O
D) CH3CH2OH + D2O
E) More than one of these
Ans: B
71
Chapter 3
Topic: Acid-Base Reactions, isotope labeling
36. A product of the reaction, CH3CH2Li + D2O ⎯⎯⎯⎯→ is
A) CH3CH2OD
B) CH3CH2CH2CH3
C) CH2=CH2
D) CH3CH2D
E) CH3CH2OCH2CH3
Ans: D
Topic: Relative acid strength
37. The compounds ethane, ethene, and ethyne exhibit this order of increasing acidity:
A) Ethyne < ethene < ethane
B) Ethene < ethyne < ethane
C) Ethane < ethyne < ethene
D) Ethane < ethene < ethyne
E) Ethene < ethane < ethyne
Ans: D
Topic: Relative acid/base strength
38. Which is an incorrect statement?
A) RSH compounds are stronger acids than ROH compounds.
B) PH3 is a weaker base than NH3.
C) NH2− is a stronger base than OH−.
D) OH− is a stronger base than OR−.
E) H− is a stronger base than OR−.
Ans: D
Topic: Relative acid/base strength
39. The correct sequence of the ions shown, in order of increasing basicity, is:
A) CH3CH2:− < CH2=CH:− < HC≡C:−
B) CH3CH2:− < HC≡C:− < CH2=CH:−
C) HC≡C:− < CH3CH2:− < CH2=CH:−
D) CH2=CH:− < HC≡C:− < CH3CH2:−
E) HC≡C:− < CH2=CH:− < CH3CH2:−
Ans: E
72
Chapter 3
Topic: Solvent effects
40. Which is a protic solvent?
A) CCl4
B) HCCl3
C) CH3OH
D) CH3(CH2)4CH3
E) CH3CH2OCH2CH3
Ans: C
Topic: Solvent effects
41. Which might be used as protic solvent?
O
A)
O
B)
C)
H2N
O
N
D)
O
O
E)
Ans: B
Topic: Solvent effects
42. Which of the following classes of substances cannot be used as protic solvents?
A) esters
B) aldehydes
C) ketones
D) carboxylic acids
E) two of the above
Ans: D
73
Chapter 3
Topic: Acid constants
43. If a 0.01 M solution of a weak acid has a pH of 4.0, the pKa of the acid is:
A) 10.0
B) 8.0
C) 6.0
D) 4.0
E) 2.0
Ans: C
Topic: General
44. Which one of the following is a true statement?
A) The stronger the acid, the larger is its pKa.
B) The conjugate base of a strong acid is a strong base.
C) Acid-base reactions always favor the formation of the stronger acid and the stronger
base.
D) Strong acids can have negative pKa values.
E) Hydrogen need not be present in the molecular formula of a Bronsted-Lowry acid.
Ans: D
Topic: Relative acid/base strengths
45. The basic species are arranged in decreasing order of basicity in the sequence:
A) F− > OCH3− > NH2− > CH3CH2−
B) OCH3− > CH3CH2− > NH2− > F−
C) CH3CH2− > NH2− > OCH3− > F−
D) NH2− > CH3CH2− > F− > OCH3−
E) NH2− > OCH3− > CH3CH2− > F−
Ans: C
Topic: pKa calculations
46. A particular acid has Ka = 2.0 x 10-5 (in aqueous solution). The evaluation of which of
these expressions gives the value for pKa?
A) 10-14/2.0 x 10-5
B) 10-14(2.0 x 10-5)
C) 5 – log 2.0
D) -5 + log 2.0
E) 2.0 x 10-5/10-14
Ans: C
74
Chapter 3
Topic: Solvent effects
47. As a consequence of the "leveling effect," the strongest acid which can exist in
appreciable concentration in aqueous solution is:
A) H3O+
B) H2SO4
C) HClO4
D) HCl
E) HNO3
Ans: A
Topic: Relative acid/base strengths
48. Based on the position of the central atom in the periodic chart, we predict that the
strongest acid of the following is:
A) H2O
B) H2S
C) H2Se
D) H2Te
Ans: D
Topic: Acid dissociation, thermodynamic calculations
49. An acid, HA, has the following thermodynamic values for its dissociation in water at
27º C: ΔH = -8.0 kJ mol-1; ΔS = -70 kJ K-1mol-1. The ΔG for the process is:
A) +29 kJ mol-1
B) +13 kJ mol-1
C) -6.1 kJ mol-1
D) -13 kJ mol-1
E) -29 kJ mol-1
Ans: B
Topic: Relative acid/base strengths, solvent effects
50. Which of these bases is the strongest one which can be used (and retains its basic
character) in aqueous solution?
A) OCH3−
B) F−
C) OH−
D) C2H3O2−
E) HSO4−
Ans: C
75
Chapter 3
Topic: Acidity constant- thermodynamic principles
51. The acidity constant, Ka, differs from the equilibrium constant, Keq, for the dissociation
of the same acid in water at the same temperature and concentration in what way?
A) Ka can be determined experimentally with less accuracy than Keq.
B) The two terms are identical numerically.
C) Ka is used for strong acids only; Keq for weak acids.
D) Ka is the reciprocal of Keq.
E) Keq = Ka/[H2O].
Ans: E
Topic: Acids/bases-general principles
52. Which of the following is an untrue statement?
A) The % dissociation of a weak acid increases with increasing dilution of the acid
solution.
B) The stronger an acid, the weaker its conjugate base.
C) The larger the value of Ka for an acid, the smaller the value of its pKa.
D) Comparison of the acidity of strong acids in solution requires the use of a solvent
less basic than water.
E) The stronger the acid, the more positive the value of ΔGº for the dissociation.
Ans: E
Topic: Acid-base reactions
53. When proton transfer reactions reach equilibrium, there have been formed:
A) the weaker acid and the weaker base.
B) the weaker acid and the stronger base.
C) the stronger acid and the weaker base.
D) the stronger acid and the stronger base.
E) All proton transfers go to completion; they are not equilibrium processes.
Ans: A
Topic: Relative acid-base strengths
54. For the simple hydrides, MHn, pKa values decrease in the order:
A) CH4 > NH3 > H2O > H2S > HBr
B) HBr > H2S > H2O > NH3 > CH4
C) HBr > H2O > NH3 > H2S > CH4
D) NH3 > H2S > CH4 > H2O > HBr
E) H2S > H2O > HBr > NH3 > CH4
Ans: A
76
Chapter 3
Topic: Relative acid-base strengths, solvent effects
55. The compound aniline, C6H5NH2, has weakly basic properties in aqueous solution. In
this other solvent, aniline would behave as a strong base.
A) CH3OH
B) CH3CH2OH
C) CF3CO2H
D) Liquid NH3
E) CH3(CH2)4CH3
Ans: C
Topic: Relative acid-base strengths
56. Which of the following organic compounds is the strongest acid?
A) C6H12 pKa = 52
B) CH3CH3 pKa = 50
C) CH3CH2OH pKa = 18
D) CH3CO2H pKa = 5
E) CF3CO2H pKa = 0
Ans: E
Topic: Relative acid-base strengths
57. Which is the strongest acid?
A) CH3CH2OH
B) CH3CO2H
C) HC≡CH
D) CH2=CH2
E) CH3CH3
Ans: B
Topic: Relative acid-base strengths
58.
A)
B)
C)
D)
E)
Which is the strongest acid?
CH2ClCH2CH2CH2CH2CO2H
CH3CHBrCH2CH2CH2CO2H
CH3CH2CH2CBr2CH2CO2H
CH3CH2CH2CHFCH2CO2H
CH3CH2CH2CF2CH2CO2H
Ans: E
77
Chapter 3
Topic: Relative acid-base strengths
59.
A)
B)
C)
D)
E)
Which is the strongest acid?
CH3CH2CH2CH2CHFCO2H
CH3CHBrCH2CH2CH2CO2H
CH3CH2CH2CHClCH2CO2H
CH3CH2CH2CHFCH2CO2H
CH3CH2CH2CHICH2CO2H
Ans: A
Topic: Relative acid-base strengths
60.
A)
B)
C)
D)
E)
Which is the weakest acid?
CH3CH2CH2CH2CHFCO2H
CH3CHCH2CH2CH2CH2OH
CH3CH2CH2CH2CH2SO3H
CH3CH2CH2CH2CH=CH2
CH3CH2CH2CH2NH2
Ans: A
Topic: Relative acid-base strengths
61.
A)
B)
C)
D)
E)
Which of the following substances has a hydrogen atom with pKa ≈25?
CH3CH2CH2CH2CO2H
CH3CHCH2C≡CCH3
CH3CH2CH2C≡CH
CH3CH2CH2CH2CH=CH2
CH3CH2CH2CH2NH2
Ans: C
78
Chapter 3
Topic: Relative acid-base strengths
62. Hydrogen atom(s) from which position(s) is (are) most likely to be abstracted when the
following substance is treated with NaH?
9
10
8
1
6
7
A)
B)
C)
D)
E)
4
5
2
3
1
1, 5, 6
1, 2
3, 4, 7, 8, 9, 10
Hydrogens from all of the positions are equally likely to be abstracted by NaH.
Ans: A
Topic: Relative acid-base strengths
63. Which of the following correctly lists the compounds in order of decreasing acidity?
A) H2O > HC≡CH > NH3 > CH3CH3
B) HC≡CH > H2O > NH3 > CH3CH3
C) CH3CH3 > HC≡CH > NH3 > H2O
D) CH3CH3 > HC≡CH > H2O > NH3
E) H2O > NH3 > HC≡CH > CH3CH3
Ans: A
Topic: Relative acid-base strengths
64. Select the strongest base.
A) OH−
B) RC≡C−
C) NH2−
D) CH2=CH−
E) CH3CH2−
Ans: E
79
Chapter 3
Topic: Relative acid-base strengths
65. A group of acids arranged in order of decreasing acidity is:
HNO3 > CH3COOH > C6H5OH > H2O > HC≡CH
What is the arrangement of the conjugate bases of these compounds in decreasing order
of basicity?
A) NO3− > CH3COO− > C6H5O− > OH− > HC≡C−
B) CH3COO− > C6H5O− > NO3− > OH− > HC≡C−
C) C6H5O− > NO3− > HC≡C− > OH− > CH3COO−
D) HC≡C− > OH− > C6H5O− > CH3COO− > NO3−
E) No prediction of relative base strength is possible.
Ans: D
Topic: Relative acid-base strengths
66. What prediction can be made of the relative strengths of the conjugate bases of: H2S,
HCl, SiH4, PH3?
A) PH2− > SiH3− > HS− > Cl−
B) SiH3− > PH2− > HS− > Cl−
C) Cl− > HS− > PH2− > SiH3−
D) HS− > Cl− > SiH3− > PH2−
E) Cl− > PH2− > SiH3− > HS−
Ans: B
Topic: Acids/bases- general
67. Which of these species is not amphoteric?
A) HC≡C−
B) HS−
C) NH3
D) CH3−
E) HPO4−
Αns: D
80
Chapter 3
Topic: Acids/bases- general
68. Which of these phosphorus-based acids is dibasic?
O
A)
P
HO
OH
OH
O
B)
HO
P
OH
H
O
C)
H
P
H
OH
O
D)
HO
P
O
O
OH
O O
E)
HO
P
P
P
OH
OH
OH
OH OH
Ans: B
Topic: Acids/bases- general
69. Why cannot one determine the relative acid strengths of HClO4 and HNO3 using
aqueous solutions of these acids?
A) The acids are insufficiently soluble for the measurements.
B) A more basic solvent than H2O must be used.
C) H2O is too basic a solvent for the distinction to be made.
D) These oxidizing acids cause redox reactions to occur.
E) Actually, the acid strengths can be determined using aqueous solutions.
Ans: C
Topic: Acids/bases- general
70. Which of these is not a diprotic acid?
A) H2S
B) H2SO4
C) H2O
D) (COOH)2
E) H2PO4Ans: C
81
Chapter 3
Topic: Acids/bases- thermodynamic principles
71. Which set contains non-equivalent members?
A) Enthalpy and heat content
B) Endothermic reaction and +ΔH
C) Exothermic reaction and -ΔH
D) Kinetic energy and energy of motion
E) High energy and high stability
Ans: E
SHORT-ANSWER QUESTIONS:
Topic: General Reactivity
72. Addition reactions are characteristic of compounds with ______________.
Ans: multiple bonds
Topic: Reaction Types
73. The four basic types of reactions are: ________________.
Ans: substitution, addition, elimination, rearrangement
Topic: Reaction Mechanisms
74. The process of bond-breaking where each fragment takes away one of the electrons
from the bond is called ____________.
Ans: homolysis
Topic: Reaction Mechanisms
75. Heterolytic bond-breaking produces __________.
Ans: charged fragments/ions
Topic: Acids and Bases
76. According to Bronsted-Lowry theory, an acid is a substance that can ____________.
Ans: donate a proton
Topic: Acids and Bases
77. According to Lewis theory, a base is a substance that can _________.
Ans: donate a lone pair of electrons
82
Chapter 3
Topic: Acids and Bases
78. The molecule or ion that is formed when an acid loses its proton is called the
______________.
Ans: conjugate base
Topic: Nucleophiles and Electrophiles
79. Reagents that seek to react with a proton or some other electron-deficient center are
called ____________.
Ans: nucleophiles
Topic: Acids and Bases
80. A substance that can donate a lone pair of electrons is a ____________ according to
____________ theory.
Ans: base; Lewis
Topic: Curved Arrow Notation
81. When drawing mechanisms, chemists generally use curved arrows. The curved arrow
begins with ______________ and points toward ______________.
Ans: a lone pair or covalent bond; a site of electron deficiency
Topic: Acids and Bases
82. Why do water-insoluble carboxylic acids dissolve in aqueous sodium hydroxide?
Ans: Because they are converted to water-soluble salts.
Topic: Relative Acidity
83. Bond polarization that takes place through space and through the bonds of the molecule
is called the _____________.
Ans: inductive effect
Topic: Energy
84. What are the two fundamental types of energy?
Ans: potential energy and kinetic energy
Topic: Types of Solvents
85. Define a protic solvent.
Ans: one that has a hydrogen atom attached to a strongly electronegative element such as
oxygen
83
Chapter 3
Topic: Relative acid strength
86. Briefly, but clearly, explain why the –OH hydrogen in acetic acid (CH3CO2H) is more
acidic than in ethanol (C2H5OH).
Ans: The greater acidity of the –OH hydrogen in acetic acid is due primarily to two
factors: resonance and inductive effects. In acetic acid, the presence of the neighboring
carbonyl group has an electron-withdrawing inductive effect on the –OH hydrogen,
increasing its acidity relative to ethanol, in which there is no carbonyl group. Also,
comparison of the respective conjugate bases shows that, unlike the ethoxide ion, the
acetate ion is stabilized by resonance. This too, is reflected in the greater acid strength
of the corresponding acid species.
O
O
H
O
(- H+)
O
Inductive effect:
electron withdrawing
O
O
H
O
resonance stabilized anion
(- H+)
O
84
Chapter 3
Topic: Relative acid strength, isotope labeling
87. Isotope labeling is an important tool in the study of reaction mechanisms. How will you
selectively deuterate the specified hydrogen atom, indicated by an arrow, in the
following compound? Use equations to clarify your answer and briefly explain your
rationale.
H
Ans: The specified hydrogen is attached to an sp-hybridized carbon and is thus more
acidic than all the other hydrogens in the molecule. By using an appropriate strong
base, such as NaH or NaNH2, the acetylenic hydrogen is selectively removed; adding
D2O then replaces the lost hydrogen atom with a deuterium atom.
H
D
i) NaH or NaNH2
ii) D2O
Topic: Acid-base reactions
88. Write an equation to show the reaction between ethanol, C2H5OH and methyllithium,
CH3Li. Draw all non-bonding electrons and show electron flow with curved arrows.
Ans:
.. H
+
O
..
..
+
O : Li
..
: CH3 Li+
+
CH4
Topic: Acid-base reactions, solvent selection
89. You are planning to carry out a reaction between propyne, CH3C≡CH and sodium
amide, NaNH2. You also need to choose an appropriate solvent for carrying out the
reaction. Would ethanol be suitable for this purpose? Explain your rationale clearly.
Ans: No, ethanol would be unsuitable for carrying out the reaction between propyne and
methyllithium. The pKa of ethanol is ~16, and sodium amide is strong enough a base to
readily abstract a proton from the solvent ( pKa of NH3 is ~38); in doing so, it becomes
deactivated and is no longer available to react with propyne (pKa ~25). Typically, the
selected solvent should be chemically inert toward all substances used in the reaction, so
that it does not interfere with the desired reaction.
85
Chapter 3
Topic: Acid-base reactions, solvent selection
90. You are planning to purify an impure sample of benzoic acid, known to be contaminated
with naphthalene. Propose a strategy for purifying this sample, making use of acid-base
principles, using equations to further clarify your answer.
Ans: Benzoic acid reacts with aqueous NaOH to form a water soluble salt, while
naphthalene, which is not soluble in water to any appreciable extent, does not react with
NaOH. So, upon shaking the impure sample with NaOH (aq), the naphthalene remains
as a solid, while the benzoic acid dissolves in the aqueous medium as sodium benzoate.
After filtration, HCl is added to the filtrate to regenerate benzoic acid, which
precipitates as almost pure crystals. Cooling maximizes the yield of the purified
product, after which pure crystals can be obtained by filtration.
Alternatively, before beginning the reaction with aqueous NaOH, the impure
sample is first dissolved in diethyl ether (benzoic acid and naphthalene are both
soluble in ether) and placed in a separatory funnel. After shaking thoroughly
with aqueous NaOH, the aqueous layer (which now contains sodium benzoate) is
collected. The rest of the procedure is as described earlier.
O
O
OH
NaOH
water insoluble
O
OH
O
ONa
water soluble
NaOH
X
No reaction
water insoluble
86
HCl
OH
water insoluble
PURE
Chapter 4
MULTIPLE CHOICE QUESTIONS
Topic: Nomenclature
1.
A)
B)
C)
D)
E)
The IUPAC name for
6-Ethyl-3,4-dimethylheptane
2-Ethyl-4,5-dimethylheptane
3,4,6-Trimethyloctane
3,5,6-Trimethyloctane
2-(1-Methylpropyl)-4-methylhexane
Ans: C
is:
Topic: Nomenclature
2.
A)
B)
C)
D)
E)
An IUPAC name for
5-Methyl-4-(1-methylpropyl)hexane
2-Methyl-3-(1-methylpropyl)hexane
2-Methyl-3-(2-methylpropyl)hexane
3-Methyl-4-(1-methylethyl)heptane
5-Methyl-4-(1-methylethyl)heptane
Ans: D
is:
Topic: Nomenclature
3. A correct IUPAC name for the following compound is:
A)
B)
C)
D)
E)
2,5-Dimethyl-3-propylheptane
3,6-Dimethyl-5-propylheptane
6-Methyl-4-(1-methylethyl)octane
2-Methyl-3-(2-methylbutyl)hexane
3-Methyl-5-(1-methylethyl)octane
Ans: E
87
Chapter 4
Topic: Nomenclature
4. A correct IUPAC name for the following compound is:
OH
A)
B)
C)
D)
E)
Ans:
Cl
4-propyl-5-chloro-3-heptanol
4-propyl-3-chloro-5-heptanol
4-(1-chloropropyl)-3-heptanol
5-chloro-4-propyl-3-heptanol
3-hydroxy-4-propyl-5-chloroheptane
D
Topic: Nomenclature
5.
A)
B)
C)
D)
E)
Which of the following pairs of compounds represent pairs of constitutional isomers?
2-Methylbutane and pentane
2-Chlorohexane and 3-chlorohexane
sec-Butyl bromide and tert-butyl bromide
Propyl chloride and isopropyl chloride
All of the above
Ans: E
88
Chapter 4
Topic: Nomenclature
6. Which of the following is bicyclo[3.2.2]nonane?
I
II
IV
A)
B)
C)
D)
E)
III
V
I
II
III
IV
V
Ans: C
Topic: Nomenclature
7. Select the systematic name for
A)
B)
C)
D)
E)
Cl
H
H
Cl
cis-1,3-Dichlorocyclopentane
trans-1,4-Dichlorocyclopentane
cis-1,2-Dichlorocyclopentane
trans-1,3-Dichlorocyclopentane
1,1-Dichlorocyclopentane
Ans: D
89
Chapter 4
Topic: Nomenclature
8. Which compound is a bicycloheptane?
I
II
IV
A)
B)
C)
D)
E)
III
V
I
II
III
IV
V
Ans: D
Topic: Nomenclature
9.
A)
B)
C)
D)
E)
Which isomer of C5H10 would you expect to have the smallest heat of combustion?
Cyclopentane
Methylcyclobutane
1,1-Dimethylcyclopropane
cis-1,2-Dimethylcyclopropane
trans-1,2-Dimethylcyclopropane
Ans: A
Topic: Nomenclature
10. Which is the correct name for the compound shown below?
A)
B)
C)
D)
E)
Bicyclo[2.2.0]hexane
Bicyclo[2.2.0]butane
Bicyclo[2.2.2]hexane
Bicyclo[2.2.1]hexane
Disquarane
Ans: A
90
Chapter 4
Topic: Nomenclature
11. What is the name of this compound?
A)
B)
C)
D)
E)
Bicyclo[2.2.2]octane
Bicyclo[3.2.1]octane
Bicyclo[4.1.1]octane
Bicyclo[4.2.0]octane
Bicyclo[3.3.0]octane
Ans: D
Topic: Nomenclature
12. What is the common name for this compound?
Br
A)
B)
C)
D)
E)
Isobutyl bromide
tert-Butyl bromide
Butyl bromide
sec-Butyl bromide
Bromo-sec-butane
Ans: A
Topic: Nomenclature
13. A correct IUPAC name for the following compound is:
Br
A)
B)
C)
D)
E)
Ans:
3,6,7-trimethyl-4-bromo-1-octene
4-bromo-3-methyl-6-isopropyl-1-heptene
4-bromo-3,6,7-trimethyl-1-octene
4-bromo-6-isopropyl-3-methyl-1-heptene
4-bromo-6-isopropyl-3,6-dimethyl-1-hexene
C
91
Chapter 4
Topic: Nomenclature
14. A correct name for the following compound is:
A)
B)
C)
D)
E)
2-Methylbicyclo[4.3.0]nonane
1-Methylbicyclo[4.3.1]nonane
7-Methylbicyclo[4.3.0]nonane
2-Methylbicyclo[4.3.1]nonane
1-Methylbicyclo[4.3.0]nonane
Ans: A
Topic: Nomenclature
Use the following to answer questions 15-17:
I
II
IV
15.
A)
B)
C)
D)
E)
III
V
Which of the above is bicyclo[3.3.1]nonane?
I
II
III
IV
V
Ans: D
Topic: Nomenclature
16.
A)
B)
C)
D)
E)
Which of the above is bicyclo[5.2.0]nonane?
I
II
III
IV
V
Ans: E
92
Chapter 4
Topic: Nomenclature
17.
A)
B)
C)
D)
E)
Which of the above is bicyclo[4.3.0]nonane?
I
II
III
IV
V
Ans: A
Topic: Nomenclature
Use the following to answer questions 18-19:
Br
H
H
Br
Br
H
H
I
Br
Br
H
Br
H
II
III
Topic: Nomenclature
18.
A)
B)
C)
D)
E)
trans-1,2-Dibromocyclohexane is represented by structure(s):
I
II
III
II and III
I and II
Ans: D
Topic: Nomenclature
19.
A)
B)
C)
D)
E)
cis-1,2-Dibromocyclohexane is represented by structure(s):
I
II
III
II and III
I and II
Ans: A
93
Chapter 4
Topic: Nomenclature
20. cis-1,3-Dibromocyclohexane is represented by structure(s):
Br
H
Br
H
H
H
I
A)
B)
C)
D)
E)
H
Br
Br
Br
Br
II
III
I
II
III
II and III
I and II
Ans: A
Topic: Nomenclature
21.
A)
B)
C)
D)
E)
CH3CHCH2−
CH2CH3 is:
An IUPAC name for the group
Isopentyl
Isoamyl
sec-Butylmethyl
2-Methylbutyl
2-Ethylpropyl
Ans: D
Topic: Nomenclature
22.
A)
B)
C)
D)
E)
H
The neopentyl group has the alternative name:
1,1-Dimethylpropyl
1,2-Dimethylpropyl
2,2-Dimethylpropyl
1-Methylbutyl
2-Methylbutyl
Ans: C
94
Chapter 4
Topic: Nomenclature
23.
A)
B)
C)
D)
E)
Br
Cl is:
The correct IUPAC name for
2-Bromo-4-chloro-4-isopropylpentane
4-Bromo-2-chloro-2-isopropylpentane
5-Bromo-3-chloro-2,3-dimethylhexane
2-Bromo-4-chloro-4,5-dimethylhexane
2-(2-Bromopropyl)-2-chloro-3-methylbutane
Ans: C
Topic: Nomenclature
24. Which of the following is a correct name which corresponds to the common name tertpentyl alcohol?
A) 2,2-Dimethyl-1-propanol
B) 2-Ethyl-2-propanol
C) 2-Methyl-2-butanol
D) 3-Methyl-1-butanol
E) Methyl tert-butanol
Ans: C
Topic: Nomenclature
25. The correct IUPAC name for the following compound is:
OH
A)
B)
C)
D)
E)
1-Hydroxy-3-sec-butylcyclopentane
3-sec-Butyl-1-cyclopentanol
1-sec-Butyl-3-cyclopentanol
4-sec-Butyl-1-cyclopentanol
3-Isobutyl-1-cyclopentanol
Ans: B
95
Chapter 4
Topic: Nomenclature
26. What is a correct name for the following compound?
A)
B)
C)
D)
E)
3-Isobutyl-2-methylheptane
3-sec-Butyl-2-methyloctane
5-Isobutyl-6-methylheptane
2-Ethyl-3-isopropyloctane
4-Isopropyl-3-methylnonane
Ans: E
Topic: Nomenclature
27. What is the correct name of the following compound?
Cl
A)
B)
C)
D)
E)
1-Chlorobicyclo[4.1.1]octane
2-Chlorobicyclo[4.1.0]octane
2-Chlorobicyclo[4.1.1]octane
2-Chlorobicyclo[4.1.1]heptane
5-Chlorobicyclo[4.1.1]octane
Ans: C
Topic: Nomenclature
28. What is the correct IUPAC name for the following compound?
HO
A)
B)
C)
D)
E)
3-Hydroxymethylheptane
3-Hydroxymethylhexane
3-Methyloxyheptane
2-Ethyl-1-hexanol
2-Ethyl-1-heptanol
Ans: D
96
Chapter 4
Topic: Nomenclature
29. Which of the following structures represents bicyclo[3.2.1]octane?
I
II
IV
A)
B)
C)
D)
E)
III
V
I
II
III
IV
V
Ans: E
Topic: Nomenclature
30.
A)
B)
C)
D)
E)
Which of these is the common name for the 1,1-dimethylpropyl group?
tert-Butyl
tert-Pentyl
Isopentyl
Neopentyl
sec-Pentyl
Ans: B
Topic: Nomenclature
31. Neglecting stereochemistry, which of these common group names is ambiguous, i.e.,
does not refer to one specific group?
A) Butyl
B) sec-Butyl
C) tert-Pentyl
D) Neopentyl
E) sec-Pentyl
Ans: E
97
Chapter 4
Topic: Nomenclature
32. What is the correct IUPAC name for the following compound?
OH
A)
B)
C)
D)
E)
5-Ethyl-3-methylhexanol
5-Ethyl-3-methyl-1-hexanol
2-Ethyl-4-methyl-6-hexanol
3,5-Dimethyl-7-heptanol
3,5-Dimethyl-1-heptanol
Ans: E
Topic: Nomenclature
33. Isopentyl is the common name for which alkyl group?
A) CH3CH2CH2CH
CH3
B) CH3CH2CHCH2
CH3
CH
CHCH
C)
3
2CH2
CH3
D) CH3CH2CH
CH2CH3
E)
CH3
CH3CCH2
CH3
Ans: C
98
Chapter 4
Topic: Nomenclature
34.
A)
B)
C)
D)
E)
An IUPAC name for
3-Isobutyl-2,4-dimethylhexane
3-sec-Butyl-2,5-dimethylhexane
4-sec-Butyl-2,5-dimethylhexane
4-Isopropyl-2,5-dimethylheptane
4-Isopropyl-3,6-dimethylheptane
Ans: D
is:
Topic: Nomenclature
35. An IUPAC name for the following compound is:
A)
B)
C)
D)
E)
4-Isobutyl-3,4-dimethylheptane
4-sec-Butyl-2,4-dimethylheptane
2,4,5-Trimethyl-4-propylheptane
3,4,6-Trimethyl-4-propylheptane
4-Isobutyl-4,5-dimethylheptane
Ans: C
Topic: Nomenclature
36. What is the correct IUPAC name for the following compound?
HO
A)
B)
C)
D)
E)
3-Hydroxymethyl-2-heptene
2-(1-methylethyl)-4-hexen-1-ol
5-(1-methylethyl)-2-hexen-6-ol
5-isopropyl-2,6-hexenol
2-(1-methylethyl)-4-hepten-1-ol
Ans: B
99
Chapter 4
Topic: Nomenclature
37. What is the correct IUPAC name for the following compound?
OH
F
A)
B)
C)
D)
E)
5-Ethyl-3-fluorohexanol
5-Ethyl-3-fluoro-1-hexanol
2-Ethyl-4-fluoro-6-hexanol
3-fluoro-5-methyl-7-heptanol
3-fluoro-5-methyl-1-heptanol
Ans: E
Topic: Nomenclature
38. A correct name for the following compound is:
Cl
A)
B)
C)
D)
E)
3-chloro-8-methylbicyclo[4.3.0]nonane
8-Methyl-3-chlorobicyclo[4.3.1]nonane
3-Methyl-7-chlorobicyclo[4.3.0]nonane
3-Methyl-7-chlorobicyclo[4.3.1]decane
3-chloro-8-methyl[4.3.0]bicyclononane
Ans: A
Topic: Nomenclature
39. A correct name for the following compound is:
Br
A)
B)
C)
D)
E)
4-bromo-3,8-dimethylbicyclo[5.2.2]nonane
3,8-dimethyl-4-bromo-bicyclo[5.2.0]nonane
4-bromo-3,8-dimethylbicyclo[5.2.1]decane
7-bromo-2,6-dimethylbicyclo[5.2.0]nonane
4-bromo-3,8-dimethylbicyclo[5.2.0]nonane
Ans: E
100
Chapter 4
Topic: Conformational Analysis
40. The least stable conformation of butane is:
H
H3C
H
H3C
H
H
H
H
CH3
CH3
H
H
H
I
H3C
H
H
II
III
H
A)
B)
C)
D)
E)
CH3
H
CH3
H
CH3
H
H
CH3
H
H
H
IV
V
CH3
H
I
II
III
IV
V
Ans: B
Topic: Conformational Analysis
41.
A)
B)
C)
D)
E)
The preferred conformation of cis-3-tert-butyl-1-methylcyclohexane is the one in which:
the tert-butyl group is axial and the methyl group is equatorial.
the methyl group is axial and the tert-butyl group is equatorial.
both groups are axial.
both groups are equatorial.
the molecule exists in a boat conformation.
Ans: D
101
Chapter 4
Topic: Conformational Analysis
42. What structure represents the most stable conformation of cis-1,3-dimethylcyclohexane?
CH3
H
H
H
CH3
H3C
H
CH3
H
H3C
I
H
II
III
H
H
CH3
H3C
CH3
H3C
H
H
V
IV
A)
B)
C)
D)
E)
CH3
I
II
III
IV
V
Ans: B
Topic: Ring Strain
43. Which cycloalkane has the largest heat of combustion per CH2 group?
I
A)
B)
C)
D)
E)
IV
I
II
III
IV
V
Ans: A
II
III
V
102
Chapter 4
Topic: Ring Strain
44. Which cycloalkane has the lowest heat of combustion per CH2 group?
I
II
IV
A)
B)
C)
D)
E)
III
V
I
II
III
IV
V
Ans: D
Topic: Ring Strain
45.
A)
B)
C)
D)
E)
Which isomer would have the largest heat of combustion?
Propylcyclopropane
Ethylcyclobutane
Methylcyclopentane
Cyclohexane
Since they are all isomers, all would have the same heat of combustion.
Ans: A
Topic: Ring Strain
46.
A)
B)
C)
D)
E)
Which is the most stable conformation of cyclohexane?
Chair
Twist
Boat
One-half chair
Staggered
Ans: A
103
Chapter 4
Topic: Ring Strain
47.
A)
B)
C)
D)
E)
Which cycloalkane has the greatest ring strain?
Cyclopropane
Cyclobutane
Cyclopentane
Cyclohexane
Cycloheptane
Ans: A
Topic: Conformational Analysis
48. The most stable conformation of butane is:
H
H3C
H
H3C
H
H
H
H
CH3
CH3
H
H
H
I
H3C
H
H
II
III
H
A)
B)
C)
D)
E)
CH3
H
CH3
H
CH3
H
CH3
H
CH3
H
H
H
H
IV
V
I
II
III
IV
V
Ans: C
104
Chapter 4
Topic: Conformational Analysis
49. The most stable conformation of 1,2-dibromoethane is:
H
Br
H
Br
H
H
H
H
Br
Br
H
H
H
I
H
Br
H
H
II
III
H
A)
B)
C)
D)
E)
Br
Br
H
Br
H
H
Br
H
H
H
IV
V
Br
H
I
II
III
IV
V
Ans: C
105
Chapter 4
Topic: Conformational Analysis
50. The most stable conformation of 2,3-dibromobutane, viewed through the C-2—C-3
bond :
CH3
Br
H
Br
H
H
H
H
Br
Br
H3C
H3C
CH3
I
H3C
Br
CH3
H
II
III
Br
Br
A)
B)
C)
D)
E)
Br
H3C
H
H
Br
H3C
H
H3C
H
Br
CH3
IV
V
I
II
III
IV
V
Ans: C
106
Chapter 4
Topic: Conformational Analysis
51. The most stable conformation of 3-bromo-2-methylpentane, viewed through the
C-3—C-4 bond (i.e., C-3 in the front, C-4 in the back):
CH3
Br
H
H
Br
H
H
CH3
CH(CH3)2
H
H
H
CH(CH3)2
I
H
H
CH3
CH(CH3)2
II
III
CH3
H
Br
CH(CH3)2
H
A)
B)
C)
D)
E)
Br
H
Br
H
CH3
CH3
H
CH2CH3
IV
V
I
II
III
IV
V
Ans: A
107
Chapter 4
Topic: Conformational Analysis
52. The most stable conformation of 3-bromo-2-methylpentane, viewed through the
C-2—C-3 bond (i.e., C-2 in the front, C-3 in the back):
H
H
CH3
Br
CH3
H
Br
H
CH3
CH2CH3
H
H3C
CH3
I
H
H
CH3
CH(CH3)2
II
III
CH3
CH3
Br
CH2CH3
H
A)
B)
C)
D)
E)
Br
H
Br
H
CH3
CH3
H
CH2CH3
IV
V
I
II
III
IV
V
Ans: E
108
Chapter 4
Topic: Conformational Analysis
53. The most stable conformation of 2,3-dimethylpentane, viewed through the
C-2—C-3 bond (i.e., C-2 in the front, C-3 in the back):
CH3
CH3
CH3
H
H
CH3
H
CH2CH3
CH3
CH2CH3
H
H
H3C
I
CH3
H
H
CH(CH3)2
II
III
CH3
CH3
H
CH3
H
H3C
CH3
CH3
CH(CH3)2
IV
V
H3CH2C
A)
B)
C)
D)
E)
H
H
H
I
II
III
IV
V
Ans: D
Topic: Ring Strain
54.
A)
B)
C)
D)
E)
Which cycloalkane has the least ring strain?
Cyclopropane
Cyclobutane
Cyclopentane
Cyclohexane
Cycloheptane
Ans: D
Topic: Conformational Analysis
55. The most stable conformation of cis-1-tert-butyl-2-methylcyclohexane is the one in
which:
A) the tert-butyl group is axial and the methyl group is equatorial.
B) the methyl group is axial and the tert-butyl group is equatorial.
C) both groups are axial.
D) both groups are equatorial.
E) the twist boat conformation is adopted.
Ans: B
109
Chapter 4
Topic: Conformational Analysis
56. The most stable conformation of trans-1-tert-butyl-2-methylcyclohexane is the one in
which:
A) the tert-butyl group is axial and the methyl group is equatorial.
B) the methyl group is axial and the tert-butyl group is equatorial.
C) both groups are axial.
D) both groups are equatorial.
E) the molecule is in the half chair conformation.
Ans: D
Topic: Conformational Analysis
57. The most stable conformation of trans-1-tert-butyl-3-methylcyclohexane is the one in
which:
A) the tert-butyl group is axial and the methyl group is equatorial.
B) the methyl group is axial and the tert-butyl group is equatorial.
C) both groups are axial.
D) both groups are equatorial.
E) the twist boat conformation is adopted.
Ans: B
Topic: Conformational Analysis
58.
A)
B)
C)
D)
E)
In the most stable conformation of cis-1,4-dimethylcyclohexane, the methyl groups are:
one axial, one equatorial.
both axial.
both equatorial.
alternating between being both axial and both equatorial.
None of the above
Ans: A
110
Chapter 4
Topic: Conformational Analysis
59. The most stable conformation for 1,2-ethanediol (ethylene glycol) is shown below. It is
the most stable conformation because:
H
H
OH
H
OH
H
A)
B)
C)
D)
E)
this corresponds to an anti conformation.
in general, gauche conformations possess the minimum energy.
it is stabilized by intramolecular hydrogen bonding.
it is a staggered conformation.
it has the highest energy of all the possibilities.
Ans: C
Topic: Conformational Analysis
Relative energy
60. The graph below is a plot of the relative energies of the various conformations of:
0o
60o 120o 180o 240o 300o 360o
Angle of rotation
A)
B)
C)
D)
E)
Ethane
Propane
Chloroethane
1-Chloropropane (C1-C2 rotation)
Butane (C1-C2 rotation)
Ans: D
111
Chapter 4
Topic: Conformational Analysis
Relative energy
61. The graph below is a plot of the relative energies of the various conformations of:
0o
A)
B)
C)
D)
E)
60o 120o 180o 240o 300o 360o
Angle of rotation
2-chloropropane
1,3-dichloropropane
2-methylpropane
Butane (C1-C2 rotation)
Butane (C2-C3 rotation)
Ans: E
Topic: Conformational Analysis
Relative energy
62. Consider the graph below, which is a plot of the relative energies of the various
conformations of hexane, viewed through the C2-C3 bond. The conformations
corresponding to the 60o and 300o are:
0o
60o 120o 180o 240o 300o 360o
Angle of rotation
A)
B)
C)
D)
E)
Eclipsed
Staggered, and gauche
Staggered and anti
More stable than the conformation at 180o
None of the above
Ans: B
112
Chapter 4
Topic: Conformational Analysis
Relative energy
63. Consider the graph below, which is a plot of the relative energies of the various
conformations of 2,3-dimethylbutane, viewed through the C2-C3 bond. The
conformations corresponding to the 120o and 240o are:
0o
A)
B)
C)
D)
E)
60o 120o 180o 240o 300o 360o
Angle of rotation
Eclipsed, more stable than the conformation at 0o
Eclipsed, more stable than the conformation at 180o
Staggered, more stable than the conformation at 0o
Staggered, less stable than the conformation at 180o
Two of the above are true
Ans: A
Topic: Conformational Analysis
64. Which conformation of trans-1-isopropyl-3-methylcyclohexane would be present in
greatest amount at equilibrium?
A) The conformation with the methyl group equatorial and the isopropyl group axial
B) The conformation with the methyl group axial and the isopropyl group equatorial
C) The conformation with both groups axial
D) The conformation with both groups equatorial
E) The twist boat conformation.
Ans: B
113
Chapter 4
Topic: Conformational Analysis
65. Which conformation represents the most stable conformation of trans-1-bromo-4methylcyclohexane?
CH3
Br
Br
Br
CH3
CH3
I
II
III
CH3
Br
H3C
Br
IV
A)
B)
C)
D)
E)
V
I
II
III
IV
V
Ans: A
Topic: Cis-Trans Isomers
66. Which of the following can be described as cis isomers?
Br
HO
Cl
Br
F
CH3
I
II
CH3
III
CH3
HO
CH2CH3
IV
A)
B)
C)
D)
E)
V
I
II, V
III, IV
I, III and IV
None of the above are cis isomers.
Ans: B
114
Chapter 4
Topic: Cis-Trans Isomers
67. Which of the following can be described as trans isomers?
Br
HO
Cl
Br
F
CH3
I
II
III
CH3
CH3
HO
CH2CH3
IV
A)
B)
C)
D)
E)
V
I
II, V
III, IV
I, III and IV
None of the above are trans isomers.
Ans: C
Topic: Conformational Analysis
68. Which of the following will have the same energy after undergoing ring flip?
OH
HO
Cl
Br
F
CH3
I
II
CH3
III
CH2CH3
HO
CH2CH3
IV
A)
B)
C)
D)
E)
V
I
II
III
IV
V.
Ans: E
115
Chapter 4
Topic: Conformational Analysis
69.
A)
B)
C)
D)
E)
The least stable conformation of cyclohexane is the:
boat.
twist boat.
chair.
half-chair.
twist chair.
Ans: D
Topic: Conformational Analysis
70. Express, quantitatively, the difference in stability of the two structures shown below.
CH3
CH3
H3C
H3C
I
A)
B)
C)
D)
E)
II
I is more stable than II by 7.5 kJ mol-1.
I is more stable than II by 15 kJ mol-1.
II is more stable than I by 7.5 kJ mol-1.
II is more stable than I by 15 kJ mol-1.
The two are equal in stability.
Ans: A
Topic: Conformational Analysis
71.
A)
B)
C)
D)
E)
The twist boat conformation is the preferred conformation for this compound.
cis-1,4-Di-tert-butylcyclohexane
trans-1,4-Di-tert-butylcyclohexane
cis-1,3-Di-tert-butylcyclohexane
trans-1,2-Di-tert-butylcyclohexane
None of these
Ans: A
116
Chapter 4
Topic: Ring Strain
72. Which of these C10H18 isomers is predicted to be the most stable?
I
A)
B)
C)
D)
E)
II
III
H
H
H
H
IV
V
I
II
III
IV
V
Ans: E
Topic: Alkane Synthesis
73.
A)
B)
C)
D)
E)
Select the reagents necessary to convert cyclopentene into cyclopentane.
H2 and Ni
H2O
Heat
Zn, H3O+
Light
Ans: A
Topic: Alkane Synthesis
74.
A)
B)
C)
D)
E)
Select the reagents necessary to convert 3-bromohexane into hexane.
Zn, H3O+
CuI
H2O
H3O+
OHAns: A
117
Chapter 4
Topic: General
75.
A)
B)
C)
D)
E)
How many constitutional isomers are possible for the formula C6H14?
2
3
4
5
6
Ans: D
Topic: General
76. How many compounds with the formula C7H16 (heptanes) contain a single 3° carbon
atom?
A) 2
B) 3
C) 4
D) 5
E) 6
Ans: C
Topic: Alkane Synthesis, Nomenclature
77.
A)
B)
C)
D)
E)
Catalytic hydrogenation of which of the following will yield 2-methylpentane?
2-methyl-1-pentene
2-methyl-2-pentene
4-methyl-2-pentene
4-methyl-1-pentene
All of the above
Ans: E
2
Topic: Alkane Synthesis, Nomenclature
78. The synthesis of an alkyne precursor to 2,2-dimethylheptane is accomplished most
effectively by the reaction between these two reagents:
A) CH3CH2CH2C≡CNa and (CH3)3CBr
B) CH3CH2C≡CNa and (CH3)3CCH2Br
C) (CH3)3CC≡CNa and CH3CH2CH2Br
D) (CH3)3CCH2CH2C≡CH and CH3CH2I
E) HC≡CNa and (CH3)3CCH2CH2Br
Ans: C
118
Chapter 4
Topic: Alkane Synthesis, Nomenclature
79. The reaction of the Na salt of 3-methyl-1-pentyne with 1-bromo-3-methylbutane
produces which of these?
A) 3,8-dimethyl-4-nonyne
B) 2,7-dimethyl-4-nonyne
C) 3,8-dimethyl-5-nonyne
D) 3,7-dimethyl-4-nonyne
E) 3,7-dimethyl-5-nonyne
Ans: A
Topic: General
80. What is the simplest alkane, i.e., the one with the smallest molecular weight, which
possesses primary, secondary and tertiary carbon atoms?
A) 2-Methylpropane
B) 2-Methylbutane
C) 2-Methylpentane
D) 3-Methylpentane
E) 2,2-Dimethylbutane
Ans: B
Topic: General
81.
A)
B)
C)
D)
E)
How many alkanes of formula C7H16 possess a quaternary carbon atom?
1
2
3
4
5
Ans: C
SHORT ANSWER QUESTIONS
Topic: General
82. To determine the parent name for an alkane, we begin by finding the
___________________________.
Ans: longest continuous chain of carbons
119
Chapter 4
Topic: Nomenclature
83. When more than one substituent is present on the parent chain, how do we organize the
names of the substituents in the name of the compound?
___________________________.
Ans: alphabetical order
Topic: General
84. A compound containing two fused or bridged rings is referred to as a
___________________________.
Ans: bicycloalkane
Topic: General
85. A compound containing a triple bond between the first two carbons in the chain is called
a ___________________________.
Ans: terminal alkyne
Topic: General
86. When non-bonded atoms/groups are closely positioned, the repulsive interactions
between their electron clouds are referred to as ___________________________.
Ans: steric hindrance
Topic: General
87. _____________________ have the same molecular formula and connectivity but
different arrangements of atoms in three-dimensional space.
Ans: Stereoisomers
Topic: Nomenclature
88. Alkanes in which some or all of the carbon atoms are arranged in a ring are called
___________________________.
Ans: cycloalkanes
Topic: Nomenclature
89. What is the fundamental principle of the IUPAC system of nomenclature for organic
compounds?
Ans: Each different compound should have an unambiguous name.
120
Chapter 4
Topic: Nomenclature
90. What are the four possible parts of a IUPAC name?
Ans: locant, prefix, parent name, suffix
Topic: General
91. In a bicyclic compound, the carbon atoms common to both rings are called
___________________________.
Ans: bridgeheads
Topic: Nomenclature
92. Give the IUPAC name corresponding to the following structure.
Br
OH
Ans: 5-bromo-6-cyclopentyl-2-hexanol
Topic: Nomenclature
93. Give the IUPAC name corresponding to the following structure:
Br
HO
Ans: 3-(2-bromobutyl)cyclopentanol
Topic: Nomenclature
94. Give the IUPAC name corresponding to the following structure:
Br
F
Ans: trans-1-bromo-2-fluorocyclohexane
Topic: Nomenclature
95. Give the IUPAC name corresponding to the following structure:
HO
Ans: 3-ethyl-6,7-dimethyl-2-octanol
121
Chapter 4
Topic: Nomenclature, Conformations
96. Draw the bond-line structural formula corresponding to the most stable conformation of
the following substance: trans-1-bromo-3-(1-methylpropyl)cyclohexane
Br
Ans:
Topic: Nomenclature
97. Draw the bond-line structural formula corresponding to the name: 2,5,9-trimethyl-2decen-7-yne
Ans:
Topic: Alkane Conformations
98. Draw the Newman projection formula for the gauche conformation of the C2-C3 bond
in pentane.
CH3
Ans:
H
CH2CH3
H
H
H
Topic: Alkane Conformations
99. Draw the Newman projection formula for the most stable conformation of the C2-C3
bond in 3-methylpentane.
CH3
Ans:
CH3
H
H
H
CH2CH3
Topic: Alkane Synthesis
100. Complete the following reaction sequence, giving structural details of A, B and C.
NaH
A
Ans:
C2H5Br
NaH
B
H2,Ni
C
C2H5Br
H2,Ni
: Na+
A
B
122
C
Chapter 5
MULTIPLE CHOICE QUESTIONS
Topic: Identifications and Comparisons
1. Which of the following is the enantiomer of the following substance?
H
H
Br
H
CH3
CH3
Br
H
Br
H
H
CH3
I
A)
B)
C)
D)
E)
Ans:
CH3
H
Br
H
II
III
I
II
III
It does not have a non-superposable enantiomer.
Both II and III
D
Topic: Identifications and Comparisons
2. Which of the following molecules is achiral?
CO2H
H
H
H3C
C C
H
C
OH
HO2C
CH3
Br
H
H
Cl
OH
I
II
III
CO2H
H
OH
HO
H
CO2H
IV
A)
B)
C)
D)
E)
Ans:
F
CH3
Br
HO
CH2CH3
V
I
II
III
IV
V
B
123
Chapter 5
Topic: Identifications and Comparisons
3. Hexane and 3-methylpentane are examples of:
A) enantiomers.
B) stereoisomers.
C) diastereomers.
D) constitutional isomers.
E) None of these
Ans: D
Topic: Identifications and Comparisons
4. I and II are:
CH2CH3
H3C
A)
B)
C)
D)
E)
Ans:
Cl
CH2Cl
H
H
H
CH2CH3
I
II
constitutional isomers.
enantiomers.
non-superposable mirror images.
diastereomers.
not isomeric.
A
124
Chapter 5
Topic: Identifications and Comparisons
5. Pairs of enantiomers are:
CH3
H3C
H
CH3
Cl
Cl
CH3
H3C
CH2CH2CH3
CH2CH2CH3
CH2CH2CH3
I
II
III
CH2Cl
CH2CH2CH3
H3C
H
Cl
H
CH2CH2CH3
H
A)
B)
C)
D)
E)
Ans:
Cl
IV
I, II and III, IV
I, II
III, IV
IV, V
None of the structures
C
V
Topic: Identifications and Comparisons
6. Chiral molecules are represented by:
CH3
H3C
CH3
Cl
Cl
H
CH3
H3C
Cl
C2H5
C2H5
C2H5
I
II
III
C2H5
H3C
Cl
CH2Cl
H
H
H
C2H5
IV
A) I, II, III, IV and V
B) I, II, III and IV
C) I and II
D) III and IV
E) IV alone
Ans: D
V
125
Chapter 5
Topic: Identifications and Comparisons
7. The molecules shown are:
H
H
H
OH
H
Br
H
Br
H
CH3
CH3
A)
B)
C)
D)
E)
Ans:
OH
constitutional isomers.
enantiomers.
diastereomers.
identical.
None of these
B
Topic: Identifications and Comparisons
The
8. The molecules shown are:
H
Cl
H3CH2C
H
H
Cl
and
Cl
CH2CH3
Cl
H
CH2CH3
CH2CH3
A)
B)
C)
D)
E)
Ans:
are
enantiomers.
diastereomers.
constitutional isomers.
two conformations of the same molecule.
not isomeric.
B
Topic: Identifications and Comparisons
9. The molecules shown are:
CH3
H
F
H
H
CH3
F
A)
B)
C)
D)
E)
Ans:
H
constitutional isomers.
enantiomers.
diastereomers.
identical.
None of these
B
126
Chapter 5
Topic: Identifications and Comparisons
10. The molecules below are:
F
H
H
H
F
H
F
F
A)
B)
C)
D)
E)
Ans:
constitutional isomers.
enantiomers.
diastereomers.
identical.
stereoisomers.
A
Topic: Identifications and Comparisons
11. The molecules shown are:
H
CH3
Cl
H
Cl
CH3
H
Cl
Cl
H
CH3
A)
B)
C)
D)
E)
Ans:
CH3
enantiomers.
diastereomers.
constitutional isomers.
two different conformations of the same molecule.
not isomeric.
A
Topic: Identifications and Comparisons
12. The molecules shown are:
H
CH3
OH
H
OH
H
H
CH3
A)
B)
C)
D)
E)
Ans:
constitutional isomers.
enantiomers.
diastereomers.
identical.
None of these
B
127
Chapter 5
Topic: Identifications and Comparisons
13. The molecules shown are:
A)
B)
C)
D)
E)
Ans:
H
H
F
F
constitutional isomers.
enantiomers.
diastereomers.
identical.
None of these
E
Topic: Identifications and Comparisons
14. The molecules below are:
CH3
H
H
H
H
CH3
CH3
CH3
A)
B)
C)
D)
E)
Ans:
structural isomers.
enantiomers.
diastereomers.
identical.
None of these
C
Topic: Identifications and Comparisons
15. Cis-trans isomers are:
A) diastereomers.
B) enantiomers.
C) stereoisomers.
D) constitutional isomers.
E) More than one of these
Ans: E
128
Chapter 5
Topic: Identifications and Comparisons
16. The molecules below are:
Cl
CH3
Cl
Br
F
A)
B)
C)
D)
E)
Ans:
F
Br
CH3
I
II
constitutional isomers.
enantiomers.
diastereomers.
identical.
None of these
D
Topic: Identifications and Comparisons
17. The molecules below are:
CH3
CH3
H
F
CH3
H2N
F
H
H3C
F
F
A)
B)
C)
D)
E)
Ans:
NH2
constitutional isomers.
enantiomers.
diastereomers.
identical.
None of these
C
Topic: Identifications and Comparisons
18. The molecules shown are:
A)
B)
C)
D)
E)
Ans:
H
CH3
F
F
constitutional isomers.
enantiomers.
diastereomers.
identical.
None of these
A
129
Chapter 5
Topic: Identifications and Comparisons
19. The molecules below are:
Cl
CH3
F
F
Cl
CH3
H
A)
B)
C)
D)
E)
Ans:
H
constitutional isomers.
enantiomers.
diastereomers.
identical.
None of these
D
Topic: Identifications and Comparisons
20. The molecules below are:
H
H
H
H
CH3
Cl
A)
B)
C)
D)
E)
Ans:
Cl
CH3
constitutional isomers.
enantiomers.
diastereomers.
identical.
None of these
B
130
Chapter 5
Topic: Identifications and Comparisons
21. Which molecule is achiral?
H
Cl
H
Cl
Br
H
Br
Cl
Br
Br
H
H
Cl
Cl
Br
Cl
Br
Br
I
II
III
A)
B)
C)
D)
E)
Ans:
I
II
III
More than one of these
None of these
A
Topic: Identifications and Comparisons
22. Which molecule is achiral?
H
Br
Br
H
Br
Cl
H
Cl
H
H
Br
Cl
Br
Br
Cl
Cl
H
H
I
II
III
A)
B)
C)
D)
E)
Ans:
I
II
III
More than one of the above
None of the above
C
131
Chapter 5
Topic: Identifications and Comparisons
23. The molecules below are:
H
H
H
F
F
H
H
F
H
H
H
H
H
F
H
H
I
II
A)
B)
C)
D)
E)
Ans:
constitutional isomers.
enantiomers.
diastereomers.
identical.
None of these
A
Topic: Identifications and Comparisons
24. The structures
H
CH3
CH3
CH3
H
H
H
CH3
represent:
A) a single compound.
B) enantiomers.
C) meso forms.
D) diastereomers.
E) conformational isomers.
Ans: A
132
Chapter 5
Topic: Identifications and Comparisons
25. Which pair of structures represents the same compound?
CH3
OH
H
H
H
OH
OH
CH3
CH3
H
OH
H
HO
OH
H
CH3
CH3
OH
H
H
HO
H
HO
CH3
CH3
H
HO
H
HO
H
OH
CH3
CH3
OH
H
OH
H
HO
H
CH3
II
III
IV
V
I
A)
B)
C)
D)
E)
Ans:
I and II
II and III
III and IV
III and V
IV and V
D
Topic: Identifications and Comparisons
26. Which structure(s) represent(s) diastereomer(s) of I?
CH3
OH
H
HO
H
HO
H
CH3
CH3
H
OH
H
HO
OH
H
CH3
I
II
A)
B)
C)
D)
E)
Ans:
CH3
OH
H
OH
H
OH
H
CH3
III
II
II and III
II and IV
III and V
IV and V
B
133
CH3
H
HO
H
HO
H
OH
CH3
CH3
OH
H
OH
H
HO
H
CH3
IV
V
Chapter 5
Topic: Identifications and Comparisons
27. The two compounds shown below are:
Br
Cl
H
F
H
Br
F
A)
B)
C)
D)
E)
Ans:
Cl
enantiomers.
diastereomers.
constitutional isomers.
identical.
different but not isomeric.
A
Topic: Identifications and Comparisons
28. The two compounds shown below are:
Br
Br
H
H
A)
B)
C)
D)
E)
Ans:
Br
H
and
H
Br
are:
identical.
enantiomers.
diastereomers.
conformational isomers.
meso forms.
B
Topic: Identifications and Comparisons
29. (2R,4S) –2,4–Dichloropentane and (2S,4R)-2,4-dichloropentane are:
A) enantiomers
B) diastereomers
C) identical
D) conformational isomers
E) constitutional isomers
Ans: C
134
Chapter 5
Topic: Meso compounds
30. Which of the following is a meso compound?
H
HO
H3C
H2
C
OH
CH3
H
H3C
HO
H
H
II
I
CH2OH
CH2OH
CH3
H
OH
CH3
OH
H
H
H
OH
CH2OH
CH2OH
CH3
III
IV
V
A)
B)
C)
D)
E)
Ans:
I
II
III
IV
V
A
Topic: Meso compounds
31. Which of the following is(are) meso compound(s)?
CH3
H
CH3
Cl
CH3
Cl
H
H
Cl
H
Cl
H
Cl
H
Cl
CH3
CH3
CH3
I
II
III
A)
B)
C)
D)
E)
Ans:
I
II
III
Both II and III
Both I and III
E
135
Chapter 5
Topic: Meso compounds
32. Which of the following molecules is achiral?
A) (2R,3R)-2,3-Dichloropentane
B) (2R,3S)-2,3-Dichloropentane
C) (2S,4S)-2,4-Dichloropentane
D) (2S,4R)-2,4-Dichloropentane
E) Two of these
Ans: D
Topic: Meso compounds
33. Which of the following is(are) meso?
Br
Br
H
H
Br
H
Br
Br
I
II
A)
B)
C)
D)
E)
Ans:
Br
Br
H
H
III
Topic: Meso compounds
34. Which molecule has a plane of symmetry?
Cl
H
H
H
Cl
Cl
H
H
CH3
I
II
A) I
B) II
C) III
D) More than one of these
E) None of these
Ans: D
H
IV
I
II
III
IV
Two of the above
E
F
Br
H
H
CH3
CH3
III
136
Chapter 5
Topic: Meso compounds
35. Which molecule has a plane of symmetry?
H
CH2CH3
CH3
HO
H
HO
CH3
I
A)
B)
C)
D)
E)
Ans:
CH3
H
H
F
F
II
III
I
II
III
More than one of these
None of these
D
Topic: Meso compounds
36. Which compound does NOT possess a plane of symmetry?
H
H
Cl
F
F
H
I
A)
B)
C)
D)
E)
Ans:
II
H
CH3
H
Br
CH3
Br
Br
H
Br
III
I, II and V
I, III and IV
II, III and IV
III and IV
V
D
137
IV
H3C
V
Chapter 5
Topic: Meso compounds
37. Which molecule is a meso compound?
H
H
CH3
H
Br
H3C
H
Br
H
OH
H3C
F
H
F
CH3
CH3
OH
CH3
I
II
III
A)
B)
C)
D)
E)
Ans:
I
II
III
More than one of the above
None of the above
D
Topic: Meso compounds
38. Which molecule is a meso compound?
H
H
H
F
H
H
H
CH3
F
CH3
OH
F
F
H
H
F
H
CH3
F
CH3
I
A)
B)
C)
D)
E)
Ans:
II
III
IV
I and II
IV and V
II and III
I, II and III
None of the above
E
138
V
Chapter 5
Topic: Meso compounds
39. Which of the following substances is achiral ?
Cl
H
H
F
H
H
CH3
F
H
Br
I
A)
B)
C)
D)
E)
Ans:
II
CH3
Br
H
III
HO
IV
I
II
III
IV
More than one of these
E
Topic: Meso compounds
40. Which statement is not true for a meso compound?
A) The specific rotation is 0º.
B) There are one or more planes of symmetry.
C) A single molecule is identical to its mirror image.
D) More than one stereogenic center must be present.
E) The stereochemical labels, (R) and (S), must be identical for each stereogenic
center.
Ans: E
Topic: Meso compounds
41. Which is a meso compound?
A) (2R,3R)-2,3-Dibromobutane
B) (2R,3S)-2,3-Dibromopentane
C) (2R,4R)-2,4-Dibromopentane
D) (2R,4S)-2,4-Dibromopentane
E) (2R,4S)-2,4-Dibromohexane
Ans: D
139
Chapter 5
Topic: Specific names
42. (R)-2-Chlorobutane is represented by:
H3C
A)
B)
C)
D)
E)
Ans:
Cl
H
CH3
CH3
Cl
CH3
H3C
CH2Cl
CH2CH3
Cl
H3C
Cl
H
H
CH2CH3
CH2CH3
CH2CH3
H
CH2CH3
I
II
III
IV
V
CH3
CH2CH3
I
II
III
IV
V
C
Topic: Specific names
43. Which of the following represent (R)-2-butanol?
H3C
A)
B)
C)
D)
E)
Ans:
H
CH3
H
OH
H
OH
HO
CH3
HO
H
H
OH
CH2CH3
CH2CH3
CH2CH3
CH2CH3
CH3
I
II
III
IV
V
III and V
I, III, IV and V
I, IV and V
I and III
I, II, IV and V
C
140
Chapter 5
Topic: Specific names
44. Which structure represents (S)-1-chloro-1-fluoroethane?
Cl
CH3
F
F
H
H
Cl
H
Cl
CH3
CH3
F
I
II
III
A)
B)
C)
D)
E)
Ans:
I
II
III
More than one of the above
None of the above
D
Topic: Specific names
45. Which structure represents (R)-1-chloro-1-fluoroethane?
Cl
CH3
F
F
H
H
Cl
H
Cl
CH3
CH3
F
I
II
III
A)
B)
C)
D)
E)
Ans:
I
II
III
More than one of the above
None of the above
B
141
Chapter 5
Topic: Specific names
46. Which structure represents (S)-2-bromobutane?
H
H
CH3
CH3
Br
Br
H
H3C
Br
CH2CH3
CH2CH3
CH2CH3
I
II
III
A)
B)
C)
D)
E)
Ans:
I
II
III
More than one of the above
None of the above
A
Topic: Specific names
CH2CH3
47.
H
Cl
H
CH3
Cl
H
CH3
A)
B)
C)
D)
E)
Ans:
is properly named:
(3R,4S,5R)- 3,5-Dichloro-4-methylhexane
(2S,3S,4S)- 2,4-Dichloro-3-methylhexane
(2S,3R,4R)- 2,4-Dichloro-3-methylhexane
(2S,3R,4S)-2,4-Dichloro-3-methylhexane
(2S,3S,4R)- 2,4-Dichloro-3-methylhexane
D
142
Chapter 5
Topic: Specific names
48. The Cahn-Ingold-Prelog stereochemical designations used for the following substance
are:
H
Cl
Cl
H
Cl
A)
B)
C)
D)
E)
Ans:
Cl
2R,4S
2S,4R
2R,4R
2S,4S
The “R,S” terminology doesn't apply in this case.
C
Topic: Specific names
49. Which of these is not a correct Fischer projection formula of the (S) form of
3-bromo-1,1-dichloro-2-propanol ?
CHCl2
H
OH
CH2Br
I
A)
B)
C)
D)
E)
Ans:
CH2Br
HO
H
CHCl2
II
H
Cl2HC
CH2Br
OH
OH
CHCl2
H
CH2Br
IV
III
OH
CHCl2
BrH2C
H
V
I
II
III
IV
V
D
Topic: Optical activity
50. What is the percent composition of a mixture of (S)-(+)-2-butanol,[α] 25 = +13.52º,
D
= -13.52º, with a specific rotation [α] 25
= +6.76º?
and (R)-(-)-2-butanol,[α] 25
D
D
A) 75%(R) 25%(S)
B) 25%(R) 75%(S)
C) 50%(R) 50%(S)
D) 67%(R) 33%(S)
E) 33%(R) 67%(S)
Ans: B
143
Chapter 5
Topic: Optical activity
51. Which one of the following can exist in optically active forms?
A) cis-1,3-Dichlorocyclohexane
B) trans-1,3-Dichlorocyclohexane
C) cis-1,4-Dichlorocyclohexane
D) trans-1,4-Dichlorocyclohexane
E) cis-1,2-Dichlorocyclohexane
Ans: B
Topic: Optical activity
52. The compounds whose molecules are shown below would have:
CH2CH3
CH3
H
F
H
F
CH2CH3
CH3
I
II
A)
B)
C)
D)
E)
Ans:
the same melting point.
different melting points.
equal but opposite optical rotations.
More than one of the above
None of the above
D
Topic: Optical activity
53. The compounds whose molecules are shown below would have:
CH3
CH3
H
F
H
CH3
F
A)
B)
C)
D)
E)
Ans:
F
H
H3C
F
H
the same melting point.
different melting points.
equal but opposite optical rotations.
More than one of the above
None of the above
B
144
Chapter 5
Topic: Optical activity
54. The compounds whose molecules are shown below would have:
H
H
H
OH
OH
A)
B)
C)
D)
E)
Ans:
the same melting point.
different melting points.
equal but opposite optical rotations.
More than one of these
None of these
B
H
OH
OH
Topic: Optical activity
55. The compounds whose structures are shown below would have:
CH3
Br
CH3
Br
Br
CH3
Br
A)
B)
C)
D)
E)
Ans:
the same melting point.
different melting points.
equal but opposite optical rotations.
More than one of these
None of these
D
CH3
145
Chapter 5
Topic: Optical activity
56. A solution of which of these allenes will rotate plane-polarized light?
H
H
C
C
H
H
C
H
C
H
C
Cl
I
H
C
C
H
H
C
C
Cl
Cl
Cl
Cl
C
C
Cl
C
Cl
V
IV
A)
B)
C)
D)
E)
Ans:
C
III
C
Cl
C
Cl
II
Cl
H
I
II
III
IV
V
C
Topic: Optical activity
57. How many optically active compounds are represented by the following generalized
formula?
BrHC
A)
B)
C)
D)
E)
Ans:
CHBr
0
1
2
3
4
C
146
Chapter 5
Topic: Optical activity
58. Which of these is a comparatively insignificant factor affecting the magnitude of
specific optical rotation?
A) Concentration of the substance of interest
B) Purity of the sample
C) Temperature of the measurement
D) Length of the sample tube
E) All of the above are equally significant.
Ans: C
Topic: Optical activity
59. What can be said with certainty if a compound has [α] 25 = -9.25º ?
D
A) The compound has the (S) configuration.
B) The compound has the (R) configuration.
C) The compound is not a meso form.
D) The compound possesses only one stereogenic center.
E) The compound has an optical purity of less than 100%.
Ans: C
Topic: Optical activity
60. An alkane which can exhibit optical activity is:
A) Neopentane
B) Isopentane
C) 3–Methylpentane
D) 3–Methylhexane
E) 2,3–Dimethylbutane
Ans: D
Topic: Optical activity
61. In the absence of specific data, it can only be said that (R)–2–bromopentane is:
A) dextrorotatory (+).
B) levorotatory (–).
C) optically inactive.
D) achiral.
E) analogous in absolute configuration to (R)–2–chloropentane.
Ans: E
147
Chapter 5
Topic: Optical activity
62. If a solution of a compound (30.0 g/100 mL of solution) has a measured rotation of +15º
in a 2 dm tube, the specific rotation is:
A) +50º
B) +25º
C) +15º
D) +7.5º
E) +4.0º
Ans: B
Topic: Optical activity
63. Which compound would show optical activity?
A) cis 1,4- Dimethylcyclohexane
B) trans 1,4- Dimethylcyclohexane
C) cis 1,4- Dimethylcycloheptane
D) trans 1,4- Dimethylcycloheptane
E) More than one of these
Ans: D
Topic: Optical activity
64. Of the compounds which correspond to the general name "dichlorocyclobutane", how
many are optically active?
A) 0
B) 1
C) 2
D) 3
E) 4
Ans: C
Topic: General definitions
65. Which of the following is true about any (R)-enantiomer?
A) It is dextrorotatory.
B) It is levorotatory.
C) It is an equal mixture of + and -.
D) It is the mirror image of the (S)-enantiomer.
E) (R) indicates a racemic mixture.
Ans: D
148
Chapter 5
Topic: General definitions
66. Which of the following is true of any (S)-enantiomer?
A) It rotates plane-polarized light to the right.
B) It rotates plane-polarized light to the left.
C) It is a racemic form.
D) It is the mirror image of the corresponding (R)-enantiomer.
E) It has the highest priority group on the left.
Ans: D
Topic: General definitions
67. Enantiomers are:
A) molecules that have a mirror image.
B) molecules that have at least one stereogenic center.
C) non-superposable molecules.
D) non-superposable constitutional isomers.
E) non-superposable molecules that are mirror images of each other.
Ans: E
Topic: General definitions
68. Which of the following is NOT true of enantiomers? They have the same:
A) boiling point.
B) melting point.
C) specific rotation.
D) density.
E) chemical reactivity toward achiral reagents.
Ans: C
Topic: General definitions
69. Which statement is true of 1,3-dimethylcyclobutane?
A) Only one form of the compound is possible.
B) Two diastereomeric forms are possible.
C) Two sets of enantiomers are possible.
D) Two enantiomeric forms and one meso compound are possible.
E) None of the previous statements is true.
Ans: B
149
Chapter 5
Topic: Reaction stereochemistry
70. In which of the following reactions is the absolute configuration of the product likely to
be the same as that of the reactant?
Br
Br
A)
PBr3
(R)
OH
Br
O
B)
Br
(R)
OH
Br
OH
O
HA
O
Br
C)
OH
H2O
OH
(R)
D) All of the above
E) Answers A) and B) only
Ans: E
OH
Topic: Reaction Stereochemistry
71. Which of the following reactions must occur with retention of configuration?
KMnO4
A)
(R)
OH
O
POCl3
B)
(R)
OH
Cl
C)
(S)
OH
(R)
O
OCH3
OH
D)
Cl
NH2
NH3
OH
O
OCH3
Cl
PCl5
O
O
Cl
E)
(S)
OH
P, Cl2
Cl
O
Cl
O
Ans: D
150
OH
Chapter 5
Topic: Reaction stereochemistry
72. Which reaction must take place with retention of configuration at the stereogenic
center?
OH
Cl
HCl
A)
F
Br
F
OH
B)
PCl3
O
C)
F
OH
F
Br
F
Cl
O
O
F
O
O
D) More than one of the above
E) None of the above
Ans: D
Topic: Reaction stereochemistry
73.
O
The reaction of
with H2/Ni forms:
A) 2-methylheptane.
B) (R)-2-methyl-5-heptanol
C) (S)-6-methyl-3-heptanol
D) (R,S)-6-methyl-3-heptanol
E) Achiral 6,6-dimethyl-3-hexanol
Ans: D
Topic: General stereochemistry
74. CH3CHBrCH2CHClCH3 is the generalized representation of what number of
stereoisomers?
A) 3
B) 4
C) 5
D) 6
E) 7
Ans: Bs
151
Chapter 5
Topic: General stereochemistry
75. For the generalized structure BrCH2CHClCH2CHClCH2Br there exists what number of
stereoisomers?
A) 2
B) 3
C) 4
D) 6
E) 8
Ans: B
Topic: General stereochemistry
76. How many discrete dimethylcyclopropanes are there?
A) 2
B) 3
C) 4
D) 5
E) 6
Ans: C
Topic: General stereochemistry
77. What is the molecular formula for the alkane of smallest molecular weight which
possesses a stereogenic center?
A) C4H10
B) C5H12
C) C6H14
D) C7H16
E) C8H18
Ans: D
Topic: General stereochemistry
78. How many chiral stereoisomers can be drawn for CH3CHFCHFCH(CH3)2?
A) 1
B) 2
C) 3
D) 4
E) 8
Ans: D
152
Chapter 5
Topic: General stereochemistry
79. How many different compounds are there which correspond to the general name "3-(1methylbutyl)cyclobutanol”?
A) 2
B) 4
C) 6
D) 8
E) None of the above
Ans: B
Topic: General stereochemistry
80. How many stereogenic centers are there in Lovastatin ( Mevacor® : a cholesterollowering drug) ?
HO
O
O
O
(Lovastatin)
A) 4
B) 5
C) 6
D) 7
E) 8
Ans: E
Topic: General stereochemistry
81. What is the total number of compounds, stereoisomers included, designated by the
general name "dichlorocyclopentane"?
A) 4
B) 5
C) 7
D) 8
E) 9
Ans: C
153
Chapter 5
Use the following to answer questions 82-85:
Consider the following compounds:
H
CH2OH
H
OH
H
H
CH3
HO
CH3
HO
H
H
CH3
HO
CH3
CH3
OH
H
H3C
H
CH3
OH
H
OH
I
II
III
IV
Topic: General stereochemistry
82. Which of the compounds above (I-IV) represent enantiomers?
A) I and II
B) II and III
C) III and IV
D) II and IV
E) III and IV
Ans: B
Topic: General stereochemistry
83. Which compound above (I-IV) is a meso compound?
A) I
B) II
C) III
D) IV
E) None of these
Ans: D
Topic: General stereochemistry
84. Which compound above (I-IV) is (2R,3R)-2,3-butanediol?
A) I
B) II
C) III
D) IV
E) None of these
Ans: C
154
Chapter 5
Topic: General stereochemistry
85. Which compounds above (I-IV) form a set of stereoisomers?
A) I, II and III
B) II, III and IV
C) II and III
D) I, III and IV
E) I, II, III and IV
Ans: B
SHORT ANSWER QUESTIONS
Topic: General stereochemistry
86. To be superposable, when one object is placed on top of another, _________________.
Ans: all parts of each object must coincide
Topic: General stereochemistry
87. An achiral molecule is one that is __________________ upon its mirror image.
Ans: superposable
Topic: General stereochemistry
88. Enantiomers are stereoisomers whose molecules are ____________________.
Ans: nonsuperposable mirror images
Topic: General stereochemistry
89. A chiral molecule is defined as one that is _____________________ on its mirror
image.
Ans: nonsuperposable
Topic: General stereochemistry
90. A meso compound has __________ chiral center(s), and is __________ upon its mirror
image.
Ans: two or more, superposable
155
Chapter 5
Topic: General stereochemistry
91. The molecule of aspartame ( Nutrasweet® ), depicted below, has ___________ ( how
many? ) stereogenic centers.
O
O
HO
HN
O
H2N
O
Aspartame
Ans: two
Topic: General stereochemistry
92. All molecules with a plane of symmetry are _________.
Ans: achiral
Topic: General stereochemistry
93. When using the Cahn-Ingold-Prelog rules for prioritizing groups on a stereogenic
carbon, priority is first assigned on the basis of ____________________.
Ans: atomic number
Topic: General stereochemistry
94. Enantiomers have ______________ physical properties.
Ans: identical
Topic: General stereochemistry
95. The device that is used for measuring the effect of optically active compounds on planepolarized light is called a ________________.
Ans: polarimeter
Topic: General stereochemistry
96. True or false: A sample consisting of the pure R enantiomer of a compound will always
rotate plane-polarized light in a clockwise direction.
A) True
B) False
Ans: B
156
Chapter 5
Topic: General stereochemistry
97. An equimolar mixture of two enantiomers is called a ________________.
Ans: racemic mixture
Topic: General stereochemistry
98. A molecule that contains stereogenic carbons but is nonetheless achiral is referred to as
a ______________.
Ans: meso compound
Topic: General stereochemistry
99. When discussing the stereochemical relationship of cis/trans isomers of cyclic
compounds, we refer to them as __________________.
Ans: diastereomers
Topic: Nomenclature
100. Draw a dash-wedge structure for (3R)-3-methyl-5-hexen-3-ol
OH
Ans:
Topic: Nomenclature
101. Draw a dash-wedge structure for (1R)-1-bromo-1,3,3-trimethylcyclohexane
Ans:
Br
Topic: Nomenclature
102. Draw a dash-wedge structure for (1R,3R)-1,3-dibromo-1,3-dimethylcyclohexane
Ans:
Br
Br
157
Chapter 5
Topic: Nomenclature
103. Draw a dash-wedge structure for (1S,3R)-1-ethyl-1,3-dimethylcyclopentane
CH3
Ans:
C2H5
H
CH3
Topic: Nomenclature
104. What is the complete IUPAC name of the following substance? (Remember to give
stereochemical details, as relevant.)
H
C2H5
H3C
CH3
Ans: (1S)-1-ethyl-1,3,3-trimethylcyclopentane
Topic: Nomenclature
105. What is the complete IUPAC name of the following substance? (Remember to give
stereochemical details, as relevant.)
H
OH
H3C
OH
Ans: (1R,3S)-1-methylcyclopentane-1,3-diol
158
Chapter 5
Topic: Fischer projections
106. Draw Fischer projection formulas for all stereoisomers of 2,4-dibromohexane, giving
stereochemical details for each structure.
CH3
CH3
CH3
CH3
Ans:
Br
H
H
Br
H
Br
H
H
H
H
H
H
H
Br
H
H
Br
Br
H
H
Br
H
H
H
H
H
H
H
H
H
Br
H
CH3
CH3
CH3
CH3
2R,4S
2S,4R
2S,4S
2R,4R
2,4-dibromohexane
Topic: Stereoisomers
107. Draw dash-wedge structures for all stereoisomers of 1-bromo-3-isopropylcyclohexane,
giving stereochemical details for each structure.
Ans:
H
H
H
H
Br
1S, 3R
H
Br
H
H
1R, 3S
Br
1S, 3S
1-bromo-3-isopropylcyclohexane
159
Br
H
1R, 3R
Chapter 5
Topic: Fischer projections
108. Draw Fischer projection formulas for all stereoisomers of 2,4-dimethyl-3-hexanol,
giving stereochemical details for each structure.
Ans:
CH3
H3C
CH3
H
H
OH
CH3
CH3
H3C
H
H3C
H
HO
H
H
OH
H3C
H
H
CH3
H
CH3
H
H
H
H
H
H
H3C
H
HO
H
H3C
H
H
H
CH3
CH3
CH3
CH3
3S,4S
3R,4R
3S,4R
3R,4S
2,4-dimethyl-3-hexanol
Topic: Fischer projections
109. Draw a Fischer projection formula of (3R)-6-Bromo-1-hexen-3-ol.
H2C
H
Ans:
*
OH
Br
H
OH
H
H
H
H
CH2Br
(3R)-6-Bromo-1-hexen-3-ol.
160
Chapter 6
MULTIPLE CHOICE QUESTIONS
Topic: Reaction kinetics
1. Consider the SN2 reaction of butyl bromide with OH- ion.
CH3CH2CH2CH2Br + OH- ⎯⎯⎯⎯→ CH3CH2CH2CH2OH + Br-
A)
B)
C)
D)
E)
Assuming no other changes, what effect on the rate would result from simultaneously
doubling the concentrations of both butyl bromide and OH- ion?
No effect.
It would double the rate.
It would triple the rate.
It would increase the rate four times.
It would increase the rate six times.
Ans: D
Topic: Reaction kinetics
−
2. Consider the SN2 reaction of 2-iodopentane with CH3CO2 ion.
I
O
O
O
NaI
ONa
Assuming no other changes, what effect on the rate would result from simultaneously
doubling the concentrations of both 2-iodopentane and CH3CO2− ion?
A)
B)
C)
D)
E)
No effect.
It would double the rate.
It would triple the rate.
It would increase the rate four times.
It would increase the rate six times.
Ans: D
161
Chapter 6
Topic: Reaction kinetics
−
3. Consider the SN2 reaction of 1-chloro-5-methylhexane with CN ion.
Cl
NaCN
CN
NaCl
Assuming no other changes, what effect on the rate would result from simultaneously
doubling the concentrations of both 1-chloro-5-methylhexane and CN− ion?
A)
B)
C)
D)
E)
No effect.
It would double the rate.
It would triple the rate.
It would increase the rate four times.
It would increase the rate six times.
Ans: D
Topic: Reaction kinetics
4. Select the rate law for the following reaction, e.g.,
CH3CH2CH2CHBrCH3 + OH− ⎯⎯⎯→ CH3CH2CH2CHOHCH3 + X−
( RBr )
A) Rate = k [RBr]
B) Rate = k [RBr] [OH−]
C) Rate = k [RBr]2 [OH−]
D) Rate = k [RBr] [OH−]2
E) Rate = k [RBr]2 [OH−]2
Ans: B
Topic: Activation Energy
5. Increasing the temperature of a chemical reaction usually increases greatly the rate of
the reaction. The most important reason for this is that increasing the temperature:
A) Increases the collision frequency.
B) Decreases the probability factor.
C) Increases the fraction of collisions with energy greater than Eact.
D) Decreases the energy of activation.
E) Makes the reaction more exothermic
Ans: C
162
Chapter 6
Topic: Activation Energy
6. Increasing the temperature of a chemical reaction usually increases greatly the rate of
the reaction. The primary reason for this is that increasing the temperature:
A) Decreases the energy of activation.
B) Increases the total number of collisions between reactants
C) Decreases the rate of the reverse reaction.
D) Favors endothermic processes.
E) None of the above properly explains the observed increase in reaction rates with
increase in temperature.
Ans: E
Topic: Reaction Kinetics
7. The rate equation for a nucleophilic substitution reaction of a tertiary alkyl bromide (RBr) with I− ion would be:
A) Rate = k [RBr]
B) Rate = k [I−]
C) Rate = k [RBr][I−]
D) Rate = k [RBr]2[I−]
E) Rate = k [RBr][I−]2
Ans: A
Topic: Reaction Kinetics
8. The rate equation for a nucleophilic substitution reaction of a secondary alkyl chloride
(R-Cl) with I− ion would be:
A) Rate = k [RCl]
B) Rate = k [I−]
C) Rate = k [RCl][I−]
D) Rate = k [RCl]2[I−]
E) Rate = k [RCl][I−]2
Ans: C
163
Chapter 6
Topic: Reaction Kinetics
9. Consider the reaction of 2-chloro-2-methylpentane with sodium iodide.
NaI
NaCl
Cl
A)
B)
C)
D)
E)
I
Assuming no other changes, how would it affect the rate if one simultaneously doubled
the concentration of 2-chloro-2-methylpentane and sodium iodide?
No effect
It would double the rate.
It would triple the rate.
It would quadruple the rate.
It would increase the rate five times.
Ans: B
Topic: Predicting mechanisms, products
10. What would you expect to be the chief organic product(s) when tert-butyl bromide
reacts with sodium acetylide, i.e.,
Br
I
HC
C:
?
Na
II
III
HC
CH
IV
A)
B)
C)
D)
E)
I
II
III
IV
None of these
Ans: D
164
Chapter 6
Topic: Predicting mechanisms, products
11. What would you expect to be the chief organic product(s) when 2-bromo-2methylpentane reacts with sodium propynide, i.e.,
Br
C:
I
?
Na
III
II
HC
CH
HC
IV
A)
B)
C)
D)
E)
CH
V
I
II
III
IV
V
Ans: D
Topic: Predicting products
12. What product(s) would you expect to obtain from the following SN2 reaction?
H
H
CH3
Br
CH3OH
H
H
H
OCH3
CH3
OCH3
CH3
H
I
A)
B)
C)
D)
E)
CH3ONa
?
CH3O
H
H
Br
II
III
I
II
An equimolar mixture of I and II.
III
None of these
Ans: B
165
Chapter 6
Topic: Predicting mechanisms, products
13. What major product(s) are likely to be obtained from the following reaction?
H
H
Cl
CH3CH2ONa
CH3CH2OH, 65oC
H
H
O
I
A)
B)
C)
D)
E)
H
?
O
O
H
H
H
Cl
II
III
I, by predominantly SN2
II, by predominantly SN2
An equimolar mixture of I and II, by predominantly SN1.
III, by substitution of the alkyl group, rather than substitution of the chloro group
Actually, none of these products are likely to be obtained as major products, because
elimination will probably predominate, leading to the formation of an alkene.
Ans: E
Topic: Predicting mechanisms, products
14. Treating (CH3)3C-Cl with a mixture of H2O and CH3OH at room temperature would
yield:
A) CH2=C(CH3)2
B) (CH3)3COH
C) (CH3)3COCH3
D) All of these
E) None of these
Ans: D
166
Chapter 6
Topic: Predicting mechanisms, products
15. Treating 1-bromo-1-methylcyclohexane with CH3OH at room temperature would yield:
A)
OCH3
B)
C)
D) All of these
E) None of these
Ans: D
Topic: Predicting products
16. The major product of the following reaction would be:
C H 2C l
H
OCH3
−
OH
SN2
?
CH3
C H 2O H
H 3C O
H
C H 2O H
H
CH3
OCH3
CH3
II
I
C H 2C l
C H 2C l
HO
A)
B)
C)
D)
E)
H
H
OH
CH3
CH3
III
IV
I
II
III
IV
An equimolar mixture of I and II.
Ans: B
167
Chapter 6
Topic: Predicting mechanisms, products
17. Which would be formed in the following reaction?
CH3
H 3C
Br
C H 3O H
?
55 o C
H
CH3
OCH3
H 3C
CH3
H 3C
OCH3
H
H
I
A)
B)
C)
D)
E)
II
CH3
CH3
CH3
CH2
III
IV
I
II
III
IV
All of the above
Ans: E
168
Chapter 6
Topic: Predicting mechanisms, products
18. Which would be formed in the following reaction?
CH3
Br
H3C
O
OH
?
50oC
H
O
CH3 O
O
H3C
CH3
H3C
H
H
A)
B)
C)
D)
E)
O
I
II
III
IV
I
II
III
IV
All of the above
Ans: E
169
Chapter 6
Topic: Predicting mechanisms, products
19. Which would be the major product of the following reaction?
CH3
H
H
CH3
(CH3)3CO−
?
(CH3)3COH
55oC
Cl
OC(CH3)3 CH3
H
H3C
H3C
H
I
I
A)
B)
C)
D)
E)
OC(CH3)3
H
II
III
I
II
III
IV
None of the above
Ans: C
170
IV
Chapter 6
Topic: Predicting mechanisms, products
20. When 1,4-diiodo-2,2-dimethylbutane (0.10 mol) is treated with 0.10 mol of NaCN in
dimethyl sulfoxide at 30°C, the product formed is:
I
A) NC
B)
CN
I
C) both A) and B).
D)
E)
CN
NC
I
Ans: B
Topic: Predicting mechanisms, products
21. The major product(s) of the following reaction is(are):
H
NaI, CH3CH2OH
H
25oC
Cl
CH3
H
I
H
H
H
I
A)
B)
C)
D)
E)
H
OCH2CH3
H
CH3
CH3
CH3
CH3
I
II
III
IV
I
II
III
IV
Equal amounts of I and II
Ans: A
171
Chapter 6
Topic: Predicting products
22. What would be the major product of the following reaction?
CH2Cl
H
CH3
OH
−
SN2
CH2OH
H3C
CH2OH
H
H
CH3
II
I
CH2Cl
HO
CH3
III
A)
B)
C)
D)
E)
CH2Cl
H
OH
IV
I
II
III
IV
An equimolar mixture of I and II
Ans: B
172
Chapter 6
Topic: Predicting mechanisms, products
23. What would be the major product(s) of the following reaction?
CH3
H3C
CH3O− / CH3OH
55oC
Br
H
CH3
H3C
OCH3
H
A)
B)
C)
D)
E)
II
CH3
III
CH3
H
I
H3C
OCH3
H3C
H3C
IV
I
II
III and IV
I and II
All of the above
Ans: C
173
Chapter 6
Topic: Predicting mechanisms, products
24. What would be the major product
of the following reaction?
−
H
CΝ
Br
25oC
H3C
H
H
H3C
CN
H
CN
H3C
Br
H
I
II
CN
H3C
H
H
H
H3C
Br
NC
III
A)
B)
C)
D)
E)
IV
I
II
III
IV
Equal amounts of I and III
Ans: C
Topic: Predicting mechanisms, products
25. What would be the major product obtained when trans-1-bromo-3-methylcyclopentane
is allowed to react with NaSH at 25oC?
H
H3C
H
SH
H3C
H
I
A)
B)
C)
D)
E)
SH H3C
SH
H3C
H
Br
HS
H
II
III
I
II
III
IV
Equal amounts of I and II
Ans: B
174
H
Br
IV
Chapter 6
Topic: Synthesis, reagent selection
26. You want to synthesize 3-methyl-2-pentene from 2-chloro-3-methylpentane. Which
reagent would you use?
A) HCl, heat
B) NH3(aq), 25oC
C) CH3CO2Na, CH3CO2H, heat
D) CH3CH2ONa, CH3CH2OH, heat
E) CH3CH2OH, heat
Ans: D
Topic: Predicting mechanisms, products
27. When tert-pentyl chloride undergoes solvolysis in aqueous ethanol at room temperature,
there is/are formed:
A)
OH
B)
OC2H5
C)
D)
E) All of these
Ans: E
Topic: Predicting mechanisms, products
28.
A)
B)
C)
D)
E)
Reaction of sodium ethoxide with 1-bromopentane at 30°C yields primarily:
CH3CH2CH2CH=CH2
CH3CH2CH=CHCH3
CH3CH2CH2CH2CH3
CH3CH2CH2CH2CH2OH
CH3CH2CH2CH2CH2OCH2CH3
Ans: E
175
Chapter 6
Topic: Predicting mechanisms, products
29. Heating an alcoholic solution of sodium hydroxide and 1-bromopentane at 60°C yields
primarily:
A)
B)
C)
OH
D)
O
E)
Ans: D
Topic: Predicting mechanisms, products
30. If 0.10 mol of HSCH2CH2OH reacts at 25°C, sequentially, with 0.20 mol of NaH, 0.10
mol of CH3CH2Br and H2O, which is the major product?
A) HSCH2CH2OCH2CH3
B) CH3CH2SCH2CH2OH
C) CH3CH2SCH2CH2OCH2CH3
D) CH2=CH2
E) CH3CH3
Ans: B
Topic: Predicting mechanisms, products
31. When 0.10 mol of ICH2CH2CH2CH2Cl reacts with 0.10 mol of NaOCH3 in CH3OH at
40°C, the major product is:
A) CH3OCH2CH2CH2CH2Cl
B) CH3OCH2CH2CH2CH2I
C) CH3OCH2CH2CH2CH2OCH3
D) CH2=CHCH2CH2Cl
E) CH2=CHCH2CH2I
Ans: A
Topic: Generalities
32.
A)
B)
C)
D)
E)
SN2 reactions of the type, Nu- + RL ⎯⎯⎯⎯→ Nu-R + L-, are favored:
when tertiary substrates are used.
by using a high concentration of the nucleophile.
by using a solvent of high polarity.
by the use of weak nucleophiles.
by none of the above.
Ans: B
176
Chapter 6
Topic: Generalities
33. Which of the following statements is (are) true of SN1 reactions of alkyl halides in
general?
A) The rate of an SN1 reaction depends on the concentration of the alkyl halide.
B) The rate of an SN1 reaction depends on the concentration of the nucleophile.
C) SN1 reactions of alkyl halides are favored by polar solvents.
D) Answers A) and C) only are true.
E) Answers A), B) and C) are true.
Ans: D
Topic: Generalities
34. Which of the following statements is (are) true of SN1 reactions of alkyl halides in
general?
A) The rate of an SN1 reaction depends on the concentration of the alkyl halide.
B) The rate of an SN1 reaction depends on the concentration of the nucleophile.
C) SN1 reactions of alkyl halides occur faster in polar aprotic solvents (compared to protic
solvents)
D) Answers A) and C) only are true.
E) Answers A), B) and C) are true.
Ans: A
Topic: Generalities
35.
A)
B)
C)
D)
E)
SN1 reactions of the type, Nu- + RL ⎯⎯⎯⎯→ Nu–R + L-, are favored:
when tertiary substrates are used.
by using a high concentration of the nucleophile.
when L- is a strong base.
by use of a non-polar solvent.
by none of the above.
Ans: A
Topic: Generalities
36. Which of the following statements is (are) true of an SN2 reaction of (R)-2-bromobutane
with hydroxide ion?
A) Doubling the hydroxide ion concentration would double the rate of the reaction.
(Assume that all other experimental conditions are unchanged.)
B) The major product would be (S)-2-butanol.
C) Doubling the concentration of (R)-2-bromobutane would double the rate of the reaction.
(Assume that all other experimental conditions are unchanged.)
D) All of the above
E) Two of the above
Ans: D
177
Chapter 6
Topic: Generalities
37. SN1 reactions of the following type,
Nu:- + R-X ⎯⎯⎯⎯→ R-Nu + :X-
A)
B)
C)
D)
E)
are favored:
by the use of tertiary substrates (as opposed to primary or secondary substrates).
by increasing the concentration of the nucleophile.
by increasing the polarity of the solvent.
by use of a weak nucleophile.
by more than one of the above.
Ans: E
Topic: Generalities
38. SN1 reactions of the following type,
Nu:- + R-X ⎯⎯⎯⎯→ R-Nu + :X-
A)
B)
C)
D)
E)
are favored:
by the use of tertiary substrates (as opposed to primary or secondary substrates).
by increasing the concentration of the nucleophile.
by increasing the polarity of the solvent.
by use of a strong base.
by more than one of the above.
Ans: A
Topic: Generalities
39.
A)
B)
C)
D)
E)
A true statement about the transition state(s) of an SN2 reaction is:
the two transition states are of unequal energy.
the transition states precede and follow an unstable reaction intermediate.
the single transition state represents the point of maximum free energy of the reaction.
existence of this transition state implies an exothermic reaction.
the transition state will always have a net charge of -1.
Ans: C
178
Chapter 6
Topic: Generalities
40.
A)
B)
C)
D)
E)
Elimination reactions are favored over nucleophilic substitution reactions:
at high temperatures.
when tert-butoxide ion is used.
when 3° alkyl halides are used as substrates.
when nucleophiles are used which are strong bases and the substrate is a 2° alkyl halide.
in all of these cases.
Ans: E
Topic: Carbocations
41.
A)
B)
C)
D)
E)
The hybridization state of the charged carbon in a carbocation is
sp4
sp3
sp2
sp
s
Ans: C
Topic: Carbocations
42.
A)
B)
C)
D)
E)
The p orbital of a methyl cation, CH3+, contains how many electrons?
1
2
3
4
0
Ans: E
Topic: Carbocations
43. The p orbital of the charged carbon in the isopropyl cation, (CH3)2CH+, contains how
many electrons?
A) 1
B) 2
C) 3
D) 4
E) 0
Ans: E
179
Chapter 6
Topic: Generalities, solvent effects
44. Considering the relative solvation of reactants and the transition states of SN reactions of
these reactants, predict which general type of reaction would be most favored by the use
of a polar solvent.
A) Y: + RX ⎯⎯⎯⎯→ RY+ + X:B) Y:- + RX ⎯⎯⎯⎯→ RY + X:C) Y: + RX+ ⎯⎯⎯⎯→ RY+ + X:
D) Y:- + RX+ ⎯⎯⎯⎯→ RY + X:
E) RX+ ⎯⎯⎯⎯→ R+ + X:
Ans: A
Topic: Generalities, solvent effects
45. Which is not a polar aprotic solvent?
O
A)
C
H3C
H
B) 3C
C)
C
O
CH3
N
S
H3C
O
D)
CH3
C
H
N
CH3
CH3
CH3
E)
CH
H3C
OH
Ans: E
Topic: Generalities, solvent effects
46.
A)
B)
C)
D)
E)
Which is a polar aprotic solvent?
2-methylhexane
CH3CN
NH3(l)
CH3CH2CH2OCH2CH2CH3
2-methyl-2-propanol
Ans: E
180
Chapter 6
Topic: Generalities, nucleophiles
47.
Identify the nucleophile in the following reaction.
A)
B)
C)
D)
E)
2 H2O + RX Æ ROH + H3O+ + X−
XH3O+
ROH
H2O
RX
Ans: D
Topic: Generalities, nucleophiles
48.
Identify the nucleophile in the following reaction.
A)
B)
C)
D)
E)
2 R’OH + RX Æ ROR’ + [ROH2]+ + X−
X−
[ROH2]+
ROR’
R’OH
RX
Ans: D
Topic: Transition State theory
49.
A)
B)
C)
D)
E)
An increase in the kinetic energy of reacting molecules results in:
a decrease in reaction rate.
an increase in the probability factor.
a decrease in the probability factor.
an increase in the reaction rate.
no changes.
Ans: D
181
Chapter 6
Topic: Generalities, leaving group
50. Identify the leaving group in the following reaction.
S−Na+
S
Na+
I
A)
B)
C)
D)
E)
+
I−
C6H5SNa+
CH3CH2I
C6H5SCH2CH3
IAns: E
Topic: Generalities, solvent effects
51.
A)
B)
C)
D)
E)
Which is the weakest nucleophile in polar aprotic solvents?
IBrClFAll of the above are equally strong nucleophiles, regardless of the type of solvent used
Ans: A
Topic: Generalities, solvent effects
52.
A)
B)
C)
D)
E)
Which is the weakest nucleophile in polar protic solvents?
IBrClFAll of the above are equally strong nucleophiles, regardless of the type of solvent used
Ans: D
Topic: Generalities, leaving group
53.
A)
B)
C)
D)
E)
Which of the following is the poorest leaving group?
H−
CH3O−
H2O
OH−
NH2−
Ans: E
182
Chapter 6
Topic: Generalities, nucleophiles
54.
A)
B)
C)
D)
E)
Which of the following is not a nucleophile?
H2O
CH3ONH3
NH4+
All are nucleophiles.
Ans: D
Topic: Generalities, solvent effects
55.
A)
B)
C)
D)
E)
Which ion is the strongest nucleophile in aqueous solution?
FClBrIAll of these are equally strong.
Ans: D
Topic: Leaving groups
56.
A)
B)
C)
D)
E)
Which of the following is not a good leaving group?
C2H5O−
Cl−
I−
CH3CO2−
All of these are good leaving groups
Ans: A
Topic: Leaving groups
57. Identify the leaving group in the following reaction:
CH3OH + CH3OH2
CH3OCH3 + H2O
H
A)
B)
C)
D)
E)
CH3OH
CH3OH2+
CH3OCH3
H2O
None of these
Ans: D
183
Chapter 6
Topic: Nucleophiles
58. Which is the strongest nucleophile?
A) OHB) CH3CH2OC)
D) CH3CH2OH
E) H2O
Ans: B
Topic: Nucleophile, solvent effects
59. Which is the most reactive nucleophile in DMF (structure shown below) ?
H
A)
B)
C)
D)
E)
O
C
CH3
N
CH3
FClBrIThey are all equally reactive.
Ans: A
Topic: Nucleophile, solvent effects
60. Which of these species, acting in a protic solvent, exhibits greater nucleophilic activity
than expected on the basis of its basicity?
A) OHB) CH3OC) SHD) ClE) H2O
Ans: C
184
Chapter 6
Topic: Nucleophilicity
61. The relative nucleophilicities of species do not necessarily parallel the relative basicities
of the same species because:
A) not all nucleophiles are bases, and vice versa.
B) experimental measurements of sufficient accuracy are not available to make the
comparisons.
C) nucleophilicity is a thermodynamic matter; basicity is a matter of kinetics.
D) basicity is a thermodynamic matter; nucleophilicity is a matter of kinetics.
E) Actually, the relative values do parallel one another.
Ans: D
Topic: Nucleophiles
62. Ambident nucleophiles are ones which can react with a substrate at either of two
nucleophilic sites. Which of the following is not an ambident nucleophile?
..
.. 2
..
:OCH
CH
S:
:CN:
2
2
:OH
..
..
..
I
A)
B)
C)
D)
E)
III
II
..
..
:O:N::O
..
..
H. . .. ..
. N:O:H
..
H.
IV
V
I
II
III
IV
V
Ans: B
Topic: Nucleophile, solvent effects
63.
A)
B)
C)
D)
E)
Which ion is the strongest nucleophile in an aprotic solvent such as dimethylsulfoxide?
IBrClFThese are all equal.
Ans: D
185
Chapter 6
Topic: Leaving group
64.
A)
B)
C)
D)
E)
Which of the following is a feasible substitution reaction?
CH3CH2Cl + NaBr ⎯⎯⎯⎯→ CH3CH2I + NaCl
CH3CH3 + NaCN ⎯⎯⎯⎯→ CH3CH2CN + NaH
CH3CH2Cl + NaOH ⎯⎯⎯⎯→ CH2=CH2 + H2O + NaCl
More than one of the above
None of the above
Ans: A
Topic: Predicting mechanism, products
65. Heating tert-butyl chloride with 1.0 M NaOH in a mixture of water and methanol would
yield mainly:
A) (CH3)3COH through an SN1 reaction.
B) (CH3)3COCH3 through an SN1 reaction.
C) (CH3)3COH through an SN2 reaction.
D) (CH3)3COCH3 through an SN2 reaction.
E) CH2=C(CH3)2 through an E2 reaction.
Ans: E
186
Chapter 6
Topic: Free energy diagrams
66. Select the potential energy diagram that represents an exothermic (exergonic) reaction.
A)
B)
C)
D)
E)
I
II
III
IV
V
Ans: E
187
Chapter 6
Topic: Free energy diagrams
67. Select the potential energy diagram that represents a single-step endothermic
(endergonic) reaction.
A)
B)
C)
D)
E)
I
II
III
IV
V
Ans: A
188
Chapter 6
Topic: Free energy diagrams
68. Select the potential energy diagram that represents a two-step endothermic (endergonic)
reaction.
A)
B)
C)
D)
E)
I
II
III
IV
V
Ans: B
Topic: Hammond-Leffler postulate
69. The Hammond-Leffler postulate when applied to nucleophilic substitutions and
elimination reactions states that:
A) a negatively-charged nucleophile is stronger than its conjugate acid.
B) polar aprotic solvents strongly accelerate the rate of SN2 processes.
C) bimolecular nucleophilic substitutions are 2nd order kinetically.
D) the transition state for an endergonic reaction step (one accompanied by an increase in
free energy) resembles the product of that step.
E) elimination reactions will always compete with nucleophilic substitution reactions.
Ans: D
189
Chapter 6
Topic: Transition state theory
70.
A)
B)
C)
D)
E)
Which will be true for any actual or potential nucleophilic substitution reaction?
ΔH° is positive.
ΔH° is negative.
ΔG‡ is positive.
ΔG° is positive.
ΔG° is negative.
Ans: C
Topic: Entropy changes
71. For the typical SN2 reaction
Y- + RX ⎯⎯⎯⎯→ RY + X-
A)
B)
C)
D)
E)
it can be predicted that ΔS‡ will be:
positive.
zero.
negative.
either positive or negative.
unpredictable as to algebraic sign.
Ans: C
Topic: Free energy changes
72. The reaction,
CH3Cl + OH−
A)
B)
C)
D)
E)
H2O
CH3OH +
Cl−
has the following thermodynamic values at 27ºC: ΔH = -75.3 kJ mol-1;
ΔS = 54.4 J K-1 mol-1. What is the value of ΔG for this reaction?
-73.8 kJ mol-1
-76.8 kJ mol-1
-59.0 kJ mol-1
+91.6 kJ mol-1
-91.6 kJ mol-1
Ans: E
190
Chapter 6
Topic: Activation energy
73.
A)
B)
C)
D)
E)
An increase in the temperature at which reaction is carried out increases:
the collision frequency.
the fraction of molecules with proper orientation.
the fraction of molecules with energy greater than Eact.
More than one of the above
None of the above
Ans: D
Topic: Free energy diagrams
74. The difference in the bond energies of reactants and the transition state of a reaction is
designated by the notation:
A) ΔH°
B) ΔH‡
C) ΔG°
D) ΔG‡
E) ΔS‡
Ans: B
Topic: Hammond-Leffler postulate
75. Which is a true statement concerning the transition state of the rate-determining step of
an SN1 reaction?
A) Structurally, it closely resembles the carbocation intermediate.
B) Both covalent bond-breaking and bond-making are occurring.
C) Formation of the transition state is an exothermic reaction.
D) Necessarily, the transition state has zero charge overall.
E) More than one of the above
Ans: A
Topic: Stereochemistry of SN/E reactions
76. Which of the following reactions proceeds with inversion of configuration at the carbon
bearing the leaving group?
A) SN2
B) SN1
C) E2
D) E1
E) All of these
Ans: A
191
Chapter 6
Topic: Predicting mechanism, stereochemistry of SN reactions
77. Consider the substitution reaction that takes place when (R)-3-bromo-3-methylhexane is
treated with methanol. Which of the following would be true?
A) The reaction would take place only with inversion of configuration at the stereogenic
center.
B) The reaction would take place only with retention of configuration at the stereogenic
center.
C) The reaction would take place with racemization.
D) No reaction would take place.
E) The alkyl halide does not possess a stereogenic center.
Ans: C
Topic: Predicting mechanism, stereochemistry of SN reactions
78. Consider the substitution reaction that takes place when (R)-3-bromo-3-methylhexane is
treated with sodium methoxide. Which of the following would be true?
A) An SN2 reaction would take place with inversion of configuration at the stereogenic
center.
B) An SN1 reaction would take place with retention of configuration at the stereogenic
center.
C) An SN1 reaction would take place with racemization of configuration at the stereogenic
center.
D) An SN1 reaction would take place, accompanied by an E1 reaction, affording a complex
mixture of products.
E) An E2 reaction would take place, during which the stereogenic center is lost.
Ans: E
Topic: Predicting mechanism, stereochemistry of SN reactions
79. Consider the substitution reaction that takes place when (R)-3-iodo-3-methylheptane is
treated with sodium acetate (CH3CO2Na). Which of the following would be true?
A) An SN2 reaction would take place with inversion of configuration at the stereogenic
center.
B) An SN1 reaction would take place with retention of configuration at the stereogenic
center.
C) An SN1 reaction would take place with racemization of configuration at the stereogenic
center.
D) An SN1 reaction would take place, accompanied by an E1 reaction, affording a complex
mixture of products.
E) An E2 reaction would take place, during which the stereogenic center is lost.
Ans: D
192
Chapter 6
Topic: Predicting products
80. Which alkyl halide, when treated with sodium ethoxide in ethanol, would afford a
product mixture consisting of more than one elimination product?
A) 1-bromo-3,3-dimethylpentane
B) 1-bromo-2,3-dimethylpentane
C) 2-bromo-3,4-dimethylpentane
D) 2-bromo-3,3-dimethylpentane
E) None of the above would yield more than one elimination product
Ans: C
Topic: Leaving groups, nucleophiles
81.
A)
B)
C)
D)
E)
Which SN2 reaction will occur most rapidly in a mixture of water and ethanol?
I- + CH3CH2–Br ⎯⎯⎯⎯→ CH3CH2–I + BrI- + CH3CH2–Cl ⎯⎯⎯⎯→ CH3CH2–I + ClI- + CH3CH2–F ⎯⎯⎯⎯→ CH3CH2–I + FBr- + CH3CH2–Cl ⎯⎯⎯⎯→ CH3CH2–Br + ClBr- + CH3CH2–F ⎯⎯⎯⎯→ CH3CH2–Br + FAns: A
Topic: Predicting mechanism, kinetics
82. Which SN2 reaction will occur most rapidly in aqueous acetone solution? Assume
concentrations and temperature are the same in each instance.
A) HO- + CH3-Cl ⎯⎯⎯⎯→ CH3OH + ClB) HO- + CH3CH2-Cl ⎯⎯⎯⎯→ CH3CH2OH + ClC) HO- + (CH3)2CH-Cl ⎯⎯⎯⎯→ (CH3)2CHOH + ClD) HO- + (CH3)3C-Cl ⎯⎯⎯⎯→ (CH3)3COH + ClE) HO- + (CH3)3CCH2-Cl ⎯⎯⎯⎯→ (CH3)3CCH2OH + ClAns: A
Topic: Kinetics
83.
A)
B)
C)
D)
E)
Which alkyl halide would you expect to undergo an SN2 reaction most slowly?
1-bromohexane
1-bromo-2-methylpentane
1-bromo-3-methylpentane
1-bromo-4-methylpentane
1-bromo-2,2-dimethylbutane
Ans: E
193
Chapter 6
Topic: Kinetics, nucleophilicity
84. Which SN2 reaction would take place most rapidly?
H2O
A)
OH− + CH3Cl
CH3OH + Cl−
o
25 C
O
O
B)
H2O
−
+ CH3Cl
+ Cl
o
−
25 C
H3C
O
H3CO
CH3
H 2O
C)
H2O + CH3Cl
CH3OH2+ + Cl−
25oC
H
D)
CH3OH
O
CH3OH + CH3Cl
+ Cl−
25oC
H3C
CH3
H2O
E)
SH− + CH3Cl
CH3SH + Cl−
o
25 C
Ans: E
Topic: Leaving groups
85.
A)
B)
C)
D)
E)
Which nucleophilic substitution reaction would be unlikely to occur?
HO− + CH3CH2–I ⎯⎯⎯⎯→ CH3CH2–OH + I−
I− + CH3CH2–H ⎯⎯⎯⎯→ CH3CH2–I + H−
CH3S− + CH3–Br ⎯⎯⎯⎯→ CH3S–CH3 + Br−
All of the above would be unlikely to occur
None of the above would be unlikely to occur
Ans: B
Topic: Predicting products
86. Which nucleophilic substitution reaction would be likely to occur (although probably
not in excellent yield, due to competing elimination) ?
A) HO− + CH3CH2CH2CHICH3 ⎯⎯⎯⎯→ CH3 CH2CH2CHOHCH3 + I−
B) Cl− + CH3CH2CH2CH2–OH ⎯⎯⎯⎯→ CH3CH2CH2CH2–Cl + OH−
C) CH3S− + CH3CH2CH2–OCH3 ⎯⎯⎯⎯→ CH3S–CH2CH2CH3 + OCH3−
D) All of the above are likely to occur
E) None of the above are likely to occur
Ans: B
194
Chapter 6
Topic: Leaving groups
87. Which alkyl halide would you expect to react most slowly when heated in aqueous
solution?
A) (CH3)3C-F
B) (CH3)3C-Cl
C) (CH3)3C-Br
D) (CH3)3C-I
E) They would all react at the same rate.
Ans: A
Topic: Nucleophilicity, leaving groups
88. Which SN2 reaction would you expect to take place most rapidly? Assume that the
concentrations of the reactants and the temperature are the same in each instance:
−
CH3SCH3 + I −
A) CH3S + CH3I
H
B)
CH3SH + CH3I
−
C) CH3O
D)
+ CH3I
CH3OH + CH3I
−
E) CH3S + CH3Cl
Ans: A
S
H3C
CH3
CH3OCH3 +
+ I−
I
−
H
O
H3C
CH3
CH3SCH3 +
+ I−
Cl −
Topic: Leaving groups
89.
A)
B)
C)
D)
E)
Which nucleophilic substitution reaction is not likely to occur?
I− + CH3CH2–Cl ⎯⎯⎯⎯→ CH3CH2–I + Cl−
I− + CH3CH2–Br ⎯⎯⎯⎯→ CH3CH2–I + Br−
I− + CH3CH2–OH ⎯⎯⎯⎯→ CH3CH2–I + OH−
CH3O− + CH3CH2–Br ⎯⎯⎯⎯→ CH3CH2–OCH3 + Br−
OH− + CH3CH2–Cl ⎯⎯⎯⎯→ CH3CH2–OH + Cl−
Ans: C
195
Chapter 6
Topic: Leaving groups
90. Which of the following would be most reactive in an SN2 reaction?
A)
Br
B)
Br
C)
Br
D)
Br
E)
Br
Ans: D
Topic: Steric hindrance
91. Which alkyl chloride, though primary, is essentially unreactive in SN2 reactions?
Cl
A)
B)
Cl
C)
Cl
Cl
D)
E)
Cl
Ans: E
196
Chapter 6
Topic: Kinetics
92. Which alkyl halide would be most reactive in an SN1 reaction?
Br
Br
Br
I
II
III
Br
Br
IV
A)
B)
C)
D)
E)
V
I
II
III
IV
V
Ans: D
Topic: Predicting mechanisms
93. Which of these compounds would give the largest E2/SN2 product ratio on reaction with
sodium ethoxide in ethanol at 55°C?
Cl
A)
B)
Cl
C)
Cl
D)
Cl
E)
Cl
Ans: D
197
Chapter 6
Topic: Multistep synthesis, acid-base reactions, SN2
94. What final product is likely to be obtained through the following series of reactions?
NaNH2
Br
A
B
NH3(l)
Ans:
Topic: Multistep synthesis, acid-base reactions, SN2, hydrogenation
95. What final product is likely to be obtained through the following series of reactions?
NaNH2
Cl
A
NH3(l)
B
excess H2
C
Ni
Ans:
Topic: Multistep synthesis, acid-base reactions, SN2
96. What final product is likely to be obtained through the following series of reactions?
NaH
Cl
OH
B
A
diethylether
Ans:
O
SHORT ANSWER QUESTIONS
Topic: Bond length, bond strength, leaving groups
97. As we go down Group 7A of the periodic table, the size of the halogen atom increases;
accordingly, the carbon-halogen bond length gets _______________ and the bond
strength gets _______________.
Ans: longer; weaker
198
Chapter 6
Topic: Transition state theory
98. In the SN2 reaction, the unstable arrangement of atoms in which both the nucleophile
and the leaving group are partially bonded to the same carbon atom is called the
______________________.
Ans: transition state
Topic: Solvolysis
99. A reaction in which the nucleophile is a molecule of the solvent is referred to as a
______________ reaction.
Ans: solvolysis
Topic: Carbocation stability
100. The substitution mechanism whose rate depends primarily on the relative stability of the
intermediate carbocation is the _________.
Ans: SN1
Topic: Steric hindrance
101. The substitution mechanism whose rate depends primarily on the degree of steric
hindrance around the leaving group is the _________.
Ans: SN2
Topic: Carbocation stability
102. The stabilizing effect of alkyl substituents on carbocations can be explained through
_____________________.
Ans: hyperconjugation
Topic: Activation energy, kinetics
103. Increasing the concentration of either of the reactants of an SN2 reaction increases the
rate of the reaction. The primary reason for this is that increasing the concentration
increases ________________________.
Ans: the frequency of collisions between reactant species, thereby statistically
increasing the chance encounters between species having the requisite Eactivation
Topic: Activation energy, transition state theory
104. In order for colliding species to react, they must _____________ and ____________.
Ans: have energy equal/greater than the activation energy, approach with the proper
orientation.
199
Chapter 6
Topic: Free energy diagrams
105. Draw the potential energy diagram that represents an exothermic reaction between a
tertiary alkyl halide and methanol. Briefly explain your rationale.
Ans: The reaction is likely to occur via an SN1 mechanism. The first step is the ratedetermining step, leading to the formation of a tertiary carbocation. The second
step is a fast step, where the methanol oxygen acts as a nucleophile, attaching to
the positively charged carbon, forming a protonated ether; the last step is a fast
loss of a proton to form the (uncharged) ether product.
Energy
time
Topic: Predicting reaction mechanisms
106. Give a detailed reaction mechanism for the reaction expected to occur when 2-bromo-2methylpentane is heated with sodium methoxide. Draw clear structural formulas of all
relevant species and use curved arrows to represent electron flow. Also indicate which
step is likely to be rate-determining.
Ans: The reaction is likely to occur via an E2 mechanism. The first step is the ratedetermining step, leading to the formation of a tertiary carbocation, followed by
base promoted elimination to afford 2 different alkene products.
..
: Br :
H
slow
..
[ - Br:
.. ]
H
200
..
:OCH
.. 3
fast
Chapter 6
Topic: SN1 kinetics
107. Typically, increasing the concentration of the nucleophile of an SN1 reaction has no
impact on the rate of the reaction. The reason for this is that
________________________.
Ans: An SN1 reaction is a multi-step process; the overall reaction rate is determined by
the rate of formation of the carbocation intermediate. Since the nucleophile is not
involved in the rate-determining step, changes in its concentration typically have
no impact on the rate of reaction.
Topic: Nucleophilicity, solvent effects
108. The relative nucleophilicity of the halide ions in polar aprotic solvents is observed to be
markedly different from that in protic solvents. Explain briefly.
Ans: The smaller, more highly electronegative fluoride ion is more effectively solvated
in a protic solvent and is therefore a weaker nucleophile compared to the larger,
less electronegative and more polarizable iodide ion. The situation is reversed in
aprotic solvents, where the nucleophile is essentially unsolvated; thus, the
nucleophilicity parallels the basicity of the halide ions, so that the fluoride ion is
the strongest nucleophile of the halide ions
Topic: Acid-base reactions, intramolecular SN2
109. When 5-bromo-1-pentanol is treated with sodium hydride in diethyl ether, the product is
analyzed to be C5H10O. Propose a likely structure for this product, suggesting a
reasonable mechanistic pathway for its formation.
Ans: The alcohol is likely to be first deprotonated by the strong base, generating an
alkoxide ion, which can then undergo intramolecular SN2 reaction to afford the
observed product, a cyclic ether.
Cl
..
OH
..
NaH
−H2
Cl
201
..
+
O
.. : Na
O
NaCl
Chapter 7
MULTIPLE CHOICE QUESTIONS
Topic: Nomenclature
1. A correct IUPAC name for the following compound is:
A)
B)
C)
D)
E)
3,3,5-trimethyl-2-hexene
3-isobutyl-3-isopropyl-2-propene
3-isobutyl-4-methyl-2-pentene
3-(1-methylethyl)-5-methyl-2-hexene
None of the above
Ans: D
Topic: Nomenclature
2. The correct IUPAC name for the following compound is:
Br
A)
B)
C)
D)
E)
2-Bromo-4-methylenehexane
2-(2-Bromopropyl)-1-butene
4-Bromo-2-ethyl-1-pentene
2-Bromo-4-ethyl-1-pentene
2-Bromo-4-ethyl-4-pentene
Ans: C
Topic: Nomenclature
3. Give the IUPAC name for
A)
B)
C)
D)
E)
3-Methyl-4-hexyne
4-Methyl-2-hexyne
2-Ethyl-3-pentyne
4-Ethyl-2-pentyne
3-Methyl-2-hexyne
Ans: B
202
Chapter 7
Topic: Nomenclature
4. The correct IUPAC name for the following compound is:
A)
B)
C)
D)
E)
4,5-Dimethyl-3-propyl-2-hexene
4,5-Dimethyl-3-propyl-1-hexene
3-(2,3-Dimethylpropyl)-1-hexene
2,3-Dimethyl-4-isopropyl-5-hexene
2,3-Dimethyl-4-propyl-5-hexene
Ans: B
Topic: Nomenclature
5. Select the structure of 4–ethyl-2,3-dimethyl-2-heptene.
A)
B)
C)
D)
E)
Ans: B
203
Chapter 7
Topic: Nomenclature
6. Select the structure for cis-3-methyl-6-vinylcyclohexene.
I
II
IV
A)
B)
C)
D)
E)
III
V
I
II
III
IV
V
Ans: B
Topic: Nomenclature
7. The correct IUPAC name for the following compound is:
Cl
Br
A)
B)
C)
D)
E)
(E)-2-Bromo-3-chloro-5-methyl-2-hexene
(E)-2-Bromo-3-chloro-5-methyl-3-hexene
(Z)-2-Bromo-3-chloro-5-methyl-3-hexene
(Z)-2-Bromo-3-chloro-5-methyl-2-hexene
(E)-2-Methyl-5-bromo-4-chloro-4-hexene
Ans: A
204
Chapter 7
Topic: Nomenclature
8.
A)
B)
C)
D)
E)
The IUPAC name for diisobutylacetylene is
2,7-Dimethyl-4-octene
2,7-Dimethyl-4-octyne
3,6-Dimethyl-4-octyne
2,5-Diethyl-3-hexyne
2,2,5,5-Tetramethyl-3-hexyne
Ans: B
Topic: Carbocation Stability, Rearrangement
9. Which of the following carbocations would NOT be likely to undergo rearrangement?
A) CH3CHCHCH3
CH3
CH3
B)
CH3CHCCH3
C)
CH3
CH3
CH3CCH2CH3
D)
CH3
CH3CHCH2
CH3
E)
CH3CCHCH2CH3
CH3
Ans: C
205
Chapter 7
Topic: Carbocation stability
10. Which alcohol would initially produce the most stable carbocation when treated with
concentrated H2SO4?
A)
OH
B)
OH
C)
OH
D)
HO
E) HO
Ans: A
Topic: Carbocation Stability, Rearrangement
11.
A)
B)
C)
D)
E)
Which statement(s) is (are) true of acid-catalyzed alcohol dehydrations?
Protonation of the alcohol is a fast step.
Formation of a carbocation from the protonated alcohol is a slow step.
Rearrangements of less stable carbocations to more stable carbocations are common.
Loss of a proton by the carbocation is a fast step.
All of the above
Ans: E
Topic: Carbocation Stability, Rearrangement
12. Carbocations are frequent intermediates in acidic reactions of alkenes, alcohols, etc.
What do carbocations usually do? They may:
A) rearrange to a more stable carbocation.
B) lose a proton to form an alkene.
C) combine with a nucleophile.
D) react with an alkene to form a larger carbocation.
E) do all of the above.
Ans: E
206
Chapter 7
Topic: Carbocation Stability, Rearrangement
13. Neopentyl alcohol, (CH3)3CCH2OH, cannot be dehydrated to an alkene without
rearrangement. What is the chief product of dehydration?
I
II
IV
A)
B)
C)
D)
E)
III
V
I
II
III
IV
V
Ans: B
Topic: Carbocation Stability, Rearrangement
14.
A)
B)
C)
D)
E)
Rearrangements are likely to occur in which of the following reaction types?
SN1 reactions
SN2 reactions
E1 reactions
E2 reactions
Both SN1 and E1 reactions
Ans: E
207
Chapter 7
Topic: Alkene Synthesis, Reaction Mechanisms
15. Which compound listed below would you expect to be the major product when 2bromo-2-methylbutane is refluxed with KOH/ethanol?
A)
OH
B)
O
C)
D)
E)
Ans: D
Topic: Alkene Synthesis, Reaction Mechanisms
16. Which product(s) would be produced by acid-catalyzed dehydration of 2-methyl-2pentanol?
A)
B)
and
C)
and
D)
E)
O
Ans: B
208
Chapter 7
Topic: Alkene Synthesis, Reaction Mechanisms
17. Which product (or products) would be formed in appreciable amount(s) when trans-1bromo-2-methylcyclohexane undergoes dehydrohalogenation upon treatment with
sodium ethoxide in ethanol?
I
A)
B)
C)
D)
E)
II
III
IV
I
II
III
IV
More than one of these
Ans: B
Topic: Alkene Reactions, Reaction Mechanisms
18. Which would be the major product of the following reaction?
D2, Ni
?
D
CH3
D
D CH3
DD
H CH3
H
D
I
H
II
III
CH3
DH
D
D
CH3
D
H
IV
A)
B)
C)
D)
E)
V
I
II
III
IV
V
Ans: A
209
Chapter 7
Topic: Alkene Synthesis, Reaction Mechanisms
19. What is the major product of the reaction,
(CH3)3COK
?
(CH3)3COH
heat
Br
A)
B)
C)
D)
E)
(CH3)2C=C(CH3)2
(CH3)3C–CH=CH2
(CH3)2C=CHCH3
(CH3)2C=CHCH2CH3
None of these
Ans: B
Topic: Alkene Synthesis, Reaction Mechanisms
20. Which compound would be the major product?
Br
(CH3)3COK
?
(CH3)3COH
heat
O
A)
B)
C)
OH
D)
E)
Ans: E
210
Chapter 7
Topic: Alkene Synthesis, Reaction Mechanisms, Carbocation Rearrangements
21. What will be the major product of the following reaction?
85% H3PO4
?
heat
OH
A)
B)
C)
D)
O
E)
Ans: A
211
Chapter 7
Topic: Alkene Synthesis, Reaction Mechanisms, Carbocation Rearrangements
22. Which alkene would you expect to be the major product of the following dehydration?
H2SO4
?
heat
OH
I
IV
A)
B)
C)
D)
E)
III
II
V
I
II
III
IV
V
Ans: C
212
Chapter 7
Topic: Alkyne Synthesis, Catalytic hydrogenation
23. The structure of the product, C, of the following sequence of reactions would be:
NaNH2
A
CH3CH2Br
NH3(l)
B
H2
Ni2B [P-2]
A)
B)
C)
Br
D)
E)
Ans: A
Topic: Dissolving Metal Reduction; trans-hydrogenation
24. The structure of the product obtained from 2-butyne and Li/C2H5NH2 is:
A)
B)
C)
HN
D)
NH
E)
Ans: A
213
C
Chapter 7
Topic: Catalytic hydrogenation
25. Which alkene would yield 3-methylpentane when subjected to catalytic hydrogenation?
A)
B)
C)
D)
E)
Ans: B
Topic: Alkyne Synthesis
26.
A)
B)
C)
D)
E)
Which reaction would yield 2-butyne?
CH3C≡C:− Na+ + CH3Br ⎯⎯⎯⎯→
CH3CH2Br + HC≡C:− Na+ ⎯⎯⎯⎯→
CH3:− Na+ + HC≡CCH3 ⎯⎯⎯⎯→
More than one of these
None of these
Ans: A
Topic: Alkyne Synthesis
27. Which of the following methods could be used to synthesize 4,4-dimethyl-2-hexyne?
CH3
A)
C2H5 C C C: Na
CH3
H3C
I
CH3
B)
H3C C C:
Na
C2H5 C Br
CH3
C)
NaNH2 ( excess)
Br
Br
NH3 (l)
D) More than one of these
E) None of these
Ans: D
214
Chapter 7
Topic: Alkene Synthesis
28. Your task is to convert 2-bromobutane to 1-butene in highest yield. Which reagents
would you use?
A) KOH/H2O
B) KOH/CH3OH
C) CH3ONa/CH3OH
D) CH3CH2ONa/CH3CH2OH
E) (CH3)3COK/(CH3)3COH
Ans: E
Topic: Alkene Synthesis, Nomenclature
29. Which of the following reactions would yield 3,3-dimethyl-1-butene in a reasonable
percentage yield (i.e., greater than 50%)?
A)
H2SO4
heat
OH
B)
(CH3)3COK, (CH3)3COH
heat
Br
C)
i) 3 NaNH2, mineral oil, heat
Br
Br
ii) H3O+
D) All of these
E) Answers B) and C) only
Ans: B
215
Chapter 7
Topic: Alkene Synthesis, Reaction Mechanisms
30. Which compound(s) would be produced by the following reaction?
CH3
CH3CH2OH
Br
H3C
55oC
?
H
OCH2CH3
CH3
OCH2CH3
H3C
H
A)
B)
C)
D)
E)
I
I
II
III
More than one of the above
All of the above
Ans: E
CH3
CH3
H3C
CH3
H
II
III
Topic: Alkene Synthesis, Nomenclature, Reaction Mechanisms
31. Dehydrohalogenation of tert-pentyl bromide will produce 2-methyl-1-butene as the
chief product when:
A) CH3COONa is employed as the base.
B) KOH/C2H5OH is employed as the base.
C) CH3CH2ONa/CH3CH2OH is employed as the base.
D) (CH3)3COK/(CH3)3COH is employed as the base.
E) any base is used, as long as the temperature is sufficiently high.
Ans: D
Topic: Alkene Synthesis, Nomenclature, Reaction Mechanisms
32.
A)
B)
C)
D)
E)
Which is not a satisfactory procedure for the synthesis of 3-methyl-1-butene?
(CH3)2CHCΗΟΗCH3 + conc. H2SO4
(CH3)2CHC≡CH + Li/liq.NH3
(CH3)2CHCH2CH2Br + CH3ONa/CH3OH
(CH3)2CHCHBrCH3 + (CH3)3COK/(CH3)3COH
(CH3)2CHC≡CH + H2/Ni2B (P-2)
Ans: A
216
Chapter 7
Topic: Alkene Synthesis, Nomenclature, Reaction Mechanisms
33.
A)
B)
C)
D)
E)
Which of these is the most satisfactory method for the preparation of cis-2-pentene?
CH3CHBrCH2CH2CH3 + (CH3)3COK/(CH3)3COH
CH3C≡CCH2CH3 + H2,Pt
CH3C≡CCH2CH3 + H2, Ni2B (P-2)
CH3C≡CCH2CH3 + Li/liq. NH3
CH3CH2CHBrCH2CH3 + CH3CH2ONa/CH3CH2OH
Ans: C
Topic: Alkyne Reduction, Nomenclature, Reaction Mechanisms
34.
A)
B)
C)
D)
E)
Which of the following reductions of an alkyne is NOT correct?
2-Pentyne + 2H2/Pt Æ pentane
2-Pentyne + H2/Ni2B Æ Z-2-Pentene
2-Pentyne + Li/NH3(l) Æ Z-2-Pentene
All of the above are correct.
None of the above is correct.
Ans: C
Topic: Alkene Synthesis, Nomenclature, Reaction Mechanisms
35. Which of the following reactions would yield 2-pentyne?
I
A)
NaNH2
(1 mol)
NaNH2
B)
Br
(1 mol)
C)
Br
Zn
CH3CO2H
Br
HA
heat
D)
OH
E)
Br
NaOC2H5
C2H5OH
Ans: B
217
Chapter 7
Topic: Alkyne Synthesis, Nomenclature, Reaction Mechanisms
36. Which reaction would not be a method for preparing 5-methyl-1-hexyne?
A)
1) NaNH2 (2 mol), liq NH3
Br
2) NH4+
B)
HC
CNa
+ Cl
C)
Li
D)
+ HC
CH
1) NaNH2 (3 mol), liq NH3
Br
Br
Br
E)
Br
2) NH4+
1) NaNH2 (3 mol), liq NH3
2) NH4+
Ans: C
Topic: Index of Hydrogen Deficiency
37.
A)
B)
C)
D)
E)
Compute the index of hydrogen deficiency for the molecule C10H8.
3
4
5
6
7
Ans: E
218
Chapter 7
Topic: Index of Hydrogen Deficiency
38. Which compound has an index of hydrogen deficiency equal to three?
I
II
III
IV
A)
B)
C)
D)
E)
V
I, III
IV, V
V
II
II, V
Ans: E
Topic: Index of Hydrogen Deficiency, Nomenclature
39.
A)
B)
C)
D)
E)
What is the index of hydrogen deficiency of bicyclo[2.2.2]octane?
1
2
3
4
5
Ans: B
Topic: Index of Hydrogen Deficiency, Catalytic Hydrogenation
40. Which is a possible structure for a compound with an index of hydrogen deficiency
equal to 3 and which absorbs one molar equivalent of hydrogen when treated with
hydrogen and a platinum catalyst?
A)
B)
C)
D)
E)
I
I
II
III
IV
V
Ans: C
II
III
IV
219
V
Chapter 7
Topic: Index of Hydrogen Deficiency, Catalytic Hydrogenation
41. Upon catalytic hydrogenation, a compound C6H6 absorbs four moles of hydrogen.
Select a structure for C6H6.
I
A)
B)
C)
D)
E)
II
III
IV
V
I, II
III
II, III
IV, V
I, IV, V
Ans: D
Topic: Index of Hydrogen Deficiency, Catalytic Hydrogenation
42. A compound, C6H10, which reacts with 2 mol of hydrogen over a platinum catalyst and
which shows an IR absorption band at approximately 3300 cm-1 could be:
I
II
IV
A)
B)
C)
D)
E)
III
V
I
II
III
IV
V
Ans: C
220
Chapter 7
Topic: Index of Hydrogen Deficiency, Catalytic Hydrogenation
43. On hydrogenation, a compound C9H12 absorbs 2 mol of hydrogen. Which of the
following is a possible structure for the compound?
I
IV
A)
B)
C)
D)
E)
III
II
V
I
II
III
IV
V
Ans: D
Topic: Index of Hydrogen Deficiency, Catalytic Hydrogenation
44. A compound with the formula C10H14 reacts with excess hydrogen and a catalyst to
yield a compound with the formula C10H18. The compound could have:
A) one ring and three double bonds.
B) two rings and two double bonds.
C) two rings and a triple bond.
D) no rings and two double bonds.
E) More than one of the above
Ans: E
Topic: Index of Hydrogen Deficiency, Catalytic Hydrogenation
45.
A)
B)
C)
D)
E)
X (C8H14)
H2, Pt
25oC
Given:
One can conclude that X has:
no rings and no double bonds.
no rings and one double bond.
one ring and one double bond.
two rings and no double bonds.
one triple bond.
Ans: C
Y (C8H16)
221
Chapter 7
Topic: Index of Hydrogen Deficiency, Catalytic Hydrogenation
46. What is the structure of a compound with formula C6H10 which has IR absorption at
approximately 3300 cm-1 and which can be catalytically reduced with hydrogen to 2methylpentane?
A)
B)
C)
D)
E)
Ans: D
Topic: Index of Hydrogen Deficiency, Catalytic Hydrogenation
47. A compound X with the formula C7H10 undergoes catalytic hydrogenation to produce a
compound Y with the formula C7H14. What might be true of X?
A) X might have one triple bond and one ring.
B) X might have two double bonds and one ring.
C) X might have one double bond and two rings.
D) X might have one double bond and one triple bond.
E) More than one of the above
Ans: E
Topic: Alkene Stability, Heats of Hydrogenation and Combustion
48. Which reaction would you expect to liberate the least heat?
A)
+ 9O2
6CO2 + 6H2O
B)
C)
+ 9O2
6CO2 + 6H2O
+ 9O2
6CO2 + 6H2O
+ 9O2
D)
E)
+ 9O2
6CO2 + 6H2O
6CO2 + 6H2O
Ans: C
222
Chapter 7
Topic: Alkene Stability, Heats of Hydrogenation and Combustion
49. One mol of each of the following alkenes is subjected to complete combustion. Which
would you expect to liberate the LEAST heat?
A) 1-Pentene
B) 2-Pentene
C) 2-Methyl-1-butene
D) 2-Methyl-2-butene
E) 3-Methyl-1-butene
Ans: D
Topic: Alkene Stability, Heats of Hydrogenation and Combustion
50. Which molecule would have the lowest heat of hydrogenation?
A)
B)
C)
D)
E)
I
I
II
III
IV
V
Ans: A
II
III
IV
V
Topic: Alkene Stability, Heats of Hydrogenation and Combustion, Nomenclature
51. One mole of each of the following alkenes is subjected to complete combustion. Which
would you expect to liberate the most heat?
A) 1-Pentene
B) cis-2-Pentene
C) 2-Methyl-1-butene
D) 2-Methyl-2-butene
E) trans-2-Pentene
Ans: A
223
Chapter 7
Topic: Alkene Stability, Heats of Hydrogenation and Combustion
52. Which reaction would you expect to liberate the most heat?
A)
+ 9O2
6CO2 + 6H2O
B)
C)
+ 9O2
6CO2 + 6H2O
+ 9O2
6CO2 + 6H2O
+ 9O2
D)
E)
+ 9O2
6CO2 + 6H2O
6CO2 + 6H2O
Ans: C
Topic: Alkene Stability, Heats of Hydrogenation and Combustion
53. Heats of hydrogenation data would be useful in comparing the relative stabilities of
which of the following substances?
I
II
III
IV
V
I, II, III
III, IV
I, II, V
Heats of hydrogenation data would not be a useful way to compare relative stabilities of
any of the substances
E) All of the above substances could effectively be compared using heats of hydrogenation
data
Ans: C
A)
B)
C)
D)
Topic: Alkene Stability, Heats of Hydrogenation and Combustion
54. Which alkene would liberate the most heat per mole when subjected to complete
combustion?
A)
B)
C)
D)
E)
Ans: A
224
Chapter 7
Topic: Alkene Stability, Heats of Hydrogenation and Combustion
55. Which alkene would liberate the most heat per mole when subjected to catalytic
hydrogenation?
A)
B)
C)
D)
E)
Ans: A
Topic: Alkene Stability, Heats of Hydrogenation and Combustion
56. One mole of each of the following alkenes is subjected to complete combustion. Which
would liberate the least heat?
I
A)
B)
C)
D)
E)
II
III
IV
V
I
II
III
IV
V
Ans: D
Topic: Alkene Stability, Heats of Hydrogenation and Combustion
57. Which alkene is most stable?
I
A)
B)
C)
D)
E)
II
III
IV
I
II
III
IV
V
Ans: A
225
V
Chapter 7
Topic: Alkene Stability, Heats of Hydrogenation and Combustion
58. Concerning the relative stabilities of alkenes, which is an untrue statement?
A) Unless hydrogenation of the alkenes gives the same alkane, heats of hydrogenation
cannot be used to measure their relative stabilities.
B) In general, the greater the number of alkyl groups attached to the carbon atoms of the
double bond, the greater the stability of the alkene.
C) The greater the quantity of heat liberated on combustion or hydrogenation of an alkene,
the greater its energy content.
D) trans-Cycloalkenes are always more stable than the cis-isomers.
E) Heats of combustion can be used to measure the relative stabilities of isomeric alkenes,
even though their hydrogenation products are not identical.
Ans: D
Topic: Relative Acidity of Alkenes, Alkynes; Acid-Base Chemistry
59.
A)
B)
C)
D)
E)
Which of the following correctly lists the compounds in order of decreasing acidity?
H2O > HC≡CH > NH3 > CH3CH3
HC≡CH > H2O > NH3 > CH3CH3
CH3CH3 > HC≡CH > NH3 > H2O
CH3CH3 > HC≡CH > H2O > NH3
H2O > NH3 > HC≡CH > CH3CH3
Ans: A
Topic: Relative Acidity of Alkenes, Alkynes; Acid-Base Chemistry
60.
A)
B)
C)
D)
E)
Select the strongest base.
OH−
RC≡C−
NH2−
CH2=CH−
CH3CH2−
Ans: E
Topic: Relative Acidity of Alkenes, Alkynes; Acid-Base Chemistry
Which ion is the weakest base?
CH3CH2: −
CH2=CH: −
HC≡C: −
.. −
NH2
..
E) OH −
..
Ans: E
..
61.
A)
B)
C)
D)
..
226
Chapter 7
Topic: Relative Acidity of Alkenes, Alkynes; Acid-Base Chemistry
62. In the following hydrocarbon, which hydrogen would have the smallest value for pKa?
CH3CH2CH=CHCH2CH2C≡CH
A) CH3CH2CH=CHCH2CH2C CH
B) CH3CH2CH=CHCH2CH2C CH
C) CH3CH2CH=CHCH2CH2C CH
D) CH3CH2CH=CHCH2CH2C CH
E) CH3CH2CH=CHCH2CH2C CH
Ans: E
Topic: Relative Acidity of Alkenes, Alkynes; Bond Lengths
63. Which of the following statements is true when ethane, ethene and acetylene are
compared with one another?
A) Acetylene is the weakest acid and has the longest C-H bond length.
B) Acetylene is the strongest acid and has the shortest C-H bond length.
C) Ethane is the strongest acid and has the longest C-H bond length.
D) Ethene is the strongest acid and has the shortest C-H bond length.
E) Ethene is the weakest acid and has the longest C-H bond length.
Ans: B
227
Chapter 7
Topic: Alkene Synthesis; Alcohol Dehydration
64. Which one of the following alcohols would dehydrate most rapidly when treated with
sulfuric acid?
OH
OH
I
OH
II
III
OH
OH
IV
A)
B)
C)
D)
E)
V
I
II
III
IV
V
Ans: B
Topic: Alkene Synthesis; Alcohol Dehydration
65. Which mechanistic step in the acid-catalyzed dehydration of 3,3-dimethyl-2-butanol is
the rate determining step?
A) Step 1:
+
H3O+
+
H2O
OH2+
OH
B) Step 2:
+
H2O
OH2+
C) Step 3:
D) Step 4a:
+
H2O
+
H3O+
+
H 2O
+
H3O
E) Step 4b:
Ans: B
228
+
Chapter 7
Topic: Alkene Synthesis; Alcohol Dehydration
66. Which alcohol would be most easily dehydrated?
CH3
A)
CH3CH2CCH2CH3
OH
CH3
B)
CH3CH2CHCHCH3
OH
CH3
C)
CH3CH2CHCH2CH2OH
CH3
D)
HOCH2CHCH2CH2CH3
CH2OH
E)
CH3CH2CHCH2CH3
Ans: A
Topic: Alkene Synthesis; Alcohol Dehydration
67. Which alcohol would be most easily dehydrated?
OH
OH
I
OH
II
III
OH
OH
IV
A)
B)
C)
D)
E)
V
I
II
III
IV
V
Ans: B
229
Chapter 7
Topic: Alkene Structure; Stereochemistry
68. What is the simplest alkene, i.e., the one with the smallest molecular weight, which can
exhibit optical activity?
A) 3-methyl-1-pentene
B) 3-methyl-2-pentene
C) 4-methyl-1-pentene
D) 3-methyl-1-butene
E) 4-methyl-2-hexene
Ans: A
Topic: Alkene Structure; Stereochemistry
69. Determine the total number of stereoisomers which correspond to this general structure:
Cl
CH3CHCHCH=CHCH3
CH3
A)
B)
C)
D)
E)
4
6
8
10
12
Ans: C
Topic: Alkene Structure; Stereochemistry
70.
A)
B)
C)
D)
E)
Which can exist as cis-trans isomers?
1-Pentene
3-Hexene
Cyclopentene
2-Methyl-2-butene
3-Ethyl-2-pentene
Ans: B
Topic: Alkene Structure; Stereochemistry
71.
A)
B)
C)
D)
E)
For which of the following is cis-trans isomerism impossible?
2-Hexene
3-Methyl-2-pentene
3-Hexene
2-Methyl-2-butene
2-Pentene
Ans: D
230
Chapter 7
Topic: Alkene Structure; Stereochemistry
72.
A)
B)
C)
D)
E)
Which of the following compounds can exhibit cis-trans isomerism?
1-Pentene
2-Pentene
2-Methyl-2-pentene
3-Methyl-1-pentene
1-Hexene
Ans: B
Topic: Alkene Structure; Stereochemistry
73. Which of the following compounds can exhibit cis-trans isomerism?
I. 1-Pentene
II. 2-Pentene
III. 2-Methyl-2-pentene
IV. 1,2-Dibromoethene
A)
B)
C)
D)
E)
V. 1,1-Dichloroethene
II, III, IV
I, III, V
II, IV
II, III
None of these
Ans: C
Topic: Alkene Structure; Stereochemistry
74.
A)
B)
C)
D)
E)
What is the total number of pentene isomers, including stereoisomers?
2
3
4
5
6
Ans: E
231
Chapter 7
Topic: Nomenclature; Stereochemistry
75. Cis-trans alkenes are often distinguished from each other by examining the spatial
orientation of identical groups relative to the double bond. While this often also
corresponds to (Z) and (E) descriptions, sometimes it does not. For which of the
following alkenes does cis- not equal (Z) or trans not equal (E)?
O
Cl
A)
B)
C)
D)
E)
Br
Cl
I
Cl
I
Br
II
Br
F
III
F
IV
OH
Cl
V
I
II
III
IV
V
Ans: C
Topic: Nomenclature; Stereochemistry
76. The most specific term used to designate the relationship of cis-3-hexene to trans-3hexene is:
A) stereoisomers.
B) enantiomers.
C) diastereomers.
D) constitutional isomers.
E) conformational isomers.
Ans: C
Topic: Nomenclature; Stereochemistry
77.
A)
B)
C)
D)
E)
How many stereoisomers can be predicted for 4–ethyl-2,3-dimethyl-2-heptene?
1
2
3
4
None of the above
Ans: B
232
Chapter 7
Topic: General Information, Bonding
78.
A)
B)
C)
D)
E)
How many sigma bonds are there in CH2=CH–C≡CH?
6
7
8
9
10
Ans: B
Topic: General Information, Bonding
79.
A)
B)
C)
D)
E)
What characteristic(s) of alkynes would make it difficult to prepare cyclohexyne?
The requirement for linearity at the triple bond center
The large electron density between carbons of a triple bond
The short carbon-carbon triple bond length
The need that the carbon-carbon triple bond be internal in the chain
All of these
Ans: A
Topic: General Information, Nomenclature
80. The ambiguous name "methylcyclohexene" does not differentiate among this number of
compounds (ignoring stereoisomers):
A) 2
B) 3
C) 4
D) 5
E) 6
Ans: B
Topic: Alkene Synthesis
81. Zaitsev's rule states that:
A) In electrophilic addition of an unsymmetrical reagent to an unsymmetrical alkene, the
more positive portion of the reagent will become attached to the carbon of the double
bond bearing the greater number of hydrogen atoms.
B) An equatorial substituent in cyclohexane results in a more stable conformation than if
that substituent were axial.
C) E2 reactions occur only if the β-hydrogen and leaving group can assume an antiperiplanar arrangement.
D) When a reaction forms an alkene, and several possibilities exist, the more (or most)
stable isomer is the one which predominates.
E) The order of reactivity of alcohols in dehydration reactions is 3º > 2º > 1º.
Ans: D
233
Chapter 7
Topic: Alkene Synthesis
82. Regarding the use of potassium tert-butoxide as a base in E2 reactions, it is incorrect to
state that:
A) this base is more effective than ethoxide ion, because it (KO-t-Bu) is the more basic of
the two.
B) it tends to give the anti-Zaitsev, i.e., Hofmann, product.
C) it is more reactive in dimethyl sulfoxide than it is in tert-butyl alcohol.
D) it favors E2 reactions over competing SN2 reactions.
E) it will form, predominantly, the more stable alkene.
Ans: E
Topic: General Information
83. When an elimination reaction gives the most stable alkene as the major product, we say
that the elimination follows ________________'s rule.
Ans: Zaitsev
Topic: General Information
84. When an elimination reaction gives the less substituted alkene as the major product, we
say that the elimination follows ________________'s rule.
Ans: Hofmann
Topic: General Information
85. In a dehydration reaction, the leaving group is _______________.
Ans: a molecule of water
Topic: General Information
86. In hydrogenation reactions, both hydrogen atoms add to the pi system from the same
face of the molecule. This is an example of a(n) _________ addition.
Ans: syn
Topic: General Information
87. Syn hydrogenation of an alkyne will produce a _________ alkene.
Ans: Cis / Z
Topic: General Information
88. Dehydration of alcohols requires a __________________ catalyst.
Ans: strong acid
234
Chapter 7
Topic: General Information
89. Carbocations have three options available for further reaction. These options include
reaction with a nucleophile, loss of a beta proton to give an alkene, and
________________.
Ans: rearrangement
Topic: General Information
90. Alkynes can be produced from either __________________ or _________________
dihalides.
Ans: vicinal/geminal
Topic: General Information
91. Conversion of alkynes to E-alkenes (trans-alkenes) can be accomplished via a
__________________ reaction.
Ans: dissolving metal reduction
Topic: Nomenclature, Bonding
92. The number of (s-sp2) bonds in a molecule of 2,3-dimethyl-2,4-heptadiene is _____.
Ans: two
Topic: Alkene Hydrogenation
93. How will you prepare cis-1,2-dimethylcyclohexane from 1,2-dimethylcyclohexene?
Ans: Catalytic hydrogenation with H2, Ni (or Pt/Pd)
H2, Ni
Topic: Alkyne Reduction
94. How will you prepare trans-2-heptene from 2-heptyne?
Ans: Dissolving metal reduction with metallic Na in NH3(l).
Na, NH3 (l)
235
Chapter 7
SHORT EXPLANATION / STRUCTURE DRAWING QUESTIONS
Topic: Nomenclature, Stereochemistry
95. Draw the structures of all stereoisomers of 2-chloro-4-methyl-3-hexene, clearly showing
all stereochemical details, using appropriate 3-D representations (dash-wedge, etc.) as
relevant.
2-chloro-4-methyl-3-hexene
Ans:
Cl
H
H
(2R, 3E)
Cl
Cl
(2S, 3E)
H
H
(2R, 3Z)
Cl
(2S, 3Z)
Topic: Alkene Nomenclature, Relative Stability
96. The ambiguous name dimethylcyclopentene does not clearly distinguish between
several structures.
a) Draw the structures of all constitutional isomers corresponding to this name.
b) Indicate which of these is likely to be the most stable, i.e, have the smallest heat of
combustion, justifying your rationale briefly.
c) Which of these structures represents an achiral molecule?
Ans: a)
dimethylcyclopentene
I
II
III
IV
V
VI
b) Structure I is likely to be the most stable, because it is a tetra-substituted
alkene: II, IV and V are tri-substituted; III and VI are di-substituted.
c) Structure I is achiral
236
Chapter 7
Topic: Alkyne Synthesis, Acid-Base reactions
97. Complete the following reaction sequence, providing a brief rationale for your answer.
O
i) PCl5
?
ii) 3 NaNH2,
mineral oil, heat
iii) H3O+
Ans:
O
Cl
: Na
H3O+
3 NaNH2,
mineral oil, heat
(-2HCl)
II
III
I
Reaction with PCl5 would afford a geminal dihalide I, which would undergo E2
elimination when heated, losing 2 moles of HCl to afford the alkyne III.
However, since III is a terminal alkyne, it has a hydrogen atom that is labile in
presence of NaNH2; hence, an extra mole equivalent of the base is required to
ensure complete reaction and convert all of the initially formed III into the sodium
alkynide intermediate, II; this, upon acidification, affords the final alkyne product
III.
PCl5
Cl
Topic: Alkyne Synthesis, Alkyne Hydrogenation
98. Complete the following reaction, giving structural details of all intermediates as well as
the final product.
?
:
Ans:
1) NaNH2, NH3 (l)
2) CH3CH2CH2Br
3) H2, Lindlar catalyst
NaNH2
NH3 (l)
Na
CH3CH2CH2Br
H2
Lindlar catalyst
237
Chapter 7
Topic: Carbocation Stability, Rearrangement
99. What “new”, more stable, carbocation(s) may be formed when the following
carbocation undergoes spontaneous rearrangement?
Ans:
Topic: Nomenclature, Multistep Synthesis
100. Propose a two-step synthetic strategy for the synthesis of 2-methylhexane from 5methyl-2-hexanol.
OH
Ans:
H2
H3O+
+
Ni
heat
Step 1:
Acid-catalyzed dehydration of 5-methyl-2-hexanol to yield an isomeric mixture of
5-methyl-1-hexene and 5-methyl-2-hexene
Step 2:
Catalytic hydrogenation of the double bond with H2, Ni to yield the desired 2methylhexane
Topic: Nomenclature, Multistep Synthesis
101. Propose a reasonable synthetic strategy for the synthesis of trans-6-methyl-3-heptene
from 4-methyl-1-pentyne.
+
Ans:
Na :
NaNH2
CH3CH2Br
NH3 (l)
Li, NH3(l)
Step 1:
Add NaNH2/NH3(l) to selectively deprotonate the alkyne hydrogen.
Step 2:
Add ethyl iodide to give 6-methyl-3-pentyne by an SN2 process.
Step 3:
Add Na(s)/NH3(l) to give the desired trans-alkene by dissolving metal reduction.
238
Chapter 7
Topic: Carbocation Rearrangement, Alkene Synthesis, Dehydration Mechanism
102. Provide a mechanistic explanation for the formation of the observed products in the
following reaction.
H3O+
OH
heat
Ans:
H
OH2+
:O H
rearrangement
:
OH2+
:O H
H
H
2)
:
OH
:
: :
1)
[-H2O]
H
:
3)
:O H
:O H
H
H
239
Chapter 7
Topic: Nomenclature, Index of Hydrogen Deficiency, IR Spectroscopy, Catalytic Hydrogenation
103. The IR spectrum of an unknown substance Q, C9H16, is found to have distinct peaks at
3310 cm-1 and 2140 cm-1. Treatment of Q with excess H2 in presence of Raney Ni
affords nearly quantitative yields of 2,3-dimethylheptane. Propose a reasonable
structure for Q, based on the above information, briefly explaining your rationale. Also
give its IUPAC name.
Ans:
excess H2
Ni
IR: 2140, 3310 cm-1
2,3-dimethylheptane
Substance Q:
5,6-dimethyl-1-heptyne
The Index of Hydrogen deficiency of Q is 2 (compared with C9H20). The IR data
suggests that Q is a terminal alkyne; thus, there must be no rings in its structure.
The catalytic hydrogenation data provides further details of the carbon skeleton.
Due to the specific substitution pattern in this carbon skeleton, there is only one
possible position for a terminal triple bond. Thus, the structure given above is
consistent with all of the given information.
IUPAC Name: 5,6-dimethyl-1-heptyne
240
Chapter 8
MULTIPLE CHOICE QUESTIONS
Topic: Structure Elucidation
1. An alkene adds hydrogen in the presence of a catalyst to give 3,4-dimethylhexane.
Ozonolysis of the alkene followed by treatment with zinc and acetic acid gives a single
organic product. The structure of the alkene is:
CH3
A)
CH3CH=C-CHCH2CH3
(cis or trans)
CH3
CH3
B)
CH3CH2C=CCH3
(cis or trans)
CH2CH3
C)
CH3
CH2=CCH2CHCH2CH3
CH3
CH2
D)
CH3CH2CCHCH2CH3
CH3
CH3
E)
CH3CH2CHCHCH=CH2
CH3
Ans: B
241
Chapter 8
Topic: Structure Elucidation
2. Ozonolysis of compound Z yields the products shown below. What is the structure of Z?
O
Z
1) O3
2) Zn, HOAc
O
2HCH + CH3CCH2CH
O
I
II
III
O
H
IV
A)
B)
C)
D)
E)
O
V
I
II
III
IV
V
Ans: B
242
Chapter 8
Topic: Structure Elucidation
3. Compound C has the molecular formula C7H12. On catalytic hydrogenation, 1 mol of C
absorbs 1 mol of hydrogen and yields a compound with the molecular formula C7H14.
On ozonolysis and subsequent treatment with zinc and acetic acid, C yields only:
O
O
The structure of C is:
I
II
IV
A)
B)
C)
D)
E)
III
V
I
II
III
IV
V
Ans: E
Topic: Structure Elucidation
4. Compound X has the molecular formula C6H10. X decolorizes bromine in carbon
tetrachloride. X also shows IR absorption at about 3300 cm-1. When treated with
excess hydrogen and a nickel catalyst, X yields 2-methylpentane. The most likely
structure for X is:
A)
B)
C)
D)
E)
Ans: D
243
Chapter 8
Topic: Structure Elucidation
5. An unknown compound, A, has the molecular formula C7H12. On oxidation with hot
aqueous potassium permanganate, A yields CH3CH2COOH and (CH3)2CHCOOH.
Which of the following structures best represents A?
A)
B)
C)
D)
E)
Ans: C
244
Chapter 8
Topic: Structure Elucidation
6. Determine a possible structure for an alkene, X, formula C9H14, on the basis of the
following information: X adds one mole of hydrogen on catalytic hydrogenation. On
treatment with hot basic KMnO4 followed by acidification, X yields the following
dicarboxylic acid.
O
HO
OH
O
A possible structure for X might be:
I
II
IV
A)
B)
C)
D)
E)
III
V
I
II
III
IV
V
Ans: C
Topic: Structure Elucidation
7. An optically active compound, Y, with the molecular formula C7H12 gives a positive test
with cold dilute KMnO4 and shows IR absorption at about 3300 cm-1. On catalytic
hydrogenation, Y yields Z(C7H16) and Z is also optically active. Which is a possible
structure for Y?
A) CH3CH2CH2CH2CH2C≡CH
B) (CH3)2CHCH2CH2C≡CH
C) CH3CH2CH(CH3)CH2C≡CH
D) CH3CH2CH(CH3)C≡CCH3
E) CH2=CHCH(CH3)CH2CH=CH2
Ans: C
245
Chapter 8
Topic: Structure Elucidation
8. One mole of an optically active compound, X, with the molecular formula C6H8 reacts
with three moles of hydrogen in the presence of a catalyst to yield an optically inactive
product that cannot be resolved. X also exhibits IR absorption at approximately 3300
cm-1. Which is a possible structure for X?
A) (E)-4-hexen-1-yne
B) (Z)-4-hexen-1-yne
C) (E)-2-hexen-4-yne
D) 2-methyl-1-penten-3-yne
E) 3-methyl-1-penten-4-yne
Ans: E
Topic: Structure Elucidation
9. An alkene with the molecular formula C10H18 is treated with ozone and then with zinc
and acetic acid. The product isolated from these reactions is:
O
O
What is the structure of the alkene?
I
II
III
IV
A)
B)
C)
D)
E)
V
I
II
III
IV
V
Ans: C
246
Chapter 8
Topic: Structure Elucidation
10. An optically active compound, A, with the molecular formula C7H12 reacts with cold
dilute KMnO4 and gives IR absorption at about 3300 cm-1. On catalytic hydrogenation,
A is converted to B (C7H16) and B is also optically active. Which is a possible structure
for A?
I
II
III
IV
A)
B)
C)
D)
E)
V
I
II
III
IV
V
Ans: E
Topic: Structure Elucidation
11. Which substance would undergo the following reaction?
?
A)
B)
C)
D)
E)
i. O3
ii. Zn, HOAc
H
O
+
4-Hexen-1-yne
3-methyl-1-hexene
(E)-2-hexene
(Z)-2-hexene
4-methyl-1-hexene
Ans: E
r
247
H
O
Chapter 8
Topic: Structure Elucidation
12. Which alkene would yield only CH3CH2COOH on oxidation with hot alkaline
potassium permanganate (followed by acid work-up)?
A) (E)-2-hexene
B) (Z)-2-hexene
C) 2-methyl-2-pentene
D) (E)-3-hexene
E) (E)-4-methyl-2-pentene
Ans: D
Topic: Structure Elucidation
13. An unknown compound, B, has the molecular formula C7H12. On catalytic
hydrogenation 1 mol of B absorbs 2 mol of hydrogen and yields 2-methylhexane. B has
significant IR absorption band at about 3300 and 2200 cm-1. Which compound best
represents B?
A) 3-methyl-1-hexyne
B) 5-methyl-2-hexyne
C) 5-methyl-1,3-hexadiene
D) 5-methyl-1-hexyne
E) 2-methyl-1,5-hexadiene
Ans: D
Topic: Structure Elucidation
14. What compound would yield an equimolar mixture of CH3CH2CH2CHO and CH3CHO
upon treatment with O3, followed by Zn/HOAc?
A) 1-Hexene
B) cis-2-Hexene
C) trans-2-Hexene
D) More than one of these
E) None of these
Ans: D
Topic: Structure Elucidation
15. An alkene with the molecular formula C8H16 undergoes ozonolysis to yield a mixture of
(CH3)2C=O and (CH3)3CCHO. The alkene is:
A) 2,2-Dimethyl-2-hexene
B) 2,3-Dimethyl-2-hexene
C) 2,4-Dimethyl-2-hexene
D) 2,4,4-Trimethyl-2-pentene
E) More than one of the above is a possible answer.
Ans: D
248
Chapter 8
Topic: Structure Elucidation
16. Which of the following would decolorize bromine in carbon tetrachloride and yield a
ketone upon reaction with sulfuric acid in the presence of catalytic amounts of HgSO4?
A) CH3CH=CHCH3
B) CH3CH=CH2
C) CH3CH2C≡CH
D) CH3CH2CH2CH3
E) All of these would give positive results in each test.
Ans: C
Topic: Structure Elucidation
17. Which of these compounds will react with cold concd. H2SO4, as well as Br2 in CCl4 ?
A)
B)
C)
D)
E)
CH3CH2CH=CHCH3
CH3CH2CH2CH=CH2
CH3CH2C≡CCH3
(CH3)2CHC≡CH
All of these
Ans: E
Topic: Structure Elucidation
18. Which of the following reagents might serve as the basis for a simple chemical test that
would distinguish between pure 1-pentene and pure pentane?
A) Bromine in carbon tetrachloride
B) Dilute aqueous potassium permanganate
C) Concentrated sulfuric acid
D) All of the above
E) Answers A) and B) only
Ans: D
Topic: Structure Elucidation
19.
A)
B)
C)
D)
E)
Which reagent or test could be used to distinguish between 3-pentyne and 1-pentyne?
Br2/CCl4
IR examination
Concd. H2SO4
KMnO4,OHNone of these
Ans: B
249
Chapter 8
Topic: Structure Elucidation
20. Which reagent or test could be used to distinguish between 2-methyl-2-pentene and 2methylpentane?
A) Br2/CCl4
B) KMnO4, OH−
C) Concd. H2SO4
D) Two of the above
E) All of the above
Ans: E
Topic: Structure Elucidation
21.
A)
B)
C)
D)
E)
Which of the following could be used to distinguish between 1-octyne and 3-octyne?
Treatment with 2 mol of HX
Addition of water
Reaction with KMnO4
Decolorization of bromine in CCl4
IR examination
Ans: E
Topic: Structure Elucidation
22. A reagent or test that could be used to distinguish between 1-pentene and 1-pentyne
would be:
A) Bromine in carbon tetrachloride
B) Dilute aqueous potassium permanganate
C) CrO3 in H2SO4
D) H2SO4
E) IR examination
Ans: E
Topic: Structure Elucidation
23. Which of the following could be used as the basis for a simple test that would
distinguish between 1-pentyne and pentane?
A) IR examination
B) Br2/CCl4
C) KMnO4/H2O
D) Two of these
E) All of these
Ans: E
250
Chapter 8
Topic: Reaction Products
24. Treating 1-methylcyclohexene with H3O+ would yield primarily which of these?
HO
HO
HO
I
II
III
HO
OH
IV
A)
B)
C)
D)
E)
V
I and V
II
III and V
IV
I, III and V
Ans: D
Topic: Reaction Products
25. What would be the major product of the following reaction?
HCl
Cl
Cl
Cl
Cl
I
II
III
Cl
Cl
IV
A)
B)
C)
D)
E)
V
I
II
III
IV
V
Ans: B
251
Chapter 8
Topic: Reaction Products
26. Treating 1-methylcyclohexene with HCl would yield primarily which of these?
H3C
H3C
Cl
Cl
I
H3C
Cl
II
H3C
Cl
III
Cl
IV
A)
B)
C)
D)
E)
V
I
II
III
IV
V
Ans: C
Topic: Reaction Products
27. Which product would you expect from the following reaction?
H3O+
H2O
OH
OH
II
I
OH
III
OH
IV
A)
B)
C)
D)
E)
V
I
II
III
IV
V
Ans: D
252
Chapter 8
Topic: Reaction Products
28. The reaction of Br2/CCl4 to cyclohexene would produce the compound(s) represented
by structure(s):
H
H
H
Br
H
Br
A)
B)
C)
D)
E)
Br
Br
I
I alone
II alone
II and III
III alone
I , II and II
Ans: C
Br
H
Br
H
II
III
Topic: Reaction Products
29. What product would you expect from addition of deuterium chloride to 2-cyclohexyl-4methyl-2-pentene?
D
Cl
D
Cl
D H
Cl
I
III
II
Cl
D
D
Cl
IV
A)
B)
C)
D)
E)
V
I
II
III
IV
V
Ans: B
253
Chapter 8
Topic: Reaction Products
30.
A)
B)
C)
D)
E)
What is the chief product of the reaction of IBr with 2-methyl-2-pentene?
2-bromo-3-iodo-2-methylpentane
3-bromo-2-iodo-2-methylpentane
1-bromo-2-iodo-2-methylpentane
2-bromo-1-iodo-2-methylpentane
All of the above
Ans: A
Topic: Reaction Products
31.
A)
B)
C)
D)
E)
What is the chief product of the acid-catalyzed hydration of 2,5-dimethyl-2-hexene?
2,5-dimethyl-1-hexanol
2,5-dimethyl-2-hexanol
2,5-dimethyl-3-hexanol
2,5-dimethyl-2,3-hexanediol
2,5-dimethyl-3,4-hexanediol
Ans: B
Topic: Reaction Products
32. What product would result from the following reaction?
KMnO4, H2O
cold, dilute
?
CO2H
O
K
I
A)
B)
C)
D)
E)
OH OH
MnO4
II
III
OH OH
OH OH
IV
V
I
II
III
IV
V
Ans: C
254
Chapter 8
Topic: Reaction Products
33.
A)
B)
C)
D)
E)
Acid-catalyzed hydration of 2-methyl-1-butene would yield which of the following?
(CH3)2C(OH)CH2CH3
CH2OHCH(CH3)CH2CH3
(CH3)2CHCHOHCH3
(CH3)2CHCH2CH2OH
CH3CH2CH(CH3)CH2OH
Ans: A
Topic: Reaction Products
34. Which of these is not formed when cyclopentene reacts with an aqueous solution of
bromine?
Br
OH
OH
Br
I
A)
B)
C)
D)
E)
Br
Br
Br
II
Br
III
OH
OH
IV
V
I
II
III
IV
V
Ans: E
Topic: Reaction Products
35. What would be the major product of the following reaction?
Br2, H2O
?
Br
OH
I
A)
B)
C)
D)
E)
Br
OH
Br
II
OH
Br
Br
Br
III
IV
I
II
III
IV
V
Ans: A
255
OH
V
Chapter 8
Topic: Reaction Products
36. Addition of hydrogen chloride to the following molecule would produce:
HCl
?
Cl
Cl
Cl
Cl
Cl
Cl
A)
B)
C)
D)
E)
Cl
Cl
Cl
Cl
Cl
I
II
III
IV
V
I and II
II and III
I and IV
V
All of the above are equally likely to be formed
Ans: A
256
Chapter 8
Topic: Reaction Products
37. In addition to the expected major product, which compound may be reasonably
anticipated as a by-product in the hydroxylation of R-2-methylcyclohexene with cold
alkaline permanganate?
H3C
i. KMnO4, H2O
H
cold, dilute
ii. H3O
H3C
H
H3C
OH
?
+
H
OH
H3C
O
H
OH
I
O
II
H
III
CH3
O
CH3
H
O
IV
A)
B)
C)
D)
E)
H
O
OH
O
OH
I
II
III
IV
V
Ans: E
257
V
Chapter 8
Topic: Reaction Products
38. Select the structure of the major product formed in the following reaction.
2 HCl
?
Cl
Cl
Cl
Cl
I
III
II
Cl
Cl
Cl
Cl
IV
A)
B)
C)
D)
E)
V
I
II
III
IV
V
Ans: D
Topic: Reaction Products
39. Which of these compounds is not formed when gaseous ethene is bubbled into an
aqueous solution of bromine, sodium chloride and sodium nitrate?
A) BrCH2CH2Br
B) BrCH2CH2Cl
C) BrCH2CH2OH
D) ClCH2CH2OH
E) BrCH2CH2ONO2
Ans: D
258
Chapter 8
Topic: Reaction Products
40. Select the structure of the major product formed in the following reaction.
2 Cl2
?
Cl
Cl
Cl
Cl
Cl
Cl
Cl
I
II
Cl
III
Cl
Cl
Cl
IV
A)
B)
C)
D)
E)
V
I
II
III
IV
V
Ans: C
259
Chapter 8
Topic: Reaction Products
41. Addition of excess HBr to 2-methyl-3-heptyne would produce:
2-methyl-3,3-dichloroheptane
2-methyl-2,3-dichloroheptane
I
II
2-methyl-4,4-dichloroheptane
2-methyl-3,4-dichloroheptane
III
IV
2-methyl-3-chloroheptene
V
A)
B)
C)
D)
E)
I and II
I and III
II and IV
V
All of the above
Ans: B
Topic: Reaction Products
42. What would be the major product of the following reaction?
Br2, CCl4
?
CH3
CH3
H
Br
Br
H
H
Br
Br
H
C3H7
I
A)
B)
C)
D)
E)
CH3
CH3
H
Br
Br
H
Br
H
H
Br
C3H7
C3H7
C3H7
II
III
IV
Equal amounts of I and II
Equal amounts of II and III
Equal amounts of III and IV
I and II as major products, III and IV as minor products
All of the above in equal amounts
Ans: A
260
Chapter 8
Topic: Reaction Products
43. What would be the major product of the following reaction?
Br2, CCl4
C2H5
C2H5
Br
Br
CH3 H3C
Br
H
Br
Br
H
Br
H
I
A)
B)
C)
D)
E)
C2H5
C2H5
H3C
C3H7
?
Br
CH3
H
Br
C3H7
C3H7
C3H7
II
III
IV
Equal amounts of I and II
Equal amounts of II and III
Equal amounts of III and IV
I and II as major products, III and IV as minor products
All of the above in equal amounts
Ans: A
Topic: Reaction Products
44. Hydroxylation of cis-2-pentene with cold alkaline KMnO4 yields
CH3
CH3
H
OH
HO
H
H
H
OH
HO
H
HO
C2H5
A)
B)
C)
D)
E)
CH3
C2H5
CH3
OH
H
C2H5
HO
H
H
OH
C2H5
I
II
III
IV
Equal amounts of I and II
Equal amounts of II and III
Equal amounts of III and IV
I and II as major products, III and IV as minor products
All of the above in equal amounts
Ans: B
261
Chapter 8
Topic: Reaction Products
45. The reaction of BrCl (bromine monochloride) with 1-methylcyclopentene will produce
as the predominant product:
CH3
CH3
Br
Cl
+
enantiomer
I
A)
B)
C)
D)
E)
Br
Cl
Br
Cl
+
enantiomer
CH3
CH2Cl
Cl
Br
Br
+
enantiomer
II
CH3
III
+
enantiomer
IV
+
enantiomer
V
I
II
III
IV
V
Ans: D
Topic: Reaction Products
46. Reaction of trans-2-hexene with a solution of Br2 in CCl4 produces:
H
Br
H
Br
H
Br
Br
H
Br
H
I
H
Br
II
III
Br
H
Br
H
IV
A)
B)
C)
D)
E)
H
Br
H
Br
V
I and II
II and V
III and IV
IV and V
V
Ans: C
262
Chapter 8
Topic: Reaction Products
47. When either cis- or trans-2-butene is treated with hydrogen chloride in ethanol, the
product mixture that results includes:
A) CH3CH2CH2CH2Cl
B) CH3CH2CH2CH2OCH2CH3
C) CH3CH2CH(CH3)OCH2CH3
D) (CH3)3CCl
E) (CH3)2CHCH2OCH2CH3
Ans: C
Topic: Reaction Products
48. What is the major product of the following reaction?
H2SO4, H2O
?
HgSO4
OH
OH
OH
OH
I
II
III
O
O
IV
A)
B)
C)
D)
E)
V
I
II
III
IV
V
Ans: D
Topic: Reaction Products
49.
A)
B)
C)
D)
E)
Addition of 2 mol of HCl to 1-butyne would yield:
CH3CH2CH2CHCl2
CH3CH2CCl2CH3
CH3CH2CHClCH2Cl
CH3CH2CH=CHCl
CH3CHClCHClCH3
Ans: B
263
Chapter 8
Topic: Reaction Products
50. Consider the addition of HCl to 3-methyl-1-butene. The major product of the reaction
would be:
A) 1-Chloro-2-methylbutane
B) 1-Chloro-3-methylbutane
C) 2-Chloro-2-methylbutane
D) 2-Chloro-3-methylbutane
E) 1-Chloropentane
Ans: C
Topic: Reaction Products
51. What is the final product of the following synthesis?
2-Butyne
HO
H
H2
Ni2B (P-2)
CH3
OH
H
CH3
I
A)
B)
C)
D)
E)
HO
HO
C4H8
CH3
H
H
CH3
i. OsO4
ii. NaHSO4
HO
H
II
CH3
H
OH
CH3
III
Final Product
H
HO
CH3
OH
H
CH3
IV
I
II
III
IV
An equimolar mixture of III and IV
Ans: B
Topic: Synthetic Strategy
52. A synthetic strategy for converting trans-2-butene into pure cis-2-butene would consist
of which of the following?
A) Reaction with H2, Ni2B (P-2)
B) Reaction with i) Br2, CCl4; ii) 2 NaNH2, NH3(l); iii) Na, NH3 (l)
C) Reaction with H3O+, heat
D) Reaction with i) Br2, CCl4; ii) 2 NaNH2, NH3(l); iii) H2, Ni2B (P-2)
E) None of these will successfully effect the desired transformation
Ans: D
264
Chapter 8
Topic: Synthetic Strategy
53. Which reagent(s) given below could be used to synthesize cis-1,2-cycloheptanediol
from cycloheptene?
A) KMnO4, OH−, 5oC
B) KMnO4, H3O+, 75oC
C) H2SO4, heat
D) All of these
E) None of these
Ans: A
Topic: Synthetic Strategy
54.
A)
B)
C)
D)
E)
Which reaction sequence would convert cis-2-butene to trans-2-butene?
Br2/CCl4; then 2 NaNH2; then H2/Ni2B(P-2)
Br2/CCl4; then 2 NaNH2; then Li/liq. NH3
H3O+, heat; then cold dilute KMnO4
HBr; then NaNH2; then H2, Pt
None of these
Ans: B
Topic: Synthetic Strategy
55. The conversion of ethylene to vinyl bromide can be accomplished by use of these
reagents in the order indicated.
A) (1) HBr; (2) NaOC2H5
B) (1) Br2; (2) NaOC2H5
C) (1) Br2; (2) H2O
D) (1) NaNH2; (2) HBr
E) (1) HBr; (2) H2SO4
Ans: B
Topic: Synthetic Strategy
56. Cyclohexene is treated with cold dilute alkaline KMnO4. Assuming syn addition, the
spatial arrangement of the two hydroxyl groups in the product would be:
A) equatorial-axial
B) axial-axial
C) equatorial-equatorial
D) coplanar
E) trans
Ans: A
265
Chapter 8
Topic: Synthetic Strategy
57.
A)
B)
C)
D)
E)
Which of the following reactions of cyclobutene would yield a meso product?
Reaction with H3O+, H2O
Reaction with Br2, CCl4
Reaction with Cl2, CCl4
Reaction with D2, Pt
Reaction with hot, alkaline KMnO4, followed by acid workup
Ans: E
Topic: Reaction Products
58. How many compounds are possible from the addition of bromine to CH2=CHCH2CH3
(counting stereoisomers separately)?
A) One
B) Two
C) Three
D) Four
E) Five
Ans: B
Topic: Reaction Products
59. Which alkene would react with cold dilute alkaline permanganate solution to form an
optically inactive and irresolvable product?
A)
B)
C)
D)
E)
Ans: C
266
Chapter 8
Topic: Reaction Products
60.
A)
B)
C)
D)
E)
Which reaction of an alkene proceeds with anti addition?
Hydroboration/oxidation
Bromination
Oxidation with cold KMnO4
Hydrogenation
Oxymercuration-demercuration
Ans: B
Topic: Reaction Products
61.
A)
B)
C)
D)
E)
A pair of enantiomers results from which of these reactions?
cyclopentene + cold, dil. KMnO4 ⎯⎯⎯⎯→
trans-2-butene + Br2 ⎯⎯⎯⎯→
1-pentene + HCl ⎯⎯⎯⎯→
cis-2-butene + D2/Pt ⎯⎯⎯⎯→
cyclobutene + OsO4, then Na2SO3 ⎯⎯⎯⎯→
Ans: C
Topic: Reaction Products
62.
A)
B)
C)
D)
E)
Which reaction would yield a racemic product?
cyclopentene + D2/Pt ⎯⎯⎯⎯→
Cyclopentene + OsO4, then Na2SO3 ⎯⎯⎯⎯→
cyclopentene + Br2/H2O ⎯⎯⎯⎯→
Cyclopentene + cold, dilute KMnO4 ⎯⎯⎯⎯→
Cyclopentene + dilute H2SO4 ⎯⎯⎯⎯→
Ans: C
Topic: Reaction Products
63. 3,3-dimethylcyclohexene is subjected to reaction with cold, dilute KMnO4, to give 3,3dimethyl-1,2-cyclohexanediol. In the most stable conformation of the product, the
hydroxyl groups would be:
A) both axial
B) both equatorial
C) axial-equatorial
D) coplanar
E) None of these
Ans: B
267
Chapter 8
Topic: Reaction Products
64. Cyclohexene reacts with bromine to yield 1,2-dibromocyclohexane. Molecules of the
product would:
A) be a racemic form and, in their most stable conformation, they would have both bromine
atoms equatorial.
B) be a racemic form and, in their most stable conformation, they would have one bromine
atom equatorial and one axial.
C) be a meso compound and, in its most stable conformation, it would have both bromine
atoms equatorial.
D) be a meso compound and, in its most stable conformation, it would have one bromine
atom equatorial and one axial.
E) be a pair of diastereomers and, in their most stable conformation, one would have the
bromines equatorial and axial, and the other would have the bromines equatorial and
equatorial.
Ans: A
Topic: Reaction Products
65. Which reaction would yield a meso compound?
Br2/CCl4
A)
cis-2-Butene
H2/Pd
B)
cis-2-Butene
i) OsO4
C) cis-2-Butene
ii) NaHSO3
dil KMnO4
D)
trans-2-Butene
5oC
E) None of these
Ans: C
Topic: Reaction Products
66.
A)
B)
C)
D)
E)
Which reaction would give a meso compound as the product?
Cyclopentene + Br2/CCl4
Cyclopentene + OsO4, then NaHSO3
Cyclopentene + RCO3H, then H3O+
Cyclopentene + Cl2, H2O
More than one of these
Ans: B
268
Chapter 8
Topic: Reaction Products
67.
A)
B)
C)
D)
E)
Which of the following reactions would yield the final product as a racemic form?
Cyclohexene + a peroxy acid, then H3O+
Cyclohexene + cold, dilute KMnO4 and OHCyclohexene + HCl
Cyclohexene + OsO4, then NaHSO3
Cyclohexene + D2/Pt
Ans: A
Topic: Reaction Products
68. (R)-3-Chloro-1-butene reacts with HCl by Markovnikov addition, and the products are
separated by gas chromatography. How many total fractions would be obtained and
how many would be optically active?
A) One optically active fraction only
B) One optically active fraction and one optically inactive
C) Two optically active fractions
D) One optically active fraction and two optically inactive
E) Two optically active fractions and one optically inactive
Ans: B
Topic: Reaction Products
69. The interaction of the π bond of an alkene with an electrophile can initially result in the
formation of a species termed a π complex. Which of these cannot combine with an
alkene to form a π complex?
A) H+
B) NH3
C) Ag+
D) Hg2+
E) BF3
Ans: B
Topic: Reaction Products
70.
A)
B)
C)
D)
E)
Markovnikov addition of HI to 2-methyl-2-butene involves:
initial attack by an iodide ion.
initial attack by an iodine atom.
isomerization of 2-iodo-2-methylbutene.
formation of a carbocation at C-2.
formation of carbocation at C-3.
Ans: D
269
Chapter 8
Topic: Reaction Products
71. Which reaction is NOT stereospecific?
trans-2-Butene
Br2/CCl4
cis-2-Pentene
I
Br2/H2O
II
1-Methylcyclohexene
H2/Pd
III
2-Methyl-2-heptene
dil KMnO4
trans-2-Hexene
5oC
V
IV
A)
B)
C)
D)
E)
I
II
III
IV
V
Ans: E
270
HBr
Chapter 8
Topic: Reaction Products
72. Which reaction is regioselective?
Cl
I
II
ICl
I
Br
Br2
Br
OH
III
KMnO4
OH
IV
A)
B)
C)
D)
E)
D2/Ni
H
D
D
H
I
II
III
IV
None of these
Ans: A
Topic: General Information
73. The thermodynamic parameters at 298 K for the following reaction are given below.
gas phase
CH2=CH2 + HCl
CH3CH2Cl
ΔHº = -64.9 kJ mol-1
ΔSº = -131 J K-1 mol-1
ΔGº = -25.8 kJ mol-1
Which of the following statements is true of the reaction?
A) Both ΔHº and ΔSº favor product formation.
B) Neither ΔHº nor ΔSº favors product formation.
C) The entropy term is unfavorable but the formation of ethyl chloride is favored.
D) The entropy term is favorable but the formation of ethyl chloride is not favored.
E) The sign of ΔGº indicates that the reaction cannot occur as written.
Ans: C
271
Chapter 8
Topic: Reaction Products
74. In general, when the addition of an unsymmetrical electrophilic reagent to an
unsymmetrical alkene forms the product predicted by Markovnikov's rule, that occurs
because:
A) the product is statistically favored.
B) steric hindrance favors its formation.
C) it is formed via the more/most stable carbocation.
D) it is the more/most stable product.
E) All of the above are reasons.
Ans: C
Topic: General Information
75. Which of these compounds belongs to the class of substances commonly known as
"halohydrins"?
A) BrCH2CH2Cl
B) ClCH2CO2H
C) ICH2CH2OH
D) FCH2CH2NH2
E) HOCH2COCl
Ans: C
Topic: General Information
76.
A)
B)
C)
D)
E)
Which alkene would you expect to be most reactive toward acid-catalyzed hydration?
1 pentene
trans-2-pentene
cis-2-pentene
2-methyl-1butene
All of these would be equally reactive.
Ans: D
Topic: General Information
77.
A)
B)
C)
D)
E)
The most resistant compound to the action of hot alkaline KMnO4 is:
Pentane
1-Pentene
2-Pentene
2-Pentyne
Cyclopentene
Ans: A
272
Chapter 8
Topic: General Information
78.
A)
B)
C)
D)
E)
2-Pentyne will not react with:
H2, Pt
Br2
NH3
H2SO4
KMnO4/H2O
Ans: C
Topic: General Information
79. Consider the ozonolysis products obtained from all the unbranched and unsymmetrical
isomers of heptene. The reaction product in each case would consist of:
A) a single aldehyde.
B) an aldehyde and a ketone.
C) two different ketones.
D) two different aldehydes.
E) a single ketone.
Ans: D
SHORT ANSWER QUESTIONS
Topic: General Information
80. The rule that correctly predicts the regiochemistry of most ionic additions to alkenes is
called ____________________.
Ans: Markovnikov's Rule
Topic: General Information
81. When a reaction that could potentially yield two or more constitutional isomers instead
produces only one as the major product, the reaction is said to be
_____________________.
Ans: regioselective
Topic: General Information
82. When a particular stereoisomer reacts in such a way that it gives a particular
stereoisomer as a product, even if more than one stereoisomer is theoretically possible,
the reaction is said to be _____________.
Ans: stereospecific
273
Chapter 8
Topic: General Information
83. A reaction in which the reactant is not necessarily chiral but still produces primarily one
stereoisomeric form of the product (or a specific subset of the possible stereoisomers) is
referred to as a _________________ reaction.
Ans: stereoselective
Topic: General Information
84. Hydroboration-oxidation is a reaction with _________ stereochemistry and
________________ regiochemistry.
Ans: syn, anti-Markovnikov
Topic: General Information
85. Even when one or more stereogenic centers are produced as the result of an addition
reaction to an alkene, the product is always formed as a racemic mixture. Why is that?
Ans: Because alkenes are planar, and the reagent can add from either face.
Topic: General Information
86. The “decolorization” of molecular bromine is often used as a functional group test to
detect the presence of ________________.
Ans: carbon-carbon multiple bonds; or, alkenes and alkynes
Topic: General Information
87. Neutral divalent carbon compounds are called ___________.
Ans: carbenes
Topic: General Information
88. Carbenes are frequently produced by α-elimination reactions. These are reactions in
which the proton being lost and the leaving group are ___________.
Ans: both attached to the same carbon
Topic: General Information
89. π bonds are quite susceptible to reaction with electron-seeking reagents, also referred to as
____________________.
Ans: electrophiles
274
Chapter 8
Topic: Reaction Sequence
90. Predict the final product(s) obtained when (2R,3R)-2-bromo-3-methylpentane is subjected to
the following sequence of reactions, briefly explaining your rationale. Provide regiochemical
and stereochemical details as relevant.
NaOC2H5/C2H5OH, heat
i)
ii)
BH3, THF
H2O2, OH−
iii)
H
Ans: Br H
E-3-methyl-2-hexene
NaOC2H5
C2H5OH
CH3
H3C H
Hydroboration/oxidation:
(E2: anti
overall
(2R,3R)
elimination)
Anti-Markovnikov,
syn addition of H2O
H
OH
HO
H
+
H3C H
H CH3
(2S,3R)
(2R,3S)
3-methyl-2-pentanol
Heating with sodium ethoxide results in E2 elimination; the anti- elimination leads to
the formation of E-3-methyl-2-hexene as the major product. Subsequent hydroboration
oxidation is regioselective (anti-Markovnikov) and stereospecific (syn), resulting in the
formation of racemic (2S,3R & 2R,3S) 3-methyl-2-pentanol. The other diastereomer
pair (2S,3S & 2R,3R) is not expected to be obtained.
Topic: Reaction Products
91. Provide a mechanistic explanation for the following observation: The same major product is
obtained when 2-ethyl-1-hexene and 3-methyl-2-heptene are allowed to react with HCl.
Ans:
HCl
Cl
HCl
most stable
carbocation
Both alkenes are unsymmetrically substituted: the major product is a consequence of
regioselective addition to the double bond. Upon reaction with HCl, electrophilic
addition of a proton results in the formation of the most stable intermediate carbocation,
which then reacts with Cl−, to give the observed product.
275
Chapter 8
Topic: Reaction Products
92. Predict the major product(s) of the following reaction, giving regiochemical and/or
stereochemical details as relevant.
Br2, H2O
?
Ans:
OH
Br
H
OH
Br
H
+
The reaction takes place with Markovnikov regioselectivity and anti stereospecificity,
giving a racemic mixture of the halohydrin as the major product.
Topic: Reaction Products
93. Draw Fischer projection formula(s) of the major product(s) of the reaction between Z-2-methyl3-hexene and cold, alkaline KMnO4.
Ans:
CH(CH )
CH(CH )
3 2
cold, dil
KMnO4
3 2
H
OH
HO
H
H
OH
+ HO
H
C2H5
C2H5
The reaction takes place with syn stereospecificity, giving a racemic mixture of the diol
product.
Topic: Reaction Products
94. Draw Fischer projection formula(s) of the major product(s) of the reaction between Z-3-methyl3-hexene and cold, alkaline KMnO4.
CH2CH3
CH2CH3
Ans:
OH
HO
CH3
cold, dil H3C
+
H
H
OH
HO
KMnO4
C2H5
C2H5
The reaction takes place with syn stereospecificity, giving a racemic mixture of the diol
product.
276
Chapter 8
Topic: Reaction Products
95. Draw Fischer projection formulas of the major product of the reaction between E-2-methyl-3hexene and aqueous Br2.
Ans:
CH(CH )
CH(CH )
CH(CH )
CH(CH )
3 2
Br2
H2O
HO
H
H
Br
3 2
H
+ Br
C2H5
3 2
OH
H
H
Br
+ HO
H
C2H5
+
3 2
Br
H
H
OH
C2H5
C2H5
The reaction takes place with anti stereospecificity. Since the alkene is symmetrically
substituted, Markovnikov rule is not applicable, and the reaction is not regioselective.
The product mixture would therefore consist of 2 pairs of enantiomers, that are
constitutional isomers of each other.
Topic: Reaction Products
96. Give a mechanistic explanation for the formation of the following product in significant yield.
What other product(s) might also be obtained? Explain clearly.
HI
Ans:
H+
+
I
fast
I−
+
I
+
I
−
I
The regioselective addition of a proton first generates a secondary carbocation, which
can undergo rearrangement to a more stable 3o carbocation by two alternate pathways.
One pathway results in ring expansion to a more stable 5-membered ring. This pathway
is thus more likely, which would then lead to the observed product. The other pathway
is less likely, since ring strain is not relieved; however, this could lead to a minor
product, as shown above.
277
Chapter 8
Topic: Structure Elucidation
97. Deduce the structure of an unknown compound A, C8H16, from the following data. Briefly, but
clearly explain your rationale. A decolorizes Br2/CCl4, and upon reaction with excess H2/Ni,
affords 1-ethyl-2-methylcyclopentane. When A is subjected to ozonolysis, the following
product is obtained.
O
O
Ans: A must be 3-ethyl-4-methylcyclopentene. This given data can be explained as shown
below:
C8H14: Index of Hydrogen Deficiency of A = 2
decolorizes Br2,CCl4: A must contain pi bonds
A
H2, Ni
A must contain some the same carbon
skeleton and one pi bond
ozonolysis
O
O
O
O
pi bond position in A deduced from the
structure of the ozonolysis product
Topic: Reaction Products
98. Predict the product(s) of the oxidation of 2,3,4-trimethyl-1,5-heptadiene with hot, alkaline
KMnO4.
Ans:
OH
hot, alkaline CO + H O +
O +
2
2
O
KMnO4
O
OH
278
Chapter 8
Topic: Reaction Products
99. Predict the structure of product obtained when cis-2-hexene is allowed to react with Zn/CH2I2.
H
Ans:
H
Zn, CH2I2
+ H
H
The Zn/CH2I2 produces a carbene which reacts with the alkene to give the
corresponding cyclopropane derivative. The reaction proceeds with retention of alkene
stereochemistry, resulting in a racemic mixture of the two cis enantiomers.
Topic: Reaction Products
100. Predict the structure of product obtained when 1-hexyne is allowed to react with aqueous
H2SO4 in the presence of catalytic amounts of HgSO4.
Ans:
HgSO4,
H2SO4, H2O
OH
O
The reaction of 1-hexyne with aqueous H2SO4, in the presence of catalytic amounts of
HgSO4, results in regioselective hydration to produce an enol, which tautomerizes to the
more stable keto form.
279
Chapter 9
MULTIPLE CHOICE QUESTIONS
Topic: Proton NMR- Chemical Shift, Splitting, Etc.
1. If all the protons of 1-fluoropentane could be discerned, which would you expect to be
at the lowest field in the 1H NMR spectrum of this compound?
CH3CH2CH2CH2CH2F
A)
B)
C)
D)
E)
V IV III II I
Protons on carbon I
Protons on carbon II
Protons on carbon III
Protons on carbon IV
Protons on carbon V
Ans: A
Topic: Proton NMR- Chemical Shift, Splitting, Etc.
2. Which proton(s) of the compound below would appear as a septet in the 1H NMR
spectrum?
I
CH3
CH3CH2CH2-O-CH
CH3
V IV III
II
I
A) The protons on carbon I
B) The proton on carbon II
C) The protons on carbon III
D) The protons on carbon IV
E) The protons on carbon V
Ans: B
Page 280
Chapter 9
Topic: Proton NMR- Chemical Shift, Splitting, Etc.
3. Which proton(s) of the compound below would appear as a doublet in the 1H NMR
spectrum?
I
CH3
CH3CH2CH2-O-CH
CH3
V IV III
II
I
A) The protons on carbon I
B) The protons on carbon II
C) The protons on carbon III
D) The protons on carbon IV
E) The protons on carbon V
Ans: A
Topic: Proton NMR- Chemical Shift, Splitting, Etc.
4. Which proton(s) of the compound below would appear as a triplet in the 1H NMR
spectrum?
I
CH3
CH3CH2CH2-O-CH
CH3
II
V IV III
I
A) The protons on carbon II
B) The protons on carbon I and V
C) The protons on carbon III and V
D) The protons on carbon III and IV
E) The protons on carbon V
Ans: C
Topic: Proton NMR- Chemical Shift, Splitting, Etc.
5. How many signals would you expect to find in the 1H NMR spectrum of
CH3OCH2CH2OCH3?
A) 1
B) 2
C) 3
D) 4
E) 5
Ans: B
281
Chapter 9
Topic: Proton NMR- Chemical Shift, Splitting, Etc.
6. Predict the splitting pattern you would observe for the proton at C3 of 2,3-dimethyl-2phenylbutane.
A)
B)
C)
D)
E)
Doublet
Singlet
Quartet
Septet
Octet
Ans: D
Topic: Proton NMR- Chemical Shift, Splitting, Etc.
7. Predict the splitting pattern you would observe for the proton at C1 of 2,3-dimethyl-2phenylbutane.
A)
B)
C)
D)
E)
Doublet
Singlet
Quartet
Septet
Octet
Ans: B
Topic: Proton NMR- Chemical Shift, Splitting, Etc.
8. Predict the splitting pattern you would observe for the proton at C4 of 2,3-dimethyl-2phenylbutane.
A)
B)
C)
D)
E)
Doublet
Singlet
Quartet
Septet
Octet
Ans: A
282
Chapter 9
Topic: Proton NMR- Chemical Shift, Splitting, Etc.
9. What feature would you expect to see in the 1H NMR spectrum of B after subjecting A
to the following reaction?
Cl2
C8H9Cl
+
HCl
heat or light
(chief product)
A
B
A)
B)
C)
D)
E)
There would be only 4 aromatic protons at low field.
The signal for the protons on the benzylic carbon would be a doublet.
The signal for the methyl protons would be a triplet.
The signal for the methyl protons would be a doublet.
The signal for the methyl protons would integrate for only 2 hydrogens.
Ans: D
Topic: Proton NMR- Chemical Shift, Splitting, Etc.
10.
A)
B)
C)
D)
E)
How many 1H NMR signals would trans-1,2-dichlorocyclopropane give?
1
2
3
4
5
Ans: B
Topic: Proton NMR- Chemical Shift, Splitting, Etc.
11. How many 1H NMR signals would the following compound give?
ClCH2CHCH3
A)
B)
C)
D)
E)
Br
1
2
3
4
5
Ans: D
283
Chapter 9
Topic: Proton NMR- Chemical Shift, Splitting, Etc.
12.
A)
B)
C)
D)
E)
How many 1H NMR signals would cis-1,2-dichlorocyclopropane give?
1
2
3
4
5
Ans: C
Topic: Proton NMR- Chemical Shift, Splitting, Etc.
13. How many 1H NMR signals would you expect from this compound?
OCH3
OCH3
A)
B)
C)
D)
E)
1
2
3
4
5
Ans: B
Topic: Proton NMR- Chemical Shift, Splitting, Etc.
14. The 1H NMR signal for which of the indicated protons occurs furthest downfield?
III
Cl
H
H
C C C
C CH3
I
H
IV
II
V
A)
B)
C)
D)
E)
I
II
III
IV
V
Ans: E
284
Chapter 9
Topic: Proton NMR- Chemical Shift, Splitting, Etc.
15. The 1H NMR spectrum of which of the compounds below, all of formula C7H12O2,
would consist of two singlets only?
O
O
O
O
O
O
I
II
III
O
O
O
O
IV
A)
B)
C)
D)
E)
V
I
II
III
IV
V
Ans: E
Topic: Proton NMR- Chemical Shift, Splitting, Etc.
16. The 1H NMR spectrum of which of the compounds below, all of formula C7H12O2,
would consist of three singlets only?
O
O
O
O
O
I
II
III
O
O
O
O
IV
A)
B)
C)
D)
E)
V
I
II
III
IV
V
Ans: B
285
O
Chapter 9
Topic: Proton NMR- Chemical Shift, Splitting, Etc.
17. The 1H NMR spectrum of which of the compounds below, all of formula C7H12O2,
would consist of a singlet, a doublet and a triplet only?
O
O
O
O
O
O
I
II
III
O
O
O
O
IV
A)
B)
C)
D)
E)
V
I
II
III
IV
V
Ans: D
Topic: Proton NMR- Chemical Shift, Splitting, Etc.
18. The 1H NMR spectrum of which of these compounds would consist of a triplet, singlet
and quartet only?
A) 2-chloro-4-methylpentane
B) 3-chloro-2-methylpentane
C) 3-chloropentane
D) 1-chloro-2,2-dimethylbutane
E) 3-chloro-3-methylpentane
Ans: E
Topic: Proton NMR- Chemical Shift, Splitting, Etc.
19. In NMR terminology, protons Ha and Hb are said to be:
H
Cl
Hb
A)
B)
C)
D)
E)
Ha
Identical
Enantiotopic
Diastereotopic
Homotopic
Mesotopic
Ans: C
286
Chapter 9
Topic: Proton NMR- Chemical Shift, Splitting, Etc.
20. Consider the 1H NMR spectrum of very pure 1-propanol. Assuming the maximum
multiplicity of signals and non-superposition of peaks, what is the expected signal
splitting pattern for each signal, in the order (a, b, c, d) ?
CH3-CH2-CH2-OH
b
c
d
a
A)
B)
C)
D)
E)
3, 6, 4, 1
3, 6, 4, 3
3, 12, 3, 1
3, 12, 3, 3
3, 12, 6, 3
Ans: C
Topic: Proton NMR- Chemical Shift, Splitting, Etc.
21. In the structure shown, Hb and Hc are classified as:
Ha
Hb
Hc
A)
B)
C)
D)
E)
homotopic protons.
vicinal protons.
enantiotopic protons.
diastereotopic protons.
isomeric protons.
Ans: D
Topic: Proton NMR- Chemical Shift, Splitting, Etc.
22. In the structure shown, Hb and Hc are classified as:
Hb
A)
B)
C)
D)
E)
Ha
homotopic protons.
vicinal protons.
enantiotopic protons.
diastereotopic protons.
isomeric protons.
Ans: C
287
Chapter 9
Topic: Proton NMR- Chemical Shift, Splitting, Etc.
23. In the structure shown, Hb and Hc are classified as:
Ha
Hb
A)
B)
C)
D)
E)
homotopic protons.
vicinal protons.
enantiotopic protons.
diastereotopic protons.
isomeric protons.
Ans: D
Topic: Proton NMR- Chemical Shift, Splitting, Etc.
24. In the structure shown, Hb and Hc are classified as:
HO H
Ha H
b
A)
B)
C)
D)
E)
homotopic protons.
vicinal protons.
enantiotopic protons.
diastereotopic protons.
isomeric protons.
Ans: D
Topic: Proton NMR- Chemical Shift, Splitting, Etc.
25. Distinction between the methine protons in the compounds I and II below should be
possible in the 1H NMR spectra if:
H
C2H5
H3C
A)
B)
C)
D)
E)
Cl
C2H5
H3C
Cl
H
I
II
a very high field instrument is used.
the spectra are determined in a chiral solvent.
a long scan time is used for each compound.
a high amplitude setting is employed.
Distinction between the enantiomers is impossible.
Ans: B
288
Chapter 9
Topic: Proton NMR- Unknown Identification.
26. A compound with the molecular formula C4H10O gives a 1H NMR spectrum consisting
only of a quartet centered at δ 3.5 and a triplet at δ 1.1. The most likely structure for the
compound is:
CH3
A)
CH3COH
B)
C)
D)
E)
CH3
CH3
CH3OCHOH
CH3CH2CH2CH2OH
CH3CH2OCH2CH3
CH3CHCH2OH
CH3
Ans: D
Topic: Proton NMR- Unknown Identification.
27. Determine the most likely structure of a compound, with the molecular formula C9H12,
which gave a 1H NMR spectrum consisting of:
a doublet at δ 1.25
a septet at δ 2.90 and
a multiplet at δ 7.25
I
II
III
IV
V
]
A)
B)
C)
D)
E)
I
II
III
IV
V
Ans: D
289
Chapter 9
Topic: Proton NMR- Unknown Identification.
28. A compound with the molecular formula C10H13Cl gave the following 1H NMR
spectrum:
singlet, δ 1.6
singlet, δ 3.1
multiplet, δ 7.2 (5H)
The most likely structure for the compound is:
Cl
Cl
I
Cl
II
III
Cl
Cl
IV
A)
B)
C)
D)
E)
V
I
II
III
IV
V
Ans: B
Topic: Proton NMR- Unknown Identification.
29. A compound with the molecular formula C8H9BrO gave the following 1H NMR
spectrum:
triplet, δ 1.4
quartet, δ 3.9
multiplet, δ 7.0 (4H)
There was no evidence of an -OH band in the IR spectrum. A possible structure for the
compound is:
O
O
Br
O
Br
Br
I
II
III
O
Br
Br
IV
A)
B)
C)
D)
E)
O
V
I
II
III
IV
V
Ans: C
290
Chapter 9
Topic: Proton NMR- Unknown Identification
30. A compound with the molecular formula C6H15N gave the following 1H NMR spectrum:
triplet, δ 0.90
quartet, δ 2.4
There were no other signals. The most likely structure for the compound is:
A) CH3NCH2CH3
B)
CH2CH2CH3
CH3NCH2CH2CH2CH3
CH3
C) CH3CH2CH2CH2CH2CH2NH2
D) CH3CH2NCH2CH3
CH2CH3
E) CH3CH2CH2NCH2CH2CH3
H
Ans: D
Topic: Proton NMR- Unknown Identification
31. A compound with the molecular formula C8H9ClO gave the following 1H NMR
spectrum:
triplet, δ 3.7
triplet, δ 4.2
multiplet, δ 7.1
There was no evidence of an -OH band in the IR spectrum. The most likely structure
for the compound is:
Cl
O
O
O
Cl
Cl
I
II
III
O
Cl
Cl
IV
A)
B)
C)
D)
E)
O
V
I
II
III
IV
V
Ans: A
291
Chapter 9
Topic: Proton NMR- Unknown Identification
32. A compound with the molecular formula C3H6Cl2 gave a 1H NMR spectrum consisting
only of a triplet centered at δ 3.7 and a quintet centered at δ 2.2. The most likely
structure for the compound is:
A) CH3CH2CHCl2
B) CH3CHClCH2Cl
C) ClCH2CHClCH3
D) ClCH2CH2CH2Cl
E) CH3CCl2CH3
Ans: D
Topic: Proton NMR- Unknown Identification
33. A compound with the molecular formula C10H14 gave the following 1H NMR spectrum:
doublet, δ 1.2
singlet, δ 2.3
septet, δ 2.8
multiplet, δ 7.1
A possible structure for the compound is:
I
II
IV
A)
B)
C)
D)
E)
III
V
I
II
III
IV
V
Ans: B
292
Chapter 9
Topic: Proton NMR- Unknown Identification
34. A compound C5H10O gave the following spectral data:
1
H NMR spectrum
IR spectrum,
doublet, δ 1.10
strong peak
singlet, δ 2.10
near 1720 cm-1
septet, δ 2.50
Which is a reasonable structure for the compound?
O
O
II
I
O
IV
A)
B)
C)
D)
E)
O
III
O
V
I
II
III
IV
V
Ans: E
Topic: Proton NMR- Unknown Identification
35. A compound C4H9Br gave the following 1H NMR spectrum:
multiplet, δ 4.1 (1H); multiplet, δ 1.8; doublet, δ 1.7;
triplet, δ 1.0 (3H)
Which is a reasonable structure for the compound?
A) CH3CH2CHBrCH3
B) CH3CH2CH2CH2Br
C) (CH3)2CHCH2Br
D) (CH3)3CBr
Ans: A
293
Chapter 9
Topic: Proton NMR- Unknown Identification
36. Determine the likely structure for a compound A (C6H10O), which is found to decolorize
bromine in carbon tetrachloride. Its spectral data is as follows:
1
H NMR
IR
-1
triplet, δ 1.0
singlet, δ2.4
2200 cm (sharp)
singlet, δ 1.4
singlet, δ3.4
3300 cm-1 (sharp)
quartet, δ 1.6
3500 cm-1 (broad)
OH
OH
I
II
OH
IV
A)
B)
C)
D)
E)
OH
III
OH
V
I
II
III
IV
V
Ans: C
294
Chapter 9
Topic: C-13 NMR- Symmetry, Chemical shift.
37. The broadband proton-decoupled 13C NMR spectrum of a hexyl chloride exhibits five
signals. Which of these structures could be the correct one for the compound?
CH3
A)
CH3CCH2CH2Cl
CH3
Cl
B)
CH3CH2CCH2CH3
CH3
CH3 CH3
C)
CH3C
CHCH3
Cl
CH3
D)
CH3CHCHCH2CH3
Cl
CH3
E)
CH3C
CHCH3
CH3 Cl
Ans: D
Topic: C-13 NMR- Symmetry, Chemical shift
38. How many signals will be recorded in the broadband proton-decoupled 13C spectrum of
4-chloro-1-ethylbenzene?
Cl
A)
B)
C)
D)
E)
2
3
4
6
7
Ans: D
295
Chapter 9
Topic: C-13 NMR- Symmetry, Chemical shift
39. How will the methyl carbon appear in the proton off-resonance decoupled 13C spectrum
of toluene?
A)
B)
C)
D)
E)
Singlet
Doublet
Triplet
Quartet
Quintet
Ans: D
Topic: C-13 NMR- Symmetry, Chemical shift
40. How many 13C signals would 1,4-dimethylbenzene give?
A)
B)
C)
D)
E)
1
2
3
4
5
Ans: C
Topic: C-13 NMR- Symmetry, Chemical shift
41. How many 13C signals would 1,3-dichlorobenzene give?
Cl
Cl
A)
B)
C)
D)
E)
1
2
3
4
5
Ans: D
296
Chapter 9
Topic: C-13 NMR- Symmetry, Chemical shift
42. How many 13C signals would 1,2-dimethylbenzene give?
A)
B)
C)
D)
E)
1
2
3
4
5
Ans: D
Topic: C-13 NMR- Symmetry, Chemical shift
43. How many 13C signals would you expect from C6H5OCH3?
O
A)
B)
C)
D)
E)
1
2
3
4
5
Ans: E
Topic: C-13 NMR- Symmetry, Chemical shift
44. The C7 compound which gives 3 signals in the broadband proton-decoupled 13C
spectrum could be:
A) Heptane
B) 2-Methylhexane
C) 3,3-Dimethylpentane
D) 2,4-Dimethylpentane
E) 2,2,3-Trimethylbutane
Ans: D
297
Chapter 9
Topic: Proton NMR- Symmetry, Splitting, Chemical shift
45. Which of these compounds will not be represented by a singlet only in the 1H NMR
spectrum?
A) Neopentane
B) Hexamethylbenzene
C) Isobutane
D) (Z)-1,2-Dichloroethene
E) (E)-1,2-Dichloroethene
Ans: C
Topic: Proton NMR- Symmetry, Splitting, Chemical shift
46. For the C2 methylene group in 1-bromopropane, the theoretical multiplicity in the 1H
NMR spectrum, presuming that Jab is sufficiently different from Jbc and that the
instrument has sufficient resolving power, is which of these?
A)
B)
C)
D)
E)
CH3
CH2
CH2
c
b
a
Br
2
5
6
8
12
Ans: E
Topic: Proton NMR- Symmetry, Splitting, Chemical shift
47. A compound C5H11Cl which exhibits only two singlets in the 1H NMR spectrum must
be:
A) 1-Chloropentane
B) 1-Chloro-2,2-dimethylpropane
C) 1-Chloro-2-methylbutane
D) 3-Chloropentane
E) 1-Chloro-3-methylbutane
Ans: B
298
Chapter 9
Topic: Proton NMR- Symmetry, Splitting, Chemical shift
48. A downfield (δ 9-10) singlet is observed in the 1H NMR spectrum of:
O
A)
B)
CH3CCH2CH3
H
C6H5CH2C
H
C)
O
(CH3)2CHC
H
O
D)
E)
(CH3)3CC
O
O
C6H5CH2CCH3
Ans: D
Topic: Proton NMR- Symmetry, Splitting, Chemical shift
49. Consider the expected splitting of signal “b” in the 1H NMR spectrum of N-methyl-1propanamine, shown below. Presuming that Jab is sufficiently different from Jbc and that
the instrument has sufficient resolving power, what is the theoretical multiplicity of
signal “b”?
d
H
b
N
a
A)
B)
C)
D)
E)
c
e
2
5
6
8
12
Ans: E
299
Chapter 9
Topic: Proton NMR- Symmetry, Splitting, Chemical shift
50. Consider the expected splitting of signal “b” in the 1H NMR spectrum of 1-methoxy-2methylpropane, shown below. Presuming that Jab is sufficiently different from Jbc and
that the instrument has sufficient resolving power, what is the theoretical multiplicity of
signal “b”?
a
a
A)
B)
C)
D)
E)
b
O
c
d
8
9
12
21
24
Ans: D
Topic: Proton NMR- Symmetry, Splitting, Chemical shift
51. Consider the expected splitting of signal “b” in the 1H NMR spectrum of 1,1-dichloro-3methylbutane, shown below. Presuming that Jab is sufficiently different from Jbc and
that the instrument has sufficient resolving power, what is the theoretical multiplicity of
signal “b”?
Cl
a
a
A)
B)
C)
D)
E)
b
c
d
Cl
8
9
12
18
21
Ans: E
300
Chapter 9
Topic: Proton NMR- Symmetry, Splitting, Chemical shift
52. Consider the expected splitting of signal “b” in the 1H NMR spectrum of 1,2dimethoxypropane, shown below. Presuming that Jbc is sufficiently different from Jbd
and that the instrument has sufficient resolving power, what is the theoretical
multiplicity of signal “b”?
c
a O
A)
B)
C)
D)
E)
b
O
e
d
2
5
6
8
12
Ans: E
Topic: Proton NMR- Symmetry, Splitting, Chemical shift
53. Consider the expected splitting of signal “c” in the 1H NMR spectrum of 2-ethyl-1butanol, shown below. Presuming that Jbc is sufficiently different from Jcd and that the
instrument has sufficient resolving power, what is the theoretical multiplicity of signal
“c”?
a
b
a
A)
B)
C)
D)
E)
b
c
OH
d
e
6
7
8
12
15
Ans: E
301
Chapter 9
Topic: Proton NMR- Symmetry, Splitting, Chemical shift
54. Consider the expected splitting of the C2 proton signal in the 1H NMR spectrum of 2ethyl-1,3-propanediol. Presuming that the coupling constants for neighboring protons
are sufficiently different, and that the instrument has sufficient resolving power, what is
the theoretical multiplicity of the C2 proton signal?
A)
B)
C)
D)
E)
6
7
8
12
15
Ans: E
Topic: Proton NMR- Symmetry, Splitting, Chemical shift
55. What is the theoretical multiplicity of the C-2 proton signal in the 1H NMR spectrum of
2-hydroxymethyl-1,3-propanediol, shown below?
OH
HO
A)
B)
C)
D)
E)
OH
1
6
7
8
9
Ans: C
Topic: Proton NMR- Symmetry, Splitting, Chemical shift
56. Consider the expected 1H NMR spectrum of 1,1,3,3-trimethylcyclopentane. Which of
the following is likely to be observed?
A)
B)
C)
D)
E)
7 signals: all singlets
7 signals: 4 singlets, 3 doublets
3 signals: all singlets
3 signals: one singlet, 2 doublets
3 signals: two singlets, one doublet
Ans: E
302
Chapter 9
Topic: Proton NMR- Symmetry, Splitting, Chemical shift
57. Consider the expected 1H NMR spectrum of 2,4-dimethyl-1,4-pentadiene. Which of the
following is likely to be observed?
A)
B)
C)
D)
E)
7 signals: all singlets
4 signals: all singlets
3 signals: all singlets
3 signals: one singlet, 2 doublets
4 signals: two singlets, two doublets
Ans: B
Topic: Proton NMR- Symmetry, Splitting, Chemical shift
58. Consider the expected splitting of the C2 proton signal in the 1H NMR spectrum of 1,1dibromopentane. Presuming that the coupling constants for neighboring protons are
sufficiently different, and that the instrument has sufficient resolving power, what is the
theoretical multiplicity of the C2 proton signal?
A) 3
B) 4
C) 5
D) 6
E) 8
Ans: D
303
Chapter 9
Topic: Carbon NMR- Symmetry, Chemical shift
59. A bromodichlorobenzene which gives four signals in the broadband proton-decoupled
13
C spectrum could be:
Br
Cl
Cl
Br
Cl
Br
Cl
Cl
Cl
I
II
III
Cl
Br
Cl
Br
Cl
IV
A)
B)
C)
D)
E)
Cl
V
I
II
III
IV
V
Ans: A
Topic: Carbon NMR- Symmetry, Chemical shift
60. In 13C NMR spectroscopy, the signal due to this type of carbon occurs furthest
downfield.
A) –C≡N
B)
C
O
C)
C
O
C
X
D)
E) –C≡C–
Ans: B
304
Chapter 9
Topic: Mass Spectrometry.
Use the following to answer Q 61-62
Topic: Mass Spectrometry.
61.
A)
B)
C)
D)
E)
Which is the base peak?
15
29
44
45
100
Ans: B
Topic: Mass Spectrometry.
62.
A)
B)
C)
D)
E)
Which is the likely molecular ion (M+•)?
15
29
44
45
100
Ans: C
Topic: Mass Spectrometry.
63. What is the molecular formula of this compound?
A)
B)
C)
D)
E)
m/z
84 M+•
85
86
C5H10O
C5H8O
C5H24
C6H12
C4H6O2
Ans: B
intensity
10.00
0.56
0.04
305
Chapter 9
Topic: Mass Spectrometry.
64. What is the molecular formula of this compound?
A)
B)
C)
D)
E)
m/z
78 M+•
79
80
81
C6H6
C3H5Cl
C6H8
C6H9
C3H7Cl
Ans: E
intensity
10.00
1
3.3
0.3
Topic: Mass Spectrometry.
65. The mass spectra of alkyl bromides and chlorides are characterized by an unusually
intense __________.
A) base peak
B) parent peak
C) M+• +1 peak
D) M+• +2 peak
E) None of these
Ans: D
Topic: Mass Spectrometry.
66.
A)
B)
C)
D)
E)
Predict the base peak for 2-chloro-2-methylpropane
m/z 15
m/z 92
m/z 43
m/z 57
m/z 77
Ans: D
306
Chapter 9
Topic: Mass Spectrometry.
67.
A)
B)
C)
D)
E)
Select the structure of a compound C6H14 with a base peak at m/z 43.
CH3CH2CH2CH2CH2CH3
(CH3CH2)2CHCH3
(CH3)3CCH2CH3
(CH3)2CHCH(CH3)2
None of these
Ans: D
Topic: Mass Spectrometry.
68. The data below from the molecular ion region of the mass spectrum of a halogencontaining compound are consistent with the presence of what halogen(s) in the original
compound?
intensity
A)
B)
C)
D)
E)
51.0
M+•
+
100.0
M • +2
+
49.0
M • +4
One Br
One Cl
One Br and one Cl
Two Br
Two Cl
Ans: D
Topic: Mass Spectrometry.
69.
A)
B)
C)
D)
E)
A prominent (M+• -18) peak suggests that the compound might be a(n):
alkane
alcohol
ether
ketone
primary amine
Ans: B
307
Chapter 9
Topic: Structure Elucidation
70. An organic compound absorbs strongly in the IR at 1687 cm-1. Its 1H NMR spectrum
consists of two signals, a singlet at 2.1 ppm and a multiplet centered at 7.1 ppm. Its
mass spectrum shows significant peaks at m/z 120, m/z 105 and m/z 77. This
information is consistent with which of the following structures?
O
O
I
II
IV
A)
B)
C)
D)
E)
O
III
V
I
II
III
IV
V
Ans: A
SHORT ANSWER QUESTIONS
Topic: General
71. Which form of electromagnetic radiation possesses the least energy?
Ans: radiofrequency radiation
Topic: General
72. “NMR” stands for _______________________.
Ans: nuclear magnetic resonance
Topic: General; Signal Integration
73. When measuring the integral for a particular peak in the NMR spectrum, we are not
interested in the peak height as much as in the ____________________.
Ans: area under the peak
308
Chapter 9
Topic: General; Chemical Shift
74. A shielded proton will absorb at a higher frequency (this is the ________ end of the
spectrum); and a deshielded proton will absorb at a lower frequency (the
_____________ end of the spectrum).
Ans: upfield; downfield
Topic: Reference Compound
75. What compound is used as the standard “zero” reference in both carbon and proton
NMR?
Ans: TMS or tetramethylsilane or (CH3)4 Si
Topic: General, Bond Rotation, Proton Exchange
76. Name two rapid processes that occur in organic molecules.
Ans: Rotation about C-C single bonds; chemical exchange of protons attached to
electronegative atoms
Topic: Molecular Ion
77. In electron impact mass spectrometry, a beam of high-energy electrons initially
dislodges one electron from the compound being studied. This produces a positively
charged ion called the ____________________.
Ans: molecular ion
Topic: Nitrogen Rule
78. What is the nitrogen rule?
Ans: If the molecular ion peak is even, the molecule must contain an even number of
nitrogen atoms (zero is an even number). If the molecular ion peak is odd, the
molecule must contain an odd number of nitrogen atoms.)
Topic: Relative Abundance
79. What can be determined from the relative abundance of the M+• +1 peak?
Ans: An approximation for the number of carbon atoms in the molecule.
Topic: Relative Abundance
80. What can be determined from the relative abundance of the M+• +2 peak?
Ans: The presence of a number of different elements, including S, Br, and Cl.
309
Chapter 9
Topic: Base Peak
81. In mass spectrometry, the most intense peak is assigned an intensity of 100%, and is
referred to as the ______________.
Ans: base peak
Topic: m/z Ratio
82. A mass spectrometer sorts ions on the basis of their _______________.
Ans: mass to charge ratio
Topic: Structure Elucidation
83. Briefly explain how you might distinguish between the following substances by
comparing their 1H-NMR spectra:
O
O
O
O
I
II
Ans: The main difference would likely be in the chemical shift of the methine proton of
the isopropyl group. The methine proton in I is likely to be more deshielded, and
produce a signal (septet) at about 3.5 ppm, while the analogous proton in II is
likely to be found, also as a septet, at about 2.3 ppm.
O
O
O
O
I
~3.5 ppm
II
~2.3 ppm
310
Chapter 9
Topic: Structure Elucidation
84. Briefly explain how you might distinguish between the following substances by
comparing their 1H-NMR spectra:
O
O
O
O
I
II
Ans: The main difference would likely be in the chemical shifts of the methine proton
of the isopropyl group and the methylene proton closest to the ester function.
The methine proton in I is likely to be more deshielded, and produce a signal
(septet) at about 3.5 ppm, while the analogous proton in II is likely to be found,
also as a septet, at about 2.3 ppm.
By contrast, the methylene proton in II is likely to be more deshielded, and
produce a signal (triplet) at about 3.4 ppm, while the analogous proton in II is
likely to be found, also as a triplet, at about 2.2 ppm.
I
II
O
O
O
O
~3.5 ppm
(septet)
~2.2 ppm
(triplet)
~2.3 ppm
(septet)
~3.4 ppm
(triplet)
311
Chapter 9
Topic: Structure Elucidation
85. Briefly explain how you might distinguish between the following substances by
comparing their 1H-NMR spectra:
O
H
N
N
H
O
I
II
Ans: The main difference would likely be in the chemical shifts of the methine proton
of the isopropyl group and the methylene proton closest to the amide function.
The methine proton in I is likely to be more deshielded, and produce a signal
(septet) at about 3.5 ppm, while the analogous proton in II is likely to be found,
also as a septet, at about 2.3 ppm.
By contrast, the methylene proton in II is likely to be more deshielded, and
produce a signal (triplet) at about 3.4 ppm, while the analogous proton in II is
likely to be found, also as a triplet, at about 2.2 ppm.
I
II
O
N
H
~3.2 ppm
(septet)
~2.2 ppm
(triplet)
H
N
O
~2.3 ppm
(septet)
~3.1 ppm
(triplet)
312
Chapter 9
Topic: Structure Elucidation
86. An unknown compound, A, has the formula C7H14O. Elucidate the structure of A by
scrutinizing its IR, 1H NMR and 13C NMR spectra, shown below.
Ans:
O
4-Heptanone
313
Chapter 9
Topic: Structure Elucidation
87.
An unknown compound, C, has the formula C7H7Br. Elucidate the structure of C by
scrutinizing its IR, 1H NMR and 13C NMR spectra, shown below.
314
Chapter 9
Ans:
Br
4-Bromotoluene
315
Chapter 9
Topic: Structure Elucidation
88. An unknown compound, E, has the formula C6H12O. Elucidate the structure of E by
scrutinizing its IR, 1H NMR and 13C NMR spectra, shown below.
Ans:
O
2-Hexanone
316
Chapter 9
Topic: Structure Elucidation
89. An unknown compound, F, has the formula C3H6O2. Elucidate the structure of F by
scrutinizing its IR, 1H NMR and 13C NMR spectra, shown below.
317
Chapter 9
Ans:
O
OH
Propanoic acid
318
Chapter 9
Topic: Structure Elucidation
90. An unknown compound, I, has the formula C3H7NO2. Elucidate the structure of I by
scrutinizing its IR, 1H NMR and 13C NMR spectra, shown below.
Ans:
NO2
1-Nitropropane
319
Chapter 9
Topic: Structure Elucidation
91. An unknown compound, L, has the formula C5H10O2. Elucidate the structure of L by
scrutinizing its IR, 1H NMR and 13C NMR spectra, shown below.
O
Ans:
O
Methyl butyrate
320
Chapter 9
Topic: Structure Elucidation
92. An unknown compound, U, has the formula C6H4Cl2O. Elucidate the structure of U by
scrutinizing its IR, 1H NMR and 13C NMR spectra, shown below.
321
Chapter 9
OH
Ans:
Cl
Cl
2,6-dichlorophenol
322
Chapter 9
Topic: Structure Elucidation
93. An unknown compound, V, has the formula C8H10O. Elucidate the structure of V by
scrutinizing its IR, 1H NMR and 13C NMR spectra, shown below.
323
Chapter 9
Ans:
OH
2,6-dimethylphenol
324
Chapter 9
Topic: Structure Elucidation
94. Examine the 1H NMR spectrum of 1-nitropropane, shown below. Assign the signals to
the respective hydrogen atoms in the molecule.
4.3 ppm
triplet
Ans:
>
>
>
1.0 ppm
triplet
NO2
2.0 ppm
sextet
Topic: 1H NMR
95. Predict the 1H NMR spectrum of 2-chloroethanal, CH2ClCHO.
9-10 ppm
triplet
Ans:
>
>
Cl
O
4-5 ppm
doublet
325
Chapter 9
Topic: 1H NMR
96. Predict the number of signals in the 1H NMR spectrum of 2-methyl-2-pentanol.
Ans: Five
1
2
1'
4
3
OH
5
Topic: 13C NMR, Structure Elucidation
97. Interpret the following 13C/DEPT spectrum of a compound C5H10Br2: elucidate the
structure and assign all the 13C resonances.
32 ppm
>
>
33 ppm
>
>
Br
27 ppm
Br
>
Ans:
33 ppm
326
Chapter 9
Topic: 1H NMR
98. Predict the 1H NMR spectrum of diethoxymethane, CH3CH2OCH2OCH2CH3.
~1.2 ppm
triplet
~1.2 ppm
triplet
>
>
~3.4 ppm
quartet
~4.9 ppm
singlet
>
O
>
O
>
Ans:
~3.4 ppm
quartet
327
Chapter 10
MULTIPLE CHOICE QUESTIONS
Topic: Delta H Calculations and Comparisons
1. The bond dissociation energies for the relevant bonds are given below each of the
species involved in the following reaction. Calculate the overall ΔH° for the reaction.
(CH3)3C-H
+
Cl-Cl
(CH3)3C-Cl
+
H-Cl
ΔHo=400 kJ/mol ΔHo=243 kJ/mol
A)
B)
C)
D)
E)
ΔHo=349 kJ/mol ΔHo=432 kJ/mol
+243 kJ / mol
-138 kJ / mol
+138 kJ / mol
-781 kJ / mol
+781 kJ / mol
Ans: B
Topic: Delta H Calculations and Comparisons
2. The bond dissociation energies for the relevant bonds are given below each of the
species involved in the following reaction. Calculate the overall ΔH° for the reaction.
CH3CH2CH2-H
+
Br-Br
CH3CH2CH2-Br
ΔHo=423 kJ/mol ΔHo=193 kJ/mol
A)
B)
C)
D)
E)
+
H-Br
ΔHo=294 kJ/mol ΔHo=366 kJ/mol
+616 kJ / mol
-101 kJ / mol
-173 kJ / mol
+57 kJ kJ / mol
-44 kJ / mol
Ans: E
328
Chapter 10
Topic: Delta H Calculations and Comparisons
3. The bond dissociation energies for the relevant bonds are given below each of the
species involved in the following reaction. Calculate the overall ΔH° for the reaction.
CH3CH2-H
+
Cl-Cl
CH3CH2-Cl
+ H-Cl
ΔH =421 kJ/mol ΔH =243 kJ/mol
o
A)
B)
C)
D)
E)
ΔH =353 kJ/mol ΔH =432 kJ/mol
o
o
o
-121 kJ / mol
+121 kJ / mol
+243 kJ / mol
+664 kJ / mol
-785 kJ / mol
Ans: A
Topic: Delta H Calculations and Comparisons
4. The bond dissociation energies for the relevant bonds are given below each of the
species involved in the following reaction. Calculate the overall ΔH° for the reaction.
(CH3)2CH-H
+
F-F
(CH3)2CH-F
+
H-F
ΔHo=413 kJ/mol ΔHo=159 kJ/mol
A)
B)
C)
D)
E)
ΔHo=439 kJ/mol ΔHo=570 kJ/mol
+437 kJ / mol
-437 kJ / mol
-411 kJ / mol
+26 kJ / mol
-1581 kJ / mol
Ans: B
Topic: Delta H Calculations and Comparisons
5.
A)
B)
C)
D)
E)
Which of the reactions listed below would have a value of ΔH° equal to zero?
H–H ⎯⎯⎯⎯→ 2H·
H· + CH3–H ⎯⎯⎯⎯→ CH3–H + H·
CH3· + CH3· ⎯⎯⎯⎯→ CH3–CH3
CH3· + CH3–H ⎯⎯⎯⎯→ CH3–H + CH3·
Reactions (B) and (D)
Ans: E
329
Chapter 10
Topic: Delta H Calculations and Comparisons
6.
A)
B)
C)
D)
E)
Which of the reactions listed below would be exothermic?
CH3–CH3 ⎯⎯⎯⎯→ 2CH3·
CH3· + CH4 ⎯⎯⎯⎯→ CH4 + CH3·
2(CH3)2CH· ⎯⎯⎯⎯→ (CH3)2CH–CH(CH3)2
H· + (CH3)3CH ⎯⎯⎯⎯→ (CH3)3CH + H·
None of the above
Ans: C
Topic: Delta H Calculations and Comparisons
7.
A)
B)
C)
D)
E)
Which of the reactions listed below would be exothermic?
H–H ⎯⎯⎯⎯→ 2H·
H· + CH3–H ⎯⎯⎯⎯→ CH3–H + H·
CH3· + CH3· ⎯⎯⎯⎯→ CH3–CH3
CH3· + CH3–H ⎯⎯⎯⎯→ CH3–H + CH3·
All of the above
Ans: C
Topic: Activation Energies
8. Which reaction would you expect to have the smallest energy of activation?
ΔH°(kJ mol-1)
A) CH3· + CH3· ⎯⎯⎯⎯→ CH3CH3
-378
B) CH4 + F· ⎯⎯⎯⎯→ CH3· + HF
-130
C) CH4 + I· ⎯⎯⎯⎯→ CH3· + HI
+142
D) CH4 + Br· ⎯⎯⎯⎯→ CH3· + HBr
+104
E) CH4 + Cl· ⎯⎯⎯⎯→ CH3· + HCl
+8
Ans: A
Topic: Activation Energies
9.
A)
B)
C)
D)
E)
Which of the following reactions would have an activation energy equal to zero?
CH3–CH3 ⎯⎯⎯⎯→ 2CH3·
H· + CH3CH3 ⎯⎯⎯⎯→ CH3CH3 + H·
2CH3CH2· ⎯⎯⎯⎯→ CH3CH2CH2CH3
CH3· + CH3CH3 ⎯⎯⎯⎯→ CH3CH3 + CH3·
None of the above
Ans: C
330
Chapter 10
Topic: Activation Energies
10.
A)
B)
C)
Which of the following reactions would have the smallest energy of activation?
CH4 + Br· ⎯⎯⎯⎯→ CH3· + HBr
CH3CH3 + Br· ⎯⎯⎯⎯→ CH3CH2· + HBr
CH3CHCH3
CH3CHCH2
+
+
Br
CH3
D)
CH3CHCH3
CH3CCH3
+
E)
+
Br
CH3
CH3
CH3
C
HBr
CH3
HBr
CH3
CH3
CH3
+
Br
CH3
CH3
C
CH2
+
HBr
CH3
Ans: D
Topic: Activation Energies
11. Which of the following reactions would have the smallest energy of activation?
.
A)
+ Br .
+ HBr
B)
+ HBr
.
+ Br
.
C)
+ Br .
D)
+ Br .
E)
+ HBr
.
.
+ HBr
. + HBr
+ Br .
Ans: C
Topic: Activation Energies
12.
A)
B)
C)
D)
E)
An example of a reaction having an Eact = 0 would be:
Br· + Br–Br ⎯⎯⎯⎯→ Br–Br + Br·
F· + CH4 ⎯⎯⎯⎯→ H–F + CH3·
CH3· + CH3CH3 ⎯⎯⎯⎯→ CH4 + CH3CH2·
Br· + H–Br ⎯⎯⎯⎯→ H–Br + Br·
CH3· + CH3· ⎯⎯⎯⎯→ CH3–CH3
Ans: E
331
Chapter 10
Topic: Activation Energies
13.
A)
B)
C)
Which of the following reactions should have the smallest energy of activation?
CH4 + Cl· ⎯⎯⎯⎯→ CH3· + HCl
CH3CH3 + Cl· ⎯⎯⎯⎯→ CH3CH2· + HCl
CH3CHCH3
CH3CHCH2
+
+
Cl
CH3
D)
CH3CHCH3
CH3CCH3
+
E)
+
Cl
CH3
CH3
CH3
C
HCl
CH3
HCl
CH3
CH3
CH3
+
CH3
Cl
CH3
C
CH2
+
CH3
Ans: D
Topic: Activation Energies
14. Which reaction would you expect to have the largest energy of activation?
ΔH° (kJ mol-1)
A) CH3· + CH3· ⎯⎯⎯⎯→ CH3CH3
-378
B) CH3· + Br· ⎯⎯⎯⎯→ CH3Br
-130
C) CH4 + I· ⎯⎯⎯⎯→ CH3· + HI
+142
D) CH4 + Br· ⎯⎯⎯⎯→ CH3· + HBr
+104
E) CH4 + Cl· ⎯⎯⎯⎯→ CH3· + HCl
+8
Ans: C
Topic: Activation Energies
15.
A)
B)
C)
D)
E)
An example of a reaction having an Eact = 0 would be:
Br· + Br–Br ⎯⎯⎯⎯→ Br–Br + Br·
F· + CH4 ⎯⎯⎯⎯→ H–F + CH3·
CH3· + Cl· ⎯⎯⎯⎯→ CH3Cl
More than one of these
None of these
Ans: C
332
HCl
Chapter 10
Topic: Activation Energies
16.
A)
B)
C)
D)
E)
Which of the following reactions would have an activation energy equal to zero?
H–H ⎯⎯⎯⎯→ 2H·
H· + CH3–H ⎯⎯⎯⎯→ CH3––H + H·
CH3· + CH3· ⎯⎯⎯⎯→ CH3–CH3
CH3· + CH3–H ⎯⎯⎯⎯→ CH3–H + CH3·
All of the above
Ans: C
Topic: Activation Energies
1. Given the following bond dissociation energies:
ΔH° (kJ mol-1)
CH3CH2–H
H–F
H–Cl
H–Br
H–I
A)
B)
C)
D)
E)
Ans:
421
570
432
366
298
predict which of the following reactions would have the highest energy of activation.
CH3CH3 + F· ⎯⎯⎯⎯→ CH3CH2· + HF
CH3CH3 + Cl· ⎯⎯⎯⎯→ CH3CH2· + HCl
CH3CH3 + Br· ⎯⎯⎯⎯→ CH3CH2· + HBr
CH3CH3 + I· ⎯⎯⎯⎯→ CH3CH2· + HI
ΔH° values are important, but not sufficient for this prediction
D
333
Chapter 10
Topic: Activation Energies
18. Given the following bond dissociation energies:
ΔH°(kJ mol-1)
CH3–H
H–F
H–Cl
H–Br
H–I
A)
B)
C)
D)
E)
440
570
432
366
298
predict which of the following reactions would have the highest energy of activation.
CH4 + F· ⎯⎯⎯⎯→ CH3· + HF
CH4 + Cl· ⎯⎯⎯⎯→ CH3· + HCl
CH4 + Br· ⎯⎯⎯⎯→ CH3· + HBr
CH4 + I· ⎯⎯⎯⎯→ CH3· + HI
ΔH° values are important, but not sufficient for this prediction
Ans: D
Topic: Activation Energies
19.
A)
B)
C)
D)
E)
For which of the following gas-phase reactions would the Eact be equal to ΔH°?
Cl–Cl ⎯⎯⎯⎯→ 2Cl·
2 Cl· ⎯⎯⎯⎯→ Cl–Cl
Cl· + CH4 ⎯⎯⎯⎯→ CH3· + H–Cl
CH3· + CH3· ⎯⎯⎯⎯→ CH3–CH3
CH3· + Cl–Cl ⎯⎯⎯⎯→ CH3–Cl + Cl·
Ans: A
Topic: Reaction Products (Isomers)
20. In the presence of light, ethane (1 mol) reacts with chlorine (1 mol) to form which
product(s)?
A) CH2ClCHCl2
B) CH3CHCl2
C) CH3CH2Cl
D) ClCH2CH2Cl
E) All of these
Ans: E
334
Chapter 10
Topic: Reaction Products (Isomers)
21. How many different monochlorobutanes (including stereoisomers) are formed in the
free radical chlorination of butane?
A) 1
B) 2
C) 3
D) 4
E) 5
Ans: C
Topic: Reaction Products (Isomers)
22. The reaction of 2-methylbutane with chlorine would yield how many monochloro
derivatives? (include stereoisomers)
A) 2
B) 3
C) 4
D) 5
E) 6
Ans: E
Topic: Reaction Products (Isomers)
23. The reaction of 2,2-dimethylbutane with chlorine would yield how many monochloro
derivatives? (include stereoisomers)
A) 1
B) 2
C) 3
D) 4
E) 5
Ans: D
Topic: Reaction Products (Isomers)
24. The free radical chlorination of pentane produces this number of monochloro
compounds, including stereoisomers.
A) 2
B) 3
C) 4
D) 5
E) 6
Ans: C
335
Chapter 10
Topic: Reaction Products (Isomers)
25.
A)
B)
C)
D)
E)
Free radical chlorination will produce but one monochloro derivative in the case of:
Propane.
Butane.
Isobutane.
Isopentane.
Neopentane.
Ans: E
Topic: Reaction Products (Isomers)
26. Free radical chlorination of hexane produces this number of monochloro derivatives
(including stereoisomers):
A) 3
B) 4
C) 5
D) 7
E) 8
Ans: C
Topic: Reaction Products (Isomers)
27. More than one monochloro compound can be obtained from the free radical chlorination
of:
A) Cyclopentane
B) Neopentane
C) Isobutane
D) Ethane
E) Methane
Ans: C
Topic: Reaction Products (Isomers)
28. What is the total number of trichloropropanes which can be produced by free radical
chlorination of propane? Include all stereoisomers.
A) 4
B) 5
C) 6
D) 7
E) 8
Ans: C
336
Chapter 10
Topic: Reaction Products (Isomers)
29. If chlorocyclopentane were chlorinated to form all possible dichloro compounds and the
product mixture subjected to precise fractional distillation, how many fractions would
be obtained (ideally)?
A) 3
B) 4
C) 5
D) 7
E) 9
Ans: C
Topic: Reaction Products (Isomers)
30. The free radical chlorination of (R)-2-chloropentane forms a mixture of
dichloropentanes which includes:
A) three optically active compounds
B) two achiral compounds.
C) two meso compounds.
D) one pair of diastereomers.
E) one racemic mixture.
Ans: B
Topic: Reaction Products (Isomers)
31. How many monochloro derivatives, including stereoisomers, can be formed in the
chlorination of 1-bromobutane?
A) 4
B) 5
C) 6
D) 7
E) 8
Ans: D
Topic: Reaction Products (Isomers)
32. The free radical chlorination of 3-chloropentane forms a mixture of dichloropentanes
which, on precise fractional distillation, affords these fractions:
A) 4 fractions, none optically active
B) 4 fractions, 2 optically active
C) 7 fractions, 4 optically active
D) 7 fractions, 6 optically active
E) 7 fractions, all optically active
Ans: A
337
Chapter 10
Topic: Reaction Products (Isomers)
33. Consider the light-initiated chlorination of (S)-2-chlorobutane followed by careful
fractional distillation (or separation by GLC) of all of the products with the formula
C4H8Cl2. How many fractions (in total) would be obtained and how many of these
fractions would be optically active?
A) Three fractions total; all optically active
B) Four fractions total; three fractions optically active
C) Five fractions total; all optically active
D) Five fractions total; four fractions optically active
E) Five fractions total; three fractions optically active
Ans: E
Topic: Reaction Products
34. What is the product of the reaction
CH3CH2CH
CH2
+
CBr4
peroxides
A) CH3CH2CH=CHCBr3
B) CH3CH2CHCH2CBr3
Br
C) CH3CH2CHCH2Br
CBr3
D) CH3CH2CH2CH2CBr3
E) No reaction occurs.
Ans: B
338
?
Chapter 10
Topic: Reaction Products
35. Select the structure of the major product formed in the following reaction.
CH3
Br2
?
hν
CH2Br
Br
CH3
CH3
Br
CH3
IV
V
Br
Br
I
A)
B)
C)
D)
E)
II
III
I
II
III
IV
V
Ans: B
Topic: Reaction Products
36. What would be the major product of the following reaction?
CH3
HBr
?
peroxides
CH3
Br
CH3
CH3
Br
CH3
Br
OR
CH3
Br
I
A)
B)
C)
D)
E)
II
III
IV
I
II
III
IV
V
Ans: A
339
V
Chapter 10
Topic: Reaction Products
37. In the presence of light at 25°C, isobutane (1 mol) and bromine (1 mol) yield which
monobromo product(s)?
A) 2-Methyl-1-bromopropane (almost exclusively)
B) 2-Methyl-2-bromopropane (almost exclusively)
C) A mixture of 50% (A) and 50% (B)
D) A mixture of 90% (A) and 10% (B)
E) Butyl bromide
Ans: B
Topic: Reaction Products
38. Which of these molecules is not expected to arise as a product of the high temperature
chlorination of methane?
A) CCl4
B) HCCl3
C) CH2Cl2
D) CH3CH3
E) CH2=CH2
Ans: E
Topic: Reaction Products
39. In a competition reaction, equimolar amounts of five alkanes compete for a deficiency
of chlorine at 300°C. The greatest amount of reaction would occur in the case of which
of these alkanes?
A) Ethane
B) Propane
C) Butane
D) Isobutane
E) Pentane
Ans: D
Topic: Reaction Products
40. Which of the following combinations of reactants can provide a demonstrable example
of anti-Markovnikov addition?
A) CH2=CHCH3 + HCl + ROOR
B) CH3CH=CH2 + H2O + Cl2
C) CH3CH=CHCH3 + HBr + ROOR
D) CH3CH2CH=CH2 + HBr + ROOR
E) CH3CH2CH=CH2 + Br2 + ROOR
Ans: D
340
Chapter 10
Topic: Reaction Products
41. 2-Methyl-2-butene reacts with HBr in the presence of peroxide to give (chiefly):
A) (CH3)2CHCH2CH2Br
B) (CH3)2CHCHCH3
Br
C) (CH3)2CCH2CH3
D)
Br
CH3
E)
BrCH2CHCH2CH3
Br
(CH3)2CCHCH3
Br
Ans: B
Topic: Reaction Products
42. What product would result from the following reaction?
+ HBr
peroxides
?
Br
Br
I
A)
B)
C)
D)
E)
Br
HOO Br
II
III
Br OOH
IV
I
II
III
IV
V
Ans: A
341
V
Chapter 10
Topic: Synthesis
43. Which of the following would serve as the best synthesis of 2-bromohexane?
peroxides
A)
B)
C)
D)
CH2
CHCH2CH2CH2CH3 + HBr
CH2
CHCH2CH2CH2CH3 + HBr
CH3CH2
CHCH2CH2CH3
+ HBr
CH3CH2
CHCH2CH2CH3
+ HBr
heat
heat
heat
peroxides
heat
E) All of the above would be equally suitable.
Ans: B
Topic: Synthesis
44. Which would be the best way to carry out the following synthesis?
?
(CH3)3COH
(CH 3)2CHCH2Br
A)
B)
C)
D)
E)
(1) HA, heat; (2) HBr
(1) HBr and peroxides; (2) Br2/CCl4
(1) HA, heat; (2) HBr and peroxides
(1) Br2/CCl4; (2) HA, heat
(1) HA, heat; (2) Br2/CCl4
Ans: C
Topic: Reaction Products
45. What sequence of reactions could be used to prepare cis-1,2-cyclopentanediol from
cyclopentane?
A) (1) Cl2, hν; (2) t-BuOK/t-BuOH; (3) OsO4; (4) NaHSO3/H2O
B) (1) t-BuOK/t-BuOH; (2) Cl2, hν; (3) NaOH/H2O
C) (1) Cl2, hν; (2) t-BuOK/t-BuOH; (3) H2O2
D) (1) NaOH/H2O; (2) Br2; (3) NaNH2(2eq.)/liq.NH3; (4) KMnO4, NaOH/H2O, 5°C
E) (1) Cl2, hν; (2) t-BuOK/t-BuOH; (3) Hg(OAc)2, H2O (4) NaBH4, H3O+
Ans: A
342
Chapter 10
Topic: Mechanisms
46. Carbocations are NOT intermediates in which one of the following reactions?
Br
A)
+ HBr
B)
OH
+ H2SO4
H
C)
H2B
+ BH3
OSO3H
D)
+ H2SO4
E)
R-O-O-R
Br
+ HBr
Ans: E
Topic: Mechanisms
47.
A)
B)
C)
D)
A chain reaction is one that:
involves a series of steps.
involves two steps of equal activation energy.
is one that can be initiated by light.
involves a series of steps, each of which generates a reactive intermediate that brings
about the next step.
E) involves free radicals that have an unusual stability and thereby cause a large quantum
yield.
Ans: D
Topic: Mechanisms
48. Which of the following statements is true when used to compare the reaction of fluorine
with 2-methylhexane and the reaction of bromine with 2-methylhexane?
A) Bromine is the less reactive and the less selective, giving 2-bromo-2-methyl hexane as
one of several products.
B) Fluorine is the less reactive and the more selective, giving 2-fluoro-2-methyl hexane as
the major product.
C) Fluorine is the more reactive and less selective, giving 2-fluoro-2-methyl hexane as one
of several products.
D) Bromine is the more reactive and the more selective, giving 2-bromo-2-methyl hexane
as the major product.
E) More than one of the above statements is true
Ans: C
343
Chapter 10
Topic: Mechanisms
49. An alternate mechanism for the chlorination of methane is:
Cl2 ⎯⎯⎯⎯→ 2Cl·
Cl· + CH4 ⎯⎯⎯⎯→ CH3Cl + H·
H· + Cl2 ⎯⎯⎯⎯→ HCl + Cl·
A)
B)
C)
D)
E)
Cl–Cl, ΔH° = 243 kJ mol-1
CH3–H, ΔH° = 440 kJ mol-1
CH3–Cl, ΔH° = 352 kJ mol-1
H–Cl, ΔH° = 432 kJ mol -1
This mechanism is unlikely because:
The overall ΔH° is highly endothermic.
The probability factor is low.
One of the chain propagating steps is non-productive.
One of the chain propagating steps has a very high Eact.
One of the chain propagating steps is highly exothermic.
Ans: D
Topic: Mechanisms
50.
A)
B)
C)
D)
E)
Which statement(s) is(are) true about the reaction of bromine with isobutane?
Bromine selectively abstracts the tertiary hydrogen.
The transition state of the rate determining step is product-like.
The major product formed from this reaction is 1-bromo-2-methylpropane.
A) and B)
A), B) and C)
Ans: D
Topic: Mechanisms
51.
A)
B)
C)
For which reaction would the transition state be most product-like?
CH4 + Br· ⎯⎯⎯⎯→ CH3· + HBr
CH3CH3 + Br· ⎯⎯⎯⎯→ CH3CH2· + HBr
CH3CHCH3
CH3CHCH2
+
+
Br
CH3
D)
CH3CHCH3
CH3CCH3
+
E)
+
Br
CH3
CH3
CH3CCH3
HBr
CH3
HBr
CH3
CH3
+
CH3CCH2
Br
CH3
CH3
Ans: A
344
+
HBr
Chapter 10
Topic: Mechanisms
52. For which of the following reactions would the transition state most resemble the
products? The following bond dissociation energies may be useful.
(CH3)2CH–H
(413 kJ mol-1)
CH3CH2CH2–H
(423 kJ mol-1)
H–Cl
(432 kJ mol-1)
H–F
(570 kJ mol-1)
H–Br
(366 kJ mol-1)
A)
B) CH3CH2CH3 + F· ⎯⎯⎯⎯→ CH3CH2CH2· + HF
C)
D) CH3CH2CH3 + Cl· ⎯⎯⎯⎯→ CH3CH2CH2· + HCl
E) CH3CH2CH3 + Br· ⎯⎯⎯⎯→ CH3CH2CH2· + HBr
Ans: E
Topic: Mechanisms
53. Which of the following gas-phase reactions is a possible chain-terminating step in the
light-initiated chlorination of methane?
A) Cl–Cl ⎯⎯⎯⎯→ 2Cl·
B) Cl· + CH4 ⎯⎯⎯⎯→ CH3· + H–Cl
C) CH3· + CH3· ⎯⎯⎯⎯→ CH3–CH3
D) CH3· + Cl–Cl ⎯⎯⎯⎯→ CH3Cl + Cl·
E) More than one of the above
Ans: C
Topic: Mechanisms
54. The reaction of 1-butene with HBr in the presence of peroxides yields 1-bromobutane.
The mechanism for the reaction involves:
A) attack on the alkene by a Br+ ion.
B) attack on the alkene by a H+ ion.
C) attack on the alkene by a bromine atom, Br·.
D) attack on the alkene by a hydrogen atom, H·.
E) isomerization of the 2-bromobutane produced initially.
Ans: C
345
Chapter 10
Topic: Mechanisms
55. The reaction of Cl2 with a methyl radical has a positive ΔH°. Which of these drawings
is the best representation of the transition state of this reaction?
H
H
A)
δCl
δCl
C
H
H
B)
Cl
Cl
C
H
H
C)
δCl
δC
Cl
H
H
H
D)
Cl
E)
H
+
δCl
Cl
Cl
C
δC
H
H
H
H
H
Ans: E
Topic: Free Radicals
56.
A)
B)
C)
D)
E)
Ans:
The hybridization state of the carbon of a methyl radical is:
sp
sp2
sp3
sp4
p3
B
Topic: Free Radicals
57.
A)
B)
C)
D)
E)
The p-orbital of a methyl radical carbon, CH3·, contains how many electrons?
1
2
3
4
0
Ans: A
346
Chapter 10
Topic: Free Radicals
58. Which of the following free radicals is the most stable?
CH2
A)
B)
CH3CHCH2CH3
CH3
C)
CH3CHCH2CH2
CH3
D)
CH2CHCH2CH3
CH3
CH3CHCHCH3
E)
CH3
CH3CCH2CH3
Ans: E
Topic: Free Radicals
59. Hydrogen atom abstraction from which position would yield the most stable free radical
intermediate during the reaction of bromine with 2,2,3-trimethylpentane?
A) C1
B) C2
C) C3
D) C4
E) C5
Ans: C
Topic: Free Radicals
60.
A)
B)
C)
D)
E)
Free radicals can be produced by:
use of high temperatures.
irradiation with light.
reaction of a molecule with another free radical.
both A) and B).
all of A), B) and C).
Ans: E
347
Chapter 10
Topic: Free Radicals
61. The ΔH° value is expected to be least for which indicated C-H bond of isopentane?
A) H CH2CHCH2CH3
B)
C)
CH3
CH3CHCH2CH3
CH2
H
H
CH3CCH2CH3
D)
CH3
H
CH3CHCHCH3
E)
CH3
CH3CHCH2CH2
H
CH3
Ans: C
Topic: Free Radicals
62.
A)
B)
C)
D)
E)
Which is true for a chain-terminating step?
A new free radical is formed.
The process is endothermic.
Eact = 0.
ΔH° is positive.
A product is formed which is immune to further reaction.
Ans: C
Topic: Free Radicals
63. At some temperatures, the relative reactivities of 3°, 2° and 1° alkane hydrogens in free
radical chlorination are in the ratio of 5:3:1. Thus, monochlorination of isopentane
should produce these percentages of 2-chloro-2-methylbutane (A), combined 1-chloro2-methylbutane and 1-chloro-3-methylbutane (B), and 2-chloro-3-methylbutane (C):
A) 8% A, 75% B, 17% C
B) 25% A, 45% B, 30% C
C) 29% A, 44% B, 18% C
D) 30% A, 35% B, 35% C
E) 36% A, 43% B, 21% C
Ans: B
348
Chapter 10
Topic: Miscellaneous
64. According to the present explanation of the role of atmospheric chlorofluorocarbons in
ozone depletion, it is this species which destroys, i.e., reacts irreversibly with, ozone.
A) F·
B) Cl·
C) O·
D) ClO·
E) FO·
Ans: B
Topic: Miscellaneous
65. If propene polymerization is initiated by the use of diacyl peroxide, this is an
intermediate species formed early in the process.
CH3 CH3
A)
B)
RCH2CHCH2CH
CH3
CH3
C)
RCHCH2CHCH2
CH3
CH3
D)
RCHCH2CH2CH
CH3
RCH2CHCHCH2
CH3
E)
CH3
RCCH2CH
CH3 CH3
Ans: A
Topic: Miscellaneous
66. As the term "peroxide" is used in Chapter 10, it can refer to which structure(s)?
A) ROOR
B) ROOH
C) O O
RCOOCR
D) Answers A) and B) only
E) Answers A), B) and C)
Ans: E
349
Chapter 10
Topic: Miscellaneous
67. When an alkane in which all hydrogen atoms are not equivalent is monosubstituted, use
of this halogen produces a ratio of isomers which is essentially statistical, i.e., dependent
only on the number of each type of hydrogen.
A) F2
B) Cl2
C) Br2
D) I2
E) All of the above
Ans: A
Topic: Multistep Reactions
68. What is the final product, C, obtained via the following reaction sequence?
Br2
hν
A t-BuOK
t-BuOH
heat
OH
A)
B)
C)
D)
E)
I
I
II
III
IV
V
Ans: A
OH
II
B
ii) H2O2, OH
OH
III
i) BH3, THF
−
OH
OH
OH
IV
350
C
V
Chapter 10
Topic: Multistep Reactions
69. What is the major product obtained from the following reaction sequence?
Br2
hν
A
OH
I
A)
B)
C)
D)
E)
t-BuOK
t-BuOH
heat
B
OH
II
i) Hg(OAc)2, H2O, THF
ii) NaBH4, OH−
OH
OH
OH
OH
III
IV
C
V
I
II
III
IV
V
Ans: B
Topic: Multistep Reactions
70. What is the major product obtained from the following reaction sequence?
Br2
HBr
NaSH
B
D
A t-BuOK
C
peroxides
t-BuOH
hν
heat
SH
SH
A)
B)
C)
D)
E)
I
I
II
III
IV
V
Ans: A
SH
II
SH
III
IV
351
V
Chapter 10
Topic: Multistep Reactions
71. What is the major product obtained from the following reaction sequence?
Br2
HBr
NaCN
B
A EtONa
C
D
peroxides
EtOH
hν
heat
CN
CN
A)
B)
C)
D)
E)
I
I
II
III
IV
V
Ans: C
CN
II
CN
CN
III
IV
V
Topic: Multistep Reactions
72. What is the major product obtained from the following reaction sequence?
i) BH3, THF
HBr
t-BuOK
B
C
A
peroxides
t-BuOH
ii) H2O2, OH−
heat
OH
OH
OH
I
II
III
HO
OH
IV
A)
B)
C)
D)
E)
V
I
II
III
IV
V
Ans: E
352
Chapter 10
Topic: Multistep Reactions
73. What is the major product obtained from the following reaction sequence?
Br2
hν
OH
A EtONa
EtOH
heat
I
I
II
III
IV
V
Ans: E
i) O3
ii) Zn, HOAc
OH
O
OH
A)
B)
C)
D)
E)
B
C
O
O
II
III
O
IV
V
SHORT ANSWER QUESTIONS
Topic: General
74. Intermediates possessing unpaired electrons are called ______________.
Ans: radicals or free radicals
Topic: General
75. A covalent bond breaking so that each of the resulting intermediates bears a single
unpaired electron is an example of __________________ bond cleavage.
Ans: homolytic
Topic: General
76. Bond formation is an ______________ process, and therefore ΔH° is ____________.
Ans: exothermic; negative
Topic: General
77. Although alkyl radicals are uncharged, the carbon bearing the odd electron is electron
___________, and alkyl groups provide a __________________ effect.
Ans: deficient; stabilizing
353
Chapter 10
Topic: General
78. The mechanism for a free-radical reaction consists of three types of steps. These are:
_______________
Ans: initiation, propagation, termination – or chain-initiating, chain-propagating, chainterminating
Topic: General
79. In a chain-initiating step, radicals are _____________.
Ans: created or formed
Topic: General
80. In a chain-terminating step, radicals are ______________.
Ans: used up or removed
Topic: General
81. In a chain-propagating step, one radical _______________.
Ans: generates another
Topic: General
82. While the electrophilic addition of HBr to an alkene is ionic and follows Markovnikov's
rule, if the reaction is done in the presence of peroxides it becomes a ___________
reaction and is ____________________ in regiochemistry.
Ans: radical or free-radical; anti-Markovnikov
Topic: General
83. Macromolecules made up of many repeating subunits are called ____________.
Ans: polymers
Topic: General
84. Two radicals that are very important in biological processes are ________________ and
__________________.
Ans: molecular oxygen; nitric oxide
354
Chapter 10
Topic: General
85. Chlorofluorocarbons, or freons, diffuse into the upper atmosphere. There, ultraviolet
light initiates a radical chain reaction that has been shown to cause extreme damage to
_______________.
Ans: the ozone layer
Topic: General
86. Unsymmetrical alkenes give Markovnikov products through the most
stable_______________ intermediates and anti-Markovnikov products through the most
stable _______________ intermediates.
Ans: Carbocation; free-radical
Topic: Reaction Products ( Isomers)
87. Draw bond-line formulas of all monochloro derivatives that might be formed when 2,3dimethylbutane is allowed to react with Cl2 under UV irradiation. For each structure,
indicate, with an asterisk, any stereocenters that might be present.
Cl
Ans:
Cl2, hν
Cl
*
+
2,3-dimethylbutane
monochloro derivatives
Topic: Reaction Products ( Isomers)
88. Draw bond-line formulas of all monochloro derivatives that might be formed when 1,1dimethylcyclobutane is allowed to react with Cl2 under UV irradiation. For each
structure, indicate, with an asterisk, any stereocenters that might be present.
Ans:
Cl2, hν Cl
+ Cl *
+ Cl
1,1-dimethylcyclobutane
monochloro derivatives
355
Chapter 10
Topic: Reaction Products ( Isomers)
89. Draw bond-line formulas of all monochloro derivatives that might be formed when 3ethylhexane is allowed to react with Cl2 under UV irradiation. For each structure,
indicate, with an asterisk, any stereocenters that might be present.
Ans:
Cl
Cl2
* *
+
+
*
Cl
*
*
hν
Cl
3-ethylhexane
Cl
*
Cl
+
*
**
*
Cl
+
Cl
monochloro derivatives
*
Topic: Structure Elucidation
90. Deduce the structure of a substance, C8H18, which gives only a single monochloro
derivative upon reaction with Cl2 under UV irradiation.
Cl
Ans:
Cl2
hν
single
C8H18
monochloro derivative
Topic: Reaction Products ( Isomers)
91. Draw bond-line formulas of all dichloro derivatives that might be formed when 1chloro-2,2,3,3,-tetramethylpentane is allowed to react with Cl2 under UV irradiation.
For each structure, indicate, with an asterisk, any stereocenters that might be present.
Cl
Ans:
Cl
*
Cl
Cl2
Cl
+ Cl
+
hν
Cl
Cl
1-chloro-2,2,3,3tetramethylpentane
Cl
Cl
Cl
+
Cl
dichloro derivatives
356
*
+
Chapter 10
Topic: Reaction Products ( Isomers)
92. Draw bond-line formulas of all monochloro derivatives that might be formed when
2,2,4-trimethylpentane is allowed to react with Cl2 under UV irradiation. For each
structure, indicate, with an asterisk, any stereocenters that might be present.
Ans:
Cl2
*
+
Cl
hν
Cl
2,2,4-trimethylpentane
+
Cl
+
*
monochloro derivatives
Cl
Topic: Multistep Reactions
93. Complete the following reaction sequence: indicate regiochemical/stereochemical
details as relevant.
i) Br2, hν
Methylcyclopentane
ii) C2H5ONa/C2H5OH, heat
?
iii) BH3, THF
iv) H2O2, OH−
Ans:
Br
Br2
hν
C2H5ONa
C2H5OH
heat
BH3, THF
H3C
H
H3C
H
OH
+
H
H
H2O2, OH
OH
−
H3C
H
H3C
H
BH2
357
+
H
H
BH2
Chapter 10
Topic: Multistep Reactions
94. Complete the following reaction sequence: indicate regiochemical/stereochemical
details as relevant.
i) Br2, hν
ii) C2H5ONa/C2H5OH, heat
?
iii) HBr, peroxides
iv) NaCN
Br2
C2H5ONa
Ans:
HBr
NaCN
Br
peroxides
C2H5OH
hν
Br
heat
CN
Topic: Multistep Reactions
95. Complete the following reaction sequence: indicate regiochemical/stereochemical
details as relevant.
i) Br2, hν
ii) C2H5ONa/C2H5OH, heat
1,1,2-trimethylcyclohexane
?
iii) Ozonolysis
O
Ans:
Br2
C H ONa
Ozonolysis
hν
Br
2 5
C2H5OH
heat
Topic: Multistep Synthesis
96. Suggest a reasonable strategy for the synthesis of 1-bromo-2-methylcyclohexane from
methylcyclohexane
Br2
C2H5ONa
Ans:
HBr
Br
peroxides
C2H5OH
hν
Br
heat
358
O
Chapter 10
Topic: Mechanisms
97. Provide a reasonable mechanistic explanation for the formation of small amounts of
3,3,4,4-teramethylhexane during the free-radical bromination of 2-methylbutane.
Ans: During the free radical bromination of 2-methylbutane, the most stable
intermediate is produced by abstraction of the tertiary hydrogen atom at C2. Two
such radicals can couple together via a termination step to afford 3,3,4,4teramethylhexane as a minor product.
Br2
minor product
hν
+
termination step: radical coupling
Topic: Mechanisms
98. What product(s) might be obtained via chain-termination step(s) during the reaction of
2-methylpropene with HBr in presence of peroxides?
Ans: The addition of HBr to 2-methylpropene takes place via the formation of the most
stable free radical intermediate, by addition of a bromine atom at C1. Two such
radicals can couple together via a termination step to afford 1,4-dibromo-2,2,3,3teramethylbutane as a minor product. Analogous coupling between this radical
and a bromine radical leads to the formation of 1,2-dibromo-2-methylpropane as
another minor product.
HBr
peroxides
Br +
Br
most stable radical intermediate
Br
Br
Br
Br
Br +
Br
Br
termination steps: radical coupling
359
Chapter 10
Topic: Reactivity / Selectivity
99. Briefly, but clearly, explain the following observation: While alkanes can be fluorinated,
chlorinated and brominated easily, they do not easily undergo iodination.
Ans: The energy of activation for the first propagation step in the free radical
mechanism for halogenation of alkanes determines the overall reactivity pattern
for the various halogens. In contrast to the reaction with the other halogens,
hydrogen atom abstraction from an alkane by an iodine atom has a very high
energy of activation- since this is the first propagation step, the overall process is
affected as well and iodination does not occur to any appreciable extent.
I . + R-H Æ H-I + R .
Eact for this step very high: reaction very slow, further steps inhibited
Topic: Reactivity / Selectivity
100. Briefly, but clearly, explain the following observation:
When 2-methylbutane reacts with Cl2/hν, the monochlorinated products consist of four
constitutional isomers in significant yields. However, when the same alkane is allowed
to react with Br2/ hν, there is only one major monobromination product.
Ans: The final product distribution is a consequence of the relative ease of hydrogen
atom abstraction from primary, secondary and tertiary positions. Chlorine is
highly reactive and therefore there is not much differentiation between the
different kinds of hydrogen atoms- as a result, all the possible free radicals are
formed, leading to the formation of all possible constitutional isomers. Bromine
is less reactive and more selective than chlorine in its reaction with alkanes; this
results in the selective abstraction of the tertiary hydrogen atom at C2, to give the
most stable radical intermediate, a tertiary radical, leading to the selective
formation of only one major monobromination product.
Br
Br2
hν
Cl2, hν
Cl
Cl
Cl
Cl
360
Chapter 11
MULTIPLE CHOICE QUESTIONS
Topic: Nomenclature
1. What is the correct IUPAC name for the following compound?
CH3
CH3CHOHCHCHCH(CH3)2
A)
B)
C)
D)
E)
CH3
4-isopropyl-3,4-dimethyl-2-butanol
2,3,4-trimethyl-4-pentanol
1,1,2,3-tetramethyl-4-pentanol
3,4,5-trimethyl-2-hexanol
3,4,5,5-tetramethyl-2-pentanol
Ans: D
Topic: Nomenclature
2.
A)
B)
C)
D)
E)
A correct IUPAC name for isobutyl alcohol is:
2-Methyl-1-propanol
2-Methyl-1-butanol
1-Methyl-1-propanol
1,1-Dimethyl-1-ethanol
3-Methyl-1-propanol
Ans: A
Topic: Nomenclature
3. What is the correct IUPAC name for the following compound?
CH3
CH3CH2C=CCH2CH3
A)
B)
C)
D)
E)
CH2CH2OH
3-methyl-4-ethyl-3-hexen-6-ol
4-ethyl-3-methyl-3,6-hexenol
3-ethyl-4-methyl-3-hexen-1-ol
3-methyl-4-(2-hydroxyethyl)-3-hexene
3-(2-hydroxyethyl)- 3-methyl-3-hexene
Ans: C
361
Chapter 11
Topic: Nomenclature
CH3
4.
CH3CCHCH
A)
B)
C)
D)
E)
The correct IUPAC substitutive name for
4-Penten-2-methyl-2-ol
4-Methyl-1-penten-2-ol
2-Methyl-4-penten-2-ol
4-Methyl-1-penten-4-ol
4-Hydroxy-4-methyl-1-pentene
Ans: C
CH3
CH2
is:
Topic: Nomenclature
5.
A)
B)
C)
D)
E)
Which is a correct IUPAC name for CH3CH2OCH2CH2CH2OCH2CH3?
1,4-Dioxane
Ethylene glycol diethyl ether
1,3-Diethoxypropane
1,2-Diethoxyethane
1,2-Diethoxymethane
Ans: C
Topic: Nomenclature
6.
CH2CH3
CH3CH2COH
A)
B)
C)
D)
E)
The IUPAC name of compound
1,1,1-Triethylmethanol
1,1-Diethyl-1-propanol
2-Ethyl-3-pentanol
3-Ethyl-3-pentanol
tert-Heptanol
Ans: D
CH2CH3
362
is:
Chapter 11
Topic: Nomenclature
7.
A)
B)
C)
D)
E)
The correct IUPAC name for tert-butyl alcohol is:
1-Butanol
2-Methyl-1-propanol
2-Methyl-2-propanol
2-Butanol
1,1-Dimethyl-1-ethanol
Ans: C
Topic: Nomenclature
8. Select the structure of benzyl methyl ether.
I
CH3
II
CH3O
III
CH3 O CH2
IV
O
CH O CH
CH3
V
A)
B)
C)
D)
E)
CH3
CH3
CH3
CH2
O
CH2
CH3
I
II
III
IV
V
Ans: C
363
Chapter 11
Topic: Nomenclature
9.
OH
OH
A)
B)
C)
D)
E)
is properly named:
cis-1,2-Cyclopentanediol
meso-1,2-Cyclopentanediol
(1R,2R)-1,2-Cyclopentanediol
(1R,2S)-1,2-Cyclopentanediol
(1S,2S)-1,2-Cyclopentanediol
Ans: C
Topic: Nomenclature
10.
A)
B)
C)
D)
E)
2,2-Dimethyl-1-propanol has the common name:
Isoamyl alcohol
Isopentyl alcohol
tert-Pentyl alcohol
Neopentyl alcohol
2-Methylisobutyl alcohol
Ans: D
Topic: Nomenclature
11. What is the most accurate name for the molecule represented by the following Fischer
projection formula?
H
H3CH2C
A)
B)
C)
D)
E)
OCH3
CH3
sec-Butyl methyl ether
Isobutyl methyl ether
tert-Butyl methyl ether
(R)-2-Methoxybutane
(S)-2-Methoxybutane
Ans: E
364
Chapter 11
Topic: Nomenclature
12. A correct name for the following Fischer projection formula is:
OH
CH3
C
C
C
CH3
H
A)
B)
C)
D)
E)
(R)-3-Pentyn-2-ol
(S)-3-Pentyn-2-ol
(R)-2-Pentyn-4-ol
(S)-2-Pentyn-4-ol
(S)-2-Hydroxy-3-pentyne
Ans: B
Topic: Nomenclature
13.
A)
B)
C)
D)
E)
Which of these, though commonly used, is an incorrect name for CH3CHOHCH3?
Isopropyl alcohol
sec-Propyl alcohol
2-Propanol
Isopropanol
More than one of these.
Ans: D
Topic: Nomenclature
14.
A)
B)
C)
D)
E)
Which of these, though commonly used, is an incorrect name for (CH3)3COH?
tert-Butyl alcohol
tert-Butanol
2-Methyl-2-propanol
More than one is incorrect.
Each is a correct name.
Ans: B
Topic: Isomerism, Stereoisomers
15.
A)
B)
C)
D)
E)
Which of the following can be described as “optically active, primary alcohol”?
CH3CH2CH2CH2CH2OH
(CH3)2CHCH2CH2OH
CH3CH2CH(CH3)CH2OH
(CH3)2CHCHOHCH3
Two of the above
Ans: C
365
Chapter 11
Topic: Isomerism, Stereoisomers
16.
A)
B)
C)
D)
E)
What is the total number of pentyl alcohols, including stereoisomers?
7
8
9
10
11
Ans: E
Topic: Isomerism, Stereoisomers
17. The number of primary alcohols corresponding to the formula C5H12O, counting
stereoisomers separately, is:
A) 1
B) 2
C) 3
D) 4
E) 5
Ans: E
Topic: Isomerism, Stereoisomers
18. The number of optically active pentyl alcohols (C5H12OH), i.e., the total number of
individual enantiomers, is:
A) 0
B) 2
C) 3
D) 4
E) 6
Ans: E
Topic: Alcohol Reactions
19. Which of the following reactions would serve as a synthesis of butyl bromide?
reflux
A) CH CH CH CH OH
+
HBr
3
2
2
2
B) CH3CH2CH2CH2OH + PBr3 ⎯⎯⎯⎯⎯→
reflux
C) CH CH CH CH OH
+
NaBr
3
2
2
2
D) CH3CH2CH2CH2OH + Br2 ⎯⎯⎯⎯⎯→
E) Answers A) and B) only
Ans: E
366
Chapter 11
Topic: Ether Reactions
20. The product(s) of the following reaction
CH2
CH2
excess HBr
CH2
CH2
is/are:
heat
O
CH3CH2OCH2CH3
CH3CH2CH2CH2OH and CH3CH2CH2CH2Br
I
II
CH2
BrCH2CH2CH2CH2OH and BrCH2CH2CH2CH2Br
CH2
CH2
O
III
A)
B)
C)
D)
E)
IV
I
II
III
IV
None of these
Ans: C
367
CHBr
Chapter 11
Topic: Alcohol Synthesis
21. Which product(s) would you expect to obtain from the following sequence of reactions?
CH3
1. BH3-THF
?
2. H2O2, NaOH
CH3
CH3
O
OH
OH
+
enantiomer
I
A)
B)
C)
D)
E)
H3C
CH3
II
OH
+
enantiomer
+
enantiomer
III
IV
I
II
III
IV
V
Ans: C
Topic: Alcohol Synthesis
22. Which would be the best way to carry out the following synthesis?
?
CH3CH2CHCH3
CH3CH2CH2CH2OH
Br
A)
B)
C)
D)
E)
CH2OH
(1) HA, heat; (2) H3O+, H2O, heat
(1) (CH3)3COK / (CH3)3COH; (2) BH3:THF, then H2O2, OH(1) (CH3)3COK / (CH3)3COH; (2) H3O+, then H2O, heat
(1) KOH, C2H5OH; (2) BH3:THF, then H2O2, OH(1) KOH, C2H5OH; (2) HA, heat; (3) H3O+, H2O, heat
Ans: B
368
V
Chapter 11
Topic: Alcohol Synthesis
23. Select the structure of the major product formed from the following reaction.
CH3
1. Hg(OOCCH3)2
THF, H2O
?
2. NaBH4, NaOH
CH3
CH3
CH2OH
HO CH3
OH
CH3
OH
OH
I
A)
B)
C)
D)
E)
II
III
IV
I
II
III
IV
V
Ans: D
369
V
Chapter 11
Topic: Alcohol Synthesis, Isotope Labeling
24. The hydroboration-oxidation procedure can be successfully employed for synthesis of
deuterated derivatives, by using BD3 instead of BH3. What product would you expect
from the following reaction?
CH3
1. (BD3)2
?
2. H2O2/NaOH
HO
H
D
H
CH3
+
enantiomer
I
A)
B)
C)
D)
E)
H
D
BD2
CH3
+
enantiomer
II
D
D
CH3
H
+
enantiomer
III
H
OH
CH3
H
D
+
enantiomer
IV
I
II
III
IV
V
Ans: A
Topic: Ether Reactions, Isotope Labeling
25. Select the structure of the major product formed in the following reaction.
HA
CH3CH
CH2
?
18
O
H2 O
18
A) CH3CH2CH2 OH
B) CH3CHCH3
C)
18OH
CH3CHCH2OH
18OH
D) CH3CH
E)
CH2
OH 18OH
CH3CHCH218OH
18OH
Ans: C
370
BD2
CH3
+
enantiomer
V
Chapter 11
Topic: Ether Reactions
26. What would be the major product of the following reaction
Concd. HBr (xs)
C6H5CH2OCH3
A)
B)
C)
D)
E)
?
heat
C6H5Br + CH3OH
C6H5CH2Br + CH3Br
C6H5CH2OH + CH3Br
C6H5CH2Br + CH3OH
C6H5CH2CH2Br
Ans: B
Topic: Ether Synthesis
27.
A)
B)
C)
D)
E)
Which is the best way to prepare 3-methoxypentane via the Williamson method?
CH3OH + CH3CH2CHOHCH2CH3 + H2SO4, 140°C
CH3OH + (CH3)2CHCH2CH2OH + H2SO4, 140°C
CH3ONa + (CH3CH2)2CHBr
CH3I + (CH3CH2)2CHONa
CH3I + (CH3)2CHCH2CH2ONa
Ans: D
Topic: Ether Synthesis
28.
A)
B)
C)
D)
E)
Which is the best method to prepare 2-ethoxy-5methylhexane?
C2H5ONa + (CH3)2CHCH2CH2Br
C2H5ONa + (CH3)2CHCH2CH2CH2CH2Br
C2H5ONa + (CH3)2CHCH2CH2CHBrCH3
C2H5Br + (CH3)2CHCH2CH2CH(CH3)ONa
C2H5OH + (CH3)2CHCH2CH2CHOHCH3 + H2SO4, 140°C
Ans: D
Topic: Alcohol Reactions
29.
A)
B)
C)
D)
E)
Which of the following could be used to synthesize 2-bromobutane?
CH3CH2C=CH2 + Br2 (aq) ⎯⎯⎯⎯→
CH3CH2CΗΟΗCH3 + HBr ⎯⎯⎯⎯→
CH3CH2C≡CH + HBr ⎯⎯⎯⎯→
CH3CH2C≡CH + Br2 ⎯⎯⎯⎯→
More than one of the above
Ans: B
371
Chapter 11
Topic: Alcohol Reactions
30.
A)
B)
C)
D)
E)
Which of the following could not be used to synthesize 2-bromopentane efficiently?
1-Pentene + HBr Æ
2-Pentene + HBr Æ
2-Pentanol + HBr Æ
2-Pentanol + PBr3 Æ
All of the above would afford good yields of 2-bromopentane
Ans: B
Topic: Alcohol Reactions
31.
A)
B)
C)
D)
E)
Which of the following could be used to synthesize 1-bromobutane efficiently?
CH3CH2CH=CH2 + HBr ⎯⎯⎯⎯→
CH3CH2CH2CH2OH + PBr3 ⎯⎯⎯⎯→
CH3CH2CH2CH2OH + HBr ⎯⎯⎯⎯→
CH3CH2CH2CH2OH + Br2 ⎯⎯⎯⎯→
More than one of these
Ans: E
Topic: Alcohol Synthesis
32. Which of the following would be a reasonable synthesis of 2-butanol?
A)
H3O+, heat
1-Butene
1. BH3-THF
B)
1-Butene
2. H2O2, NaOH
1. Hg(OAc)2, THF, H2O
C)
1-Butene
2. NaBH4, NaOH
D) More than one of these
E) None of these
Ans: D
372
Chapter 11
Topic: Epoxide Reactions
33. What would be the final product?
O
CH3
RCOOH
CH2
H3CC
product
CH3OH, HA
final product
A) (CH3)2CHCH2OCH3
B) (CH3)2CCH3
C)
OCH3
(CH3)2CCH2OH
OCH3
D) (CH3)2CCH2OCH3
E)
OH
(CH3)2CCH2OCH3
OCH3
Ans: C
Topic: Alcohol Synthesis
34. Which of the following would be a reasonable synthesis of CH3CH2CH2CH2OH?
A)
H3O+, heat
1-Butene
1. BH3-THF
B)
1-Butene
2. H2O2, NaOH
1. Hg(OAc)2, THF, H2O
C)
1-Butene
2. NaBH4, NaOH
D) More than one of these
E) None of these
Ans: B
Topic: Alcohol Synthesis
35. Which would be the best method for converting 3,3-dimethyl-1-pentene into 3,3dimethyl-2-pentanol?
A) H3O+, heat
B) BH3:THF; then H2O2, OHC) concd. H2SO4; then H2O, heat
D) Hg(OAc)2/THF-H2O; then NaBH4,OHE) HBr; then NaOH/H2O
Ans: D
373
Chapter 11
Topic: Alcohol Reactions
36. trans-3-Methylcyclopentanol is treated with CH3SO2Cl in the presence of base. The
product of this reaction is then heated with KI in methanol. What is the final product?
A) trans-1-Iodo-3-methylcyclopentane
B) cis-1-Iodo-3-methylcyclopentane
C) 1-Methylcyclopentene
D) 2-Methylcyclopentene
E) 3-Methylcyclopentene
Ans: B
Topic: Ether Synthesis
37. Which method would provide the best synthesis of ethyl isopropyl ether?
A) (CH3)2CHONa + CH3CH2Br ⎯⎯⎯⎯⎯→
B) CH3CH2ONa + (CH3)2CHBr ⎯⎯⎯⎯⎯→
C)
H2SO4, 140 oC
CH3CH2OH
+ (CH3)2CHOH
D)
H2SO4, 180 oC
CH3CH2OH
+ (CH3)2CHOH
E) CH3CH2ONa + (CH3)2CHOH ⎯⎯⎯⎯⎯→
Ans: A
Topic: Alcohol Reactions
38.
A)
B)
C)
D)
E)
Which of the following could be used to synthesize 1-bromopentane?
CH3CH2CH2CH=CH2 + HBr ⎯⎯⎯⎯→
CH3CH2CH2CH2CH2OH + PBr3 ⎯⎯⎯⎯→
CH3CH2CH2CH2CH2OH + NaBr ⎯⎯⎯⎯→
CH3CH2CH2CH2CH2OH + Br2 ⎯⎯⎯⎯→
CH3CH2CH2CH=CH2 + Br2 ⎯⎯⎯⎯→
Ans: B
374
Chapter 11
Topic: Epoxide Reactions
39. Which compound (or compounds) would be produced when trans-2-butene is treated
first with a peroxy acid to form an epoxide, and then the epoxide is subjected to acidcatalyzed hydrolysis?
H
HO
CH3
CH3
CH3
H
H
OH
C
C
C
C
C
C
H
CH3
OH
A)
B)
C)
D)
E)
HO
HO
CH3
H
H3C
H
OH
I
II
An equimolar mixture of I and II
An equimolar mixture of II and III
I alone
II alone
III alone
Ans: E
III
Topic: Ether Reactions
40.
A)
B)
C)
D)
E)
Heating 2-ethoxyhexane with excess concentrated HBr would produce:
CH3CH2OCH2CH2CH2CH2CH2CH2Br
BrCH2CH2OCH2CH2CH2CH2CH2CH3
CH3CH2OH and CH3CH2CH2CH2CHBrCH3
CH3CH2Br and CH3CH2CHOHCH2CH2CH3
CH3CH2Br and CH3CH2CH2CH2CHBrCH3
Ans: E
Topic: Ether Synthesis
41.
A)
B)
C)
D)
E)
Which is the best method for the synthesis of tert-butyl methyl ether?
CH3ONa + (CH3)3CBr ⎯⎯⎯⎯→
(CH3)3CONa + CH3I ⎯⎯⎯⎯→
CH3OH + (CH3)3COH + H2SO4 at 140° C ⎯⎯⎯⎯→
(CH3)3CONa + CH3OCH3 ⎯⎯⎯⎯→
CH3ONa + (CH3)3COH ⎯⎯⎯⎯→
Ans: B
375
Chapter 11
Topic: Alcohol Reactions
42. What would be the major product of the following reaction?
H
OH
CH3SO2Cl
CH3
base
mesylate
NaI
?
ethanol
H
H
I
I
CH3
A)
B)
C)
D)
E)
H
H
H
OSO2I
I
CH3
CH3
CH3
H
H
I
H
I
II
III
IV
I
II
III
IV
An equimolar mixture of I and II
Ans: B
Topic: Alcohol Reactions
43. cis-3-Methylcyclopentanol is treated with CH3SO2Cl in the presence of a base. The
product of the reaction then is allowed to react with KI in methanol. What is the final
product?
A) trans-1-Iodo-3-methylcyclopentane
B) cis-1-Iodo-3-methylcyclopentane
C) 1-Methylcyclopentene
D) 2-Methylcyclopentene
E) 3-Methylcyclopentene
Ans: A
376
Chapter 11
Topic: Alcohol Reactions
44. The major product of the following reaction would be:
CH3
H
OH
CH3SO2Cl
Product
base
CH3CO2-
?
C2H5
CH3
H3CCO2
H
C2H5
I
A)
B)
C)
D)
E)
CH3
CH3
H
O2CCH3
H3CCO2
OSO2CH3
C2H5
C2H5
III
II
I
II
III
Equal amounts of I and II
None of these
Ans: A
377
Chapter 11
Topic: Alcohol Synthesis
45. Which would be the major product of the reaction shown?
CH2CH3
1. Hg(OAc)2, THF, H2O
?
2. NaBH4, NaOH
H
OH
OHCH2CH3
A)
B)
C)
D)
E)
H
H
CH2CH3
H
CH2CH3
H CH2CH3
H CH2CH2OH
H
H
H
OH
H
I
II
III
IV
V
I
II
III
IV
V
Ans: B
Topic: Alcohol Reactions
46.
A)
B)
C)
D)
E)
O
Which reagent(s) would transform propyl alcohol into propyl bromide?
Concd. HBr and heat
PBr3
NaBr/H2O and heat
More than one of these
All of these
Ans: D
378
Chapter 11
Topic: Epoxide Reactions
47. Epoxidation followed by reaction with aqueous base converts cyclopentene into which
of these?
H
OH
I
A)
B)
C)
D)
E)
H
H
OH
H
OH
OH
OH
H
H
OH
II
III
IV
OH
H
I
II
III
IV
Equal amounts of III and IV
Ans: E
Topic: Ether Synthesis
48. When 3-methyl-2-pentene is treated with mercuric acetate, Hg(O2CCH3)2, in a THFethanol mixture and the resulting product reacted with NaBH4 in basic solution, the
principal product formed is which of these?
A) 3-methyl-3-pentanol
B) 3-ethoxy-3-methylpentane
C) 3-methyl-2-pentanol
D) 2-ethoxy-3-methylpentane
E) 1-ethoxy-3-methylpentane
Ans: B
Topic: Alcohol Synthesis
49. Anti-Markovnikov hydration of the carbon-carbon double bond occurs when an alkene
reacts with:
A) BH3:THF; then H2O2/OHB) BH3:THF; then CH3COOH
C) Hg(OAc)2, THF, H2O; then NaBH4, OHD) Hg(OAc)2, THF, CH3OH; then NaBH4, OHE) Hg(OAc)2, THF, H2O; then BH3:THF
Ans: A
379
Chapter 11
Topic: Alcohol Reactions
CH3
50.
H3C
A)
B)
C)
D)
E)
CH
CH3
CH
CH3
to
H3C
CH
CH
CH3
OH
Br
is best
The conversion of
achieved through use of which of these reagents in a low temperature reaction?
Concd. HBr
Br2
NaBr, H2SO4
PBr3
HBr, peroxide
Ans: D
Topic: Alcohol Reactions
51. The conversion of 3-methyl-1-octanol to 1-chloro-3-methyloctane is best achieved
through use of which of these reagents?
A) Concd. HCl
B) SO2Cl2
C) NaCl, H2SO4
D) PCl3
E) SOCl2
Ans: D
Topic: Alcohol Reactions
52. What is the product of the reaction of propyl alcohol with (CH3)3SiCl in the presence of
a tertiary amine?
A) CH3CH2CH2Si(CH3)3
B) (CH3)2CHSi(CH3)3
C) CH3CH2CH2OSi(CH3)3
D) (CH3)2CHOSi(CH3)3
E) (CH3CH2CH2)3SiOH
Ans: C
380
Chapter 11
Topic: Alcohol Synthesis
53. What is the major product of the reaction:
i) BH3, THF
ii) H2O2,NaOH
CH3
CH3
?
CH3
OH
+
enantiomer
I
A)
B)
C)
D)
E)
II
CH3
OH
OH
III
+
enantiomer
IV
I
II
III
IV
Both III and IV
Ans: D
Topic: Alcohol Synthesis
54. What is the major product of the reaction when (R)-3-methylcyclohexene is subjected to
the hydroboration-oxidation sequence?
A) (1R,2R)-2-methylcyclohexanol + (1S,2R)-2-methylcyclohexanol
B) (1S,2R)-2-methylcyclohexanol + (1S,2S)-2-methylcyclohexanol
C) (1R,3R)-3-methylcyclohexanol + (1S,3R)-3-methylcyclohexanol
D) (1S,3R)-3-methylcyclohexanol (1S,3S)-3-methylcyclohexanol
E) Two of the above
Ans: E
381
Chapter 11
Topic: Alcohol Synthesis
55. Oxymercuration-demercuration of 3-methylcyclopentene produces this/these product(s):
HO CH3
CH2OH
CH3
CH3
OH
OH
I
A)
B)
C)
D)
E)
II
III
IV
I
II
III
IV
Both III and IV
Ans: E
382
Chapter 11
Topic: Alcohol Reactions
O
56.
CH3
S
OH,
O
Methanesulfonic acid,
is treated, in turn, with PCl5 and (R)-2-butanol.
Which of the following Fischer formulas is a stereochemically correct representation of
the final product?
CH3
O
CH3
S
O
C
H
CH3
CH2
O
H
O
S
O
C
CH3
CH3
CH2
O
CH3
S
C
O
CH2
CH3
II
O
H
S
C
O
CH2
CH3
III
CH3
O
CH3
S
C
O
CH2
CH3
CH3
IV
A)
B)
C)
D)
E)
CH3
CH3
I
CH3
O
V
I
II
III
IV
V
Ans: A
383
Cl
H
Chapter 11
Topic: Epoxide Reactions
57. cis-3-Hexene is treated with magnesium monoperoxyphthalate and the product is then
subjected to acid-catalyzed hydrolysis. What is the final product?
H
H
A)
B)
C)
D)
E)
CH2CH3
H
OH
CH2CH3
CH2CH3
H
H
HO
CH2CH3
H
CH2CH3
CH2CH3
OH
H
HO
H
OH
HO
H
CH2CH3
CH2CH3
H
II
I
equal amounts I and II
equal amounts I, II and V
equal amounts III, IV and V
equal amounts, III and IV
Only V
Ans: D
III
IV
H
H
CH2CH3
OH
OH
CH2CH3
V
Topic: Chemical Tests
58. Which of the following reagents might serve as the basis for a simple chemical test that
would distinguish between pure 1-butene and CH3CH2CH2CH2OH?
A) Bromine in carbon tetrachloride
B) Dilute aqueous potassium permanganate
C) Chromic oxide in aqueous sulfuric acid
D) All of these
E) None of these
Ans: D
Topic: Chemical Tests
59. Which of the reagents listed below would serve as the basis for a simple chemical test to
distinguish between
and
?
O
A)
B)
C)
D)
E)
AgNO3 in alcohol
NaOH in H2O
Br2 in CCl4
Cold concd. H2SO4
KMnO4 in H2O
Ans: D
384
Chapter 11
Topic: Chemical Tests
60. Which of the reagents listed below would serve as the basis for a simple chemical test to
distinguish between
?
and
O
A)
B)
C)
D)
E)
Br
AgNO3 in C2H5OH
Dilute HCl
Br2 in CCl4
NaOH in H2O
KMnO4 in H2O
Ans: A
Topic: Physical Properties, Comparison
61. Which of the compounds listed below would you expect to have the highest boiling
point? (They all have approximately the same molecular weight.)
A) CH3CH2CH2CH2CH3
B) CH3CH2CH2CH2OH
C) CH3CH2CH2OCH3
D) CH3CH2CH2Cl
E) CH3CH2OCH2CH3
Ans: B
Topic: Physical Properties, Comparison
62.
A)
B)
C)
D)
E)
Which compound would have the highest boiling point?
CH3CH2CH2CH3
CH3CH2OCH3
CH3CH2CH2OH
(CH3)2CHOH
HOCH2CH2OH
Ans: E
Topic: Physical Properties, Comparison
63.
A)
B)
C)
D)
E)
Which compound would have the lowest solubility in water?
Diethyl ether
Methyl propyl ether
1-Butanol
2-Butanol
Pentane
Ans: E
385
Chapter 11
Topic: Physical Properties, Comparison
64. Which compound would have the lowest boiling point?
O
I
A)
B)
C)
D)
E)
CH2OH
OH
OH
II
III
IV
OH
V
I
II
III
IV
V
Ans: A
Topic: Physical Properties, Comparison
65. Which compound would have the highest boiling point?
O
O
O
O
O
OH
I
II
III
OH
O
O
IV
A)
B)
C)
D)
E)
OH
V
I
II
III
IV
V
Ans: E
Topic: Alcohol Reaction Rates
66.
A)
B)
C)
D)
E)
Which of the alcohols listed below would you expect to react most rapidly with HBr?
CH3CH2CH2CH2CH2CH2OH
(CH3CH2)2CH2CH2OH
(CH3CH2)2CHOHCH3
CH3CH2CH2CH2CH2OH
(CH3CH2)2C(CH3)OH
Ans: E
386
Chapter 11
Topic: Alcohol Reaction Rates
67.
A)
B)
C)
D)
E)
Which alcohol would undergo acid-catalyzed dehydration most rapidly?
3,3-dimethyl-1-butanol
2,2-dimethyl-1-butanol
3,3-dimethyl-2-butanol
2-methyl-2-butanol
All would undergo dehydration equally rapidly.
Ans: D
Topic: Ether Reaction Rates
68. Which of these ethers is most resistant to peroxide formation on exposure to
atmospheric oxygen?
A) CH3OCH2CH3
B) CH3CH2OCH2CH3
C) (CH3)2CHOCH(CH3)2
D) (CH3)2CHOCH2CH3
E) CH3OC(CH3)3
Ans: E
Topic: Ether Reaction Rates
69. Which of these ethers is least likely to undergo significant cleavage by hot aqueous
H2SO4 ?
A)
B)
C)
D)
E)
CH3OCH3
CH3OCH(CH3)2
(CH3)2CHOCH(CH3)2
I
II
III
(CH3)3COC(CH3)3
O
IV
V
I
II
III
IV
V
Ans: A
387
Chapter 11
Topic: Alcohol Reaction Rates
70.
A)
B)
C)
D)
E)
Which of these alkyl halide syntheses is predicted to occur at the greatest rate?
CH3CH2CH2CH2OH + HI ⎯⎯⎯⎯→
(CH3)2CHCH2OH + HBr ⎯⎯⎯⎯→
CH3CHOHCH2CH3 + HCl ⎯⎯⎯⎯→
CH3CHOHCH2CH3 + HBr ⎯⎯⎯⎯→
(CH3)3COH + HI ⎯⎯⎯⎯→
Ans: E
Topic: Reaction Mechanisms
71. The following reaction,
HBr
CH3CH2CH2CH2OH
A)
B)
C)
D)
E)
CH3CH2CH2CH2Br
heat
+
H2 O
is probably:
An SN1-type reaction involving the protonated alcohol as the substrate.
An SN2-type reaction involving the protonated alcohol as the substrate.
An E1-type reaction involving the protonated alcohol as the substrate.
An E2-type reaction involving the protonated alcohol as the substrate.
An epoxidation reaction.
Ans: B
Topic: Reaction Mechanisms
72. The reaction between 1-pentanol and HBr to yield 1-bromopentanol is probably:
A)
B)
C)
D)
E)
an SN1-type reaction involving the protonated alcohol as the substrate.
an SN2-type reaction involving the protonated alcohol as the substrate.
an E1-type reaction involving the protonated alcohol as the substrate.
an E2-type reaction involving the protonated alcohol as the substrate.
an epoxidation reaction.
Ans: B
Topic: Reaction Mechanisms
73. The reaction between 2-methyl-2-pentanol and HBr to yield 2-bromo-2-methylpentane
is probably:
A) an SN1-type reaction involving the protonated alcohol as the substrate.
B) an SN2-type reaction involving the protonated alcohol as the substrate.
C) an E1-type reaction involving the protonated alcohol as the substrate.
D) an E2-type reaction involving the protonated alcohol as the substrate.
E) an epoxidation reaction.
Ans: A
388
Chapter 11
Topic: Reaction Mechanisms
74. The reaction between 2-methyl-2-pentanol and HBr to yield 2-methyl-2-pentene is
probably:
A) an SN1-type reaction involving the protonated alcohol as the substrate.
B) an SN2-type reaction involving the protonated alcohol as the substrate.
C) an E1-type reaction involving the protonated alcohol as the substrate.
D) an E2-type reaction involving the protonated alcohol as the substrate.
E) an epoxidation reaction.
Ans: C
Topic: Reaction Mechanisms
75. The reaction between 4-methyl-1-pentanol and HBr to yield 4-methyl-1-pentene is
probably:
A) an SN1-type reaction involving the protonated alcohol as the substrate.
B) an SN2-type reaction involving the protonated alcohol as the substrate.
C) an E1-type reaction involving the protonated alcohol as the substrate.
D) an E2-type reaction involving the protonated alcohol as the substrate.
E) an epoxidation reaction.
Ans: D
Topic: Reaction Mechanisms
76. Which statement is true concerning the formation of alcohols by the hydroborationoxidation sequence?
A) Overall, the process results in syn addition and anti-Markovnikov orientation.
B) Overall, the process results in anti addition and anti-Markovnikov orientation.
C) Overall, the process results in syn addition and Markovnikov orientation.
D) Overall, the process results in anti addition and Markovnikov orientation.
E) The stereochemistry and orientation are unpredictable.
Ans: A
389
Chapter 11
Topic: Reaction Mechanisms
77. What is the electrophilic species involved in the initial step of the reaction below?
Hg(OAc)2
OH
HgOAc
THF, H2O
A)
B)
C)
D)
E)
+
OH
HgOAc
H3O+
THF
the THF/H2O complex
Ans: B
+
Topic: Reaction Mechanisms
78. The following reaction,
2 CH3CH2CH2CH2OH
A)
B)
C)
D)
E)
H2SO4
(CH3CH2CH2CH2)2O
heat
is probably:
an SN1-type reaction involving the protonated alcohol as the substrate.
an SN2-type reaction involving the protonated alcohol as the substrate.
an E1-type reaction involving the protonated alcohol as the substrate.
an E2-type reaction involving the protonated alcohol as the substrate.
an epoxidation reaction.
Ans: B
Topic: Stereoisomerism
79. What is the relationship between alcohols I and II?
H
H
CH3
OH
H
A)
B)
C)
D)
E)
H
CH3
OH
I
II
They are:
different conformations of the same compound.
constitutional isomers.
enantiomers.
diastereomers.
identical.
Ans: D
390
+
H2O
Chapter 11
Topic: Stereoisomerism
80. What is the relationship between alcohols I and II?
H
CH3
CH3
OH
H
A)
B)
C)
D)
E)
H
H
OH
I
II
They are:
different conformations of the same compound.
constitutional isomers.
enantiomers.
diastereomers.
identical.
Ans: C
Topic: Ether Reactivity
81. Which of the following statements is NOT true of ethers?
A) Ethers are generally unreactive molecules toward reagents other than strong acids.
B) Ethers generally have lower boiling points than alcohols of a corresponding molecular
weight.
C) Ethers generally have much lower water solubilities than alcohols with a corresponding
molecular weight.
D) Ethers can generally be cleaved by heating them with strong acids.
E) Ethers form peroxides when allowed to stand in the presence of oxygen.
Ans: C
391
Chapter 11
Topic: General
82. Which compound is a tosylate?
O
CH3
O
S O CH3
Br
O
O
S O CH2 CH3
CH3
S O CH3
O
I
II
III
O
O
CH3
S CH3
CH3
O
S
CH3
O
IV
A)
B)
C)
D)
E)
V
I
II
III
IV
V
Ans: A
Topic: General
83.
A)
B)
C)
D)
E)
The major industrial process in use today for the production of methanol is the:
hydration of ethyne.
distillation of wood.
hydrogenation of carbon dioxide.
reduction of methanal.
catalytic reduction of carbon monoxide.
Ans: E
Topic: General
84.
A)
B)
C)
D)
E)
Today, most industrial ethanol is made in the U.S. by the:
fermentation of grain.
hydrolysis of ethyl bromide.
hydration of ethylene.
reduction of acetaldehyde.
hydration of acetylene.
Ans: C
392
Chapter 11
Topic: Ether Reactivity
85. Long-term storage of ethers can be dangerous, because most ethers react slowly with
oxygen by a radical process called __________________. This process forms peroxides
and hydroperoxides which are dangerously _______________.
Ans: autooxidation, explosive
Topic: General, Alcohol Synthesis
86. We have learned three different methods to convert alkenes into alcohols.
a. The method that takes place with Markovnikov regioselectivity but is prone to
rearrangement is ________________.
b. The method that takes place with Markovnikov regioselectivity and does not lead to
rearrangement is _____________.
c. The method that takes place with anti-Markovnikov regioselectivity and syn
stereoselectivity is _______________.
Ans: a. acid-catalyzed hydration; b. oxymercuration-demercuration; c. hydroborationoxidation
Topic: General, Alcohol Reactions
87. Reactions of alcohols can be broadly categorized into three types.
a. Those that utilize the hydroxyl oxygen as a ____________.
b. Those that utilize the hydroxyl proton as an _________.
c. Those that convert the hydroxyl group into a _____________.
Ans: a nucleophile or base; b. acid; c. leaving group
Topic: General
88. Stereochemically speaking, conversion of an alcohol into a tosylate occurs with
_____________ of configuration.
Ans: retention
Topic: General
89. When an alcohol in which the OH is attached to a stereogenic carbon reacts with thionyl
chloride (SOCl2) in the presence of a 3° amine, the resulting alkyl chloride is produced
with _____________ of configuration.
Ans: inversion
Topic: General
90. The SN2 reaction between an alkoxide and an alkyl halide is commonly referred to as
the _________________ synthesis.
Ans: Williamson
393
Chapter 11
Topic: General
91. Three-membered rings containing oxygen are called _______________ or
______________.
Ans: oxiranes; epoxides
Topic: General
92. The development of a procedure called __________________ has made the use of
nonpolar solvents possible in reactions involving polar reagents.
Ans: phase transfer catalysis
Topic: General
93. The relationship between a crown ether and the ion it transports is known as the
____________________ relationship.
Ans: host-guest
Topic: General
94. A compound or ion that prefers a nonpolar environment to an aqueous one is said to be
______________.
Ans: lipophilic
Topic: Isomerism
95. Draw structures for all possible ethers having the formula C4H10O
O
Ans:
O
O
C4H10O: ethers
Topic: Ether Synthesis
96. Which is the most efficient way to prepare isopropyl methyl ether via the Williamson
method?
Ans: CH3I + (CH3)2CHONa
Topic: Ether Synthesis
97. Which is the best method to prepare ethoxycyclopentane via the Williamson method?
Ans:
O
ONa
+
I
394
Chapter 11
Topic: Alcohol Synthesis
98. Draw the structures of all the products formed when (3R)-3-methylcyclopentene is
subjected to the hydroboration-oxidation sequence.
Ans: The alkene is symmetrically substituted: thus, Markovnikov rule does not apply.
Hence, all four products shown below are likely to be formed in roughly equal
proportions.
H3C
H
hydroboration-oxidation
(3R)-3-methylcyclopentene
H3C
H3C
H
H3C
H
H3C
H
H
OH
OH
+
+
+
OH
OH
Topic: Ether Synthesis
99. Provide a reasonable synthetic strategy for the synthesis of trans-1,2-cyclohexanediol
from bromocyclohexane
OH
Ans:
Br CH ONa
MMPP
H3O+
3
O
CH3OH
OH
heat
+
enantiomer
Topic: Nomenclature
100. Give the correct IUPAC name corresponding to the following structure:
OH
Ans: 6-cyclohexyl-2,7-dimethyl-3-octanol
395
Chapter 11
Topic: Nomenclature
101. Give the correct IUPAC name corresponding to the following structure:
OH
Ans: (R)-3-ethyl-2-methyl-3-hexanol
Topic: Nomenclature
102. Give the correct IUPAC name corresponding to the following structure:
O
O
Ans: 1,3-diethoxyhexane
Topic: Nomenclature
103. Give the correct IUPAC name corresponding to the following structure:
Br
H
CH3
OH
Ans: (1R,2R)-2-bromo-1-methylcyclopentanol
Topic: Multistep Synthetic Strategy
104. Provide a reasonable synthetic strategy for the synthesis of a racemic mixture of
(1R,2R) and (1S,2S)-2-bromo-1-methylcyclopentanol from methylcyclohexane.
CH3
Ans:
CH3 Br
CH3
2
CH3ONa
Br
CH3OH
hν
heat
Br2, H2O
Br
H
Br
CH3
OH
+
H
CH3
OH
(1R,2R)
(1S,2S)
2-bromo-1-methylcyclopentanol
396
Chapter 11
Topic: Multistep Reactions
105. Complete the following reaction sequence, giving structural details of all key
intermediates:
i) Li, NH3
?
ii) KMnO4 ,OH−, H2O
Ans:
iii) H3O+
Li
NH3
i) KMnO4 ,OH−, H2O
ii) H3O+
H
H
OH
OH
+
HO
H
HO
2S,3R
H
2R,3S
Topic: Multistep Reactions
106. Complete the following reaction sequence, giving structural details of all key
intermediates:
i) H2, Lindlar catalyst
?
ii) KMnO4 ,OH−, H2O
iii) H3O+
Ans:
H2
Lindlar
catalyst
−
H
OH
i) KMnO4 ,OH , H2O
ii) H3O+
HO
H
meso
397
Chapter 11
Topic: Multistep Reactions
107. Complete the following reaction sequence, giving structural details of all key
intermediates:
i) Li, NH3
?
ii) MMPP
Ans:
iii) H3O+, H2O
Li
NH3
i) MMPP
ii) H3O+, H2O
H
H
OH
OH
+
H
OH
H
2S, 3S
OH
2R, 3R
398
Chapter 12
MULTIPLE CHOICE QUESTIONS
Topic: General Carbonyl Information
1. Which of the following resonance structures is not a significant contributor to the hybrid
for the carbonyl group?
C
O
I
A)
B)
C)
D)
E)
C
O
C
II
O
III
I
II
III
Neither II nor III is important.
All are significant contributors.
Ans: B
Topic: General Carbonyl Information
2. When nucleophilic addition to a carbonyl group occurs, the carbon attacked undergoes
this hybridization change:
A) sp2 ⎯⎯⎯⎯→ sp3
B) sp ⎯⎯⎯⎯→ sp2
C) sp ⎯⎯⎯⎯→ sp3
D) sp3 ⎯⎯⎯⎯→ sp2
E) sp2 ⎯⎯⎯⎯→ sp
Ans: A
Topic: Functional Group Tests
3.
A)
B)
C)
D)
E)
Which reagent(s) will distinguish between cyclopentanol and cyclopentane?
Br2/CCl4
KMnO4 (cold)
CrO3/aqueous H2SO4
NaOH (aq)
A) and B)
Ans: C
Page 399
Chapter 12
Topic: Functional Group Tests
4. Which reagent(s) will distinguish between 2-methylcyclopentanol and 1methylcyclopentanol?
A) Br2/CCl4
B) KMnO4
C) CrO3/aqueous H2SO4
D) NaOH (aq)
E) B) and C)
Ans: E
Topic: Functional Group Tests
5. Which of the following reagents might serve as the basis for a simple chemical test that
would distinguish between CH3CH=CHCH3 and CH3CHOHCH2CH3?
A) CrO3 in H2SO4
B) KMnO4
C) Br2 in CCl4
D) Two of these
E) All of these
Ans: D
Topic: Functional Group Tests
6. Which of the reagents listed below would serve as the basis for a simple chemical test to
distinguish between CH3CH=CHCH2OH and CH3CH2CH2CH2OH ?
A) CrO3 in H2SO4
B) Cold conc. H2SO4
C) Br2 in CCl4
D) NaOH/H2O
E) NaBH4
Ans: C
Topic: Functional Group Tests
7. Which of the reagents listed below would serve as the basis for a simple chemical test to
distinguish between (CH3)2C=CHCH2CH2OH and (CH3)2CHCH2CH2CH2OH ?
A) CrO3 in H2SO4
B) Cold conc. H2SO4
C) Br2 in CCl4
D) NaOH/H2O
E) NaBH4
Ans: C
400
Chapter 12
Topic: Functional Group Tests
8. Which of the reagents/techniques listed below would serve as the basis for a simple
chemical test to distinguish between 2-methyl-3-hexyn-1-ol and 2-methyl-5-hexyn-1-ol?
A) CrO3 in H2SO4
B) Cold conc. H2SO4
C) Br2 in CCl4
D) NaBH4
E) IR spectroscopy
Ans: E
Topic: Functional Group Tests
9. Which of the reagents/techniques listed below would serve as the basis for a simple
chemical test to distinguish between 2-methyl-3-hexyn-2-ol and 2-methyl-5-hexyn-1-ol?
A) CrO3 in H2SO4
B) IR spectroscopy
C) Br2 in CCl4
D) A) and B)
E) All of the above
Ans: D
Topic: Functional Group Tests
10. Which of the reagents/techniques listed below would serve as the basis for a simple
chemical test to distinguish between CH3CH2CH=CHCH2CH2CHO and
CH3CH2CH=CHCH2CH2CH2OH?
A) KMnO4
B) IR spectroscopy
C) Br2 in CCl4
D) A) and B)
E) All of the above
Ans: B
Topic: Functional Group Tests
11. Which of the reagents listed below would serve as the basis for a simple chemical test to
distinguish between (CH3)3COH and (CH3)2CHCH2OH ?
A) NaH
B) NaOH/H2O
C) Br2 in CCl4
D) Cold conc. H2SO4
E) CrO3 in H2SO4
Ans: E
401
Chapter 12
Topic: Functional Group Tests
12. CrO3 in H2SO4/H2O will fail to give a positive test with which of these compounds?
A) CH3CH2CH2CH2OH
B) CH3CHCH2CH3
OH
C) (CH3)3COH
H
D)
CH3CH2CH2C
O
E) More than one of these
Ans: C
Topic: Functional Group Tests
13.
A)
B)
C)
D)
E)
CH3
Consider the molecule CH3CHC CHCH2OH.
Which reagent will not give a positive test with this compound?
Cold conc. H2SO4
Br2/CCl4
NaOH (aq)
CrO3/H2SO4
Dilute KMnO4/H2O
Ans: C
402
Chapter 12
Topic: Reactions and Reaction Sequences
14. The final product, D, in the following reaction sequence,
CH2
CH2
CH3CHOH
PBr3
CH3
A
Mg
ether
A) CH3CHOCH2CH2OH
B)
C)
CH3
CH3CHCH2CH2OH
D)
CH3
CH3CHOCH2CH3
E)
CH3
CH3CHCH2CH3
C
H3O+
D
,
would be?
CH3
CH3CHCH2CH2Br
B
O
CH3
Ans: C
403
Chapter 12
Topic: Reactions and Reaction Sequences
15. What is the product, A, that would be obtained from the following reaction sequence?
O
C CH
CH3CH2MgBr
CH3
C CCH2CH3
C CCH
I
CH3CH
C CCH
A
CH3
CHOCH2CH3
CH3
CH3CH2
II
CH3
C CCH
III
CH3
CH3
CHOH
C C
IV
A)
B)
C)
D)
E)
H3O+
CHCH3
O
V
I
II
III
IV
V
Ans: D
404
CHCH2CH3
CH3
CHOH
Chapter 12
Topic: Reactions and Reaction Sequences
16. What is the product, A, that would be obtained from the following reaction sequence?
CH2
CH2
CH3CH2MgBr
C CH
C CCH2CH3
C CCH2CH2OCH2CH3
I
H3O+
O
A
CH3CH2
II
C CCH2CH2OH
III
CH3
C CCH2CH2OCH3
C COCHCH2CH3
IV
A)
B)
C)
D)
E)
V
I
II
III
IV
V
Ans: C
Topic: Reactions and Reaction Sequences
17. What would be the product, C, of the following reaction sequence?
CH3
CH3
Li
CH3CCH2Br
ether
CuI
A
B
CH3
A)
B)
C)
D)
E)
2,6-Dimethylheptane
2,2-Dimethylpropane
2-Methylpentane
2,2,5-Trimethylhexane
2,2,6-Trimethylheptane
Ans: E
405
CH3CHCH2CH2Br
C
Chapter 12
Topic: Reactions and Reaction Sequences
18. What would be the product, O, of the following reaction sequence?
CH3CH2CHCH3
Mg
ether
Br
D2O
N
O
A) CH3CH2CH2CH3
B) CH3CH2CHCH3
C)
D
CH3CH2CHCH3
OD
D) CH3CH2CH2CH2OD
E) CH3CH2CH2CH2D
Ans: B
Topic: Reactions and Reaction Sequences
19. What is the product, A, that would be obtained from the following reaction sequence?
OH
PBr3
Li
NH4Cl
(CH3)2C=O
10oC
A
OH
OH
I
II
III
HO
IV
A)
B)
C)
D)
E)
V
I
II
III
IV
V
Ans: D
406
Chapter 12
Topic: Reactions and Reaction Sequences
20. What is the product, A, that would be obtained from the following reaction sequence?
OH
PBr3
H3O+
heat
(CH3)2C=O
Mg, Et2O
A
OH
OH
I
II
III
HO
IV
A)
B)
C)
D)
E)
V
I
II
III
IV
V
Ans: C
Topic: Reactions and Reaction Sequences
21. What would be the final product, A, in the following reaction sequence?
O
OH
PBr3
Mg, Et2O
OH
O
I
II
Br
III
O
IV
V
I
II
III
IV
V
Ans: C
407
A
OH
OH
A)
B)
C)
D)
E)
H3O+
heat
Chapter 12
Topic: Reactions and Reaction Sequences
22. What is the final product of the following reaction sequence?
O
i)
Mg, Et2O
CH3I
H2CrO4
acetone
+
ii) H3O
A
O
OH
II
III
O
O
I
OH
O
IV
A)
B)
C)
D)
E)
V
I
II
III
IV
V
Ans: E
408
Chapter 12
Topic: Reactions and Reaction Sequences
23. What would be the alkane C that is the product of the following reaction sequence?
CH3
CH3CHBr
A)
CH3
CH3
Li
ether
CuI
A
B
CH3CHCHCH2CH3
B)
CH3
CH3CH
CHCH2CH3
CH3
C)
CH3
CH3
CH3CHCH2CHCH3
CH3
CH3
D)
CH3CH
CH3
CCH3
CH3
CH3
E)
CH3CH2CH2CH2CHCH3
Ans: C
409
CH3CHCH2Br
C
Chapter 12
Topic: Reactions and Reaction Sequences
24. What would be the major product of the following reaction?
O
O
i) NaBH4
ii) ethylene oxide, HA
OH
I
HO
HO
O
O
O
III
II
O
A
O
OH
OH
IV
A)
B)
C)
D)
E)
V
I
II
III
IV
V
Ans: A
Topic: Reactions and Reaction Sequences
25. What would be the major product of the following reaction?
O
i) NaBH4
ii) NaH, Et2O
iii)
O
S
A
O
OCH2CH3
A)
B)
C)
D)
E)
CH3CH2OCH(CH3)CH2CH2CH2CH3
(CH3CH2O)2CHCHOHCH2CH2CH3
(CH3CH2)2CHOHCH2CH2CHOHCH3
CH3OCH(C2H5)CH2CH2CH2CH3
CH3CH2CH(OCH3)CH2CH2CHOHCH3
Ans: A
410
OH
Chapter 12
Topic: Reactions and Reaction Sequences
26. What would be the major product of the following reaction?
Cl
A)
B)
C)
D)
E)
Li, Et2O
CuI
H
Br
CH3
A
(R)-3-ethyl-5-methylheptane
(R,S)-3-ethyl-5-methylheptane
(S)-3-ethyl-5-methylheptane
(3R,5S)-5-ethyl-3-methylheptane
(3S,5R)-5-ethyl-3-methylheptane
Ans: C
Topic: Reactions and Reaction Sequences
27. What is the principal product(s) formed when 1 mol of methylmagnesium iodide reacts
with 1 mol of p-hydroxyacetophenone ?
O
CH3MgI
A
HO
O
O
OMgI
+ CH4
IMgO
HO
O
II
I
III
O
O
O
MgI
IMgO
IV
A)
B)
C)
D)
E)
V
I
II
III
IV
V
Ans: A
411
Chapter 12
Topic: Reactions and Reaction Sequences
28. What is (are) the principal product(s) formed when 1 mol of ethylmagnesium bromide
reacts with 1 mol of 3-(N-methylamino)cyclopentanone ?
H
N
O
H3C
CH3
BrMgN
O
+ CH4
CH3CH2MgBr
H
N
H
N
CH2CH3
OMgBr
H3C
H
N
CH2CH3
N
OMgBr
O
H3C
IV
V
I
II
III
IV
V
Ans: A
Topic: Reactions and Reaction Sequences
29. What would be the product, A, of the following reaction ?
i) NaBD4
A
ii) H2O
O
A)
OD
B)
D
OH
C)
D
OD
D)
OH
E) D
O
Ans: B
412
CH2CH3
III
H3CH2C
H3C
O
H3C
II
I
A)
B)
C)
D)
E)
A
Chapter 12
Topic: Reactions and Reaction Sequences
30. What is the predominant product from the reaction of 2-hexanol with H2CrO4?
A) CH3CO2H
B) CH3(CH2)3CO2H
O
C)
CH3(CH2)3CCH3
D) CH3(CH2)4CO2H
E) A) and B)
Ans: C
Topic: Synthetic Strategy, Redox
31. Which of the reagents listed below would efficiently accomplish the transformation of
CH3CH2CH=CHCH2CH2CHO into CH3CH2CH=CHCH2CH2CH2OH?
A) KMnO4
B) NaBH4
C) Br2 in CCl4
D) H2, Ni
E) Two of the above
Ans: B
Topic: Synthetic Strategy, Redox
32. Which of the reagents listed below would efficiently accomplish the transformation of
CH3CH2CH=CHCH2CH2CHO into CH3CH2CH2CH2CH2CH2CH2OH?
A) KMnO4
B) NaBH4
C) Br2 in CCl4
D) H2, Ni
E) Two of the above
Ans: D
Topic: Synthetic Strategy, Redox
33. Which of the reagents listed below would efficiently accomplish the transformation of
CH3CH2C≡CCH2CH2CHO into CH3CH2C≡CCH2CH2CH2OH?
A) KMnO4
B) NaBH4
C) Br2 in CCl4
D) H2, Ni
E) Two of the above
Ans: B
413
Chapter 12
Topic: Synthetic Strategy, Redox
34. Which of the reagents listed below would efficiently accomplish the transformation of
CH3CH2CH=CHCH2CH2CH2OH into CH3CH2CH=CHCH2CH2CHO ?
A) KMnO4(aq, alkaline)
B) CrO3 / H2SO4
C) PCC in CH2Cl2
D) Br2, CCl4
E) Two of the above
Ans: C
Topic: Synthetic Strategy
35. Which of the reagents listed below would efficiently accomplish the transformation of
2-methyl-3-cyclopentenol into 2-methyl-3-cyclopentenone?
OH
O
?
A)
B)
C)
D)
E)
KMnO4(aq, alkaline)
CrO3/H2SO4
H2, Pt
Br2, CCl4
Two of the above
Ans: B
Topic: Synthetic Strategy, Redox
36. Which of the reagents listed below would efficiently accomplish the transformation of
3-methyl-3-cyclopentenone into 3-methyl-3-cyclopentenol?
OH
O
?
A)
B)
C)
D)
E)
i) LiAlH4; ii) H2O
NaBH4,H2O
H2, Pd
A) and B)
A), B) and C) of the above
Ans: D
414
Chapter 12
Topic: Synthetic Strategy, Redox
37. Which of the reagents listed below would efficiently accomplish the transformation of
ethyl-3-pentenoate into 3-penten-1-ol?
HO
O
?
O
A)
B)
C)
D)
E)
i) LiAlH4; ii) H2O
NaBH4,H2O
H2, Pd
A) and B)
A), B) and C) of the above
Ans: A
Topic: Reactions and Reaction Sequences
38.
A)
B)
C)
D)
E)
When 2-pentanol is treated with chromic acid,
The color changes from orange to blue
The color changes from purple to colorless
The color changes from blue to red
The color changes from orange to green
The color changes from green to orange
Ans: D
Topic: Reactions and Reaction Sequences
39.
A)
B)
C)
D)
E)
When 1-pentanol is treated with alkaline potassium permanganate,
The color changes from orange to blue
The color changes from purple to colorless
The color changes from blue to red
The color changes from orange to green
The color changes from green to orange
Ans: B
Topic: Reactions and Reaction Sequences
40.
A)
B)
C)
D)
E)
When 3-methyl-3-pentanol is treated with chromic acid,
The color changes from orange to blue
The color changes from purple to colorless
The purple color remains unchanged
The color changes from orange to green
The orange color remains unchanged
Ans: E
415
Chapter 12
Topic: Reactions and Reaction Sequences
41. What is the final product?
O
CH3C
A)
CH3CCH3
CH3I
liq. NH3
HO
CH3CC
B)
NaNH2
CH
CCH3
CH3
OCH3
CH3CC
CCH3
CH3
OCH3
C)
D)
CH3C COCH2CHCH3
CH3
CH3CC
CONa
CH3
E) None of these
Ans: B
Topic: Reactions and Reaction Sequences
42. What product(s) is/are formed in the following reaction?
O
D2O
LiAlD4
?
ether
CH3CH2CH2COCH2CH3
A) CH3CH2CH2CH2OD + CH3CH2OD
B) CH3CH2CH2CD2OD + CH3CH2OD
C) CH3CH2CH2CD2OH + CH3CH2OH
D) CH3CH2CH2CHDOD + CH3CH2OD
OD
E)
CH3CH2CH2CDOCH2CH3
Ans: B
416
?
Chapter 12
Topic: Reactions and Reaction Sequences
43. What product(s) is/are produced in the 1:1 reaction of sec-butylmagnesium bromide
with
OH
H
?
CH3CHCH2CH2C
OH
A)
O
OMgBr
CH3CHCH2CH2CHCCH2CH3
CH3
B)
OMgBr
H
+
CH3CH2CH2CH3
C)
CH3CHCH2CH2C
O
OH
OMgBr
D)
CH3CHCH2CH2CHCH2CH(CH3)2
H
OMgBr
O
E)
CH3CHCH2CH2C
H
CH3CHCH2CH2C
O
+
(CH3)3CH
OCHCH2CH3
CH3
Ans: B
417
Chapter 12
Topic: Reactions and Reaction Sequences
44. What is the principal product of the following reaction:
O
+ CH3CH2CH2CH2MgBr/ether; then H2O ⎯⎯⎯⎯→ ?
A) CH3CHCH2CH2CH2CH3
CH3CH
B)
CH2
CH2OH
OH
C)
CH3CHCH2CH2CH2CH2CH3
OH
D)
CH3CHCH2OCH2CH2CH2CH3
CH3CHOCH2CH2CH2CH3
E)
CH2OH
CH3CHCH2OCH2CH2CH2CH3
OCH2CH2CH2CH3
Ans: B
Topic: Reactions and Reaction Sequences
45.
A)
B)
C)
D)
E)
The reaction of lithium di-sec-butylcuprate with isopentyl bromide yields:
2,5-Dimethylheptane
2,6-Dimethylheptane
3,5-Dimethylheptane
3,4-Dimethylheptane
3,6-Dimethylheptane
Ans: A
Topic: Reactions and Reaction Sequences
46.
A)
B)
C)
D)
E)
The reaction of lithium diethylcuprate with 1-bromo-4,4-dimethylhexane yields:
3,3-Dimethylheptane
3,3-Dimethyloctane
1-Ethyl-4,4-dimethylhexane
Di-4,4-dimethylhexylcuprate
None of the above
Ans: B
418
Chapter 12
Topic: Reactions and Reaction Sequences
47. What compound(s) result(s) from the reaction of CH3CH2CH2MgBr with
CH3CH2CH2CH2CO2H (1:1 mole ratio)?
A) (CH3CH2CH2)2CCH2CH2CH2CH3
OH
O
B)
C)
CH3CH2CH2CCH2CH2CH2CH3
O
CH3CH2CH2CH2COCH2CH2CH3
D) CH3CH2CH3 + CH3CH2CH2CH2CO2MgBr
O O
E)
CH3CH2COCCH2CH2CH2CH3
Ans: D
Topic: Synthetic Strategy
C6H5
48.
CH3CH2CH2CCH3
OH
Your task is to synthesize
through a Grignard synthesis. Which pairs
of compounds listed below would you choose as starting materials?
O
A)
CH3CH2CH2Br and
B)
C)
CH3CC6H5
O
CH3CH2CH2CH and C6H5Br
O
CH3CH2CHCH3
and
C6H5CH
Br
D) More than one of these
E) None of these
Ans: A
419
Chapter 12
Topic: Synthetic Strategy
49. Which of the following synthetic procedures would be employed most effectively to
transform ethanol into ethyl propyl ether?
A) Ethanol + HBr, then Mg/ether, then H3O+, then NaH, then CH3CH2Br
B) Ethanol + HBr, then Mg/ether, then HCHO, then H3O+, then NaH, then CH3CH2Br
C) Ethanol + CH3CH2CH2OH + H2SO4/140°C
D) Ethanol + NaH, then HCHO, then H3O+, then HBr, then Mg/ether, then CH3CH2CH2Br
E) Ethanol + H2SO4/180°C, then CH3CH2CH2Br
Ans: B
Topic: Synthetic Strategy
50. Your task is to synthesize 2-phenyl-2-hexanol through a Grignard synthesis. Which
pair(s) of compounds listed below would you choose as starting materials?
O
A)
CH3CH2CH2Br and
B) CH3CHCH2Br
CH3CC6H5
O
and
CH3CC6H5
CH3
O
C)
CH3CH2CH2CH2CCH3
and C6H5Br
D) Answers A) or B)
E) Answers A) or C)
Ans: E
Topic: Synthetic Strategy
51. How could the following synthesis be accomplished?
O
OH
CH2CH
A)
O
(1) SOCl2, (2) Mg, ether, (3) CH3CH, then H3O+
B) (1) SOCl2, (2) Li, ether, (3) (CH3CH2)2CuLi, (4) KMnO4, OHO
C)
1) PBr3, (2) Mg, ether, (3) CH2
D) More than one of the above
E) None of the above
Ans: C
CH2, then
420
H3O+ (4) PCC,CH2Cl2
Chapter 12
Topic: Synthetic Strategy, Redox
52. Select the correct reagent(s) for the following reaction:
O
OH
CH3CCH2CH2CO2CH3
A)
B)
C)
D)
E)
CH3CHCH2CH2CH2OH
+
LiAlH4/ether; then H3O
NaBH4; then H3O+
H2 with Pt/C
A) and B)
A), B) and C)
Ans: A
Topic: Synthetic Strategy, Redox
53. Select the correct reagent(s) for the following reaction:
O
OH
CH3CCH2CH2CO2CH3
A)
B)
C)
D)
E)
CH3CHCH2CH2CO2CH3
+
LiAlH4/ether; then H3O
NaBH4; then H3O+
H2 with Pt/C
B) and C)
A), B) and C)
Ans: D
Topic: Synthetic Strategy, Redox
54.
A)
B)
C)
D)
E)
Which reagent(s) would you use to convert CH3(CH2)6CO2H to CH3(CH2)6CH2OH?
NaBH4/H2O
LiAlH4/ether, then H3O+
PCC/CH2Cl2
Zn, H3O+
H2, Pt
Ans: B
421
Chapter 12
Topic: Synthetic Strategy
OH
55.
Which method would give (CH3)2CC CH ?
A) CH3CHCH3
H2CrO4
NH4+
HC CNa
acetic
acid
CH3CH2Li
OH
B) CH3CCH3
C)
O
CH3CC
H3O+
H2CrO4
acetone
CH
2 CH3MgBr
NH4+
O
D) More than one of the above
E) None of the above
Ans: D
Topic: Synthetic Strategy
56. Which synthesis of a Grignard reagent would fail to occur as written?
Mg, ether
A)
CH3OCH2CH2Br
CH3OCH2CH2MgBr
Mg, ether
B)
CH3CH2CH2I
CH3CH2CH2MgI
Mg, ether
C)
HO2CCH2CH2I
HO2CCH2CH2MgI
Mg, ether
D)
C6H5Br
C6H5MgBr
E) All of the above will succeed.
Ans: C
Topic: Synthetic Strategy
57. Which combination of reagents is to be preferred for the synthesis of 2,4dimethylhexane by the Corey-Posner, Whitesides-House procedure?
A) Lithium diisobutylcuprate + sec-butyl bromide
B) Lithium dimethylcuprate + 2-bromo-4-methylhexane
C) Lithium dimethylcuprate + 4-bromo-2-methylhexane
D) Lithium diisopropylcuprate + 1-bromo-2-methylbutane
E) Lithium di(2-methylbutyl)cuprate + isopropyl bromide
Ans: D
422
Chapter 12
Topic: Synthetic Strategy
58. Which of these reactions will not produce a 1° alcohol?
1. LiAlH4, Et2O
A)
(CH3)2CHCH2CH2MgBr
2. H3O+
CH2
1. CH2
B)
(CH3)2CHCH2CH2MgBr
C)
O
C6H5CCH3
D)
E)
O
O+
2. H3
1. NaBH4
2. H3O+
CH3CH2CH2CH2Li
CH3(CH2)5COOH
1. H2C=O
2. H3O+
1. LiAlH4, Et2O
2. H3O+
Ans: C
423
Chapter 12
Topic: Synthetic Strategy, Stereochemistry
59. Which of the following would serve as a synthesis of racemic 2-methyl-1-phenyl-2butanol?
CH3
CCH2CH3
?
OH
I
CH3CH2CCH3
+
CH2MgCl
O
O
II
+
CH2CCH3
III
CH2CCH2CH3
CH3CH2MgBr
+
CH3MgI
O
A)
B)
C)
D)
E)
I
II
III
All of the above
None of the above
Ans: D
424
1. Et2O
2. NH4+
1. Et2O
2. NH4+
1. Et2O
2. NH4+
Chapter 12
Topic: Synthetic Strategy, Stereochemistry
60. Which of the following reactions would serve as a reasonable synthesis of the following
racemic alcohol (2-phenyl-2-butanol)?
OH
CH3CC6H5
C2H5
O
A)
B)
CH3CC6H5
O
C)
C2H5CC6H5
O
+
C2H5MgBr
+
CH3MgBr
+
C6H5MgBr
1. Et2O
2. NH4+
1. Et2O
2. NH4+
1. Et2O
2. NH4+
CH3CC2H5
D) Answers A) and B) only
E) Answers A), B) and C)
Ans: E
Topic: Synthetic Strategy, Stereochemistry
61. Which Grignard synthesis will produce an optically active product or product mixture?
A)
MgBr
B)
O
+
O
ClMg
+
O
C)
+
O
D)
E)
MgI
H CH3
O
+
H3C
MgI
H
+
MgBr
O
Ans: E
425
Chapter 12
Topic: Synthetic Strategy, Stereochemistry
62. Which reaction leads to an optically active product?
A)
+
NaBH
4
O
O
B)
ClMg
+
C)
H
CO2H
H2, Pt
+
D)
E)
+
H OH
H3C CHO
+
O
O
KMnO4
LiAlH4 / Et2O
(excess)
Ans: C
Topic: Acid/Base
63.
A)
B)
C)
D)
Which of the following is the strongest acid?
RMgX
Mg(OH)X
RH
H2O
Ans: D
Topic: Acid/Base
64.
A)
B)
C)
D)
Which of the following is the strongest base?
RMgX
Mg(OH)X
RH
H2O
Ans: A
Topic: Acid/Base
65. In which of the following series are the compounds arranged in order of decreasing
basicity?
A) CH3CH2MgBr > NaNH2 > HC≡CNa > NaOH > CH3CH2ONa
B) CH3CH2MgBr > NaNH2 > HC≡CNa > CH3CH2ONa > NaOH
C) HC≡CNa > CH3CH2MgBr > NaNH2 > CH3CH2ONa > NaOH
D) NaNH2 > CH3CH2MgBr > HC≡CNa > CH3CH2ONa > NaOH
E) None of these
Ans: B
426
Chapter 12
Topic: Redox
66.
A)
B)
C)
D)
E)
Which reaction is an oxidation?
RCHO ⎯⎯⎯⎯→ RCO2H
RCH2OH ⎯⎯⎯⎯→ RCHO
RCH2OH ⎯⎯⎯⎯→ RCO2H
Two of these
All of these
Ans: E
Topic: Synthetic Strategy, Redox
67. Which of these compounds will not be reduced by LiAlH4?
A) CH3CH2CH2CH=CH2
H
B)
C)
CH3CH2CH2C
O
O
D)
CH3CH2CH2COH
O
E)
CH3CH2CH2COCH3
O
CH3CH2CH2CCH3
Ans: A
Topic: Synthetic Strategy, Redox
68. Which reagent(s) is/are capable of reducing the following ester to a primary alcohol ?
O
O
A)
B)
C)
D)
E)
LiAlH4 / ether
NaBH4 / H2O
Na / C2H5OH
H2 / Ni, atmospheric pressure
All can be used successfully.
Ans: A
427
Chapter 12
Topic: Synthetic Strategy, Redox
69. Which reagent(s) is/are capable of reducing the following ester to an aldehyde ?
O
O
A)
B)
C)
D)
E)
LiAlH4 / ether
NaBH4 / H2O
Na / C2H5OH
H2 / Ni, atmospheric pressure
None of the above can be used successfully.
Ans: E
Topic: Redox
70.
A)
B)
C)
D)
Which of these transformations cannot be classified as a reduction?
RCH2Cl ⎯⎯⎯⎯→ RCH3
RCH=CH2 ⎯⎯⎯⎯→ RCH2CH3
RCOOH ⎯⎯⎯⎯→ RCH2OH
O
RCH2OH + R'OH
E) All of these are reductions.
Ans: E
RCOR'
Topic: Synthetic Strategy, Redox
71. Which of these compounds cannot be reduced by sodium borohydride?
H
A)
B)
(CH3)2CHC
O
C)
CH3CH2CCH3
O
O
C6H5COH
D)
O
CH3(CH2)4COCH3
E) Neither C) nor D) can be reduced.
Ans: E
428
Chapter 12
Topic: Redox, Reaction Mechanisms
72.
A)
B)
C)
D)
E)
In the reaction of carbonyl compounds with LiAlH4, the effective reducing species is:
Li+
Al+3
AlH4AlH3
HAns: E
Topic: Redox, Reaction Mechanisms
73. The success in converting low molecular weight 1° alcohols to aldehydes by use of
K2Cr2O7/H2SO4 as oxidant can be attributed to the fact that:
A) dichromate is a relatively weak oxidizing agent.
B) the presence of H2SO4 limits the oxidation.
C) the aldehyde can be separated, as formed, by distillation.
D) aldehydes are not oxidized by the K2Cr2O7/H2SO4 mixture.
E) hydrogen bonding occurs between the alcohol and the acid present.
Ans: C
Topic: Redox, Reaction Mechanisms
74.
A)
B)
C)
D)
E)
Fundamentally, 2-methyl-2-pentanol does not undergo oxidation by H2CrO4 because:
the intermediate chromate ester is not formed.
the oxidant isn't in a sufficiently high oxidation state.
the alcohol undergoes dehydration.
the intermediate chromate ester cannot lose hydrogen.
Actually, this oxidation does occur.
Ans: D
429
Chapter 12
Topic: Synthetic Strategy, Redox
75. Which of these reduction reactions is unsuccessful?
O
A)
B)
CH3CH2CH2COCH2CH2CH3
H
O
C6H5CH2C
+ NaBH4, CH3OH
O
C)
D)
+ LiAlH4, ether
CH3(CH2)10COH
O
+ H2, Pt, high pressure
+ NaBH4, CH3CH2OH
E) All of these are successful reductions.
Ans: C
C6H5CCH2CH2CH2CH3
Topic: Synthesis of Grignards and Other Organometallics
76. Grignard reagents react with oxirane (ethylene oxide) to form 1° alcohols but can be
prepared in tetrahydrofuran solvent. Why is this difference in behavior observed?
A) Steric hindrance in the case of tetrahydrofuran precludes reaction with the Grignard.
B) There is a better leaving group in the oxirane molecule.
C) The oxirane ring is the more highly strained.
D) It is easier to obtain tetrahydrofuran in anhydrous condition.
E) Oxirane is a cyclic ether, while tetrahydrofuran is a hydrocarbon.
Ans: C
Topic: Synthesis of Grignards and Other Organometallics
77. Which of these compounds cannot be used to prepare the corresponding Grignard
reagent?
A) CH3OCH2CH2CH2Br
B) (CH3)3CCl
C) CH2=CHCH2Br
D) (CH3)2NCH2CH2Br
H
E)
O
CCH2CH2I
Ans: E
430
Chapter 12
Topic: Synthesis of Grignards and Other Organometallics
78.
A)
B)
C)
D)
E)
Which of these compounds can be used to prepare the corresponding Grignard reagent?
CH3CHOHCH2CH2CH2CH2Br
(CH3)3CHCHBrCH2CH2CO2H
BrCH=CHCH2CH2CH3
CH3NHCH2CH2Br
None of the above can be used to prepare the corresponding Grignard reagent
Ans: C
Topic: Reactivity of Grignards and Other Organometallics
79.
A)
B)
C)
D)
E)
Which of these is the least reactive type of organometallic compound?
RK
R2Hg
RLi
R2Zn
R3Al
Ans: B
Topic: Synthesis of Grignards and Other Organometallics
80. If the role of the solvent is to assist in the preparation and stabilization of the Grignard
reagent by coordination with the magnesium, which of these solvents should be least
effective?
CH3CH2OCH2CH3
I
A)
B)
C)
D)
E)
(CH3CH2)3N
O
II
III
CH3(CH2)4CH3
CH3OCH2CH2OCH3
IV
V
I
II
III
IV
V
Ans: D
431
Chapter 12
Topic: Synthesis of Grignards and Other Organometallics
81. Which of these is most likely to be a successful synthesis of an organometallic
compound?
A) CH3CH2CH2MgBr + LiCl ⎯⎯⎯⎯→ CH3CH2CH2Li + MgBrCl
B) 2 CH3CH2CH2CH2Li + ZnCl2 ⎯⎯⎯⎯→ (CH3CH2CH2CH2)2Zn + 2 LiCl
C) 3 (CH3CH2)2Hg + 2 AlCl3 ⎯⎯⎯⎯→ 2 (CH3CH2)3Al + 3 HgCl2
D) (CH3CH2)3Al + 3 NaCl ⎯⎯⎯⎯→ 3 CH3CH2Na + AlCl3
E) (CH3)2Cu + MgBr2 ⎯⎯⎯⎯→ (CH3)2Mg + CuBr2
Ans: B
SHORT ANSWER QUESTIONS
Topic: General
82. Because the carbonyl carbon bears a partial positive charge, it is susceptible to
________________ attack.
Ans: nucleophilic
Topic: General, Redox
83. A reaction which increases the hydrogen content or decreases the oxygen content of an
organic molecule is called a(n) _____________.
Ans: reduction
Topic: General, Redox
84. A reaction which increases the oxygen content or decreases the hydrogen content of an
organic molecule is called a(n) _____________.
Ans: oxidation
Topic: General, Redox
85. LAH is the abbreviation for the powerful reducing agent ______________.
Ans: lithium aluminum hydride
Topic: General, Redox
86. A negatively charged hydrogen atom is called a ______________.
Ans: hydride ion
Topic: General, Redox Reagents
87. A solution of CrO3 in aqueous acetone is called the _______________.
Ans: Jones reagent
432
Chapter 12
Topic: Functional Group Tests
88. Primary and secondary alcohols can be distinguished from most other compounds (with
the exception of aldehydes) because they will react rapidly with a solution of CrO3 in
aqueous sulfuric acid, causing a color change from ______________ to
_____________.
Ans: orange; green
Topic: General
89. Compounds that contain carbon-metal bonds are known as _________________.
Ans: organometallic compounds
Topic: General
90. Organomagnesium halides are known as _______________.
Ans: Grignard reagents
Topic: Synthesis of Grignards and Other Organometallics
91. Why can we not prepare a Grignard reagent from a molecule such as 4-bromo-1butanol?
Ans: Because Grignard reagents are extremely strong bases and will react with the
alcohol's acidic proton.
Topic: Reactions of Grignards and Other Organometallics
92. A Grignard reagent will produce a primary alcohol when reacted with ____________.
Ans: Formaldehyde and ethylene oxide
Topic: Reactivity of Grignards and Other Organometallics
93. The reactivity of organometallic compounds increases with the _____________ of the
carbon-metal bond.
Ans: percent ionic character
Topic: Reactions of Grignards and Other Organometallics
94. A Grignard reagent will produce a secondary alcohol when reacted with ____________.
Ans: Any aldehyde except formaldehyde
433
Chapter 12
Topic: Reactions of Grignards and Other Organometallics
95. Reaction of an alkyllithium with a ketone produces, after acid work-up, a
_____________ alcohol.
Ans: tertiary
Topic: Reactions of Grignards and Other Organometallics
96. Reaction of excess methyllithium with ethyl acetate (CH3CO2C2H5) produces, after acid
work-up, ____________.
Ans: t-butyl alcohol: (CH3)3COH
Topic: Reactions of Grignards and Other Organometallics
97. Reaction of excess ethyl magnesium bromide with ethyl benzoate (C6H5CO2C2H5)
produces, after acid work-up, ____________.
OH
Ans:
3-phenyl-3-pentanol:
Topic: Reactions of Grignards and Other Organometallics
98. Reaction of excess ethyl magnesium iodide with a ethyl acetate (CH3CO2C2H5)
produces, after acid work-up, ____________ .
Ans: 3-methyl-3-pentanol CH3CH2C(CH3)(OH)CH2CH3
Topic: Redox Reactions
99. In the reaction of carbonyl compounds with LiAlH4, the effective reducing species is:
Ans: H− (hydride)
Topic: Redox Reactions
100. In the reaction of carbonyl compounds with NaBH4, the effective reducing species is:
Ans: H− (hydride)
Topic: Grignard Reactions
101. NH4Cl is sometimes preferred instead of HCl or H2SO4 for “acid” work-up after
Grignard reactions, particularly when the expected and desired product is a tertiary
alcohol: why?
Ans: NH4Cl provides mildly acidic conditions via partial hydrolysis of NH4Cl. Under
strongly acid conditions, a tertiary alcohol product can undergo facile dehydration
to an alkene.
434
Chapter 12
Topic: Synthesis of Grignards and Other Organometallics
102. Diethyl ether, the most commonly used solvent for Grignard reactions, also acts as an
‘internal cooling agent’, preventing the overheating of the reaction mixture: what
specific property of diethyl ether is exploited here and why is it important to prevent the
overheating of the reaction mixture?
Ans: Diethyl ether is very volatile (35-36oC): its low boiling point ensures that
reactions carried out in diethyl ether solvent cannot become overheated (the
maximum temperature cannot exceed the b.p. of the solvent): this is important in
Grignard reactions, because Grignard reagents decompose at higher temperatures.
Thus, the decomposition of the newly formed Grignard reagent is minimized by
use of this ‘internal cooling agent’.
Topic: General
102. When nucleophilic addition to a carbonyl group occurs, the carbon attacked undergoes
this hybridization change:
Ans: sp2 Æ sp3
Topic: General, Redox
103. PCC and CrO3 are both Cr(VI) reagents. However, PCC/CH2Cl2 is useful in oxidizing
primary alcohols to aldehydes, while the analogous reaction with CrO3/H2SO4 will
typically produce carboxylic acids. Why?
Ans: Both reagents first oxidize the alcohol to the corresponding aldehyde. Further
oxidation of aldehydes, to carboxylic acids, is facilitated in aqueous solutions, via
formation of the intermediate aldehyde hydrates (CrO3/H2SO4). When the solvent
is non-aqueous, such as CH2Cl2, these key intermediates cannot be formed,
thereby preventing further oxidation of aldehydes (with PCC/CH2Cl2).
Topic: Multistep Reactions
104. What would be the major product, A, of the following reaction?
i) NaBH4
ii) NaH, Et2O
iii)
CF3
O
O
S
A
O
OCH2CH3
Ans:
O
435
Chapter 12
Topic: Multistep Synthetic Strategy
105. Suggest a reasonable synthetic strategy for the synthesis of 3-methyl-3-hexanol from 3hexanol.
Ans: 3-hexanol-Æ i) PCC; ii) CH3MgBr (or CH3Li); iii) NH4Cl
OH
O
PCC
CH2Cl2
i) CH3MgBr, Et2O
ii) NH4Cl
OH
Topic: Multistep Synthetic Strategy
106. Suggest a reasonable synthetic strategy for the synthesis of 2-methyl-3-hexanol from 1butanol.
Ans: 1-butanol Æ i) PCC; ii) (CH3)2CHMgBr (or (CH3)2CHLi); iii) NH4Cl (H3O+)
HO
PCC
CH2Cl2
i) (CH3)2CHMgBr, Et2O
ii) NH4Cl
O
OH
Topic: Multistep Synthetic Strategy
107. Suggest a reasonable synthetic strategy for the synthesis of 4-heptanol from 1-pentene.
Ans: 1-pentene Æ i)MMPP; ii) CH3CH2MgBr (or CH3CH2Li); iii) NH4Cl (H3O+)
MMPP
O
i) CH3CH2MgBr, Et2O
ii) NH4Cl
OH
Topic: Multistep Synthetic Strategy
108. Suggest a reasonable synthetic strategy for the synthesis of 3-ethyl-3-heptanol from
pentanoic acid, CH3CH2CH2CH2CO2H.
Ans: CH3CH2CH2CH2CO2H Æ i)SOCl2; ii) 2 equivalents CH3CH2MgBr (or
CH3CH2Li); iii) NH4Cl (H3O+)
i) CH3CH2MgBr
O
O
OH
SOCl2
(2 equivalents), Et2O
HO
Cl
ii) NH4Cl
436
Chapter 12
Topic: Multistep Reaction Sequence
109. Complete the following reaction sequence, giving structural details of all key
intermediates.
1-hexene------------------Æ
i) HBr
ii) Li
iii) (CH3CH2)2C=O
iv) C2H5I
Br
Li
Ans:
HBr
Li
O
OLi
O
C2H5I
Topic: Multistep Reaction Sequence
110. Complete the following reaction sequence, giving structural details of all key
intermediates.
2-methyl-1-pentene------------------Æ
i) BH3, THF
ii) H2O2,NaOH
iii) PCC, CH2Cl2
iv) CH3CH2C≡C:−Na+
v) NH4Cl
Ans:
i) BH3,THF
HO
ii) H2O2, NaOH
PCC,
CH2Cl2
i)
HO
:− Na+
O
ii) NH4Cl
437
Chapter 12
Topic: Multistep Reaction Sequence
111. Complete the following reaction sequence, giving structural details of all key
intermediates.
4-methyl-1-hexene------------------Æ
i) BH3, THF
ii) H2O2,NaOH
iii) SOCl2
iv) Mg, Et2O
iv) ethylene oxide
v) NH4Cl
Ans:
i) BH3,THF
HO
ii) H2O2, NaOH
SOCl2
Mg
Et2O
ClMg
i)
Cl
O
ii) NH4Cl
HO
438
Chapter 12
Topic: Multistep Reaction Sequence
112. Complete the following reaction sequence, giving structural details of all key
intermediates.
Bromocyclohexane------------------Æ
i) Li
ii) CuI
iii) 4-methyl-1-bromohexane
Ans:
Br
Li
Li
CuI
Br
439
CuLi
2
Chapter 13
MULTIPLE CHOICE QUESTIONS
Topic: Nomenclature
1.
A)
B)
C)
D)
E)
What is an IUPAC name for
2,3-methylhept-2-en-4-yne
1,1,3-trimethylhex-3-yn-1-ene
1,1,3-trimethyl-3-hexyn-1-ene
2,3-dimethyl-2-hepten-4-yne
5,6-dimethyl-5-hepten-3-yne
Ans: D
Topic: Nomenclature
2. What is an IUPAC name for this triene?
H
CH3CH2
A)
B)
C)
D)
E)
H H
CH3
H
H
H
(2E,4Z,6E)–2,4,6–Nonatriene
(2Z,4E,6Z)–2,4,6–Nonatriene
(2E,4Z,6Z)–2,4,6–Nonatriene
(3Z,5Z,7E)–3,5,7–Nonatriene
(3Z,5E,7E)–3,5,7–Nonatriene
Ans: C
Topic: Nomenclature
3. What is an IUPAC name for this triene?
A)
B)
C)
D)
E)
(2E,4Z,6E)–3,4,7,8–tetramethyl-2,4,6–Nonatriene
(2Z,4E,6E)–3,4,7,8–tetramethyl-2,4,6–Nonatriene
(2E,4Z,6E)–2,3,6,7–tetramethyl-3,5,7–Nonatriene
(2E,4Z,6E)– 2,3,6,7–tetramethyl-3,5,7–Nonatriene
(2E,4E,6E)–3,4,7,8–tetramethyl-2,4,6–Nonatriene
Ans: E
440
Chapter 13
Topic: Nomenclature
4. A correct IUPAC name of the compound below is:
Br
A)
B)
C)
D)
E)
1-Bromo-1-methyl-2,5-cyclohexadiene
3-Bromo-3-methyl-1,4-cyclohexadiene
6-Bromo-6-methyl-1,4-cyclohexadiene
2-Bromo-2-methyl-1,3-cyclohexadiene
None of these
Ans: B
Topic: Stability (Alkenes, Cations, Radicals)
5.
A)
B)
C)
D)
E)
Which of the following dienes would you expect to be the most stable?
CH3CH=CHCH=CHCH3
CH3CH=CHCH2CH=CH2
CH2=CHCH2CH2CH=CH2
CH2=CHCH(CH3)CH=CH2
CH3CH=C=CHCH2CH3
Ans: A
Topic: Stability (Alkenes, Cations, Radicals)
6.
A)
B)
C)
D)
E)
Which of the following dienes would you expect to be the most stable?
CH3CH2CH=CHCH2CH=CHCH3
CH3CH=CHCH=CHCH2CH3
CH2=CHCH2CH2CH2CH=CH2
CH2=CHCH=CHCH2CH2CH3
CH3CH2CH=C=CHCH2CH3
Ans: B
441
Chapter 13
Topic: Stability (Alkenes, Cations, Radicals)
7. Which of the following compounds would be the most stable?
I
A)
B)
C)
D)
E)
II
III
IV
I
II
III
IV
They are all of equal stability.
Ans: A
Topic: Stability (Alkenes, Cations, Radicals)
8. Considering both configurational and conformational factors, select the most stable
form of 2,4-hexadiene.
CH3
H
H
H
H
H
I
A)
B)
C)
D)
E)
CH3
H
H
H
CH3
H
CH3
CH3
II
H
III
I
II
III
IV
V
Ans: A
442
H
H
CH3
CH3
H
H
H
CH3
IV
H
CH3
CH3
V
Chapter 13
Topic: Stability (Alkenes, Cations, Radicals)
9. Which alkene would you expect to be most stable?
A) CH2=CHCH2CH2CH=CH2
H
B) CH2 CHCH2
C
C)
C
H
CH2CH
CH3
C
CH3
CH2
C
H
H
H
D)
CH3
H
C
C
C
C
H
CH3
H
H
E) CH3
C
C
H
H
C
H
C
CH3
Ans: E
Topic: Stability (Alkenes, Cations, Radicals)
10. Which diene would be least stable?
I
A)
B)
C)
D)
E)
II
III
IV
I
II
III
IV
V
Ans: C
443
V
Chapter 13
Topic: Stability (Alkenes, Cations, Radicals)
11. Arrange these hexadienes in order of expected decreasing stability.
I
II
IV
A)
B)
C)
D)
E)
III
V
V > II > I > III > IV
III > IV > II > I > V
IV > III > II > V > I
IV > III > I > II > V
I > II > IV > III > V
Ans: D
Topic: Stability (Alkenes, Cations, Radicals)
12. Which carbocation would be most stable?
A)
B)
C)
D)
E)
Ans: C
444
Chapter 13
Topic: Stability (Alkenes, Cations, Radicals)
13. Which carbocation would be most stable?
A)
B)
C)
D)
E)
CH2
CH3
CH3
CH3
CH3
I
II
III
IV
V
I
II
III
IV
V
Ans: C
Topic: Stability (Alkenes, Cations, Radicals)
14. Which free radical would be most stable?
CH3
A)
CH3CC
B)
C)
CH2
CH3
CH2
CCH2CH2
CH3
CH3CCH2CH3
D)
E)
CH3
CH2CHC
CH3
CH2
CH3CHCHCH3
Ans: A
445
Chapter 13
Topic: Stability (Alkenes, Cations, Radicals)
V
15.
CH3
CH3
A)
B)
C)
D)
E)
CH
CH
CH
II III
Which hydrogen atom(s) of I
susceptible to abstraction by free radicals?
I
II
III
IV
V
Ans: D
IV
CH
CH
CH3
is/are most
Topic: Heat of Hydrogenation
16. Which alkene would you expect to have the smallest heat of hydrogenation?
A) CH2=CHCH2CH2CH=CH2
H
B) CH2 CHCH2
C
C
H
CH2CH
C) CH3
C
CH3
CH2
C
H
H
H
D)
CH3
C
C
C
C
H
CH3
H
H
E) CH3
C
H
C
H
H
C
H
C
CH3
Ans: E
446
Chapter 13
Topic: Bond Lengths
17.
A)
B)
C)
D)
E)
Which compound would have the shortest carbon-carbon single bond?
CH3–CH3
CH2=CH–CH3
HC≡C–C≡CH
CH2=CH–C≡CH
CH2=CH–CH=CH2
Ans: C
Topic: Bond Lengths
18. Which compound would have the shortest carbon-carbon single bond?
A)
B)
C)
D)
E)
CH≡C–CH=CH–CH2–CH3
CH2=CH–CH=CH–CH3
HC≡C–CH2–C≡C–CH3
CH2=CH–C≡C–CH2–CH3
CH3–C≡C–C≡C–CH3
Ans: E
Topic: Bond Lengths
19. Which carbon-carbon bond in the following compound would you expect to be shortest?
H
A)
B)
C)
D)
E)
C
C
CH
I II
I
II
III
IV
V
Ans: A
CH
III
CH2
IV
CH3
V
447
Chapter 13
Topic: Bond Lengths
20. Which carbon-carbon bond in the following compound would you expect to be longest?
H
A)
B)
C)
D)
E)
C
I
I
II
III
IV
V
Ans: E
C
CH
II
CH
III
CH2
IV
CH3
V
Topic:Alkene Classification
21. Select the structure of the conjugated diene.
I
A)
B)
C)
D)
E)
II
III
IV
V
I
II
III
IV
V
Ans: B
448
Chapter 13
Topic:Alkene Classification
22. Select the structure(s) of the conjugated diene(s).
II
I
IV
A)
B)
C)
D)
E)
III
V
I and II
II and III
III and IV
I, II and V
V
Ans: B
Topic: Alkene Classification
23.
A)
B)
C)
D)
E)
Which of the following dienes is a cumulated diene?
CH2=CHCH2CH2CH=CH2
CH2=CHCH=CHCH2CH3
CH3CH=C=CHCH2CH3
CH3CH=CHCH=CHCH3
CH3CH=CHCH2CH=CH2
Ans: C
Topic: Alkene Classification
24.
A)
B)
C)
D)
E)
Which of the following dienes is a cumulated diene?
CH2=CHCH2CH2CH2CH=CH2
CH2=CHCH=CHCH2CH2CH3
CH3CH=C=CHCH2CH2CH3
CH3CH=CHCH=CHCH2CH3
CH3CH2CH=CHCH2CH=CH2
Ans: C
449
Chapter 13
Topic: Molecular Orbitals
25.
A)
B)
C)
D)
E)
The allyl radical has how many bonding π molecular orbitals?
1
2
3
4
5
Ans: A
Topic: Molecular Orbitals
26.
A)
B)
C)
D)
E)
The allyl cation has how many electrons in bonding π molecular orbitals?
1
2
3
4
5
Ans: B
Topic: Molecular Orbitals
27.
A)
B)
C)
D)
E)
1,3-Pentadiene has how many bonding π molecular orbitals?
1
2
3
4
0
Ans: B
450
Chapter 13
Topic: Heat of Hydrogenation
28. Estimate the stabilization energy for 1,3-butadiene using the heats of hydrogenation in
Table 1.
Table 1.
Heats of Hydrogenation for Selected Compounds
Compound
A)
B)
C)
D)
E)
1-Butene
1-Pentene
1,3-Butadiene
1,3-Pentadiene
13 kJ mol-1
15 kJ mol-1
28 kJ mol-1
239 kJ mol-1
112 kJ mol-1
Ans: B
Moles H2
ΔH(kJ mol-1)
1
1
2
2
-127
-126
-239
-226
Topic: Resonance
29. Which is not an example of resonance?
I
CH2=CH−CH2 .
II
CH2
+
CH2
CH2
III
. CH2−CH=CH2
+
CH3
+
.
.
IV
A)
B)
C)
D)
E)
CH2−CH=CH−CH2CCl3
CH2=CH−CH−CH2CCl3
I
II
III
IV
None of these are examples of resonance
Ans: C
451
Chapter 13
Topic: Resonance
30. Which is not a proper resonance structure for 1,3-butadiene?
A) CH2=CH–CH=CH2
B)
CH2
CH
CH
CH2
C)
CH2
CH
CH
CH2
D)
CH2
CH
CH
CH2
E) All are correct
Ans: B
Topic: Resonance
31. Which pair does not represent a pair of resonance structures?
H
I
H
H
H
H
and
H
H
H
H
H
II
III
IV
A)
B)
C)
D)
E)
H
H
H
H
H
and
H
CH2
CH2
H
H
H
and
and
H
CH2
CH3
I
II
III
IV
All of these represent pairs of resonance structures.
Ans: D
452
Chapter 13
Topic: Reaction Mechanisms and Control
32.
A)
B)
C)
D)
E)
A thermodynamically-controlled reaction will yield predominantly:
the more/most stable product.
the product whose formation requires the smallest free energy of activation.
the product that can be formed in the fewest steps.
the product that is formed at the fastest rate.
the product which possesses the greatest potential energy.
Ans: A
Topic: Reaction Mechanisms and Control
33.
A)
B)
C)
D)
E)
A reaction under kinetic (or rate) control will yield predominantly:
the most stable product.
the product that can be formed in the fewest steps.
the product whose formation requires the smallest free energy of activation.
the product with the greatest potential energy.
the product with the least potential energy.
Ans: C
Topic: Reaction Mechanisms and Control
34. The accompanying diagram, which describes the fate of the intermediate in a reversible
reaction, implies that:
E
B
A
A)
B)
C)
D)
E)
reaction coordinate
the less stable product forms more rapidly.
the more stable product forms more rapidly.
product B will predominate at equilibrium.
the intermediate has a short lifetime.
No conclusions can be drawn as to either reaction rate or product stability.
Ans: B
453
Chapter 13
Topic: Reaction Mechanisms and Control
35. The accompanying diagram implies that:
E
B
A
A)
B)
C)
D)
E)
reaction coordinate
The formation of B from A would be favored at high temperature.
The more stable product forms more rapidly from the intermediate species.
The formation of the intermediate from A is the rate-limiting step in the transformation
of A into B
The formation of B from A is not a concerted reaction.
All of the above statements are true
Ans: E
Topic: Diels-Alder Reaction
36.
A)
B)
C)
D)
E)
Which is an untrue statement concerning the Diels-Alder reaction?
The reaction is a syn addition.
The diene must be in the s-cis conformation to react.
Most Diels-Alder reactions are reversible.
Generally, the adduct formed most rapidly is the exo product.
Depending on the nature of the dienophile, both electron-releasing and electronwithdrawing groups in the diene can favor adduct formation.
Ans: D
Topic: Diels-Alder Reaction
37.
A)
B)
C)
D)
E)
does not undergo the Diels-Alder reaction because:
ring systems cannot function as the diene component.
it cannot adopt the s-cis conformation.
it lacks electron-withdrawing groups.
it lacks strong electron-releasing groups.
the two double bonds are further apart than in a non-cyclic conjugated system.
Ans: B
454
Chapter 13
Topic: Diels-Alder Reaction
38.
A)
B)
C)
D)
E)
does not undergo the Diels-Alder reaction because:
ring systems cannot function as the diene component.
it cannot adopt the s-cis conformation.
it lacks electron-withdrawing groups.
it lacks strong electron-releasing groups.
the two double bonds are further apart than in a non-cyclic conjugated system.
Ans: B
Topic: Diels-Alder Reaction
39.
A)
B)
C)
D)
E)
Which of these dienes is the most reactive in the Diels-Alder reaction?
1,3-Butadiene
1,4-Pentadiene
Cyclopentadiene
1,2-Butadiene
1,4-Cyclohexadiene
Ans: C
Topic: Diels-Alder Reaction
40.
A)
B)
C)
D)
E)
Which of these dienes can undergo the Diels-Alder reaction?
1,3-Pentadiene
1,4-Pentadiene
1,2-Butadiene
1,4-Cyclohexadiene
All of the above can undergo the Diels-Alder reaction
Ans: A
Topic: Diels-Alder Reaction
41.
A)
B)
C)
D)
E)
Which of these dienes can undergo the Diels-Alder reaction?
1,2-Heptadiene
1,3-Heptadiene
1,4-Heptadiene
1,5-Heptadiene
1,6-Heptadiene
Ans: B
455
Chapter 13
Topic: Diels-Alder Reaction
42. Which diene and dienophile would you choose to synthesize the following compound?
H
COCH3
O
O
and
CH2
and
CH2
HC
I
CCOCH3
II
O
O
and
and
CH2
CH2
CHCOCH3
III
A)
B)
C)
D)
E)
CHCOCH3
IV
I
II
III
IV
None of these
Ans: C
Topic: Diels-Alder Reaction
43. Which diene would be least reactive toward Diels-Alder addition of maleic anhydride?
CH3
CH2
CH
CH
CH2
CH2
I
CH
C
CH2
CH2
II
IV
A)
B)
C)
D)
E)
CH3 CH3
456
C
III
V
I
II
III
IV
V
Ans: E
C
CH2
Chapter 13
Topic: Diels-Alder Reaction
44. What would be the product of the following reaction?
O
+
CH2
A)
B)
C)
D)
E)
O
O
C CH
3
C
I
CHCCH3
O
CH3
CH
heat
C
C
CH2
CH3
II
III
I
II
III
IV
All of these
Ans: C
457
IV
CH3
Chapter 13
Topic: Diels-Alder Reaction
45. How would you synthesize:
H
O
COCH3
O
O
O
I
1,3-Cyclohexadiene + CH2
CHCOCH3,
then
O
II
RCOOH
O
1,3-Cyclohexadiene + RCOOH,
then CH2
CHCOCH3,
O
COCH3
III
+
CH2
CH2
O
O
IV
1,4-Cyclohexadiene + CH2
CHCOCH3,
O
COCH3
V
+
CH2
CH2
O
A)
B)
C)
D)
E)
I
II
III
IV
V
Ans: A
458
CH2
CH2
then
O
Chapter 13
Topic: Diels-Alder Reaction
46. Which of the following would afford a synthesis of the following compound?
CHO
CHO
I
2 CH3CH
II
CH3CH
CHCH3
CH2
+
+
CHO
A)
B)
C)
D)
E)
III
+
CH2
CHCHO
IV
+
CH2
CHCHO
I
II
III
IV
None of these
Ans: D
459
Chapter 13
Topic: Diels-Alder Reaction
47. Which diene would you expect to react most rapidly with maleic anhydride?
O
CH3
CH3
CH3C
CH3
CH3
CH3C
O
I
A)
B)
C)
D)
E)
II
III
IV
V
I
II
III
IV
V
Ans: D
Topic: Diels-Alder Reaction
48. Which of these conjugated dienes can undergo a Diels-Alder reaction?
(CH3)3C
CH2
CH2
CH2
I
A)
B)
C)
D)
E)
II
III
I
II
III
IV
V
Ans: C
460
IV
C(CH3)3
V
Chapter 13
Topic: Diels-Alder Reaction
49. Which of these conjugated dienes can undergo a Diels-Alder reaction?
II
I
IV
A)
B)
C)
D)
E)
III
V
I and V
I, II and V
III
IV
III and IV
Ans: D
461
Chapter 13
Topic: Diels-Alder Reaction
50. Which diene and dienophile would you choose to synthesize the following compound?
O
H
COCH3
O
O
COCH3
and O2
O
O
and
HC
I
C
II
O
O
O
and
O
CH2
CH
COCH3
and
CH2
III
A)
B)
C)
D)
E)
COCH3
IV
I
II
III
IV
None of these
Ans: C
462
CH
COCH3
Chapter 13
Topic: Diels-Alder Reaction
51. Which diene and dienophile would you choose to synthesize the following compound?
H
CHO
H
CHO
I
CHO
CHO
CHO
CHO
II
III
OHC
CHO
CHO
A)
B)
C)
D)
E)
IV
I and VI
II and V
III and IV
IV and V
I and III
Ans: E
CHO
V
VI
463
Chapter 13
Topic: Diels-Alder Reaction
52. Which compounds could be used in a Diels-Alder synthesis of
CH3
CO2CH3
?
CH3
CH3
CH3
CH2
CH3
A)
B)
C)
D)
CHCO2CH3
HC
CCO2CH3
CH3
I
I and III
I and IV
II and III
II and IV
Ans: C
II
III
IV
Topic: Diels-Alder Reaction
53. Which is the major product of the following reaction?
O
H
C
C
OH
?
+
C
H
C
O
OH
H
CO2H
I
A)
B)
C)
D)
E)
H
H
H
CO2H
CO2H
H
H
CO2H
CO2H
CO2H
H
CO2H
H
II
III
I
II
III
IV
None of these
Ans: D
464
CO2H
IV
Chapter 13
Topic: Diels-Alder Reaction
54. Which is the major product of the following reaction?
CHO
?
+
CHO
H
CHO
H
CHO
I
A)
B)
C)
D)
E)
CHO
H
CHO
H
H
CHO
CHO
H
II
III
CHO
H
H
CHO
IV
H
CHO
H
CHO
V
I
II
III
IV
V
Ans: A
Topic: Diels-Alder Reaction
55. Which is the major product of the following reaction?
+
I
A)
B)
C)
D)
E)
H
CHO
CHO
H
CHO
H
CHO
H
H
CHO
H
CHO
II
?
CHO
OHC
III
I
II
III
IV
V
Ans: D
465
CHO
CHO
IV
CHO
CHO
V
Chapter 13
Topic: Diels-Alder Reaction
56. Which is the major product of the following reaction?
+
CHO
H
CHO
H
H
CHO
H
CHO
I
A)
B)
C)
D)
E)
II
?
CHO
CHO
III
I
II
III
IV
V
Ans: III
466
CHO
IV
CHO
V
Chapter 13
Topic: Diels-Alder Reaction
57. Which diene and dienophile would you choose to synthesize the following compound?
O
COCH3
O
O
COCH3
and O2
O
O
and
HC
I
C
II
O
O
and
O
CH2
CH
and
COCH3
CH2
III
A)
B)
C)
D)
E)
IV
I
II
III
IV
None of these
Ans: B
Topic: Diels-Alder Reaction
58. Which of the following can undergo the Diels Alder reaction?
II
I
IV
A)
B)
C)
D)
E)
COCH3
III
V
I and II
II and III
III and IV
I, II and V
V
Ans: C
467
CH
COCH3
Chapter 13
Topic: Diels-Alder Reaction
59. From the standpoint of reactivity, which is the poorest choice of dienophile to react with
2,3-dimethyl-1,3-butadiene in a Diels-Alder reaction?
O
O
I
O
II
H3CO
III
O
O
H3CO
O
OCH3
IV
A)
B)
C)
D)
E)
OCH3
O
V
I
II
III
IV
V
Ans: B
468
Chapter 13
Topic: Diels-Alder Reaction
60. What is the product of the following reaction?
CH
CH2
O
+
O
?
CH3C
C
C
CCH3
COCH3
COCH3
COCH3
COCH3
COCH3
I
COCH3
II
III
COCH3
COCH3
COCH3
COCH3
IV
A)
B)
C)
D)
E)
V
I
II
III
IV
V
Ans: A
469
Chapter 13
Topic: Diels-Alder Reaction
61. Which would be the best synthesis of the following compound?
CO2CH3
CO2CH3
CO2CH3
CO2CH3
CO2CH3
+
+
+
CO2CH3
CO2CH3
I
CH3O2C
II
III
CO2CH3
+
+ 2 HCO2CH3
CO2CH3
IV
A)
B)
C)
D)
E)
V
I
II
III
IV
V
Ans: B
470
Chapter 13
Topic: Diels-Alder Reaction
62. Which reaction would produce the following compound?
H
CO2CH3
CO2CH3
H
H
CO2CH3
CO2CH3
H
CO2CH3
+
+
CH3O2C
H
I
H
II
CO2CH3
+
H
CO2CH3
H
CO2CH3
+
H
CO2CH3
III
A)
B)
C)
D)
E)
H
IV
I
II
III
IV
None of the above
Ans: D
471
Chapter 13
Topic: Diels-Alder Reaction
63. Which of the following pairs of compounds could be used as the basis for a Diels-Alder
synthesis of the compound shown below?
CO2CH3
CO2CH3
I
2 CH3CH
CHCH3
+
II
III
CH3CH
CH2
+
CH2
CHCO2CH3
+
CO2CH3
IV
A)
B)
C)
D)
E)
+
CH2
CHCO2CH3
I
II
III
IV
More than one of the above
Ans: D
472
Chapter 13
Topic: Multi-step Synthesis
64. How could the following synthesis be carried out?
CH2CH3
A)
B)
C)
D)
E)
(1) Br2/CCl4; (2) CH3CH2MgCl, ether; (3) CH3ONa/CH3OH
(1) HBr, 80°C; (2) (CH3CH2)2CuLi, ether
(1) HBr, 80°C; (2) Mg, ether; (3) CH3CH2OH, then H3O+
More than one of the above
All of the above
Ans: B
Topic: Multi-step Reaction Sequence
65. How could the following synthesis be carried out?
A)
B)
C)
D)
E)
(1) Br2/CCl4; (2) CH3MgCl, ether; (3) CH3ONa/CH3OH
(1) HBr (1 eq); (2) (CH3)2CuLi, ether
(1) HBr (1eq); (2) Li, ether; (3) CuI; (4) CH3Br
More than one of the above
All of the above
Ans: C
Topic: Allylic Substitution
66. What product(s) would you expect from the following substitution reaction of 14Clabeled propene?
Cl2
14CH
2
CH
CH3
?
500 oC
A) 14CH2=CH–CH2Cl alone
B) 14CH2=CH–CH2Cl and CH2=CH–14CH2Cl, in equal amounts
C) CH2=CH–14CH2Cl alone
D) more 14CH2=CHCH2Cl, but a little CH2=CH–14CH2Cl
E) more CH2=CH–14CH2Cl, but a little 14CH2=CHCH2Cl
Ans: B
473
Chapter 13
Topic: Allylic Substitution
67. What product(s) would you expect from the following substitution reaction?
* Cl2
?
500oC
Cl
Cl
Cl
*
I
A)
B)
C)
D)
E)
*
II
Cl
*
III
*
IV
I
II
III
IV
More than one of the above
Ans: E
Topic: Synthesis of Allylic Halides
68. Which of the following could be used to synthesize 3-bromopropene?
A)
25 oC
CH3CH
CH2
+ Br2
CCl4
ROOR
B)
CH3CH
CH2
+ N-bromosuccinimide
CCl4
C) CH2=CHCH2OH + PBr3 ⎯⎯⎯⎯→
D) More than one of these
E) None of these
Ans: D
Topic: Synthesis of Allylic Halides
69.
A)
B)
C)
D)
E)
Which of the following could be used to synthesize 3-bromocyclopentene?
Cyclopentene + Br2, CCl4 , 25o
Cyclopentene + NBS, CCl4 (ROOR)
3-Cyclopentenol + PBr3
Both A) and B)
Both B) and C)
Ans: E
474
Chapter 13
Topic: Synthesis of Allylic Halides
70. Treatment of 2-butene (cis or trans) with N-bromosuccinimide in CCl4 would yield
mainly:
A) CH3CHBrCHBrCH3 alone
B) CH3CH=CBrCH3 alone
C) CH3CH=CHCH2Br alone
D) CH2BrCH2CH=CH2 and CH3CH=CHCH2Br
E) CH3CHBrCH=CH2 and CH3CH=CHCH2Br
Ans: E
Topic: Synthesis of Allylic Halides
71. Treatment of 4-methylcyclohexene with N-bromosuccinimide in CCl4 would yield
mainly:
Br
Br
Br
Br
I
A)
B)
C)
D)
E)
II
III
I
II
III
IV
All of the above
Ans: E
475
IV
Chapter 13
Topic: Multistep Reactions
72. Which would be the best synthesis?
?
CH2
CHCH3
CH2CHCH2Cl
Br Br
A)
Propene
B)
Propene
C)
D)
E)
Propene
Propene
Propene
Ans: B
Cl2, hν
Br2, CCl4
Cl2, 400 oC
Br2, CCl4
Br2, hν
HCl
NBS, CCl4
Cl2, CCl4
NBS, CCl4
Cl2, CCl4
Topic: Ozonolysis of Polyenes
73. Which is the diene that yields on ozonolysis (O3, followed by Zn/HOAc) an equimolar
CHO
CH2
mixture of CH2O, CH3CHO, and CHO
CH2
CH
CH
CH
CH2
CH3
CH3
I
CH2
III
II
CH
CH2
CH
CH
CH3
CH2
IV
A)
B)
C)
D)
E)
CH
CH2
CH2
V
I
II
III
IV
V
Ans: D
476
CH
CH2
Chapter 13
Topic: Allylic Halogenation
74. What is (are) the product(s) of the following reaction?
Cl2
?
400oC
Cl
Cl
Cl
I
A)
B)
C)
D)
E)
II
III
I
II
III
a mixture of I and II
a mixture of II and III
Ans: E
Topic: Allylic Halogenation
75.
A)
B)
C)
D)
E)
Which set of conditions does not result in allylic halogenation of an alkene?
Cl2 at 400°C
Cl2 in CCl4 at 25°C
Cl2, ROOR,hν
Br2 at low concentration in CCl4
N-Bromosuccinimide in CCl4, ROOR
Ans: B
Topic: Allylic Halogenation
CH3
76.
CH3
A)
B)
C)
D)
E)
CH
CH2
CH
CH2
IV
III
II
I
is predicted to be the major site of
Which carbon of V
substitution when this alkene reacts with chlorine at 400°C?
I
II
III
IV
V
Ans: C
477
Chapter 13
Topic: Allylic Halogenation
77. Which carbon is predicted to be the major site of substitution when this alkene reacts
with NBS?
V
I
A)
B)
C)
D)
E)
III
IV
II
I
II
III
IV
V
Ans: B
Topic: Diene Synthesis
78.
A)
B)
C)
D)
E)
Which of these is not a useful method for the synthesis of 1,3-pentadiene?
1,4-pentanediol + H2SO4 at 180oC
2,4-dibromopentane + (CH3)3COK , (CH3)3COH at 75oC
2,4-pentanediol + H2SO4 at 180oC
HC≡CCH=CHCH3 + H2, Ni2B (P-2)
1,4-dibromopentane + CH3CH2ONa , CH3CH2OH at 75oC
Ans: B
478
Chapter 13
Topic: Conjugate Addition Reactions
79. Ignoring stereochemistry, the 1:1 reaction of bromine with 1,3-cyclohexadiene at 25°C
in the dark and in the absence of peroxide forms which of these?
Br
Br
Br
Br
Br
I
A)
B)
C)
D)
E)
Br
II
III
Br
IV
I
II
III
IV
Both I and II
Ans: E
Topic: Conjugate Addition Reactions
80. Which is the only compound which can be completely ruled out as a product of the
reaction of 1,3-butadiene with HCl?
A) (S)-3-chloro-1-butene
B) (R)-3-chloro-1-butene
C) (E)-1-chloro-2-butene
D) (Z)-1-chloro-2-butene
E) (Z)-2-chloro-2-butene
Ans: E
Topic: Diene Synthesis
81. Hot alumina can be used as an effective dehydrating agent: when 3-methyl-2,3pentanediol vapor is passed over hot alumina (Al2O3), the chief product to be expected
is which of these?
A) 2-Ethyl-1,3-butadiene
B) 3-Methyl-1,3-pentadiene
C) 3-Methyl-1,2-pentadiene
D) 3-Methyl-2,3-pentadiene
E) 3-Methyl-1,4-pentadiene
Ans: B
479
Chapter 13
Topic: Allylic Halogenation
82.
A)
B)
C)
D)
E)
An unsaturated product results from the reaction of cyclohexene with which of these?
Br2/CCl4 at 25°C
NBS/CCl4, ROOR
HCl, ROOR
HCl, no peroxides
More than one of these
Ans: B
Topic: Conjugate Addition Reactions
83. Which of the following dienes might react with bromine in CCl4 to yield 2,5-dibromo-3hexene?
A) CH2=CHCH2CH2CH=CH2
B) CH2=CHCH=CHCH2CH3
C) CH3CH=C=CHCH2CH3
D) CH3CH=CHCH=CHCH3
E) CH3CH=CHCH2CH=CH2
Ans: D
Allylic Halogenation
84. What product(s) would you expect from the following substitution reaction of 14Clabeled propene?
N-bromosuccinimide
14CH
2
CH
?
CH3
ROOR, CCl4
A) 14CH2=CH–CH2Br alone
B) 14CH2=CH–CH2Br and CH2=CH–14CH2Br in equal amounts
C) CH2=CH–14CH2Br alone
D) More 14CH2=CHCH2Br but a little CH2=CH–14CH2Br
E) More CH2=CH14CH2Br but a little 14CH2=CHCH2Br
Ans: B
480
Chapter 13
Allylic Halogenation
85.
A)
B)
C)
D)
E)
Treatment of 2-butene (cis or trans) with Cl2 at 400°C would yield mainly:
CH2ClCHClCH2CH3
CH3CHClCH2CH3
CH3CH=CClCH3
CH3CH=CHCH2Cl and CH3CHClCH=CH2
CH3CHClCHClCH3
Ans: D
Topic: Conjugate Addition Reactions
86. Indicate which products would be obtained from the chlorination of 1,5-hexadiene at
high temperature (500°C), using a 1:1 mole ratio of the reactants.
ClCH2CHCH2CH2CH
ClCH2CH
CH2
CHCH
CHCH2Cl
Cl
I
II
ClCH2CH
CHCH2CH
CH2
III
CH2
CHCH2CHCH
Cl
CH2
ClCH2CHCH2CH2CHCH2Cl
IV
A)
B)
C)
D)
E)
Cl
Cl
V
I and II
II and III
III and IV
IV and V
V and I
Ans: C
481
Chapter 13
Topic: 1,2 Reactions of Conjugated Dienes
87. Which reagent would convert 1,3-pentadiene into 3-penten-2-ol?
A)
B)
C)
D)
E)
KMnO4/-OH
OsO4
H2O2, then H3O+
Cl2/H2O
H3O+
Ans: E
Topic: 1,2 Reactions of Conjugated Dienes
88.
A)
B)
C)
D)
E)
Which reagent would convert 1,3-octadiene into 3-octen-2-ol?
KMnO4/-OH
OsO4
H2O2, then H3O+
Cl2/H2O
H3O+
Ans: E
Topic: UV-Vis Spectroscopy
89. Which compound would have an UV absorption band at longest wavelength?
CHCH
CH
CH
CHCH3
I
II
CH2CH
CH2
III
CH
IV
A)
B)
C)
D)
E)
CH2
V
I
II
III
IV
V
Ans: B
482
CHCH3
CHCH3
Chapter 13
Topic: UV-Vis Spectroscopy
90.
A)
B)
C)
D)
E)
Select the most energetically favorable UV transition for 1,3-butadiene.
n ⎯⎯⎯→ σ*
n ⎯⎯⎯→ π*
π2 ⎯⎯⎯→ π3*
σ ⎯⎯⎯→ σ*
π1 ⎯⎯⎯→ π4*
Ans: C
Topic: UV-Vis Spectroscopy
91. Which compound would have an UV absorption band at longest wavelength?
CH
CHCH
CH2
CH
I
CHCH
CH
II
CH2CH2CH
CH2
CHCH
CH2
III
CH
IV
A)
B)
C)
D)
E)
CH2
CHCH
CH2
V
I
II
III
IV
V
Ans: A
Topic: UV-Vis Spectroscopy
92. Which of these symbols is used in connection with the intensity of absorption in the
UV-visible region?
A) ∝
B) ν
C) ε
D) λ
E) θ
Ans: C
483
Chapter 13
Topic: Allylic Halogenation
93. Which of the following compounds is not formed as a result of a chain-termination step
in the free radical chlorination of propene?
A) CH2=CHCH2Cl
B) Cl2
C) CH2=CHCH2CH2CH=CH2
D) HCl
E) All can be formed in chain-termination steps.
Ans: D
SHORT ANSWER QUESTIONS
Topic: General
94. Systems that have a p orbital on an atom adjacent to a double bond are called
______________ systems.
Ans: conjugated unsaturated
Topic: General
95. The hydrogen atom on a saturated (sp3) carbon adjacent to a double bond is called an
______________ hydrogen.
Ans: allylic
Topic: General
96. The hydrogen atom on a carbon that is part of a double bond is called a ___________
hydrogen.
Ans: vinyl or vinylic
Topic: General
97. Allylic radicals are ___________ stable than tertiary radicals.
Ans: more
Topic: General
98. Vinyl cations are _____________ stable than tertiary cations.
Ans: less
484
Chapter 13
Topic: General
99. There are three types of polyenes (molecules containing two or more double bonds).
They are: _________________.
Ans: conjugated, cumulated, isolated
Topic: General
100. Polyenes in which single and double bonds alternate along the carbon chain are called
___________.
Ans: conjugated
Topic: General
101. UV-vis spectroscopy can be used to indicate whether _________________ is present.
Ans: conjugation
Topic: General
102. When the product distribution for a particular reaction reflects the relative stabilities of
the products, the reaction is said to be under ______________ control.
Ans: thermodynamic
Topic: General
103. When the product distribution for a particular reaction reflects the relative rate at which
the products are formed, the reaction is said to be under _______________ control.
Ans: kinetic
Topic: General
104. The Diels-Alder reaction is a cycloaddition between a conjugated ________ and a
___________.
Ans: diene; dienophile
Topic: General
105. A thermodynamically-controlled reaction will yield predominantly:___________.
Ans: the more/most stable product.
Topic: General
106. A reaction under kinetic (or rate) control will yield predominantly:___________.
Ans: the product whose formation requires the smallest free energy of activation.
485
Chapter 13
Topic: General
107. Stereochemically speaking, the Diels-Alder reaction is ___________ and occurs with
___________ of the dienophile stereochemistry.
Ans: stereospecific; retention
Topic: General
108. Conjugated dienes routinely undergo 1,2 and 1,4 addition reactions with a variety of
electrophilic reagents; this suggests that ___________ are likely intermediates during
these reactions.
Ans: allylic carbocations
Topic: Diels –Alder Reactions
109. During Diels Alder reactions, when two stereoisomer products, exo and endo, are
possible, the ___________ product is typically obtained as the major product.
Ans: endo
Topic: Nomenclature
110. Draw the structural formula for (2E,4Z,6E)–3,4,7,8–tetramethyl-2,4,6–nonatriene,
clearly indicating stereochemical details.
Ans:
Topic: Nomenclature
111. Draw the structural formula for (2Z,4Z,6Z)-3,4,8-trimethyl-2,4,6-nonatriene,clearly
indicating stereochemical details.
Ans:
Topic: Nomenclature
112. Draw the structural formula for (R)-5-bromo-2,5-dimethyl-1,3-cyclopentadiene, clearly
indicating stereochemical details.
CH3
Ans:
Br
486
Chapter 13
Topic: Multistep Reactions
113. Complete the following sequence of reactions, giving structural details of all key
intermediates.
O
heat
+
A
O
H2
Ni
B
Ans:
O
O
O
heat
+
+
O
O
O
H2, Ni
O
O
+
O
O
+
+
O
O
+
O
487
O
Chapter 13
Topic: Multistep Reactions
114. Complete the following sequence of reactions, giving structural details of all key
intermediates.
i) NBS
ii) (CH3)3COK, (CH3)3COH, heat
iii) trans CH3CH=CHCHO
?
Br
Ans:
(CH3)3COK,
(CH3)3COH,
heat
NBS
O
O
+
O
Topic: Multistep Reactions
115. Complete the following sequence of reactions, giving structural details of all key
intermediates.
i) cis-CH3CH=CHCHO
?
ii) CH3CH2MgBr
+
iii) H3O
Ans:
H
H
CHO
CH3
H
i) CH3CH2MgBr
CH3
H
ii) H3O+
O
HO
Topic: Diels Alder Reaction
116. What reagents would be needed to synthesize the following substance via the DielsAlder reaction? Give stereochemical details, as relevant.
O
O
O
O
Ans:
+
O
O
488
Chapter 14
MULTIPLE CHOICE QUESTIONS
Topic: Bonding and Resonance
1. Which of the following is NOT true of benzene?
A) Benzene tends to undergo substitution rather than addition reactions, even though it has
a high index of hydrogen deficiency.
B) All of the hydrogen atoms of benzene are equivalent.
C) The carbon-carbon bonds of benzene are alternately short and long around the ring.
D) Only one o-dichlorobenzene has ever been found.
E) Benzene is more stable than the hypothetical compound 1,3,5-cyclohexatriene.
Ans: C
Topic: Bonding and Resonance
2.
A)
B)
C)
D)
E)
Which of the following is true of benzene?
Benzene tends to undergo addition rather than substitution reactions.
All of the hydrogen atoms of benzene are equivalent.
The carbon-carbon bonds of benzene are alternately short and long around the ring.
The benzene ring is a distorted hexagon.
Benzene has the stability expected for cyclohexatriene.
Ans: B
Topic: Bonding and Resonance
3.
A)
B)
C)
D)
E)
The carbon-carbon bonds in benzene are:
of equal length and are shorter than the double bond of ethene.
of equal length and are intermediate between a double bond and a single bond.
of unequal length and are alternately short and long around the ring.
due only to p-orbital overlap.
of equal length and intermediate between the carbon-carbon bond lengths in ethene and
ethyne.
Ans: B
Topic: Nomenclature, Bonding and Resonance
4. In which of the following compounds would the shortest carbon-carbon bond(s) be
found?
A) Toluene
B) 2-Ethylcyclopentene
C) 4-Methyl-1,3-cyclohexadiene
D) 3-Methyl-1-hexyne
E) 3-Methyl-1,3-hexadiene
Ans: D
Page 489
Chapter 14
Topic: Bonding and Resonance
5. We now know that the two Kekule structures for benzene are related in the following
way:
A) They are each equally correct as a structure for benzene.
B) Benzene is sometimes one structure and sometimes the other.
C) The two structures are in a state of rapid equilibrium.
D) Neither of the two structures adequately describes benzene; benzene is a resonance
hybrid of the two.
E) None of the above
Ans: D
Topic: Nomenclature, Bonding and Resonance
6. In which of the following compounds would the longest carbon-carbon bond(s) be
found?
A) 2-bromobenzaldehyde
B) Vinylbenzene
C) 1,3,5-heptatriene
D) 2,4,6-octatriene
E) 2-Ethylbenzoic acid
Ans: D
Topic: Aromatic/ Antiaromatic/ Nonaromatic
7.
A)
B)
C)
D)
E)
Which of the following statements regarding the cyclopentadienyl radical is correct?
It is aromatic.
It is not aromatic.
It obeys Huckel's rule.
It undergoes reactions characteristic of benzene.
It has a closed shell of 6 pi-electrons.
Ans: B
490
Chapter 14
Topic: Aromaticity, Huckel Rule
8. Which of the following would you expect to be aromatic?
H
H
A)
B)
C)
D)
E)
I
I
II
III
IV
V
Ans: E
II
III
IV
V
Topic: Aromaticity, Huckel Rule
9. Which of these would you expect to have significant resonance stabilization energy?
N
H
A)
B)
C)
D)
E)
N
I
II
I
II
III
All of the above
None of the above
Ans: D
III
491
Chapter 14
Topic: Aromaticity, Huckel Rule
10. Of the following C-10 compounds, which is expected to possess the greatest resonance
(delocalization) energy?
I
A)
B)
C)
D)
E)
II
III
IV
I
II
III
IV
V
Ans: C
Topic: Aromaticity, Huckel Rule
11. Which of the following would you expect to be aromatic?
H H
A)
B)
C)
D)
E)
I
I
II
III
IV
None of these
Ans: B
II
III
IV
492
V
Chapter 14
Topic: Aromaticity, Huckel Rule
12. Which compound would you NOT expect to be aromatic?
N
A)
B)
C)
D)
E)
I
I
II
III
IV
V
Ans: E
S
N
H
O
B
R
II
III
IV
V
Topic: Aromaticity, Huckel Rule
13.
A)
B)
C)
D)
E)
Which annulene would you NOT expect to be aromatic?
[6]-Annulene
[14]-Annulene
[16]-Annulene
[18]-Annulene
[22]-Annulene
Ans: C
Topic: Aromaticity, Huckel Rule
14. Which of the following would you expect to be aromatic?
A)
B)
C)
D)
E)
I
II
I
II
III
IV
All of these
Ans: A
III
IV
493
Chapter 14
Topic: Aromaticity, Huckel Rule
15. Which of these species is aromatic?
A)
B)
C)
D)
E)
O
O
N
H
O
N
H
O
S
I
I
II
III
IV
V
Ans: B
II
III
IV
V
Topic: Aromaticity, Huckel Rule
16. Which of the following structures would be aromatic?
K
-2
K
Br
Br
2K
A)
B)
C)
D)
E)
I
I
II
III
IV
V
Ans: B
II
III
494
IV
V
Chapter 14
Topic: Aromaticity, Huckel Rule
17. Which of the following would you expect to be antiaromatic?
A)
B)
C)
D)
E)
I
I
II
III
IV
V
Ans: C
II
III
IV
V
Topic: Aromaticity, Huckel Rule
18. Which of the following would you expect to be aromatic?
H H
I
A)
B)
C)
D)
E)
II
III
IV
I
II
III
IV
None of the above
Ans: C
495
Chapter 14
Topic: Aromaticity, Huckel Rule
19. On the basis of molecular orbital theory and Huckel's rule, which molecules and/or ions
should be aromatic?
I
H
H H
II
III
H
IV
V
N
H
A)
B)
C)
D)
E)
H
VI
VII
I and V
III and VIII
IV, VII and IX
IV, VI, VII and IX
All of the structures, I-IX
Ans: D
H
H
VIII
IX
Topic: Aromaticity, Huckel Rule
20. On the basis of molecular orbital theory and Huckel's rule, which of these compounds
should be aromatic?
H H
A)
B)
C)
D)
E)
I
I
II
III
IV
V
Ans: E
II
III
IV
V
496
Chapter 14
Topic: Aromaticity, Huckel Rule
21. Of Huckel's requirements for aromatic character, only this one is waived in the case of
certain compounds considered to be aromatic.
A) The ring system must be planar.
B) The system must be monocyclic.
C) There must be (4n + 2) π electrons.
D) The Huckel number of electrons must be completely delocalized.
E) None. All of these rules must apply in every case.
Ans: B
Topic: Aromaticity, Huckel Rule
22. Which of these is an aromatic molecule?
N
O
B
CH3
A)
B)
C)
D)
E)
I
I
II
III
IV
V
Ans: A
CH3
II
III
N
S
H
O
IV
V
Topic: Aromaticity, Huckel Rule
23. In which case is the indicated unshared pair of electrons NOT a contributor to the π
aromatic system?
N
H
A)
B)
C)
D)
E)
O
N
I
II
I
II
III
IV
None of these
Ans: B
H
III
IV
497
Chapter 14
Topic: Chemical Tests and Reactions
24. Which reagent(s) would serve as the basis for a simple chemical test that would
distinguish between benzene and 1-hexene?
A) NaOH in H2O
B) Br2 in CCl4
C) AgNO3 in C2H5OH
D) NaHSO3 in H2O
E) None of these
Ans: B
Topic: Chemical Tests and Reactions
25. Which reagent(s) would serve as the basis for a simple chemical test that would
distinguish between ethylbenzene and vinylcyclohexane?
A) H2CrO4
B) LAH
C) NaBH4, H2O
D) KMnO4, OH−, H2O
E) Two of the above
Ans: D
Topic: Chemical Tests and Reactions
26. Which reagent(s) would serve as the basis for a simple chemical test that would
distinguish between naphthalene and 2,4,6-decatriene?
A) NaOH in H2O
B) KMnO4, OH−, H2O
C) NaBH4, H2O
D) H2CrO4
E) None of these
Ans: B
Topic: Chemical and Spectroscopic Analysis
27. Which reagent(s)/technique would serve to distinguish between azulene and
bicyclo[5.3.0]decane?
A)
B)
C)
D)
E)
Br2, CCl4
KMnO4, OH−, H2O
NMR Spectroscopy
UV Spectroscopy
Two of these
Ans: E
498
Chapter 14
Topic: Chemical Tests and Reactions
28. Which is the only one of these reagents which will react with benzene under the
specified conditions?
A) Cl2, FeCl3, heat
B) H2, 25°C
C) Br2/CCl4, 25°C, dark
D) KMnO4/H2O, 25°C
E) H3O+, heat
Ans: A
Topic: Nomenclature and Isomerism
29.
A)
B)
C)
D)
E)
Which dibromobenzene can, in theory, yield three mononitro derivatives?
o-Dibromobenzene
m-Dibromobenzene
p-Dibromobenzene
All of these
None of these
Ans: B
Topic: Nomenclature and Isomerism
30. If thiophene is an aromatic molecule and reacts similarly to benzene, how many
(neutral) monobromothiophenes could be obtained in the following reaction?
S
A)
B)
C)
D)
E)
Br2
FeBr3
1
2
3
4
5
Ans: B
499
Chapter 14
Topic: Nomenclature and Isomerism
31. 2-Bromo-4-nitroaniline is:
Br
Br
NH2
NO2
Br
NO2
NH2
NO2
I
II
H2N
III
OCH3
Br
Br
A)
B)
C)
D)
E)
H3CO
NO2
NO2
IV
V
I
II
III
IV
V
Ans: C
Topic: Nomenclature and Isomerism
32. 4-Bromo-2-nitroaniline is:
Br
Br
NH2
NO2
Br
NO2
NH2
NO2
I
II
H2N
III
OCH3
Br
A)
B)
C)
D)
E)
Br
H3CO
NO2
NO2
IV
V
I
II
III
IV
V
Ans: A
500
Chapter 14
Topic: Nomenclature and Isomerism
33. 4-Amino-2-chlorophenol is:
Cl
OH
Cl
O
O
Cl
NH2
NO2
I
II
H2N
III
OH
OH
Cl
A)
B)
C)
D)
E)
Cl
NH2
NO2
IV
V
I
II
III
IV
V
Ans: D
Topic: Nomenclature and Isomerism
34. 4-Chloro-3-nitro-benzaldehyde is:
Cl
OH
Cl
O
O
Cl
NH2
NO2
I
II
H2N
III
OH
Cl
A)
B)
C)
D)
E)
OH
Cl
NH2
NO2
IV
V
I
II
III
IV
V
Ans: B
501
Chapter 14
Topic: Nomenclature and Isomerism
35. 4-amino-2-chlorobenzaldehyde is:
Cl
OH
Cl
O
O
Cl
NH2
NO2
I
II
H2N
III
OH
OH
Cl
A)
B)
C)
D)
E)
Cl
NH2
NO2
IV
V
I
II
III
IV
V
Ans: C
Topic: Nomenclature and Isomerism
36. 4-amino-2-bromoanisole is:
Br
Br
Br
OCH3
O
H3CO
NH2
NO2
I
II
H2N
III
OCH3
Br
A)
B)
C)
D)
E)
OCH3
Br
NH2
NO2
IV
V
I
II
III
IV
V
Ans: D
502
Chapter 14
Topic: Nomenclature and Isomerism
37. 4-bromomethyl-2-nitroanisole is:
Br
Br
Br
OCH3
O
H3CO
NH2
NO2
I
II
H2N
III
OCH3
Br
A)
B)
C)
D)
E)
OCH3
Br
NH2
NO2
IV
V
I
II
III
IV
V
Ans: B
Topic: Nomenclature and Isomerism
38.
A)
B)
C)
D)
E)
How many different dibromophenols are possible?
8
7
6
5
4
Ans: C
Topic: Nomenclature and Isomerism
39. The correct name for the compound shown below is:
Br
Br
NH2
A)
B)
C)
D)
E)
3,4-Dibromoaniline
2,4-Dibromoaniline
2,5-Dibromoaniline
3,6-Dibromoaniline
2,6-Dibromoaniline
Ans: C
503
Chapter 14
Topic: Nomenclature and Isomerism
40.
A)
B)
C)
D)
E)
Which dibromobenzene can yield only one mononitro derivative?
o-Dibromobenzene
m-Dibromobenzene
p-Dibromobenzene
More than one of these
None of these
Ans: C
Topic: Nomenclature and Isomerism
41. A dichlorobenzene that on reaction with nitric acid and sulfuric acid might theoretically
yield three mononitro products would be:
A) o-Dichlorobenzene
B) m-Dichlorobenzene
C) p-Dichlorobenzene
D) None of these
E) All of these
Ans: B
Topic: Nomenclature and Isomerism
42.
A)
B)
C)
D)
E)
In theory, a single molecule of this compound will rotate plane-polarized light.
Butylbenzene
Isobutylbenzene
sec-Butylbenzene
tert-Butylbenzene
None of these
Ans: C
504
Chapter 14
Topic: Nomenclature and Isomerism
43. Which of the following is NOT 2-bromo-5-nitrobenzoic acid?
Br
Br
HO
OH
O2 N
NO2
O
O
O
I
Br
II
Br
III
HO
HO
NO2
IV
A)
B)
C)
D)
E)
V
I
II
III
IV
V
Ans: D
Topic: Nomenclature and Isomerism
44.
A)
B)
C)
D)
E)
Toluene is the name commonly assigned to:
Hydroxybenzene
Aminobenzene
Methylbenzene
Ethylbenzene
Methoxybenzene
Ans: C
Topic: Nomenclature and Isomerism
45.
A)
B)
C)
D)
E)
O
Br
O
NO2
HO
Anisole is the name commonly assigned to:
Hydroxybenzene
Aminobenzene
Methylbenzene
Ethylbenzene
Methoxybenzene
Ans: E
505
Chapter 14
Topic: Nomenclature and Isomerism
46.
A)
B)
C)
D)
E)
Phenol is the name commonly assigned to:
Hydroxybenzene
Aminobenzene
Methylbenzene
Ethylbenzene
Methoxybenzene
Ans: A
Topic: Nomenclature and Isomerism
47.
A)
B)
C)
D)
E)
Aniline is the name commonly assigned to:
Hydroxybenzene
Aminobenzene
Methylbenzene
Ethylbenzene
Methoxybenzene
Ans: B
Topic: Nomenclature and Isomerism
48. The complete name for the following compound is:
CH3
H
A)
B)
C)
D)
E)
sec-Hexylbenzene
2-Phenylhexane
(R)-2-Phenylhexane
(S)-2-Phenylhexane
Butylmethylphenylmethane
Ans: D
Topic: Nomenclature and Isomerism
49.
A)
B)
C)
D)
E)
How many dichloronaphthalenes are possible?
7
8
9
10
12
Ans: D
506
Chapter 14
Topic: Nomenclature and Isomerism
50. Which of these is the single best representation for naphthalene?
II
I
IV
A)
B)
C)
D)
E)
III
V
I
II
III
IV
V
Ans: A
Topic: Molecular Orbital Theory
51. How many equivalent resonance structures can be written for the cyclopentadienyl
anion?
A) 3
B) 4
C) 5
D) 6
E) 7
Ans: C
Topic: Molecular Orbital Theory
52. In the molecular orbital model of benzene, the six p-orbitals combine to form how many
molecular orbitals?
A) 6
B) 5
C) 4
D) 3
E) 2
Ans: A
507
Chapter 14
Topic: Molecular Orbital Theory
53. Consider the molecular orbital model of benzene. In the ground state how many
molecular orbitals are filled with electrons?
A) 1
B) 2
C) 3
D) 4
E) 5
Ans: C
Topic: Molecular Orbital Theory
54. In the molecular orbital model of benzene, how many pi-electrons are delocalized about
the ring?
A) 2
B) 3
C) 4
D) 5
E) 6
Ans: E
Topic: Molecular Orbital Theory
55. In the molecular orbital model of benzene, how many pi-electrons are in bonding
molecular orbitals?
A) 6
B) 5
C) 4
D) 3
E) 2
Ans: A
Topic: Molecular Orbital Theory
56. Consider the molecular orbital model of cyclopentadienyl anion. In the ground state
how many molecular orbitals are filled with electrons?
A) 1
B) 2
C) 3
D) 4
E) 5
Ans: C
508
Chapter 14
Topic: Molecular Orbital Theory
57. In the molecular orbital model of cyclopentadienyl anion, how many pi-electrons are
delocalized about the ring?
A) 2
B) 3
C) 4
D) 5
E) 6
Ans: E
Topic: Molecular Orbital Theory
58. In the molecular orbital model of cyclopentadienyl anion, how many pi-electrons are in
bonding molecular orbitals?
A) 6
B) 5
C) 4
D) 3
E) 2
Ans: A
Topic: Molecular Orbital Theory
59. In the molecular orbital model of cycloheptadienyl cation, how many pi-electrons are in
bonding molecular orbitals?
A) 6
B) 5
C) 4
D) 3
E) 2
Ans: A
Topic: Molecular Orbital Theory
60. In the molecular orbital model of cycloheptadienyl cation, how many pi-electrons are
delocalized about the ring?
A) 6
B) 5
C) 4
D) 3
E) 2
Ans: A
509
Chapter 14
Topic: Molecular Orbital Theory
61. In the molecular orbital model of 1,3-cyclobutadiene, how many electrons are in nonbonding molecular orbitals?
A) 6
B) 5
C) 4
D) 3
E) 2
Ans: E
Topic: Molecular Orbital Theory
62. In the molecular orbital model of which of the following substance(s) are non-bonding
orbitals found?
A) 1,3-Cyclobutadiene
B) Cyclopentadienyl anion
C) 1,3,5,7-Cyclooctatetraene
D) Two of the above
E) All of the above
Ans: D
Topic: Molecular Orbital Theory
63. Which cyclization(s) should occur with a decrease in pi-electron energy?
A)
B)
C)
D)
E)
I
CH2
CH
CH
II
CH2
CH
CH2
III CH2
CH
CH2
IV CH2
CH
CH2
CH2
+ H2
+ H2
+ H2
+ H2
I
II
III
IV
All of the above
Ans: B
510
Chapter 14
Topic: Molecular Orbital Theory
64. Application of the polygon-and-circle technique reveals that single electrons occupy
each of the two nonbonding orbitals in the molecular orbital diagram of:
A) Cyclobutadiene
B) Benzene
C) Cyclopropenyl cation
D) Cyclopentadienyl anion
E) Cycloheptatrienyl cation
Ans: A
Topic: Explanations and Predictions
65.
A)
B)
C)
D)
E)
Why would 1,3-cyclohexadiene undergo dehydrogenation readily?
It is easily reduced.
Hydrogen is a small molecule.
1,3-Cyclohexadiene has no resonance energy.
It would gain considerable stability by becoming benzene.
It would not undergo dehydrogenation.
Ans: D
Topic: Explanations and Predictions
66.
A)
B)
C)
D)
E)
Which of the following statements about cyclooctatetraene is NOT true?
The compound rapidly decolorizes Br2/CCl4 solutions.
The compound rapidly decolorizes aqueous solutions of KMnO4.
The compound readily adds hydrogen.
The compound is nonplanar.
The compound is comparable to benzene in stability.
Ans: E
Topic: Explanations and Predictions
67. Cyclopentadiene is unusually acidic for a hydrocarbon. An explanation for this is the
following statement.
A) The carbon atoms of cyclopentadiene are all sp2-hybridized.
B) Cyclopentadiene is aromatic.
C) Removal of a proton from cyclopentadiene yields an aromatic anion.
D) Removal of a hydrogen atom from cyclopentadiene yields a highly stable free radical.
E) Removal of a hydride ion from cyclopentadiene produces an aromatic cation.
Ans: C
511
Chapter 14
Topic: Explanations and Predictions
68. Recalling that benzene has a resonance energy of 152 kJ mol-1 and naphthalene has a
resonance energy of 255 kJ mol-1, predict the positions which would be occupied by
bromine when phenanthrene (below) undergoes addition of Br2.
3
4
5
6
2
7
1
8
10
A)
B)
C)
D)
E)
9
1, 2
1, 4
3, 4
7, 8
9, 10
Ans: E
Topic: Spectroscopy
69. Which isomer of C7H7Cl exhibits strong IR absorbances at 690 and 750 cm1?
CH3
CH3
CH3
CH2Cl
Cl
Cl
Cl
I
A)
B)
C)
D)
E)
II
III
IV
I
II
III
IV
II and IV
Ans: E
512
Chapter 14
Topic: Spectroscopy
70. Which of these compounds absorbs at the longest wavelength in the UV-visible region?
I
A)
B)
C)
D)
E)
II
III
IV
V
I
II
III
IV
V
Ans: B
Topic: Spectroscopy
71. A compound has the formula C8H9Br. Its 1H NMR spectrum consists of:
doublet, δ 2.0
quartet, δ 5.15
multiplet, δ 7.35
The IR spectrum shows two peaks in the 680-840 cm-1 region; one is between 690 and
710 cm-1 and the other is between 730 and 770 cm-1. Which is a possible structure for
the compound?
Br
Br
Br
II
I
III
Br
Br
IV
A)
B)
C)
D)
E)
V
I
II
III
IV
V
Ans: D
513
Chapter 14
Topic: Spectroscopy
72. Which of the following substances, formula C8H9Br, might exhibit the following 1H
NMR spectrum ?
triplet, δ 2.8
triplet, δ 4.65
multiplet, δ 7.2
Br
Br
Br
II
I
III
Br
Br
IV
A)
B)
C)
D)
E)
V
I
II
III
IV
V
Ans: C
514
Chapter 14
Topic: Spectroscopy
73. Which of the following substances, formula C8H9Br, might exhibit the following 1H
NMR spectrum ?
triplet, δ 1.2
quartet, δ 2.45
multiplet, δ 7.4
Br
Br
Br
II
I
III
Br
Br
IV
A)
B)
C)
D)
E)
V
I
II
III
IV
V
Ans: A
515
Chapter 14
Topic: Spectroscopy
74. Which of the following substances, formula C8H9Br, might exhibit the following 1H
NMR spectrum ?
singlet, δ 2.45
singlet, δ 2.5
multiplet, δ 7.35
Br
Br
Br
II
I
III
Br
Br
V
IV
A)
B)
C)
D)
E)
I
II
III
IV
V
Ans: E
Topic: Spectroscopy
75. Which of the following substances, C8H9Cl, would exhibit five signals in its 13C NMR
spectrum?
Cl
Cl
Cl
I
III
II
Cl
Cl
IV
A)
B)
C)
D)
E)
V
I
II
III
IV
V
Ans: C
516
Chapter 14
Topic: Spectroscopy
76. Which of the following substances, C9H10O, would exhibit seven signals in its 13C NMR
spectrum?
O
O
O
I
II
III
HO
O
V
IV
A)
B)
C)
D)
E)
I
II
III
IV
V
Ans: A
Topic: Spectroscopy
77. For which of the following substances, C9H10O, would the aromatic multiplet, in its 1H
NMR spectrum, consist of two doublets?
O
O
O
I
II
III
HO
O
IV
A)
B)
C)
D)
E)
V
I
II
III
IV
V
Ans: A
517
Chapter 14
Topic: Spectroscopy
78. In the 1H NMR spectra of which of the following substances, C9H10O, would you expect
to see two singlets in the range of δ 2.0-2.5 ppm?
O
O
I
O
II
III
HO
O
IV
A)
B)
C)
D)
E)
V
I and III
I, II, and III
IV
V
All of the above substances would exhibit two singlets δ 2.0-2.5 ppm in their 1H NMR
spectra
Ans: B
SHORT ANSWER QUESTIONS
Topic: General Concepts
79. A compound that reacts like an alkane, alkene, alkyne, or one of their derivatives, is
called an _____________ compound, while one that reacts like benzene is called an
_____________ compound.
Ans: aliphatic, aromatic
Topic: Nomenclature
80. When a benzene ring is used as a substituent, it is called a _________ group.
Ans: phenyl
Topic: General Concepts
81. The difference between the amount of heat actually released upon hydrogenation of
benzene and that calculated on the basis of the Kekule structure is called the
____________________ of benzene.
Ans: resonance energy
518
Chapter 14
Topic: General Concepts
82. Molecular orbitals of equal energy are referred to as _________ orbitals.
Ans: degenerate
Topic: Huckel’s rule
83. Hückel's Rule requires _________ π electrons for an aromatic compound.
Ans: 4n+2
Topic: General Concepts
84. Monocyclic compounds with alternating single and double bonds are called
__________.
Ans: annulenes
Topic: General Concepts
85. Cagelike molecules with the geometry of a truncated icosahedron are called
___________.
Ans: fullerenes
Topic: Aromaticity, Reactivity
86. Pyrrole is not particularly basic, because the lone pair on the nitrogen is
______________.
Ans: part of the aromatic system
Topic: General Concepts
87. Humans do not have the biochemical ability to synthesize the benzene ring. Therefore,
two amino acids, _____________ and ___________, are essential to the human diet.
Ans: tryptophan, phenylalanine
Topic: Aromaticity, Reactivity
88. Benzene, while unusually unreactive, will react under certain conditions. However,
when it does react, it does so by ___________ rather than by addition.
Ans: substitution
519
Chapter 14
Topic: Nomenclature
89. Draw the structure corresponding to the following name: 2-bromo-5-nitrophenol
OH
Ans:
Br
O2 N
Topic: Nomenclature
90. Draw the structure corresponding to the following name:
3-bromo 5-phenylacetophenone
NH2
Ans:
Br
Topic: Nomenclature
91. Draw the structure corresponding to the following name: 2-hydroxy-4-(3methylcyclopentyl)benzoic acid
HO
Ans:
HO2C
Topic: Nomenclature
92. Draw the structure corresponding to the following name: 3-nitro-4-iodoanisole
O
Ans: O2N
I
Topic: Nomenclature
93. Give the IUPAC name for the following substance:
OH
O
Ans: 2-methyl-4-(3-methylbutyl)benzoic acid
Topic: Resonance
94. Draw all significant resonance structures for pyridine, C6H5N
Ans:
..N
N
..
520
Chapter 14
Topic: Resonance
95. Draw all significant resonance structures for azulene, C10H8
Ans:
Topic: Aromaticity
96. Briefly explain why the aromatic hydrocarbon azulene, C10H8, possesses a significant
dipole moment. Use diagrams as needed to illustrate/clarify your answer.
Ans: Azulene has the bicyclic structure shown below. Since the “non-polar” resonance
hybrid meets the Huckel rule for aromaticity, it accounts for the observed
aromatic properties of azulene. However, closer scrutiny suggests that a shift in
electron density, in which the 7-membered ring is electron deficient and the 5membered ring is electron-rich would be especially stable: each ring is now
independently aromatic, an aromatic cycloheptatrienyl cation fused with an
aromatic cyclopentadienyl anion. Thus, the most stable pi-electron distribution
would suggest a polar structure for azulene, accounting for this substance having a
significant dipole moment.
non-polar
polar
Topic: Aromaticity, Molecular Orbital Theory
97. Use the “polygon-in-circle” method to draw an MO diagram showing the pi-electron
distribution pattern in the cyclopentadienyl anion. How is this diagram useful in
explaining the aromatic properties of this anion?
Ans: The following MO diagram can be generated using the polygon-in-circle method.
The six pi electrons of the cyclopentadienyl anion in the bonding molecular
orbitals form a closed bonding shell, accounting for its aromatic properties.
Antibonding MO
Bonding MO
6 pi electrons
"closed bonding shell"
521
Chapter 14
Topic: Aromaticity, Reactivity
H
..
98. Explain briefly why cyclopentadiene readily reacts with strong bases.
Ans: The acidity of a substance can be often be correlated with the stability of its
conjugate base: substances that have exceptionally stable conjugate bases tend to
be strong acids. The conjugate base of cyclopentadiene, formed during reaction
with a base, is exceptionally stable, by virtue of being an aromatic anion (6 pi
electrons, planar ring), thus accounting for the readiness with which
cyclopentadiene reacts with bases.
B:−
H
H
Aromatic
anion
Topic: Resonance, Bond Lengths
99. Although all bond lengths in benzene are identical, that is not always true for all
aromatic compounds. In naphthalene, for instance, the C1-C2 bond is shorter than the
C2-C3 bond. Explain, using resonance theory.
Ans: The major resonance structures for naphthalene are drawn below. The observed
bond lengths reflect the contribution of each structure to the overall resonance
hybrid:
The C1-C2 bond: two double bonds (I and II), and one single bond (III)
The C2-C3 bond: one double bond (III), and two single bonds (I and II)
Overall: C1-C2 bond more double-bond-like, thus shorter.
shorter
1
I
1
1
1
2
2
2
2
3
3
3
3
II
III
522
IV
longer
Chapter 14
Topic: Aromaticity, Resonance Stabilization Energy
100. The heats of hydrogenation for cyclohexene and benzene are given below. Calculate the
resonance stabilization energy of benzene.
ΔHo = -120 kJ/mol
C6H10 + H2 Æ C6H12
C6H6 + 3H2 Æ C6H12
ΔHo = -208 kJ/mol
Ans: The resonance stabilization energy of benzene is the difference between the
theoretical and observed ΔHo values.
Hydrogenation of three double bonds should release 3 times the energy released
when one double bond is hydrogenated. Thus, the theoretical ΔHo for benzene
can be calculated as [3x (-120)] kJ/mol = -360 kJ/mol
Therefore,
Resonance stabilization energy of benzene = (-208) – (-360) kJ/mol
= 152 kJ/mol
523
Chapter 14
Topic: Aromaticity, Reactivity
101. The Lewis structures of both pyridine and pyrrole have an electron pair on the nitrogen
atom, suggesting that both substances might possess basic properties. However, only
pyridine reacts readily with HCl to form pyridinium chloride, while pyrrole is rather
unreactive. Explain clearly, using relevant diagrams to illustrate/clarify your answer.
Ans:
+ HCl
N+ −
Cl
H
N
..
2
sp
pyridinium chloride
pyridinium chloride
p
..
N H
+ HCl
H
−
N + Cl
H
X
not aromatic
pyrrole
electron pair:
part of aromatic pi sextet
Both pyridine and pyrrole are known to exhibit aromatic properties; the nitrogen
atom must therefore be sp2 hybridized in both cases.
In the pyridine molecule, this places the electron pair in an sp2 hybrid orbital, and
not involved in the aromatic π sextet needed for aromatic stability; the electron
pair is thus “available” for reacting with HCl, forming pyridinium chloride, while
preserving aromatic stability.
By contrast, the electron pair in pyrrole must be placed in the p orbital, since all
three sp2 orbitals are used in sigma bonds with neighboring atoms. This allows
the electron pair to contribute to aromatic stability by becoming part of the
aromatic sextet. Thus, the electron pair on nitrogen is not “available” for reacting
with HCl, because doing so would destroy the aromaticity of the ring.
524
Chapter 14
Topic: Aromaticity, Bonding
102. Consider the oxygen atom in the molecules of furan and tetrahydrofuran. Bearing in
mind that furan exhibits aromatic properties, compare the hybridization of oxygen in
both species, explaining why it must be different, although at first glance, it may appear
identical.
..
..
O.
O.
.
.
Furan
Ans:
THF
p
..
O
..
.. sp2
O
2
..
sp
3
sp
3
3
Furan: sp hybridized
one electron pair in p orbital:
Tetrahydrofuran: sp hybridized
both electron pairs
part of aromatic pi sextet
in sp orbitals
3
Oxygen has 2 lone electron pairs in both species and, at first glance, it might
appear that the hybridization patterns are identical. In THF, the oxygen atom is
sp3 hybridized and both pairs can be placed in sp3 hybrid orbitals. In furan, there
are two possibilities that must be considered: i) the oxygen may be sp3 hybridized,
with both electron pairs sp3 hybrid orbitals, as in THF, or, ii) oxygen may be sp2
hybridized, with one electron pair in an sp3 hybrid orbital, the other in a p orbital.
In the latter case, the electrons in the p orbital can then become part of an
aromatic sextet, allowing the molecule to gain aromatic stability. Since furan is
known to exhibit markedly aromatic properties, the oxygen must be sp2
hybridized.
525
Chapter 15
MULTIPLE CHOICE QUESTIONS
Topic: Reaction Products
1. The major product(s), A, of the following reaction,
Cl2, hν
CH3CH2
CH3CH2
A
400 - 600 oC
Cl
CH3CH2
ClCH2CH2
CH3CH
Cl
Cl
I
II
III
IV
would be:
A) I
B) II
C) A mixture of I and II
D) III
E) IV
Ans: E
Topic: Reaction Products
2. What would be the major product(s) of the following reaction?
CO2H
HNO3
NO2 H2SO4, heat
CO2H
NO2
NO2
I
A)
B)
C)
D)
E)
Ans:
CO2H
NO2
NO2
?
CO2H
O2N
II
O2N
CO2H
NO2
NO2
IV
III
I
II
III
IV
Equal amounts of II and IV
C
Page 526
Chapter 15
Topic: Reaction Products
3. The major product(s), C, of the following reaction,
Cl
Cl
Cl
SO3
H2SO4
C
Cl
Cl
SO3H
Cl
I
Cl
SO3H
HO3S
II
Cl
III
would be:
A) I
B) II
C) III
D) Equal amounts of I and II
E) Equal amounts of I and III
Ans: B
527
Chapter 15
Topic: Reaction Products
4. The major product(s), D, of the following reaction,
O
C CH2
Br
Br2
D
FeBr3
O
O
C CH2
C CH2
Br
I
II
O
O
C CH2
C CH2
Br
III
IV
would be:
A) I
B) II
C) III
D) IV
E) Equal amounts of I and II
Ans: D
528
Br
Chapter 15
Topic: Reaction Products
5. What would be the product of the following reaction sequence?
OH
i) SOCl2
?
ii) C6H6, AlCl3
O
iii) Zn(Hg), HCl, heat
O
O
OH
OH
II
I
IV
A)
B)
C)
D)
E)
Ans:
III
V
I
II
III
IV
V
E
529
Chapter 15
Topic: Reaction Products
6. What would you expect to be the major product obtained from the following reaction?
COOH
Br2
SO3H
COOH
Br
?
FeBr3
COOH
COOH
COOH
Br
SO3H
SO3H
Br
SO3H
SO3H
Br
I
A)
B)
C)
D)
E)
Ans:
II
III
I
II
III
IV
Equal amounts of II and IV
C
IV
Topic: Reaction Products
7. What would be the major product of the following reaction?
Cl2
CH2CH2CH3
Cl
CH2CH2CH3
400 oC
?
CH2CH2CH3
Cl
Cl
I
II
CH2CHCH3
III
CH2CH2CH2Cl
Cl
IV
A)
B)
C)
D)
E)
Ans:
CHCH2CH3
V
I
II
III
IV
V
C
530
Chapter 15
Topic: Reaction Products
8. What would you expect to be the major product obtained from the following reaction?
NO2
Br2
NO2
?
FeBr3
SO3H
NO2
NO2
NO2
Br
Br
SO3H
SO3H
Br
SO3H
SO3H
Br
I
A)
B)
C)
D)
E)
Ans:
II
III
I
II
III
IV
Equal amounts of II and IV
C
IV
Topic: Reaction Products
9. What would you expect to be the major product obtained from the following reaction?
O
HNO3
H2SO4, heat
HN
O
O2N
HN
?
O
NO2
O
HN
HN
II
III
O2N
I
O
O
HN
HN
IV
A)
B)
C)
D)
E)
Ans:
NO2
V
I
II
III
IV
V
E
531
NO2
Chapter 15
Topic: Reaction Products
10. What would be the major product(s) of the following reaction?
O
H2SO4
?
NO2
O
O
O
O
HO3S
SO3H
NO2
I
A)
B)
C)
D)
E)
Ans:
NO2
SO3H
II
NO2
NO2
HO3S
IV
III
I
II
III
Equal amounts of IV
Equal amounts of II and IV
C
Topic: Reaction Products
11. What would you expect to be the major product obtained from the mononitration of mdichlorobenzene?
Cl
Cl
Cl
NO2
Cl
I
A)
B)
C)
D)
E)
Ans:
Cl
NO2
O2N
Cl
II
III
I
II
III
Equal amounts of I and II
Equal amounts of I, II and III
B
532
Chapter 15
Topic: Reaction Products
12. What would you expect to be the major product obtained from the following reaction?
HO
O
HNO3
H2SO4 , heat
SO3H
HO
HO
O
O
HO
HO
O
I
A)
B)
C)
D)
E)
Ans:
O
O2N
NO2
SO3H
?
SO3H
O2N
NO2
II
SO3H
SO3H
IV
III
I
II
III
IV
Equal amounts of I and II
C
533
Chapter 15
Topic: Reaction Products
13. What product would result from the following series of reactions,
i) AlCl3
Cl
ii) N-Bromosuccinimide, peroxides
?
+
iii) CH3ONa, CH3OH, heat
iv) MMPP
v) CH3MgBr
vi) NH4Cl
Br
HO
HO
I
II
HO
IV
A)
B)
C)
D)
E)
Ans:
III
HO
V
I
II
III
IV
V
D
534
Chapter 15
Topic: Reaction Products
14. The major product(s), D, of the following reaction
HNO3
O
D
H2SO4
O
O2N
O
I
NO2 II
O
NO2
O
NO2
III
IV
would be:
A) I
B) II
C) III
D) IV
E) Equal amounts of I and II
Ans: D
535
Chapter 15
Topic: Reaction Products
15. What would be the major product of the following reaction?
NBS, ROOR
?
light
Br
Br
Br
I
II
III
Br
Br
IV
A)
B)
C)
D)
E)
Ans:
V
I
II
III
IV
V
C
Topic: Reaction Products
16. Which would be the major product(s) of the following reaction?
HO
O
HNO3
H2SO4
HO
HO
O
?
O
HO
O
NO2
NO2
I
A)
B)
C)
D)
E)
Ans:
HO3S
II
III
I
II
III
I and II in roughly equal amounts
I and III in roughly equal amounts
A
536
Chapter 15
Topic: Reaction Products
17. Which would be the product, X, of the following reaction sequence?
O
i) SOCl2
?
OH ii) benzene, AlCl3
iii) Zn(Hg), HCl
SO2
CO2
CH2
I
II
III
O
IV
A)
B)
C)
D)
E)
Ans:
V
I
II
III
IV
V
C
537
Chapter 15
Topic: Reaction Products
18. Toluene is subjected to the action of the following reagents in the order given: (1)
KMnO4,OH-, heat; then H3O+ (2) HNO3, H2SO4 (3) Br2, FeBr3
What is the final product of this sequence?
CO2H
Br
CO2H
NO2
CO2H
NO2
Br
I
NO2
Br
II
III
Br
CH3
Br
A)
B)
C)
D)
E)
Ans:
NO2
NO2
CO2H
IV
V
I
II
III
IV
V
A
Topic: Reaction Products
19. When toluene is reacted in turn with 1) Cl2 (large excess), heat, and light and 2)
Br2,FeBr3, the chief product is:
Br
CCl3
CCl3
CCl3
Br
Br
Cl
I
A)
B)
C)
D)
E)
Ans:
Br
II
III
IV
I
II
III
IV
A mixture of III and IV
B
538
Chapter 15
Topic: Reaction Products
20. The reaction of benzene with (CH3)3CCH2Cl in the presence of anhydrous aluminum
chloride produces principally which of these?
II
I
IV
A)
B)
C)
D)
E)
Ans:
III
V
I
II
III
IV
V
E
Topic: Reaction Products
21. What might be predicted to happen when the following substance undergoes FriedelCrafts acylation?
O2N
A)
B)
C)
D)
E)
Ans:
CH2
A
B
Substitution occurs in ring B, p- to the methylene group
Substitution occurs in ring A, o- to the nitro group
Substitution occurs in ring A, o- to the methylene group
Substitution occurs in ring B, m- to the methylene group
None of the above. No reaction will occur.
A
539
Chapter 15
Topic: Nomenclature, Reaction Products
22. A mixture of chlorobenzene (1 mol) and acetanilide (1mol) is allowed to react with Br2
(0.5 mol) in the presence of trace amounts of FeBr3. What is the principal product of
the competing reactions?
A) 1-bromo-4-chlorobenzene
B) 1-bromo-2-chlorobenzene
C) 1-bromo-3-chlorobenzene
D) 4-bromoacetanilide
E) 3-bromoacetanilide
Ans: D
Topic: Nomenclature, Reaction Products
23. What is the chief product of the Friedel-Crafts alkylation of benzene with 1-butene and
AlCl3?
A) butylbenzene
B) 2-phenylbutane
C) 2-methyl-1-phenylpropane
D) t-butylbenzene
E) 2,2-diphenylbutane
Ans: B
540
Chapter 15
Topic: Reaction Products
24. Which of the following reactions would give the product(s) indicated in substantial
amounts (i.e., in greater than 50% yield)?
NH2
NH2
+ CH3Cl
I
AlCl3
NH2
and
CH3
CH3
NH2
II
NH2
+ CH3Cl
AlCl3
CH3
III
A)
B)
C)
D)
E)
Ans:
+ CH3CH2CH2Cl
AlCl3
CH2CH2CH3
I
II
III
All of these
None of these
E
Topic: Reagents
25. Which reagent(s) would you use to carry out the following transformation?
toluene ⎯⎯⎯→ benzoic acid
A) Br2, heat, and light
B) Cl2, FeCl3
C) KMnO4, OH-, heat (then H3O+)
D) HNO3/H2SO4
E) SO3/H2SO4
Ans: C
541
Chapter 15
Topic: Reagents
26. Which reagent(s) would you use to carry out the following transformation?
ethylbenzene ⎯⎯⎯→ 1-chloro-1-phenylethane
A) Cl2, light, and heat
B) Cl2, FeCl3
C) SOCl2
D) C2H5Cl, AlCl3
E) HCl, O2
Ans: A
Topic: Reagents
27. Which reagent(s) would you use to carry out the following transformation?
isopropylbenzene ⎯⎯⎯→ 2- and 4-chloro-1-isopropylbenzene
A) Cl2, light, and heat
B) Cl2, FeCl3
C) SOCl2
D) C2H5Cl, AlCl3
E) HCl, peroxides
Ans: B
Topic: Reagents
28. Which reagent would you use to carry out the following transformation?
tert-butylbenzene ⎯⎯⎯→
p-tert-butylbenzenesulfonic acid
+
o-tert-butylbenzenesulfonic acid
A) HNO3/H2SO4
B) tert-C4H9Cl/AlCl3
C) H2SO3/peroxides
D) SO3/H2SO4
E) SO2/H2SO3
Ans: D
Topic: Reagents
29. Which reagent(s) would you use to carry out the following transformation?
ethylbenzene ⎯⎯⎯⎯→ benzoic acid
A) Cl2, light, and heat
B) Cl2, FeCl3
C) KMnO4, OH-, heat (then H3O+)
D) HNO3/H2SO4
E) SO3/H2SO4
Ans: C
542
Chapter 15
Topic: Reagents
30. Which reagent(s) would you use to carry out the following transformation?
ethylbenzene ⎯⎯→ 2- and 4-chloro-1-ethylbenzene
A) Cl2, light, and heat
B) Cl2, FeCl3
C) SOCl2
D) C2H5Cl, AlCl3
E) None of these
Ans: B
Topic: Reagents
31. Which reagent(s) would you use to carry out the following transformation?
t-butylbenzene ⎯⎯⎯→ p-chloro substitution product
A) Cl2, light, and heat
B) Cl2, FeCl3
C) SOCl2
D) C2H5Cl, AlCl3
E) HCl, peroxide
Ans: B
Topic: Reagents
32. Which reagent(s) would you use to carry out the following transformation?
toluene ⎯⎯⎯→ benzyl bromide
A) Br2, FeBr3
B) N-Bromosuccinimide, ROOR, hν
C) HBr
D) Br2/CCl4
E) NaBr, H2SO4
Ans: B
Topic: Reagents
33. Which reagent(s) would you use to synthesize 2- and 4-bromo-1-cyclopentylbenzene
from cyclopentylbenzene?
A) N-Bromosuccinimide (NBS), CCl4, light
B) PBr3
C) Br2, FeBr3
D) CH3CH2Br, AlBr3
E) HBr, ROOR
Ans: C
543
Chapter 15
Topic: Reagents
34. The compound 4-bromo-1-propylbenzene is best made from benzene by the application
of these reagents in the order shown:
A) (1) Br2,Fe (2) CH3CH2CH2Cl, AlCl3
B) (1) CH3CH2CH2Cl, AlCl3 (2) Br2,Fe
C)
1) CH3CH2CCl , AlCl3 2) Br2, Fe 3) Zn(Hg), HCl
O
D)
1) CH3CH2CCl , AlCl3
2) Zn(Hg), HCl
3) Br2, Fe
O
E) (1) (CH3)2CHCl, AlCl3 (2) Br2,Fe
Ans: D
Topic: Functional Group Tests
35. Which reagent would you use as the basis for a simple chemical test that would
distinguish between toluene and vinylbenzene (C6H5CH=CH2)?
A) NaOH/H2O
B) Br2/CCl4
C) NaBH4
D) HCl/H2O
E) NaI in acetone
Ans: B
Topic: Nomenclature, Functional Group Tests
36. Which reagent or test could you use to distinguish between phenylacetylene and
vinylbenzene?
A) NaOH/H2O
B) Br2/CCl4
C) IR Spectroscopy
D) CrO3/H2SO4
E) Concd. H2SO4
Ans: C
544
Chapter 15
Topic: Nomenclature, Synthesis
37. Which of the following would be the best synthesis of 1,4-dimethyl-2-nitrobenzene?
A)
HNO3
2 CH3Cl
Benzene
product
2 AlCl3
H2SO4
HNO3
CH3Cl
B)
Toluene
product
AlCl3
H2SO4
HNO
C)
3
p-Xylene
H2SO4
CH3Cl
D)
m-Nitrotoluene
AlCl3
E) All of these are equally good.
Ans: C
Topic: Synthesis
38. Starting with benzene, the best method for preparing p-nitrobenzoic acid is:
A) HNO3/H2SO4; then CH3Cl/AlCl3; then separation of isomers; then KMnO4/
OH/heat, followed by H3O+.
B) CH3Cl/AlCl3; then HNO3/H2SO4; then separation of isomers; then KMnO4/
OH/heat, followed by H3O+.
C) CH3Cl/AlCl3; then KMnO4/-OH/heat, followed by H3O+; then HNO3/H2SO4.
D) HNO3/H2SO4; then KMnO4/-OH/heat, followed by H3O+; then CH3Cl/AlCl3.
E) HNO3/H2SO4; then CO2, followed by H3O+.
Ans: B
545
Chapter 15
Topic: Synthesis
39. A good synthesis of
O
(CH3)3C
CCH3
would be:
A)
Benzene
O
CH3CCl
(CH3)3CCl
AlCl3
AlCl3
O
B)
Benzene
(CH3)3CCl
CH3CCl
AlCl3
AlCl3
O
C)
Benzene
CH3CCl
AlCl3
D) More than one of these
E) None of these
Ans: B
(CH3)2C
CH2
HF
Topic: Synthesis
40. Which would be a good synthesis of m-nitrobenzoic acid?
A)
HNO3/H2SO4
CH3Cl
1. KMnO4, NaOH, heat
Benzene
heat
AlCl3
2. H3O+
B)
Toluene
C)
Toluene
HNO3/H2SO4
1. KMnO4, NaOH, heat
2. H3O+
1. KMnO4, NaOH, heat
HNO3/H2SO4
heat
2. H3O+
D) More than one of the above
E) None of the above
Ans: C
heat
546
Chapter 15
Topic: Synthesis
41. Benzoic acid can be prepared by the oxidation of all of the following compounds except
this one:
A) C6H5CH=CH2
B) C6H5C≡CH
C) C6H5COCH3
D) C6H5CH2CH2CH3
E) C6H5C(CH3)3
Ans: E
Topic: Synthesis
42. Which of the following reactions would yield isopropylbenzene as the major product?
CH3CH CH2
A)
Benzene
H2SO4
CH
B)
3CHCH3
Benzene
H2SO4
CH3CHCH3
C)
Benzene
D)
OH
Benzene
Cl
AlCl3
CH3CH2CH2Cl
AlCl3
E) All of these
Ans: E
Topic: Synthesis
43. Which of the following reactions could be used to synthesize tert-butylbenzene?
H2SO4
A)
C6H6 + CH2=C(CH3)2
H2SO4
B)
C6H6 + (CH3)3COH
AlCl3
C)
C6H6 + (CH3)3CCl
D) All of the above
E) None of the above
Ans: D
547
Chapter 15
Topic: Synthesis
44. Which of these is a satisfactory synthesis of 1-chloro-2-phenylethane?
A) C6H5CH2CH3 + Cl2, Fe ⎯⎯→
B) C6H5CH2CH3 + Cl2, 400°C ⎯⎯→
C) C6H5CH2CH2OH + SOCl2 ⎯⎯→
D) C6H5CH=CH2 + HCl, peroxide ⎯⎯→
E) C6H6 + CH3CH2Cl, AlCl3 ⎯⎯→
Ans: C
Topic: Synthesis
45. Which of the following reactions would produce isopropylbenzene?
Cl
A)
Benzene
AlCl3
B)
Benzene
HF
OH
C)
Benzene
H2SO4
Br
D)
Benzene
AlCl3
E) All of these
Ans: E
548
Chapter 15
Topic: Synthesis
46. How might the following synthesis be carried out:
CH2CH3
Benzene
A)
B)
C6H6
several
steps
CH3CH2Cl
C6H6
AlCl3
Cl2
FeCl3
?
Cl
Cl2
FeCl3
CH3CH2Cl
AlCl3
product
product
O
C)
C6H6
CH3CCl
Cl2
Zn(Hg)
AlCl3
FeCl3
HCl
Zn(Hg)
Cl2
product
O
D)
C6H6
CH3CCl
HCl
AlCl3
E) None of these syntheses is satisfactory.
Ans: C
FeCl3
product
Topic: Synthesis
47. Which is the best sequence of reactions for the preparation of p-bromostyrene from
ethylbenzene?
A)
NBS, hν
Br2
KOH
product
ethylbenzene
Fe
CCl4
CH3CH2OH
B)
NBS, hν
Br2
KOH
product
ethylbenzene
Fe
CCl4
CH3CH2OH
C)
NBS, hν
KOH
Br2
product
ethylbenzene
Fe
CCl4
CH3CH2OH
ZnO
Br2
D)
product
ethylbenzene
Fe
630 oC
E) None of these syntheses is satisfactory.
Ans: B
549
Chapter 15
Topic: Synthesis
48. Which is the best sequence of reactions for the following transformation?
O
NO2
A)
B)
C)
D)
E)
Ans:
i) HNO3, H2SO4; ii) CH3MgBr, Et2O; iii) H3O+, heat
i) CH3MgBr, Et2O; ii) H3O+, heat; iii) HNO3, H2SO4
i) HNO3, H2SO4; ii) NaBH4, H2O; iii) H3O+, heat
i) HNO3, H2SO4; ii) NaBH4, H2O; iii) H3O+, heat
None of these syntheses is satisfactory.
A
Topic: Synthesis
49. Which is the best sequence of reactions for the following transformation?
OH
O
OCH3
NO2
A)
B)
C)
D)
E)
Ans:
i) HNO3, H2SO4; ii) 2EtMgBr, Et2O; iii) NH4Cl
i) 2EtMgBr, Et2O; ii) H3O+; iii) HNO3, H2SO4
i) HNO3, H2SO4; ii) NaBH4, H2O; iii) 2EtMgBr, Et2O
i) LAH; ii) 2EtLi, Et2O; iii) HNO3, H2SO4, heat
None of these syntheses is satisfactory.
A
Topic: Synthesis
50. Which is the best sequence of reactions for the following transformation?
O
OH
NO2
A)
B)
C)
D)
E)
Ans:
i) SOCl2; ii) C6H6, AlCl3; iii) Zn(Hg),HCl; iv) HNO3, H2SO4
i) SOCl2; ii) HNO3, H2SO4; iii) C6H6, AlCl3; iv) H2, Ni
i) HNO3, H2SO4; ii) SOCl2; iii) C6H6, AlCl3; iv) Zn(Hg),HCl
i) PBr3; ii) HNO3, H2SO4; iii) C6H6, AlCl3; iv) NaBH4, H2O
two of these syntheses are satisfactory.
C
550
Chapter 15
Topic: Relative Reactivities
51. Which of the following compounds would be most reactive toward electrophilic
substitution?
OH
CH3
I
II
A)
B)
C)
D)
E)
Ans:
Br
III
CHO
NO2
IV
V
I
II
III
IV
V
A
Topic: Relative Reactivities
52. Which of the following compounds would be most reactive to ring bromination?
CH3
I
A)
B)
C)
D)
E)
Ans:
NO2
II
CF3
III
Cl
NH2
IV
V
I
II
III
IV
V
E
Topic: Relative Reactivities
53. Which of the following compounds would you expect to be most reactive toward ring
nitration?
A) Benzene
B) Toluene
C) m-Xylene
D) p-Xylene
E) Benzoic acid
Ans: C
551
Chapter 15
Topic: Relative Reactivities
54. Which of the following compounds would be least reactive toward electrophilic
substitution?
O
O
NO2
NH
NO2
I
A)
B)
C)
D)
E)
Ans:
II
IV
III
V
I
II
III
IV
V
B
Topic: Relative Reactivities
55. Which of the following compounds would be most reactive toward ring nitration?
O
O
O
I
A)
B)
C)
D)
E)
Ans:
II
CF3
III
I
II
III
IV
V
D
552
OH
NH
IV
V
Chapter 15
Topic: Relative Reactivities
56. Which of the compounds listed below would you expect to give the greatest amount of
meta-product when subjected to ring bromination?
HO
OH
O
NH2
II
I
A)
B)
C)
D)
E)
Ans:
III
OCH3
CH3
IV
V
I
II
III
IV
V
B
Topic: Relative Reactivities
57. Which of the following compounds would be most reactive toward ring bromination?
O
HO
O
O
NH2
OCH3
O
O
O
II
I
A)
B)
C)
D)
E)
Ans:
IV
III
I
II
III
IV
V
C
553
V
Chapter 15
Topic: Relative Reactivities
58. Which of the compounds listed below would you expect to give the greatest amount of
meta-product when subjected to ring nitration?
O
I
A)
B)
C)
D)
E)
Ans:
II
O
NH
III
OCH3
CH3
IV
V
I
II
III
IV
V
B
Topic: Mechanisms and Intermediates
59. Which of the following structures contribute(s) to the resonance hybrid of the
intermediate formed when bromobenzene undergoes para-chlorination?
..
..
..
..
: Br :
: Br +
: Br :
: Br :
+
+
+
H Cl
H Cl
H Cl
H Cl
I
II
III
IV
A)
B)
C)
D)
E)
Ans:
I
II
III
IV
All of the above
E
554
Chapter 15
Topic: Mechanisms and Intermediates
60. Which of the following contributors to the resonance stabilized hybrid formed when
aniline undergoes para-chlorination would be exceptionally stable?
+NH2
: NH2
: NH2
: NH2
+
+
+
H Cl
H Cl
H Cl
H Cl
I
II
III
IV
A)
B)
C)
D)
E)
Ans:
I
II
III
IV
None of these
B
Topic: Mechanisms and Intermediates
61. Consider the structures given below. Which of them would be a relatively stable
contributor to the hybrid formed when toluene undergoes para bromination?
CH3
CH3
CH3
CH3
CH3
+
+
H Br
Br
II
I
A)
B)
C)
D)
E)
Ans:
+
+
H Br
H Br
III
IV
I
II
III
IV
V
C
555
Br+
IV
Chapter 15
Topic: Mechanisms and Intermediates
62. Which of the following structures does not contribute to the resonance hybrid of the
intermediate formed when nitrobenzene undergoes meta-chlorination?
..
..
..
..
..
..
:O : O:
: O : O:
: O : O:
N
N
+
I
A)
B)
C)
D)
E)
Ans:
H
Cl
+
N
+
H
Cl
II
H
Cl
III
I
II
III
None of these contribute
All of these contribute.
E
Topic: Mechanisms and Intermediates
63. Which of these is the rate-determining step in the nitration of benzene?
A) Protonation of nitric acid by sulfuric acid
B) Protonation of sulfuric acid by nitric acid
C) Loss of a water molecule by the protonated species to produce the nitronium ion
D) Addition of nitronium to benzene to produce the arenium ion
E) Loss of a proton by arenium ion to form nitrobenzene
Ans: D
Topic: Directing Effects
64. The ortho/para product ratio is expected to be the smallest for the bromination of which
of these?
A) Toluene
B) Isopropylbenzene
C) Butylbenzene
D) sec-Butylbenzene
E) tert-Butylbenzene
Ans: E
556
Chapter 15
Topic: Directing Effects
65. Which of the following is not a meta-directing substituent when present on the benzene
ring?
A) -NHCOCH3
B) -NO2
C) -N(CH3)3+
D) -C≡N
E) -CO2H
Ans: A
Topic: Directing Effects
66. Each of the five disubstituted benzenes shown below is nitrated. In which of these cases
does the arrow not indicate the chief position of nitration.
Cl
COOH
Cl
NHCOCH3
O
CCH3
Cl
NO2
OH
A)
B)
C)
D)
E)
Ans:
I
I
II
III
IV
V
C
II
III
557
CH3
CH2CH3
IV
V
Chapter 15
Topic: Directing Effects
67. Which of the following compounds would yield the greatest amount of meta product
when subjected to ring nitration?
HO
O
HO
O
HO
O
I
A)
B)
C)
D)
E)
Ans:
O
O
HO
OH
II
IV
III
V
I
II
III
IV
V
A
Topic: Directing Effects
68. Which of the following is not an ortho-para director in electrophilic aromatic
substitution?
A) -CF3
B) -OCH3
C) -CH3
D) -F
E) -NH2
Ans: A
Topic: Directing Effects
69. Which of these compounds gives essentially a single product on electrophilic
substitution of a third group?
A) p-chlorotoluene
B) m-ethylanisole
C) 1-bromo-2-chlorobenzene
D) m-xylene
E) 3-hydroxybenzoic acid
Ans: D
558
Chapter 15
Topic: Directing Effects
70. What is a feature found in all ortho-para directing groups?
A) An oxygen atom is directly attached to the aromatic ring.
B) The atom attached to the aromatic ring possesses an unshared pair of electrons.
C) The group has the ability to delocalize the positive charge of the arenium ion.
D) The atom directly attached to the aromatic ring is more electronegative than
carbon.
E) The group contains a multiple bond.
Ans: C
Topic: Directing Effects
71. Which is the best prediction of the site(s) of substitution when 3-methylphenol is
nitrated?
A) C-2
B) C-4
C) C-5
D) C-6
E) C-4 and C-6
Ans: E
Topic: Directing Effects
72. (Trifluoromethyl)benzene, C6H5CF3, will
A) nitrate rapidly in the ortho-para positions.
B) nitrate slowly in the ortho-para positions.
C) nitrate rapidly in the meta position.
D) nitrate slowly in the meta position.
E) fail to nitrate under any conditions.
Ans: D
Topic: Directing Effects
73. What feature is common to all meta-directing groups?
A) The atom directly attached to the ring has a full or well-developed partial positive
charge.
B) The atom directly attached to the ring is doubly bonded to oxygen.
C) One or more halogen atoms are present in the group.
D) One or more oxygen atoms are present in the group.
E) The group is attached to the ring through a carbon atom.
Ans: A
559
Chapter 15
Topic: Directing Effects
74. Arrange the following compounds in order of decreasing reactivity in electrophilic
substitution:
O
I
A)
B)
C)
D)
E)
Ans:
O
II
V > II > I > III > IV
II > V > III > I > IV
IV > I > III > V > II
III > II > I > IV > V
IV > V > II > I > III
B
Br
NO2
IV
III
V
Topic: Other EAS Considerations
75. Which of these liquids would be unsuitable as an inert solvent for a Friedel-Crafts
reaction?
A) Chlorobenzene
B) Nitrobenzene
C) Acetophenone
D) (Trifluoromethyl)benzene
E) All could be used.
Ans: A
Topic: Other EAS Considerations
76. Undesired polysubstitution of an aromatic nucleus is most likely to be encountered in
the case of:
A) Friedel-Crafts alkylation
B) Friedel-Crafts acylation
C) Nitration
D) Sulfonation
E) Chlorination
Ans: A
560
Chapter 15
Topic: Other EAS Considerations
77. This molecule cannot participate as a reactant in a Friedel-Crafts reaction.
A) Benzene
B) Chlorobenzene
C) Nitrobenzene
D) Toluene
E) tert-Butylbenzene
Ans: C
Topic: Other EAS Considerations
78. Which one of these molecules can be a reactant in a Friedel-Crafts reaction?
A) Aniline
B) Nitrobenzene
C) Chloroethene
D) Bromobenzene
E) p-Bromonitrobenzene
Ans: D
Topic: Other EAS Considerations
79. This substituent deactivates the benzene ring towards electrophilic substitution but
directs the incoming group chiefly to the ortho and para positions.
A) -OCH2CH3
B) -NO2
C) -F
D) -CF3
E) -NHCOCH3
Ans: C
Topic: Other EAS Considerations
80. In electrophilic aromatic substitution, the attacking species (the electrophile) necessarily
is a:
A) Neutral species.
B) Positively charged species.
C) Lewis acid.
D) Proton.
E) Carbocation.
Ans: C
561
Chapter 15
Topic: Other EAS Considerations
81. The electrophilic bromination or chlorination of benzene requires, in addition to the
halogen:
A) Hydroxide ion.
B) A Lewis base.
C) A Lewis acid.
D) Peroxide.
E) Ultraviolet light.
Ans: C
Topic: Non-EAS Reactions
82. What is the Birch reduction product of the following reaction?
Na, NH3(l)
EtOH
II
I
A)
B)
C)
D)
E)
Ans:
?
IV
III
I
II
III
IV
V
D
Topic: Non-EAS Reactions
83. SN1 solvolysis of C6H5CH=CHCH2Cl in water produces:
A) C6H5CH2C(OH)=CH2
B) C6H5CH=CHCH2OH
C) C6H5CHOHCH=CH2
D) A mixture of B) and C)
E) A mixture of A), B) and C)
Ans: D
562
V
Chapter 15
Topic: Non-EAS Reactions
84. Which compound is capable of undergoing both SN1 and SN2 reactions in ordinary
nonacidic solvents?
Br
Br
Br
Br
Br
I
A)
B)
C)
D)
E)
Ans:
II
III
IV
V
I
II
III
IV
V
D
Topic: Non-EAS Reactions
85. Which alkyl halide can undergo both SN1 and SN2 reactions in nonacidic solvents?
A) CH3Br
B) CH3CH2CH2Br
CH2
CHCHCH3
C)
Br
D)
E)
C6H5CH2CH2CH2Br
CH3
C6H5CCH2Br
CH3
Ans: C
Topic: Non-EAS Relative Rates and Intermediates
86. The rate of solvolysis in ethanol is least for which of these compounds?
A) C6H5CH2Cl
C6H5C(CH3)2
B)
Cl
C)
D)
(C6H5)2CHCl
C6H5CHCl
CH3
E) (C6H5)3CCl
Ans: A
563
Chapter 15
Topic: Non-EAS Relative Rates and Intermediates
87. Which of the following structures would not be a contributor to the resonance hybrid of
the benzyl cation?
CH3
H
H
+
I
A)
B)
C)
D)
E)
Ans:
CH3
+CH2
CH2
+
CH3
+
+
II
IV
III
IV
I
II
III
IV
V
A
Topic: Non-EAS Relative Rates and Intermediates
88. Which of these species is/are proposed as intermediate(s) in the mechanism for the
Birch reduction?
A) Radical
B) Carbanion
C) Radical anion
D) Both A) and B)
E) All of A), B) and C)
Ans: E
564
Chapter 15
Topic: Non-EAS Relative Rates and Intermediates
89. Which of the following carbocations would be most stable?
CH3
A)
C6H5CH2CH2CHCH2
B)
C)
CH2
C6H5CH2CH2CHCH3
CH3
C6H5CH2CH2CCH3
CH3
D)
E)
C6H5CH2CHCHCH3
CH3
C6H5CHCH2CHCH3
Ans: E
Topic: Non-EAS Relative Rates and Intermediates
90. Which alkyl halide would be most reactive in an SN2 reaction?
A) C6H5CH2CH2CH2Br
C6H5CHCH2Br
B)
C)
CH3
C6H5CH2CHCH3
Br
CH3
D)
C6H5CCH2Br
CH3
Br
E)
C6H5CCH3
CH3
Ans: A
565
Chapter 15
Topic: Non-EAS Relative Rates and Intermediates
91. Which alkyl halide would be most reactive in an SN1 reaction?
A) C6H5CH2CH2CH2Br
C6H5CH2CHCH3
B)
Br
CH3
C)
C6H5CH2CBr
CH3
Br
D)
C6H5CCH3
CH3
CH3
E)
C6H5CCH2Br
CH3
Ans: D
SHORT ANSWER QUESTIONS
Topic: General
92. Aromatic hydrocarbons are generally known as ____________.
Ans: arenes
Topic: General Reactivity
93. Benzene is susceptible to electrophilic attack primarily because of its _____________.
Ans: exposed π system
Topic: General Reactivity
94. Although benzene is susceptible to electrophilic attack, its special stability causes it to
undergo ______________ rather than ____________ reactions.
Ans: substitution; addition
566
Chapter 15
Topic: EAS Mechanism
95. In general, there are three steps to an electrophilic aromatic substitution reaction. These
are:
a) formation of an _____________;
b) reaction with an aromatic ring to form an ___________; and
c) loss of a _________ to reform the aromatic system.
Ans: electrophile; arenium ion; proton
Topic: General Reactivity
96. Aryl and vinyl halides cannot be used in the Friedel-Crafts alkylation because they do
not __________.
Ans: form carbocations readily
Topic: General EAS Reactivity
97. Substituent groups affect both _____________ and _______________ in electrophilic
aromatic substitution reactions.
Ans: reactivity; orientation
Topic: General EAS Reactivity
98. Electron-donating groups increase the electron density on the aromatic ring, causing it
to be more reactive than benzene itself towards EAS. These types of groups are called
_____________ groups.
Ans: activating
Topic: General EAS Reactivity
99. Electron-withdrawing groups decrease the electron density on the aromatic ring, causing
it to be less reactive than benzene itself towards EAS. These types of groups are called
_____________ groups.
Ans: deactivating
Topic: General EAS Reactivity
100. With the exception of halogens, activating groups are ____________ directors and
deactivating groups are _________ directors.
Ans: ortho/para; meta
567
Chapter 15
Topic: General EAS Reactivity
101. When two different groups are present on a benzene ring, the ____________ generally
determines the outcome of an EAS reaction.
Ans: more powerful activating group
Topic: EAS/non-EAS Reactivity
102. When toluene reacts with Cl2, the identity of the major product(s) obtained depends on
whether the reaction is carried out in the presence of catalytic amounts of FeCl3, or
without catalyst, at high temp. Explain clearly.
Ans: Reaction with Cl2/FeCl3 results in electrophilic aromatic substitution at the o- and
p- positions, since the methyl group in toluene is an activating, o-,p- director. In
contrast, when the reaction is carried out without catalyst, at high temperature, the
methyl group undergoes free-radical halogenation via the benzyl radical
intermediate.
Cl2
FeCl3
Cl
+
Cl
Cl
Cl2
heat
568
Chapter 15
Topic: General EAS Reactivity
103. When the following substance is treated with Br2/FeBr3, the major product is obtained in
good yield, and only very small amounts of minor products are found. What is this
major product and why are the minor products not formed to any significant degree?
Explain clearly.
O
Br2
FeBr3
HN
O
?
Ans: The major products are produced by brominating Ring B. Ring B is more
electron-rich due to the electron-donation of both the nitrogen and methyl groups
attached versus the two electron-withdrawing carbonyl groups attached to Ring A,
which make it inherently electron-poor. Therefore, bromination mainly occurs on
the more electron-rich benzene ring affording a mixture of the following two
compounds. Due to steric effects, bromination ortho to both the nitrogen and
methyl group would lead to a minor product. Also, because Ring A is electronpoor, bromination of this ring would also lead to minor products.
Ring
Ring
A
A
O
O
HN
O
Br
+
Ring
B
major products
HN
O
Br
Ring
B
Topic: Synthetic Strategy
104. Suggest a reasonable synthetic strategy for the synthesis of 3,5-dinitrobenzyl alcohol
from benzaldehyde.
NO2
NO2
Ans:
HNO3 (excess)
NaBH4
O
H2SO4
O H2O
OH
O2 N
O2 N
heat
benzaldehyde
3,5-dinitrobenzyl alcohol
Topic: Synthetic Strategy
105. Suggest a reasonable synthetic strategy for the synthesis of 4-methyl-1-phenyl-1pentanol from 4-methylpentanoyl chloride, (CH3)2CHCH2CH2COCl
O
OH
Ans:
NaBH4
Cl
H2O
AlCl3
O
4-methylpentanoyl chloride
4-methyl-1-phenyl-1-pentanol
569
Chapter 15
Topic: Synthetic Strategy
106. Suggest a reasonable synthetic strategy for the synthesis of 6-methyl-3-phenyl-3heptanol from 4-methylpentanoyl chloride, (CH3)2CHCH2CH2COCl
Ans:
O
OH
O
i) C2H5MgBr
ii) NH4Cl
Cl
AlCl3
4-methylpentanoyl chloride
6-methyl-3-phenyl-3-heptanol
Topic: Mechanism, Carbocation Stability
107. Acid-catalyzed hydration of 1-phenyl-1-pentene gives 1-phenyl-1-pentanol almost
exclusively; the other possible hydration product, 1-phenyl-2-pentanol, is not detected at
all. Explain clearly.
Ans: Acid catalyzed hydration of an alkene occurs through a carbocation intermediate.
Although 1-phenyl-1-pentene is symmetrically substituted at the C=C, the relative
stabilities of the two possible carbocation intermediates is significantly different:
although both are secondary carbocations, one is also benzylic and thus stabilized
by resonance. The observation that 1-phenyl-1-pentene gives 1-phenyl-1pentanol, almost exclusively, is in accordance with the general trend that acid
catalyzed hydration chiefly tends to afford products formed via the most stable
intermediate carbocation.
H3O+
H2O
H
+
H
+
+
2o, benzylic:
stabilized by
resonance,formed
2o, but not benzylic:
almost exclusively
appreciable degree
less stable, not
formed to any
OH
H
chief product
570
H
+
OH
not detected
Chapter 15
Topic: Spectroscopic Analysis
108. How might 1H NMR spectroscopy be useful in distinguishing between the following
two substances?
O
O
O
O
A
B
Ans: The chemical shift of the methyl protons is likely to be markedly different in the
two substances. In A, the signal may be expected at about 3.5-4.0 ppm
(attachment to O), while in B, it is likely to show up at about 2.0 ppm (attachment
to >C=O).
571
Chapter 16
MULTIPLE CHOICE QUESTIONS
Topic: Nomenclature
1. What is the correct IUPAC name for the following compound?
O
A)
B)
C)
D)
E)
Ans:
2-Methyl-5-heptanone
7-Methyl-4-octanone
6-Isopropyl-4-octanone
Isobutyl propyl ketone
1,1-Dimethyl-4-heptanone
B
Topic: Nomenclature
2. What is the correct IUPAC name for the following compound?
O
A)
B)
C)
D)
E)
Ans:
5,5-Dimethyl-2-heptanone
5-Ethyl-5,5-dimethyl-Methyl-2-octanone
5-Ethyl-5-methyl- 2-hexanone
5,5-Dimethyl-2-octanone
3,3-Dimethyl-6-heptanone
A
Topic: Nomenclature
3. What is the correct IUPAC name for the following compound?
O
A)
B)
C)
D)
E)
Ans:
1,1,2-Trimethyl-1,3-hexenone
1,2-Dimethyl-1,3-hexenone
2,3-Dimethyl-1,3-heptenone
2,3-Dimethyl-2-hepten-4-one
5,6-Dimethyl-5-hepten-4-one
C
572
Chapter 16
Topic: Nomenclature
O
4.
A correct name for C6H5CH2CH2CH is?
A) 3-Benzylpropanone
B) 3-Phenylpropanal
C) 3-Benzylpropanal
D) Nonanone
E) Nonanal
Ans: B
Topic: Nomenclature
5. A correct name for the following compound would be which of those below?
O
H
A)
B)
C)
D)
E)
Ans:
2,5-Dimethyl-6-hexanal
2,5-Dimethylhexanal
2-Aldehydoisohexane
3,5-Dimethylheptanone
1-Hydro-2,5-dimethyl-1-hexanone
B
Topic: Nomenclature
O
6.
CH3
CH
CH2
OH
What is the IUPAC name for
A) 4-Oxo-5-phenyl-2-hexanol
B) 5-Hydroxy-2-phenyl-3-hexanone
C) 2-Hydroxy-5-phenyl-4-hexanone
D) 2-Hydroxypropyl-1-phenylethyl ketone
E) 5-Hydroxy-3-keto-2-phenylhexane
Ans: B
573
C
CH
C 6H 5
CH3
?
Chapter 16
Topic: Nomenclature
7. Which is the proper name for the structure shown?
CHO
Cl
CH3
A)
B)
C)
D)
E)
Ans:
2-Chloro-5-aldehydotoluene
6-Chloro-3-aldehydotoluene
2-Methyl-4-aldehydochlorobenzene
4-Chloro-3-methylbenzaldehyde
4-Methyl-5-chloro-2-benzaldehyde
D
Topic: Nomenclature
8. Which is the proper name for the structure shown?
O2 N
O
NO2
A)
B)
C)
D)
E)
Ans:
3-Methyl-5-(1,4-dinitrobenzyl)-1-pentanal
3-Methyl-1-(2,5-dinitrophenyl)-5-pentanal
3-Methyl-5-(2,5-dinitrophenyl)pentanal
3-Methyl-5-(2,5-dinitrophenyl)pentanaldehyde
3-Methyl-5-(2,5-dinitrophenyl)pentaldehyde
C
Topic: Nomenclature
9. Which is the proper name for the structure shown?
O
Cl
A)
B)
C)
D)
E)
Ans:
3,5-Dimethyl-5-(4-chlorobenzyl)-1-pentanal
3-Methyl-5-(4-chlorophenyl)hexanol
3,5-Dimethyl-5-(4-chlorophenyl)-1-pentanaldehyde
3-Methyl-5-(4-chlorophenyl)-hexanaldehyde
3-Methyl-5-(4-chlorophenyl)hexanal
E
574
Chapter 16
Topic: Nomenclature
10. Which is the proper name for the structure shown below?
Br
O
A)
B)
C)
D)
E)
Ans:
7-Bromo-1,4-octynal
7-Bromo-1,4-octynone
2-Bromo-5,7-octynal
7-Bromo-7-methyl-1-heptyne-3-ketone
7-Bromo-1-octyn-4-one
E
Topic: Nomenclature
11. Which is the IUPAC name for the structure shown below?
Br
H
O
A)
B)
C)
D)
E)
Ans:
(R)-5-Bromo-2-heptanal
(S)-5-Bromo-2-heptanal
(R)-5-Bromo-2-heptanone
(S)-5-Bromo-2-heptanone
(R)-3-Bromopentyl methyl ketone
C
Topic: Nomenclature
12. Which is the IUPAC name for the structure shown below?
O
A)
B)
C)
D)
E)
Ans:
5-Cyclohexyl-2-hexanal
5-Cyclohexyl-2-hexanone
5-Cyclohexyl-5-methyl-2-pentanone
5-(1-Methylcyclohexyl)-2-pentanone
4-(1-Methylcyclohexyl)-2-butanone
E
575
Chapter 16
Topic: New Functional Groups
13. Which of the following compounds is an acetal?
O
OCH3
A)
B)
C)
D)
E)
Ans:
O
O
O
O
I
I
II
III
IV
None of these
C
OH
II
O
O
O
IV
III
Topic: New Functional Groups
14. Which is the general formula for a thioacetal?
H
A)
B)
RC S
RCHSR'
C)
OH
RCHSR'
D)
SH
RCHSR'
E)
SR'
RCHOR'
SR'
Ans: D
Topic: New Functional Groups
15. Which compound is an acetal?
O
O
O
I
A)
B)
C)
D)
E)
Ans:
OH
O
II
OCH3
OH
O
III
I
II
III
IV
All of the above
B
576
IV
Chapter 16
Topic: Reaction Products
16. The product, C, of the following reaction sequence,
O
CH3CCH3
+
A)
CH3
CH2
B)
C)
CN
HCN
C4H7NO
H2O
H2SO4
heat
C
would be:
CCOOH
CH3CH2COOCH3
CH3CHCH3
CN
D) CH3CH=CHCOOH
E) None of these
Ans: A
Topic: Reaction Products
17. The product, H, of the following reaction sequence,
O
C6H5CH2Br + (C6H5)3P
would be:
A)
F
CH3
C6H5CH2CC6H5
B)
OH
O
C)
C6H5CH2CC6H5
C6H5CH CHCC6H5
O
D)
E)
C6H5CH2CH=CHC6H5
C6H5CH
CCH3
C 6H 5
Ans: E
577
C6H5Li
ether
G
C6H5CCH3
H
Chapter 16
Topic: Reaction Products
18. What would be the product of the following reaction sequence?
OH
i) PCl3
?
ii) C6H6, AlCl3
O
iii) Zn(Hg), HCl, heat
O
O
OH
OH
I
II
IV
A)
B)
C)
D)
E)
Ans:
III
V
I
II
III
IV
V
E
Topic: Reaction Products
19. What new compound will be formed when HCl is added to a solution of pentanal in
methanol?
O
A)
O
O
B)
Cl
OH
C)
D)
OH
O
O
Cl
E)
O
Ans: A
578
Chapter 16
Topic: Reaction Products
20. What would be the product of the following reaction sequence?
OH
O
i) PCl3
ii) C6H6, AlCl3
iii) NaBH4, H2O
?
O
OH
II
I
III
OH
OH
IV
A)
B)
C)
D)
E)
Ans:
V
I
II
III
IV
V
D
579
Chapter 16
Topic: Reaction Products
21. The product, K, of the following sequence of reactions
CH3
Benzene
CH3CHCOCl
I
AlCl3
OH
CH3CH2MgBr
ether
H3O+
J
K
would be:
OH
HO
O
O
I
A)
B)
C)
D)
E)
Ans:
II
III
IV
V
I
II
III
IV
V
B
Topic: Reaction Products
22. Dissolving benzaldehyde in methyl alcohol establishes an equilibrium with what
compound?
OH
A)
B)
C6H5CHOCH3
O
C6H5COCH3
C)
D)
E)
Ans:
C6H5C(OCH3)3
Answers A) and B)
Answers B) and C)
A
580
Chapter 16
Topic: Reaction Products
23. Predict the major organic product of the following reaction:
H
Ag(NH3)2+
O
?
H2O
O
O
O
HO2C
O
O
H3C
I
A)
B)
C)
D)
E)
Ans:
H
OH
O
II
H
H 3C
H 3C
III
O
O
IV
V
I
II
III
IV
V
B
Topic: Reaction Products
24. The product, E, of the following reaction sequence,
PCl5
C6H6
CH3CH2COOH
C
D
AlCl3
CH2CH2CH3
I
A)
B)
C)
D)
E)
Ans:
NaBH4
CH3OH
E
would be:
O
OH
O
COCH2CH3
CHCH2CH3
CCH2CH3
CH(CH3)2
IV
V
II
III
I
II
III
IV
V
C
581
Chapter 16
Topic: Reaction Products
25. Select the structure of the major product in the following reaction.
i) BH3
?
ii) H2O2,OH-, H2O
A)
B)
C)
D)
E)
Ans:
Ethylbenzene
1-Phenylethanol
Acetophenone
2-Phenylethanal
Vinylbenzene
D
Topic: Reaction Products
26. Select the structure of the major product in the following reaction.
H2SO4
HgSO4, H2O
A)
B)
C)
D)
E)
Ans:
?
Ethylbenzene
1-Phenylethanol
Acetophenone
2-Phenylethanal
Vinylbenzene
C
Topic: Reaction Products
27. Select the structure of the major product in the following reaction.
i) H2, Lindlar catalyst
ii) O3
iii)Zn, CH3CO2H
A)
B)
C)
D)
E)
Ans:
4-methylhexanal
4-methyl-1-hexanol
3-methylhexanal
4,10-dimethyldodecane-6,7-dione
4,10-dimethyldodecane-6,7-diol
A
582
?
Chapter 16
Topic: Reaction Products
28. The product, J, of the following reaction sequence,
CH3CHCOOH
SOCl2
(CH3)2CuLi
I
ether
CH3
O
A)
CH3CHCOCH3
CH3
OH O
B)
CH3C
CCH3
C)
CH3
CH3CHCOCu
D)
CH3
O
CH3CHCCH3
CH3
O
E)
CH3CHCCHCH3
CH3 CH3
Ans: D
583
J
would be:
Chapter 16
Topic: Reaction Products
29. What is the major product of the following reaction sequence?
O
HCN
H2SO4
H2O
heat
CN
NOSO3H
CN
COH
H
OSO3H
CNH2
O
I
A)
B)
C)
D)
E)
Ans:
II
III
I
II
III
IV
V
B
584
IV
V
Chapter 16
Topic: Reaction Products
30. The product, D, of the following sequence of reactions
O
SOCl2
CH3CH2COH
A
toluene
AlCl3
Zn(Hg)
B
D
HCl
+
C (discard)
would be:
O
CH3
CH2CH2CH3
CH3CH2CCH2
I
CH3
II
CH3
CH2CH2CH2Cl
CH2CH2CCl
III
CH3
CH
CH3
CH3
IV
A)
B)
C)
D)
E)
Ans:
O
V
I
II
III
IV
V
A
Topic: Reaction Products
31. What new compound will be formed when gaseous HCl is added to a solution of
propanal in methanol?
O
A)
B)
CH3CH2COCH3
O
CH3CH2COH
C)
D)
CH3CH2CH(OCH3)2
CH3CH2CHOCH3
OH
E) None of the above
Ans: C
585
Chapter 16
Topic: Reaction Products
32. What is the product of the following reaction sequence?
PCC
C6H5CH2OH
A)
B)
BrCH2CO2Et, Zn
CH2Cl2
C6H5CH2OCH2CO2Et
H 3O +
benzene
O
C)
C6H5COCH2CO2Et
OH
D)
C6H5CHCH2CO2Et
OH
C6H5CHOCH2CO2Et
E) C6H5CH2CH2CO2Et
Ans: C
Topic: Reaction Products
33. What would be the final product?
O
(C6H5)3P
CH3I
A)
C6H5Li
C6H5CCH3
ether
C6H5C
CH2
CH3
C6H5
B)
C6H5CCH3
CH3
C)
D)
C6H5CH=CHCH3
OH
C6H5CCH3
E)
CH3
C6H5CH
CC6H5
CH3
Ans: A
586
?
Chapter 16
Topic: Reaction Products
34. What is the major product of the following reaction sequence?
OH, HCl
OCH3 i)
HO
?
O
ii) LAH, Et2O
O
iii) H2O, H3O
+
O
OH
OH
O
HO
O
HO
I
II
III
OH
OH
O
O
IV
A)
B)
C)
D)
E)
Ans:
V
I
II
III
IV
V
E
Topic: Reaction Products
35. An aldehyde results from the reaction of which of these compounds with aqueous base?
A) CH3CH2CH2Cl
B) CH3CHClCH2Cl
C) CH3CH=CCl2
D) CH3CH2CHCl2
E) CH3CCl2CH3
Ans: D
587
Chapter 16
Topic: Reaction Products
36. The Baeyer-Villiger oxidation of propiophenone (ethyl phenyl ketone) produces chiefly:
O
A)
B)
CH3CH2OCC6H5
O
C)
CH3CH2OCOC6H5
O
D)
p-HOC6H4CCH2CH3
O
E)
C6H5OCCH2CH3
O
C6H5CCH2CH2OH
Ans: D
Topic: Reaction Products
37. What is compound V in the following synthesis?
1. ether
O
Cl
+ V
(C6H5)3P + W
A)
B)
C)
D)
E)
Ans:
O
2. H3O+
C6H5Li
ether
Wittig
reagent
CH3CH2COCl
CH3CH2MgBr
CH3COCl
(CH3)2CuLi
(CH3CH2)2CuLi
E
588
Chapter 16
Topic: Reaction Products
38. What is the reactant W in the synthesis given below?
1. ether
O
O
+ V
Cl
2. H3O+
(C6H5)3P + W
A)
B)
C)
D)
E)
Ans:
C6H5Li
Wittig
reagent
ether
Cyclopentanone
Cyclopentene
Cyclopentanol
Bromocyclopentane
Triphenylphosphine oxide
D
Topic: Reaction Products
39. What is the final product, Z, of the following synthesis?
1. KMnO4,
CH3
heat
X
2. H3O+
SOCl2
Y
1. LiAlH(O-t-Bu)3
ether, -78 oC
2. H2O
O
O
O
O
O
COAl(t-Bu)2
COCH3
CH
COLi
COH
I
A)
B)
C)
D)
E)
Ans:
OH-,
II
III
I
II
III
IV
V
C
589
IV
V
Z
Chapter 16
Topic: Reaction Products
40. What is the final product of this synthetic sequence?
Br2
Mg
FeCl3
ether
Benzene
1. C6H5CHO
H2Cr2O4
2. H3O+
acetone
?
O
A)
C6H5CC6H5
B)
C)
D)
p-BrC6H4CH2C6H5
C6H5CH2COOH
O
C6H5CH2CC6H5
E) C6H5CH2C6H5
Ans: A
Topic: Reaction Products
41. The reaction of tert-butyl methyl ketone with a peroxy acid produces which of these as
the principal product(s)?
O
A)
B)
(CH3)3COCCH3
O
C)
(CH3)3CCOCH3
CH3
(CH3)2C
COCH3
O
D)
E)
(CH3)3COH + HO2CCH3
O
(CH3)3COCOCH3
Ans: A
590
Chapter 16
Topic: Synthesis
42. Which of the following reactions would yield benzaldehyde?
OHA)
C6H5CH2Cl
H2O
heat
B)
H3O+
C6H5CH(OCH3)2
C)
1. LiAlH4, ether
C6H5COOH
2. H2O
D) Answers A) and B)
E) Answers A), B), and C)
Ans: B
Topic: Synthesis
43. Which of the reactions listed below would serve as a synthesis of acetophenone,
O
?
C6H5CCH3
A)
O
+ (CH3)2CuLi
AlCl3
B)
C6H6 + CH3COCl
ether
C)
C6H5CN + CH3Li
D) Answers A) and B) only
E) Answers A), B), and C)
Ans: E
C6H5CCl
H 3O +
Topic: Synthesis
44. Which of the following would yield 3-pentanone as the major product?
ether
A)
H 3O +
CH3CH2CN + CH3CH2Li
Product
ether
B)
H 3O +
CH3CH2CO2H + 2 CH3CH2Li
Product
O
C)
ether
CH3CH2CCl + (CH3CH2)2CuLi
D) Two of these
E) All of these
Ans: E
591
Chapter 16
Topic: Synthesis
O
45.
Which of the following procedures would not yield (CH3)2CHCH as a product?
PCC
I
OH
CH2Cl2
SOCl2
II
CO2H
Zn
HOAc
KMnO4
IV
V
ether, -78 oC
O3
III
LiAlH(O-t-Bu)3
OH
O
H3O+
NaOH
H3O+
O
A)
B)
C)
D)
E)
Ans:
I
II
III
IV
V
D
592
Chapter 16
Topic: Synthesis
46. Which of the following procedures would not yield 3-pentanone as a major product?
CH3CH2MgBr
A)
H3O+
CH3CH2CN
ether
O
B)
2 CH3CH2Li
H2O
CH3CH2COH
C)
CH3CH2CN
H3O+
ether
O
D)
CH3CH2CCl
E)
ether
CH3CH2Li
CH3CH2CO2H
(CH3CH2)2CuLi
ether
CH3CH2MgBr
H3 O+
ether
Ans: E
Topic: Synthesis
47. How could the following synthetic conversion be accomplished?
O
CH3CH2CCH3
CH3CH2CH2CHO
A) HgSO4/H2SO4; then PCl5/0°C; then NaNH2, liq. NH3
B) PCl5/0°C; then NaNH2, liq. NH3; then Sia2BH; then H2O2
C) PCl5/0°C; then NaNH2, liq. NH3; then HgSO4, H2SO4/H2O
D) NaNH2, liq. NH3; then PCl5/0°C; then HgSO4, H2SO4/H2O
E) H2O2; then PCl5/0°C; then NaNH2, liq. NH3; then Sia2BH
Ans: C
593
Chapter 16
Topic: Synthesis
48. Which sequence of reactions would be utilized to convert
O
CO2CH3
into
CH3
C CH3
O
?
OH
A)
B)
C)
D)
E)
Ans:
2CH3MgBr, then NH4+
HOCH2CH2OH, H3O+; LiAlH4, ether; 2CH3MgBr, then NH4+
HOCH2CH2OH, H3O+; 2CH3MgBr, then NH4+
HOCH2CH2OH, H3O+; H2, Pt; CH3OH, NH4+
None of the above
C
Topic: Synthesis
49. Which reaction sequence would be used to prepare
?
I
O
+
MgBr
O
II
H
III
IV
A)
B)
C)
D)
E)
Ans:
+ 2 CH3MgBr
(C6H5)3P
1. ether
H2
2. H3O+
Ni
1. ether
H2
2. H3O+
Ni
C6H5Li
cyclopentanone
Cl
H2
Ni
O
CH3MgBr
H3O+
ether
I
II
III
IV
All of the above would yield the product.
C
594
Chapter 16
Topic: Synthesis
50. Which Wittig reagent would be used to synthesize
?
CH3CH=P(C6H5)3
A)
B)
C)
D)
E)
Ans:
CH=P(C6H5)3
I
II
I
II
III
IV
Either I or II could be used.
E
P(C6H5)3
III
595
CH3CH2CH=P(C6H5)3
IV
Chapter 16
Topic: Synthesis
51. A good synthesis of
H
O
would be:
H
H2O, H2SO4
I
HgSO4
H
1. Sia2BH
II
2. H2O2, NaOH
H
1. O3
III
2. Zn, HOAc
OH
IV
A)
B)
C)
D)
E)
Ans:
1. KMnO4, NaOH, heat
2. H3O+
I
II
III
IV
All of these are equally useful.
B
596
Chapter 16
Topic: Synthesis
52. Which of the following is not a synthesis of benzophenone,
O
C6H5CC6H5
?
O
A)
C6H6 + C6H5CCl
B)
C)
(C6H5)2CHOH
(C6H5)2CH=CH2
AlCl3
H2CrO4
acetone
1. O3
2. Zn, HOAc
1. ether
D)
(C6H5)2CO2H + 2 C6H5Li
2. H2O
E) All of the above will give benzophenone.
Ans: E
Topic: Synthesis
53. Which of the following is a synthesis of 3-heptanone?
A)
PCC
1. CH3CH2MgBr
H2Cr2O4
CH3CH2CH2CH2CH2OH
acetone
CH2Cl2
2. H3O+
+
CH3(CH2)2CH2Li
B)
H3 O
CH3CH2CN
ether
PBr3
CH3CH2MgBr
C)
H3 O+
NaCN
CH3CH2CH2CH2OH
ether
ether
+
KMnO4
2 CH3CH2Li
D)
H2 O
H3O
CH3CH2CH2CH2CH2OH
OH-,
ether
E) All of the above are syntheses of 3-heptanone.
Ans: E
597
Chapter 16
Topic: Synthesis
54. Which synthesis or syntheses would yield propanal?
PCC
A)
CH3CH2CH2OH
CH2Cl2
O
B)
LiAlH[OC(CH3)3]3
CH3CH2CCl
C)
CH3C
CH
ether, -78 oC
1. Sia2BH
2. H2O2, OH-
D) All of these
E) None of these
Ans: D
Topic: Reagents
55. LiAlH4 (LAH) cannot be used to convert carboxylic acids to the corresponding
aldehydes because:
A) LAH is not sufficiently reactive.
B) RCOOH is converted into RCOOLi.
C) RCOOH is reduced to RCH2OH.
D) RCOOH is reduced to RCH3.
E) RCOOH is converted into R2C=O.
Ans: C
Topic: Reagents
56. The following reduction can be carried out with which reagent(s)?
?
O
A)
B)
C)
D)
E)
Ans:
Zn(Hg), HCl
i) HSCH2CH2SH, BF3 ; ii)Raney Ni (H2)
NaBH4, CH3OH
A&B
A&C
D
598
Chapter 16
Topic: Reagents
57. Identify the reagent(s) that would bring about the following reaction:
CH3CH2CH2COCl ⎯⎯⎯→ CH3CH2CH2CHO
A) H2/Ni
B) Li/liq.NH3
C) LiAlH[OC(CH3)3]3, ether
D) NaBH4, CH3OH
E) LiAlH4, ether
Ans: C
Topic: Reagents
58. Which reagent will not differentiate between 3-butenal and 2-butanone?
A) Br2/CCl4
B) Ag2O,OHC) H2NNHC6H5
D) KMnO4,OHE) None of these
Ans: C
Topic: Reagents
59. The Reformatsky reaction involves the reaction of an aldehyde or ketone with:
A) RLi
B) R2CuLi
R
C)
XZnCHCO2R'
D) RC≡CNa
E) R3P
Ans: C
Topic: Reagents
60. Acetals are unstable in the presence of an aqueous solution of which of these?
A) HCl
B) NaOH
C) KHCO3
D) Na2CO3
E) NaCl
Ans: A
599
Chapter 16
Topic: Reagents
H
61.
H
The compound C6H5C N N CC6H5 is produced by the reaction of an excess of
benzaldehyde with which compound?
A) Ammonia
B) Hydrazine
C) Nitrogen
D) Phenylhydrazine
E) Hydroxylamine
Ans: B
Topic: Reagents
62. Which of these will not catalyze the reaction of a weak nucleophile with the carbonyl
group of an aldehyde or ketone?
A) IB) H3O+
C) AlCl3
D) BF3
E) ZnCl2
Ans: A
Topic: Reagents
63. Which reagent(s) could be used to carry out the following transformation?
O
?
A)
B)
C)
D)
E)
Ans:
Zn(Hg), HCl, reflux
LiAlH4, ether
HSCH2CH2SH, BF3; then Raney Ni (H2)
All of the above
Two of the above
E
600
Chapter 16
Topic: Physical Properties, Relationships, Stereochemistry
64. Which of the compounds listed below would you expect to have the highest boiling
point? (They all have approximately the same molecular weight.)
A) pentane
B) 1-butanol
C) Butanal
D) 1-fluorobutane
E) diethylether
Ans: B
Topic: Physical Properties, Relationships, Stereochemistry
65. What is the order of decreasing reactivity of aldehydes, esters and ketones towards the
organozinc compounds used in the Reformatsky reaction?
A) Aldehydes > esters > ketones
B) Esters > aldehydes > ketones
C) Ketones > esters > aldehydes
D) Ketones > aldehydes > esters
E) Aldehydes > ketones > esters
Ans: E
Topic: Physical Properties, Relationships, Stereochemistry
66. What, in general, is the order of decreasing reactivity of these carbonyl compounds
towards nucleophilic reagents?
O
CH3CCH3
H
CH3C
O
O
I
II
A) I > III > V > II > IV
B) IV > II > I > III > V
C) V > III > I > II > IV
D) II > I > V > III > IV
E) III > V > IV > II > I
Ans: B
(CH3)3CCCH3
III
601
H
HC
IV
O
O
(CH3)3CCC(CH3)3
V
Chapter 16
Topic: Physical Properties, Relationships, Stereochemistry
67. Which of these gem-diols is expected to be the most stable?
OH
A)
CF3CCF3
OH
B)
C)
CH3CH(OH)2
OH
CH3CCH3
OH
D) C6H5CH(OH)2
E) ClCH2CH(OH)2
Ans: A
Topic: Physical Properties, Relationships, Stereochemistry
68. The relationship of 3-methyl-2-heptanone and 3-methyl-2-hepten-2-ol is designated by
the term:
A) Tautomers
B) Conformational isomers
C) Diastereomers
D) Resonance structures
E) Stereoisomers
Ans: A
602
Chapter 16
Topic: Physical Properties, Relationships, Stereochemistry
.
69. In which of the following reactions is the final product expected to be formed as a
racemic mixture?
H
A)
C6H5MgBr + C6H5C
O
O
B)
HCN +
C)
H
D)
CH3CH2C
H
CH3C
CH3CCH3
O
O
+
BrZnCH2COC2H5
+ xs CH3OH + HCl
O
O
E)
CH3CHCH2COC2H5
+ (i-Bu)2AlH
CH3
Ans: C
Topic: Physical Properties, Relationships, Stereochemistry
70. Stereoisomers can exist in the case of which of the following?
A) The hydrazone of butanone
B) The oxime of acetone
C) The phenylhydrazone of cyclohexanone
D) The cyclic acetal formed from propanal and ethane-1,2-diol
E) The imine of cyclopentanone
Ans: A
Topic: Physical Properties, Relationships, Stereochemistry
71. Stereoisomers can exist in the case of which of the following?
A) The hydrazone of acetophenone
B) The oxime of 2-methylcyclopentanone
C) The phenylhydrazone of 4-methylcyclohexanone
D) The imine of cyclopentanone
E) Two of the above
Ans: B
603
Chapter 16
Topic: Reactions of Aldehydes and Ketones
72. Which of the following is formed when 3-methylcyclopentanone reacts with
hydroxylamine?
N-NH-C6H5
N-NH2
I
N-OH
II
III
NO2
O
N
N
H
N
NH2
N
H
IV
A)
B)
C)
D)
E)
Ans:
NO2
V
I
II
III
IV
V
C
Topic: Reactions of Aldehydes and Ketones
73. Which of the following is formed when 3-methylcyclopentanone reacts with
semicarbazide?
N-NH-C6H5
N-NH2
I
N-OH
II
III
NO2
O
N
N
H
NH2
N
N
H
IV
A)
B)
C)
D)
E)
Ans:
V
I
II
III
IV
V
D
604
NO2
Chapter 16
Topic: Reactions of Aldehydes and Ketones
74. Which of the following is formed when 4-methylpentanal reacts with phenylhydrazine?
N-C6H5
N-NH-C6H5
I
N-NH2
II
III
O
N
N
H
N=N-C6H5
NH2
V
IV
A)
B)
C)
D)
E)
Ans:
I
II
III
IV
V
A
Topic: Reactions of Aldehydes and Ketones
75. Which of the following is formed when 2,2,3-trimethylcyclohexanone reacts with
hydroxylamine?
N-NH-C6H5
N-NH2
I
N-OH
III
II
NO2
O
N
N
H
NH2
N
N
H
IV
A)
B)
C)
D)
E)
Ans:
V
I
II
III
IV
V
C
605
NO2
Chapter 16
Topic: Reactions of Aldehydes and Ketones
76. Which of the following is formed when 2,2,3-trimethylcyclohexanone reacts with
semicarbazide?
N-NH-C6H5
N-NH2
I
N-OH
III
II
NO2
O
N
N
H
N
NH2
N
H
IV
A)
B)
C)
D)
E)
Ans:
NO2
V
I
II
III
IV
V
D
Topic: Reactions of Aldehydes and Ketones
77. Which of the following is formed when 2,2,3-trimethylcyclohexanone reacts with
hydrazine?
N-NH-C6H5
N-NH2
I
N-OH
III
II
NO2
O
N
N
H
NH2
N
N
H
IV
A)
B)
C)
D)
E)
Ans:
V
I
II
III
IV
V
B
606
NO2
Chapter 16
Topic: NMR
78. A compound with formula C5H10O gives two signals only, both singlets, in the 1H NMR
spectrum. Which of these structures is a possible one for this compound?
O
A)
B)
CH3CH2CCH2CH3
O
C)
(CH3)2CHCCH3
H
(CH3)3CC
D)
O
H
CH3CH2CHC
O
CH3
H
E)
(CH3)2CHCH2C
O
Ans: C
Topic: NMR
79. The 1H NMR spectrum of a compound with formula C7H14O gives three signals. Which
of these structures is a possible one for this compound?
A) 2-heptanone
B) 3-heptanone
C) 2,4-dimethyl-3-pentanone
D) 2,2-dimethyl-3-pentanone
E) two of the above
Ans: E
607
Chapter 16
Topic: NMR
80. The 13C NMR spectrum of a compound with formula C7H14O gives three signals.
Which of these structures is a possible one for this compound?
A) 2-heptanone
B) 3-heptanone
C) 2,2-dimethyl-3-pentanone
D) 2,4-dimethyl-3-pentanone
E) two of the above
Ans: D
Topic: NMR
81. The 1H NMR spectrum of a compound with formula C7H14O gives a doublet at 9.2 ppm.
Which of these structures is a possible one for this compound?
A) 2-methyl-3-hexanone
B) 2-methylhexanal
C) 2,2 -dimethylpentanal
D) 2,2-dimethyl-3-pentanone
E) two of the above
Ans: B
SHORT ANSWER QUESTIONS
Topic: Relative Reactivity
82. Aldehydes are more reactive than ketones towards nucleophilic substitution because of
both steric and electronic factors. Briefly explain.
a. Steric reasons: ______________.
b. Electronic reasons: __________.
Ans: a. aldehydes have the small hydrogen substituent on one side, so less steric
crowding than ketones
b. aldehydes have only one electron-releasing group to partly neutralize/stabilize
the partial positive charge at the carbonyl carbon while ketones have two
Topic: General
83. A compound with an OH and OR group attached to the same carbon is known as a(n)
_____________.
Ans: hemiacetal
608
Chapter 16
Topic: General
84. A compound with two OH groups attached to the same carbon is known as a(n)
_________.
Ans: hydrate or gem-diol
Topic: General
85. A compound with two OR groups attached to the same carbon is known as a(n)
_________.
Ans: acetal
Topic: General
86. A compound containing a C=N double bond is known as a(n) __________.
Ans: imine
Topic: General
87. A compound with an OH and CN group attached to the same carbon is known as a(n)
_____________.
Ans: cyanohydrin
Topic: Wittig Reaction
88. The reaction between a phosphorus ylide and an aldehyde or ketone is called the
___________ reaction.
Ans: Wittig
Topic: Wittig Reaction
89. The driving force for the Wittig reaction is the formation of the very strong
_______________ bond in _______________.
Ans: P-O bond; triphenylphosphine oxide
Topic: General
90. A “green” version of the __________________ uses a tin-zeolite catalyst with hydrogen
peroxide.
Ans: Baeyer-Villiger oxidation
609
Chapter 16
Topic: Functional Group Tests
91. 2,4-DNP is a reagent frequently used for functional group tests; it detects the presence
of _____________.
Ans: aldehydes and ketones
Topic: Functional Group Tests
92. The Tollens test, used in conjunction with 2,4-DNP, can distinguish between
______________ and _______________.
Ans: aldehydes and ketones
Topic: Functional Group Tests
93. What reagent would help you distinguish between acetophenone and 2methylbenzaldehyde?
Ans: Tollen’s Reagent
Topic: Functional Group Tests
94. Oximes are formed when aldehydes and ketones react with _______________.
Ans: Hydroxylamine, NH2OH
Topic: Synthetic Strategy
95. Suggest a synthetic strategy for the synthesis of 3-heptanone from 1-bromobutane.
i. C2H5Li, Et2O
Ans:
NaCN
Br
CN
ii. H3O+
O
Topic: Synthetic Strategy
96. Suggest a synthetic strategy for the synthesis of 4-methylpentanal from 3-methyl-1bromobutane.
Ans:
i. DIBAL-H
O
NaCN
CN
Br
ii. H2O
610
Chapter 16
Topic: Synthetic Strategy
97. Suggest a synthetic strategy for the synthesis of 6-methyl-3-heptanol from 4methylpentanenitrile, (CH3)2CHCH2CH2CN.
Ans:
i. DIBAL-H
CN
ii. H2O
OH
iii. C2H5MgBr, Et2O
iv. H3O+
OR
i. C2H5MgBr, Et2O
CN
ii. H3O+
OH
iii. NaBH4, H2O
Topic: Reaction Sequence
98. Complete the following reaction sequence, giving structural details of all significant
intermediates.
i. Br2, hν
?
ii. Mg, CH3CH2CN
iii. H3O+
iv. RCO3H
O
Ans:
Br2, hν
Br i. Mg, CH CH CN
3
2
ii. H3O+
RCO3H
O
O
611
Chapter 16
Topic: Reaction Sequence
99. Complete the following reaction sequence, giving structural details of all significant
intermediates.
i. O3
i. EtMgBr
?
+
ii.Zn, CH3CO2H
ii. H3O , heat
O
Ans:
O
i. EtMgBr
ii. H3O+, heat
i. O3
ii.Zn, CH3CO2H
HO
O
O
Topic: NMR
100. Explain clearly how you might use 1H NMR to distinguish between
2-phenylacetaldehyde and acetophenone
Ans: A quick inspection of 1H NMR of the two substances would allow easy
identification and distinction between them: the expected 1H NMR spectra of the
two substances are described below.
2-Phenylacetaldehyde
Acetophenone
and
C6H5COCH3
C6H5CH2CHO
3 signals
2 signals
approximate δ ppm
approximate δ ppm
singlet (-CH3) 2.0
doublet (-CH2-) 2.5
multiplet (C6H5) 7.2
multiplet (C6H5) 7
doublet (-CHO) 9
612
Chapter 17
MULTIPLE CHOICE QUESTIONS
Topic: Acid/Base
1. Which compound would be most acidic?
A) CH3CH2CH3
B) CH3CH=CH2
C) Cyclohexane (CH3)2C=O
D) (CH3)2C=O
E) Benzene
Ans: D
Topic: Reaction Products
2. What would be the major product of the following reaction?
O
C6H5CH
O
+
CH3CH
OHheat
O
A)
B)
C6H5CH2CCH3
O
O
C)
C6H5CCH2CH
OH
OH
D)
C6H5CHCH2CH2
O
E)
C6H5CH2CH2CH
O
C6H5CH=CHCH
Ans: E
613
Chapter 17
Topic: Reaction Products
3. The product, C, of the following sequence of reactions,
CH3
CH3CCHO
+ CH3CH2CHO
OH-
A
CH3
would be:
CH3
A)
CH3CCH=CHCH2CH2OH
CH3
CH3
B)
CH3CCH=CCH2OH
CH3 CH3
CH3
C)
CH3CCH2CH2CH2OH
CH3
CH3
D)
CH3CCH2CH2CH2CH2OH
CH3
CH3
E)
CH3CCH2CHCHO
CH3 CH3
Ans: B
614
H3O+
(-H2O)
B
NaBH4
CH3OH
C
Chapter 17
Topic: Reaction Products
4. What would be the major product of the following reaction?
O
O
CH3
CH3CCH3
+ OH-
CH
O
CH3
C
CH
?
100 oC
O
CH3
CH
CH3
CH
CH3
C
CH3
CH3
I
OH
II
III
O
CH3
CH
OH
CHCCH3
CH3
CHC(CH3)2
OH
IV
A)
B)
C)
D)
E)
Ans:
V
I
II
III
IV
V
D
Topic: Reaction Products
5. What would be the major product of the following reaction?
O
OH-
?
H2O
CH3CH2CH2OH + CH3CH2COO-
CH3CH2CH
A)
B)
CH3CH2CHCH2CHO
OH
O
C)
D)
CH3CH2CH2OCCH2CH3
CH3CH2CHCHCHO
HO CH3
O
E)
CH3CH2CHCCH2CH3
HO
Ans: D
615
CHCH2CCH3
Chapter 17
Topic: Reaction Products
6. What would be the major product of the following reaction?
OHO
?
+
O
OH
O
O
O
O
OH
I
II
III
OH
OH
O
O
IV
A)
B)
C)
D)
E)
Ans:
V
I
II
III
IV
V
D
616
Chapter 17
Topic: Reaction Products
7. What would be the major product of the following reaction?
O
+
O
OHheat
O
O
HO
O
O
OH
I
III
II
HO
O
IV
A)
B)
C)
D)
E)
Ans:
?
V
I
II
III
IV
V
D
617
O
Chapter 17
Topic: Reaction Products
8. What would be the major product, B, of the following reaction sequence?
O
+
OHheat
O
A
i. CH3Cu
ii H3O+
B
O
O
O
OH
I
II
III
O
OH
IV
A)
B)
C)
D)
E)
Ans:
V
I
II
III
IV
V
C
618
Chapter 17
Topic: Reaction Products
9. What would be the major product, B, of the following reaction sequence?
O
+
OHheat
O
A
B
O
OH
OH
OH
I
OH
II
III
OH
OH
IV
A)
B)
C)
D)
E)
Ans:
NaBH4
H2O
V
I
II
III
IV
V
C
619
Chapter 17
Topic: Reaction Products
10. What would be the major product of the following reaction?
O CH3
OD?
+ D2O
room
C6H5C CHCH3
temp.
O
CD
A)
3
B)
C 6 H 5C
O
CHCH3
CH3
C)
C 6H 5 C
O
CDCH3
CH3
D)
C 6D 5 C
O
CDCH3
CD3
E)
C 6H 5 C
CDCD3
OD
C6H5CDCHCH3
CH3
Ans: B
Topic: Reaction Products
11. What would be the product of the following reaction?
NaCN
?
CH3CH=CO2C2H5
C2H5OH, CH3COOH
CH3CH2CHCO2C2H5
A)
CN
B)
C)
D)
CH3CH=CHCO2CN
CH3CH=CHCN
CH3CHCH2CO2C2H5
CN
CN
E)
CH3CH=CHCOC2H5
OH
Ans: D
620
Chapter 17
Topic: Reaction Products
12. Select the structure of the major product formed in the following reaction.
1. CH3Cu
(CH3)2C=CHCCH3
2. H2O
(CH3)3CCH2COCH3
O
A)
B)
C)
(CH3)2C=CHC(CH3)2
OH
(CH3)2CHCHCOCH3
CH3
D)
2 CH3CCH3
O
OH
E)
(CH3)2C=CHCCH3
H
Ans: A
Topic: Reaction Products
13. What would be the product of the following reaction?
O
+ CH3CH2CN
C6H5CH=CHCH2CN
C6H5CH
A)
B)
EtO-, EtOH
O
C6H5CCHCN
C)
CH3
C6H5CH=CCN
D)
OH
CH3
E)
C6H5CHCH2CH2CN
OH
C6H5CCH2CH2CN
OCH2CH3
Ans: C
621
?
Chapter 17
Topic: Reaction Products
14. What would be the final product of the following reaction sequence?
O
O
C6H5CH
A)
B)
+
CH3CCH3
OH-, heat
O
C)
C6H5CHCH2CCH3
O
D)
C6H5CH2CH2CCH3
OH
E)
C6H5CH=CHCHCH3
OH
OH
LiAlH4
ether
C6H5CH2CH2CH2CH3
OH
A
C6H5CHCH2CHCH3
Ans: D
622
Final
Product
Chapter 17
Topic: Reaction Products
15. What would be the major product of the following reaction?
O
C6H5CCHCH2CH3
+ Br2
H3O+
CH3
O
A)
C6H5CCBrCH2CH3
CH3
O Br
B)
C6H5CCHCHCH3
CH3
O OH
C)
C6H5CCHCHCH3
D)
CH3
C6H5CBr2CHCH2CH3
CH3
O
E)
m-BrC6H4CCHCH2CH3
CH3
Ans: A
623
?
Chapter 17
Topic: Reaction Products
16. What would be the major product of the following reaction?
O
C6H5CH
O
+
CH3CCH3
OH-
?
heat
O
A)
OH
B)
C6H5CCH2CHCH3
O
O
C)
C6H5CCH2CCH3
OH
OH
D)
C6H5CHCH2CHCH3
O
O
E)
C6H5CCH2CCH3
O
C6H5CH=CHCCH3
Ans: E
Topic: Reaction Products
17. What would be the product, C, of the following reaction sequence?
O
O
H2
H3O+
OHA
B
(CH3)3CCH + CH3CH2CH
Ni
heat
A)
B)
(CH3)3CCH2CH2CH2OH
O
OH
C)
(CH3)3CCHCH2CH2CH
(CH3)3CCH2CHCH2OH
D)
CH3
(CH3)3CCH=CCH2OH
CH3
O
E)
(CH3)3CCH=CCH
CH3
Ans: C
624
C
Chapter 17
Topic: Reaction Products
18. What would be the product, C, of the following reaction sequence?
O
O
+ CH3CH2CH
(CH3)3CCH
A)
B)
OH-
A
H3O+
heat
B
CH2
(CH3)3CCH2CH2CH2OH
OH
C)
(CH3)3CCHCH2CH=CH2
(CH3)3CCH2C=CH2
D)
CH3
(CH3)3CCH=CCH=CH2
CH3
OH
E)
(CH3)3CCHCHCH=CH2
CH3
Ans: D
Topic: Reaction Products
19. What would be the product, B, of the following reaction sequence?
O
C6H5CH
A)
B)
+ CH3CN
EtO-, EtOH
A (C9H7N)
C6H5CH=CHCO2H
O
C6H5CCH2CO2H
C)
D)
E)
C6H5CH2CH2CO2H
OH
C6H5CHCH2CO2H
OH
C6H5CHCH2CN
Ans: A
625
H3O+
heat
B
P(C6H5)3
C
Chapter 17
Topic: Reaction Products
20. What would be the product, C, of the following reaction sequence?
O
O
NaBH4
H3O+, heat
OHA
B
C6H5CH + CH3CH2CH
CH3OH
(-H2O)
25 oC
O
A)
C6H5CCHCH2OH
B)
CH3
C6H5CHCHCH2OH
HO CH3
O
C)
C6H5CH2CHCH
CH3
O
D)
C6H5CH=CCH
E)
CH3
C6H5CH=CCH2OH
CH3
hi
Ans: E
626
C
Chapter 17
Topic: Reaction Products
21. What would be the product, C, of the following reaction sequence?
O
O
H2, Ni
H3O+, heat
OHA
B
C6H5CH + CH3CH2CH
(-H2O)
high
25 oC
pressure
O
A)
C6H5CCHCH2OH
B)
CH3
C6H5CH2CHCH2OH
CH3
O
C)
C6H5CH2CHCH
CH3
O
D)
C6H5CH=CCH
E)
CH3
C6H5CH=CCH2OH
CH3
Ans: B
627
C
Chapter 17
Topic: Reaction Products
22. What would be the major product of the following reaction?
O
+ CN-
C6H5CH=CHCCH3
CH3CH2OH
CH3COOH
O
A)
C6H5CH2CHCCH3
CN
O
B)
C6H5CHCH2CCH3
CN
OH
C)
C6H5CH2CH2CCH3
CN
O
D)
E)
C6H5CH2CH2CCN
OH
C6H5CHCH=CCH3
CN
Ans: B
628
?
Chapter 17
Topic: Reaction Products
23. What would be the major product(s) of the following reaction?
O
i.
Cu
ii. H3O+
O
OH
I
O
III
II
OH
A)
B)
C)
D)
E)
Ans:
?
IV
I and V
II and IV
I and III
III and IV
None of these
A
O
V
629
Chapter 17
Topic: Reaction Products
24. What would be the major product of the following reaction?
O
O
?
OHOH
O
O
O
O
I
O
III
II
O
O
IV
A)
B)
C)
D)
E)
Ans:
O
V
I
II
III
IV
V
E
630
Chapter 17
Topic: Reaction Products
25. What would be the major product of the following reaction?
i. Br2, NaOH
?
ii. H3O+
O
Br
HO
O
O
I
O
II
Br
Br
O
A)
B)
C)
D)
E)
Ans:
IV
O
V
I
II
III
IV
V
B
631
III
Chapter 17
Topic: Reaction Products
26. What would be the major product of the following reaction sequence?
O
i. LDA
ii. C2H5I
?
O
O
O
I
II
III
O
V
IV
A)
B)
C)
D)
E)
Ans:
I
II
III
IV
V
I
Topic: Synthetic Strategy
27. What synthetic strategy would accomplish the following transformation?
?
O
A)
B)
C)
D)
E)
Ans:
O
i) Br2, H3O+; ii) NaOC2H5, C2H5OH, heat
i) Cl2, FeCl3; ii) NaOC2H5, C2H5OH, heat
i) HCN; ii) H3O+, heat
i) Br2, hν; ii) (CH3)3COK, (CH3)3COH, heat
i) Br2, H3O+; ii) (CH3)3COK, (CH3)3COH, heat
A
632
Chapter 17
Topic: Reaction Products
28. What would be the major product of the following reaction?
O
CCHCH3
OH-
+ Br2
?
CH3
O
CCHCH3
Br
CH3
Br
O
O
CCHCH3
CCBrCH3
CH3
I
II
III
O
O
CCHCH2Br
CCHCH2Br
CH3
CH2Br
IV
A)
B)
C)
D)
E)
Ans:
CH3
V
I
II
III
IV
V
C
Topic: Reaction Products
29. Which compound would be formed when 2-methylbutanal is treated with a solution of
NaOD in D2O?
CH3CH2CDCHO
A)
B)
CH3
CH3CH2CHCDO
C)
CH3
CH3CHDCHCHO
D)
CH3
CH2DCH2CHCHO
E)
CH3
CH3CH2CDCDO
CH3
Ans: A
633
Chapter 17
Topic: Reaction Products
30. Which compound would be formed when 2,3-dimethylhexanal is treated with a solution
of NaOD in D2O?
D
A)
O
D
B)
O
D
D
C)
O
D)
O
D
E)
O
D
Ans E
Topic: Reaction Products
31. What is the final product of this reaction sequence?
CH3CH2NO2
H2, Ni
?
H2C=O
OHNH2
A)
CH3C=CH2
B)
C)
CH3CH2CH2NH2
NH2
CH3CHCH3
D)
E)
CH3CH2CH2NO2
NO2
CH3CHCH3
Ans: C
634
Chapter 17
Topic: Reaction Products
32. Which of these is a product of the reaction of C6H5MgBr with
O
C6H5CH=CHCCH3
O
A)
?
C6H5CH=CCCH3
C 6H 5
OH
B)
C6H5CH=CHCCH3
C6H5
O
C)
C6H5CH=CHCCH2C6H5
O
D)
p-C6H5
E)
C6H4 CH=CHCCH3
OC6H5
C6H5CH=CHCHCH3
Ans: B
Topic: Reaction Products
33. What is the product of the reaction below?
O
1. (C6H5)2CuLi
?
2. H2O
O
O
HO C6H5
C6H5
C6H5
A)
B)
C)
D)
E)
Ans:
I
I
II
III
IV
V
B
II
O
O
C6H5
C6H5
III
635
IV
V
Chapter 17
Topic: The Aldol Reaction/Condensation (Explicitly Identified)
34. Which reagents would you use to synthesize this compound by an aldol condensation?
O
C6H5CH=CHCC6H5
O
O
A)
and
C6H5CH
B)
and
C6H5CH2CH
C6H5CCH3
O
C)
D)
C6H5CH2CH
O
O
C6H5CH=CHCH
O
C6H5CCH3
and
and
C6H5OH
O
C6H5CH
O
E)
(C6H5)2CuLi and CH2=CHCC6H5
Ans: D
Topic: The Aldol Reaction/Condensation (Explicitly Identified)
35. What starting compound(s) would you use in an aldol reaction to prepare
O
CH
OH
O
O
?
O
O
O
CH3CHCH2CH2CCH3
CH3CCH2CH2CCH3
CH3CCH2CH2CH2CH
I
II
III
O
O
O
HCCH2CH2CH2CH
IV
A)
B)
C)
D)
E)
Ans:
+ H2C=O
V
I
II
III
IV
V
D
636
Chapter 17
Topic: The Aldol Reaction/Condensation (Explicitly Identified)
36. Which reagents would you use to synthesize this compound by an aldol condensation?
O
C6H5CH=CCH
C6H5
O
A)
O
and
C6H5CH
and
C6H5CH2CH
C6H5CCH3
O
C)
D)
C6H5CH2CH
O
O
B)
C6H5CH=CHCH
O
C6H5CCH3
E)
and
and
C6H5OH
O
C6H5CH
O
C6H5CH2Cl and C6H5CHCH
ONa
Ans: A
Topic: The Aldol Reaction/Condensation (Explicitly Identified)
37. The aldol condensation product formed from 3-pentanone in the presence of base has
the IUPAC name:
A) 5-Ethyl-4-methyl-4-hepten-3-one
B) 5-Ethyl-4-methyl-5-hepten-3-one
C) 4-Methyl-4-nonen-3,7-dione
D) 3-Ethyl-4-methyl-3-hepten-5-one
E) 3-Ethyl-4-methyl-2-hepten-5-one
Ans: A
637
Chapter 17
Topic: The Aldol Reaction/Condensation (Explicitly Identified)
38. Which of these is not among the reaction products when a crossed aldol addition occurs
between ethanal and butanal?
OH
H
A)
B)
CH3CHCH2C=O
OH
H
CH3CH2CH2CHCHCH2C=O
CH2CH3
H
OH
C)
CH3CHCHC=O
D)
CH2CH3
OH
E)
CH3CH2CH2CHCH2C=O
OH H
H
CH3CH2CH2CHCHC=O
CH2CH3
Ans: B
Topic: The Aldol Reaction/Condensation (Explicitly Identified)
H
39.
C6H5CH=CC=O
CH2CH3
The retro-aldol reaction of
H
A)
B)
C6H5CH2CH2C=O
H
C)
C6H5CH2C=O
H
C6H5C=O
H
+
+
+
CH3C=O
H
CH3CH2C=O
O
CH3CH2CC=O
H
O
D)
C6H5CH2CCH2CH3
H
H
E)
C6H5C=O
+ H2C=O
+
CH3CH2CH2C=O
Ans: E
638
gives:
Chapter 17
Topic: The Aldol Reaction/Condensation (Explicitly Identified)
40. If butanal is added slowly to an aqueous solution of sodium hydroxide and 2,2dimethylpropanal at 25°C, the principal product is which of these?
OH H
A)
(CH3)3CCHCHC=O
CH2CH3
OH H
B)
CH3CH2CH2CHCHC=O
CH2CH3
OH
C)
(CH3)3CCHCC(CH3)3
O
OH
D)
E)
CH3CH2CH2CHC(CH3)3
OH H
CH3CH2CH2CHCHC=O
C(CH3)3
Ans: A
639
Chapter 17
Topic: The Aldol Reaction/Condensation (Explicitly Identified)
41. What compound results from the aldol cyclization of
O
O
CH3CCH2CH2CHCH2CCH3
?
CH3
CH3
CH3
CH3
O
CH3
O
CH3
CH3
O
CH3
CH3
CH3
CH3
I
A)
B)
C)
D)
E)
Ans:
O
II
III
IV
I
II
III
IV
Both III and IV
E
Topic: The Aldol Reaction/Condensation (Explicitly Identified)
O
42.
H
The aldol cyclization of CH3CCH2CH2CH2C=O produces which of these?
O
C CH3
I
A) I
B) II
C) III
D) IV
E) V
Ans: D
H
CH3
H
O
C=O
II
O
C=O
III
640
CH3
O
IV
O
V
Chapter 17
Topic: The Aldol Reaction/Condensation (Explicitly Identified)
43. What product results from the intramolecular aldol reaction of 2,5-hexanedione?
O
O
O
CH3
CH3
CCH3
CH3
I
A)
B)
C)
D)
E)
Ans:
CH3
CH3
CCH3
O
II
III
O
IV
V
I
II
III
IV
V
A
Topic: The Aldol Reaction/Condensation (Explicitly Identified)
O
44.
O
The reaction of CH3CCH2CH2CH2CCH3 with base affords which of these products?
H
CH3
C=O
H
CH3
O
O
A)
B)
C)
D)
E)
Ans:
O
C=O
CH3
I
O
II
III
I
II
III
IV
V
D
641
CH3
IV
V
Chapter 17
Topic: The Aldol Reaction/Condensation (Explicitly Identified)
45. The aldol reaction of cyclohexanone produces which of these self-condensation
products?
O
O
A)
B)
C)
D)
E)
Ans:
O
I
I
II
III
IV
V
B
O
II
O O
III
IV
V
Topic: The Aldol Reaction/Condensation (Explicitly Identified)
46. Which is the only one of these compounds which cannot self-condense in the presence
of dilute aqueous alkali?
A) Phenylethanal
B) Propanal
C) 2-Methylpropanal
D) 3-Methylpentanal
E) 2,2-Dimethylpropanal
Ans: E
Topic: The Aldol Reaction/Condensation (Explicitly Identified)
47. Which is the only one of these compounds which cannot self-condense in the presence
of dilute aqueous alkali?
A) Phenylethanal
B) 2-methyl-3-pentanone
C) 2-Methylpentanal
D) Acetophenone
E) 2,2-Dimethylpentanal
Ans: E
642
Chapter 17
Topic: The Aldol Reaction/Condensation (Explicitly Identified)
48. Which of these compounds cannot self-condense in the presence of dilute aqueous
alkali?
A) 3-(4-Nitrophenyl)propanal
B) 2-methyl-3-pentanone
C) 2-(4-Nitrophenyl)propanal
D) 3-(4-Nitrophenyl)-2-butanone
E) All of the above compounds can self-condense
Ans: E
Topic: The Aldol Reaction/Condensation (Explicitly Identified)
49. Which of these compounds can react with 4-methylhexanal to afford good yields of the
crossed aldol product?
A) 3-(4-Nitrophenyl)propanal
B) 2-methyl-3-pentanone
C) 2-(4-Nitrophenyl)propanal
D) 3-(4-Nitrophenyl)-2-butanone
E) None of the above compounds will give good yields of the crossed aldol product
with 4-methylhexanal
Ans: E
Topic: The Aldol Reaction/Condensation (Explicitly Identified)
50. Which of these compounds can react with 4-methylpentanal to afford good yields of the
crossed aldol product?
A) 3-(4-Nitrophenyl)propanal
B) 2ethyl-2-methylheptanal
C) 2-(4-Nitrophenyl)propanal
D) 3-(4-Nitrophenyl)-2-butanone
E) None of the above compounds will give good yields of the crossed aldol product
with 4-methylhexanal
Ans: B
643
Chapter 17
Topic: The Aldol Reaction/Condensation (Explicitly Identified)
O
51.
O
When CH3CCH2CH2CH2CH2CH2CCH3 cyclizes in basic solution, which of these
compounds will be formed?
O
C
CH3
O
C
CH3
C
CH3
I
A)
B)
C)
D)
E)
Ans:
O
CH3
CH3
O
O
O
CH2
II
CH3
CH3
III
IV
V
I
II
III
IV
V
A
Topic: The Aldol Reaction/Condensation (Explicitly Identified)
52. What final product is obtained when 2,8-nonanone is treated with base, followed by
reaction with sodium borohydride?
OH
OH
O
I
III
II
OH
O
IV
A)
B)
C)
D)
E)
Ans:
V
I
II
III
IV
V
A
644
Chapter 17
Topic: Chemical Tests
53. Which reagent would best serve as the basis for a simple chemical test to distinguish
between 2-pentanone and 3-pentanone?
A) NaOI (I2 in NaOH)
B) Br2/CCl4
C) CrO3/H2SO4
D) NaHCO3/H2O
E) Ag(NH3)2+
Ans: A
Topic: Chemical Tests
54. Which reagent would best serve as the basis for a simple chemical test to distinguish
between CH3CHO and CH3COCH3?
A) NaOI (I2 in NaOH)
B) Br2/CCl4
C) C6H5NHNH2
D) NaHCO3/H2O
E) Ag(NH3)2+
Ans: E
Topic: Chemical Tests
55. Which reagent would best serve as the basis for a simple chemical test to distinguish
between
O
C
A)
B)
C)
D)
E)
Ans:
O
CH3
C
and
CH3
?
NaOI (I2 in NaOH)
Br2/CCl4
CrO3/H2SO4
NaHCO3/H2O
Ag(NH3)2+
B
645
Chapter 17
Topic: Chemical Tests
56. Which of the reagents listed below would serve as the basis for a simple chemical test to
distinguish between 2-methyl-1-propanol and 2-butanol?
A) NaOI (I2 in NaOH)
B) KMnO4 in H2O
C) Br2 in CCl4
D) Cold concd H2SO4
E) CrO3 in H2SO4
Ans: A
Topic: Chemical Tests
57. A compound, C5H10O, reacts with phenylhydrazine and gives a positive iodoform test.
The compound could be which of these?
H
A)
B)
CH3CH2CH2CH2C=O
H
CH3CHCH2C=O
CH3
C)
D)
CH2=CHCH2CHOHCH3
O
CH3CHCCH3
CH3
O
E)
CH3CH2CCH2CH3
Ans: D
Topic: Chemical Tests
58. A negative iodoform test will be observed in the case of which of these?
A) Acetone
B) Ethanal
C) Ethanol
D) 2-Butanol
E) All of these will give a positive test.
Ans: E
646
Chapter 17
Topic: Tautomerization and Keto-Enol Equilibrium
59. Which of the following represent tautomers?
A)
CH3CH=CH O
and CH3CH CH=O
B)
CH3 CH=CH2
and CH3 C CH2
O
C)
CH3
O
CH=CH2
O
and
CH3
C
CH3
O
H
D) All of these
E) None of these
Ans: C
Topic: Tautomerization and Keto-Enol Equilibrium
60. Which of these compounds would exist in an enol form to the greatest extent?
O
A)
B)
CH3COC2H5
O
O
C)
CH3CCH2CCH3
O
D)
CH3CCH3
CH3CH
E)
O
CH3CCH2CH2CCH3
O
O
Ans: B
647
Chapter 17
Topic: Tautomerization and Keto-Enol Equilibrium
61. Which of the following is a keto-enol tautomeric pair?
O
OH
and
O
OH
and
O
and
OH
OH
I
II
O
and
III
OH
O
IV
A)
B)
C)
D)
E)
Ans:
OH
and
V
I
II
III
IV
V
B
Topic: Tautomerization and Keto-Enol Equilibrium
62. Which of the following would undergo racemization in base?
CH3
O
C
CC6H5
O
CH3CH2
C6H5CC6H5
CH3CH2CH2
I
CH3CH2
II
H
O
C
CC6H5
CH3
CH3
O
C
CCH2C6H5
CH3
III
A)
B)
C)
D)
E)
Ans:
CH3
IV
I
II
III
IV
Both III and IV
C
648
Chapter 17
Topic: Tautomerization and Keto-Enol Equilibrium
63. Which of the following would undergo racemization in base?
O
O
H3C
I
A)
B)
C)
D)
E)
Ans:
O
CH3
H3C
H
H3C
II
CH3
III
O
O
H
H
IV
V
I
II and III
IV and V
I and V
I, IV and V
D
Topic: Tautomerization and Keto-Enol Equilibrium
64. Which of the following would not undergo racemization in base?
A) (R)-3-methyl-4-heptanone
B) (R)-3-methyl-2-heptanone
C) (R)-4-methyl-2-heptanone
D) (R)-2,4-dimethyl-3-heptanone
E) All of the above will undergo racemization in base
Ans: C
Topic: Tautomerization and Keto-Enol Equilibrium
65. Which of the following would not undergo racemization in base?
A) (S)-2-phenylbutanal
B) (S)-3-phenylbutanal
C) (S)-3-phenyl-2-butanone
D) (S)-3-methyl-2-phenylbutanal
E) All of the above will undergo racemization in base
Ans: C
649
Chapter 17
Topic: Tautomerization and Keto-Enol Equilibrium
66. Which of the following represent keto-enol tautomers?
O
OH
A)
B)
CH3CCH2CH3
OH
C)
CH2=CCH2CH3
O
and
HOCH2CCH=CH2
CH3C=CHCH3
O
and
and
D) More than one of these
E) None of these
Ans: D
CH3CCH2CH3
O
CH3CCH2CH3
Topic: Tautomerization and Keto-Enol Equilibrium
67. Simple enols are less stable than the tautomeric keto forms because:
A) severe angle strain exists in the enol forms.
B) fewer atoms are coplanar in the keto form.
C) the enol cannot be chiral.
D) the C-C π bond is weaker than the C-O π bond.
E) Actually, simple enols are the more stable.
Ans: D
650
Chapter 17
Topic: Tautomerization and Keto-Enol Equilibrium
68. Which would be formed when 2-methylpropanal is dissolved in D2O containing NaOD?
O
A)
CH3CHCD
CH3
O
B)
CH3CHCH
CH2D
O
C)
CH3CDCH
CH3
O
D)
CH3CDCD
CH3
D
E)
CH3CHCHOD
CH3
Ans: C
651
Chapter 17
Topic: Reagents, Intermediates, and Spectroscopy
69. A compound, X, C9H10O, which reacts with I2/NaOH to give a pale yellow precipitate,
gives the following 1H NMR spectrum.
δ 2.0
δ 3.0
δ 7.7
Singlet,
Singlet,
Multiplet,
Which is a possible structure for X?
O
O
O
I
II
III
O
O
IV
A)
B)
C)
D)
E)
Ans:
V
I
II
III
IV
V
B
652
Chapter 17
Topic: Reagents, Intermediates, and Spectroscopy
70. What is the missing reagent?
OH-
?
heat
O
A)
O
O
B)
CH3CCH2CH2CCH3
O
O
C)
CH3CCH2CH2CH2CH
O
O
D)
CH3CCH2CH2CH2CH2CH
O
O
E)
CH3CCH2CH2CH2COEt
H
O
O=CCH2CH2CH2CH2COEt
Ans: B
Topic: Reagents, Intermediates, and Spectroscopy
O
O
71.
The conversion of C6H5CCH3
oxidizing agent?
A) Ag(NH3)2+
B) O3
C) NaOI (I2 in NaOH)
O
D)
to
C6H5COH
is accomplished by the use of which
RCOOH
E) Cu++
Ans: C
Use the following to answer questions 72-73:
H
O
A
OH-,warm
(-H2O)
B (C11H12O)
H2/Ni
high
pressure
653
OH
Chapter 17
Topic: Reagents, Intermediates, and Spectroscopy
72. Consider the synthesis above in answering this question. What is compound A?
A) Butanone
B) Butanal
C) Propanal
D) 1-Butanol
E) 2-Methylpropanal
Ans: B
Topic: Reagents, Intermediates, and Spectroscopy
73. What is the intermediate B in the synthesis shown above?
OH
I
A)
B)
C)
D)
E)
Ans:
OH
O
II
III
O
IV
O
V
I
II
III
IV
V
D
Topic: Reagents, Intermediates, and Spectroscopy
74. Which of these would not be an intermediate or final product when C6H5CH2CN and
HCHO react in basic solution and the product is treated with acid?
CH2OH
A)
B)
C6H5CHCN
CH2
C)
C6H5CCN
C6H5CHCN
D)
C6H5CH2C=NH
C6H5CHCN
C6H5CH2C=O
E) Each of these would be either an intermediate or a final product.
Ans: E
654
Chapter 17
Topic: Reagents, Intermediates, and Spectroscopy
75. The Robinson annulation reaction which produces
CH3
O
uses which of the following as starting materials?
CH3
H
O
CH3
+
CH2=CHC=O
CH2=CHCCH3
CH2
I
+
CH2
O
II
O
III
O
O
+
+
CH3CH=CHCCH3
O
O
CH3CH=CHCCH3
IV
A)
B)
C)
D)
E)
Ans:
CH3
CH=CH2
HCHO +
V
I
II
III
IV
V
B
Topic: Reagents, Intermediates, and Spectroscopy
76. Which compound could be subjected to a haloform reaction to produce m-chlorobenzoic
acid?
CH2OH
CO2H
Cl
I
A)
B)
C)
D)
E)
Ans:
O
O
CCH2CH3
CCH3
Cl
II
III
I
II
III
IV
V
D
655
CH2Cl
Cl
Cl
IV
V
Chapter 17
SHORT ANSWER QUESTIONS
Topic: General, Active Hydrogens
77. The α-hydrogens on carbonyl carbons are unusually acidic (pKa 19-20) because:
_____________.
Ans: the corresponding anion is resonance-stabilized by the carbonyl
Topic: Keto-Enol Tautomerism
78. The keto and enol forms of carbonyl compounds are a special type of __________
isomer called ___________.
Ans: constitutional; tautomers
Topic: Keto-Enol Tautomerism
79. The interconversion of the keto and enol forms of a carbonyl compound is called
_____________.
Ans: tautomerization
Topic: Epimers
80. Diastereomers that differ in configuration at only one stereogenic center are sometimes
called ___________, and their interconversion is called __________.
Ans: epimers; epimerization
Topic: General
81. Dissolving a halogen in aqueous sodium hydroxide provides a solution of
_____________.
Ans: sodium hypohalite, NaOX
Topic: Haloform Reaction
82. The haloform reaction is an efficient method for converting methyl ketones to
____________.
Ans: carboxylic acids
Topic: General
83. The initial product of the aldol reaction is a β-hydroxycarbonyl compound. These
dehydrate readily when heated because the product _______________.
Ans: is stabilized by having conjugated double bonds
656
Chapter 17
Topic: Aldol Reaction
84. An aldol reaction that starts with two different carbonyl compounds is called a
_______________.
Ans: crossed aldol reaction
Topic: Kinetic/Thermodynamic Enolates
85. An unsymmetrical ketone can form two possible enolates. The enolate with the more
highly substituted double bond is called the ____________.
Ans: thermodynamic enolate
Topic: Kinetic/Thermodynamic Enolates
86. The kinetic enolate (the less highly substituted double bond) is favored when
__________ is used as a base.
Ans: LDA or lithium diisopropylamide
Topic: 1,2- /1,4- Addition
87. When α,β-unsaturated aldehydes and ketones react with nucleophiles, they do so in one
of two ways. What are the two ways?
Ans: simple or 1,2-addition; conjugate or 1,4-addition
Topic: Kinetic/Thermodynamic Enolates
88. When α,β-unsaturated aldehydes and ketones react with nucleophiles, simple addition is
favored when _________ nucleophiles are used, while conjugate addition is preferred
by ___________ nucleophiles.
Ans: strong; weaker
Topic: Kinetic/Thermodynamic Enolates
89. The sequence of a Michael addition followed by an intramolecular aldol condensation is
known as a ___________.
Ans: Robinson annulation
657
Chapter 17
Topic: 1,2- /1,4- Addition
90. When 5-methyl-3-hepten-2-one reacts with CH3Cu, the major product expected
is___________ (structure and IUPAC name).
Ans: The 1,4-addition product:
O
4,5-dimethyl-2-heptanone
Topic: Reaction Products: Stereochemical Outcomes
91. When optically active 2-methyl-1-phenyl-1-butanone undergoes reaction with Br2/H3O+,
the product is observed to exhibit no optical activity. Explain clearly.
Ans: An achiral enol is formed in presence of H3O+, which can then undergo reaction
with Br2 from both faces, to produce a racemic mixture of the brominated product;
hence, no observed optical activity. The reaction with (R)-2-methyl-1-phenyl-1butanone is shown below; the same enol and products can be formed from the (S)
enantiomer.
O
H CH3
H3O+
+ OH
OH
CH3
H CH3
achiral enol
Br2
O
Br CH3
(S)
658
O
+
Br
CH3
(R)
Chapter 17
Topic: Multistep Reaction Sequence:
92. What is the final product of the following reaction sequence? Give structural details of
all significant intermediates.
O
O
O
i. Zn,
Br
?
ii. H3O+, heat
iii. DIBAL-H
iv. H2O
v. (CH3)2CuLi
vi. H2O
O
Ans:
O
O
O
i. Zn,
Br
O
ii. H3O+, heat
i. DIBAL-H
ii. H2O
i. (CH3)2CuLi
ii. H2O
O
O
Topic: Multistep Reaction Sequence:
93. What is the final product of the following reaction sequence? Give structural details of
all significant intermediates.
CN
O
i.
, EtO-, EtOH
?
+
ii.H3O , heat
Ans:
O
CN
+
-
CN
EtO , EtOH
659
H3O
heat
CO2H
Chapter 17
Topic: Multistep Reaction Sequence:
94. What is the final product of the following reaction sequence? Give structural details of
all significant intermediates.
i. Cl2, FeCl3
O
?
NO2 , EtO-, EtOH
ii.
iii.H3O+
OH
Ans:
O
Cl2, FeCl3
O
NO2
i.
EtO-, EtOH
ii. H3O+
Cl
NO2
Cl
Topic: Synthetic Strategy
95. Suggest a reasonable synthetic strategy to carry out the following transformation.
O
OH
Br
OCH3
Ans:
O Br2, FeBr3
O
i.
O
-
EtO , EtOH
ii. H3O+, heat
Br
O
OCH3
Br
i. LAH
ii. H2O
OH
Br
660
Chapter 17
Topic: Multistep Reaction Sequence
96. What is the final product of the following reaction sequence? Give structural details of
all significant intermediates.
O
O
i. EtO-, EtOH
?
ii. CH3CH2NH2
Ans:
O
EtO-, EtOH
H
N
CH3CH2NH2
O
O
O
Topic: Reaction Products
97. There are several possible cyclization products that might be formed when 5-methyl-6oxoheptanal undergoes intramolecular aldol condensation in presence of ethoxide.
Draw the structures of all these possible products. Comment on which of these is likely
to be the major product, clearly explaining your rationale.
Ans: There are 3 possible products (I-III) that might be formed from the 3 theoretically
possible enolates. The rationale for predicting the major product is the following:
i) aldehydes are typically more reactive toward nucleophilic addition than
ketones, so I and III are more likely than II; ii) since 5-membered rings are formed
much more readily than 7-membered rings, I is more likely than III.
-
EtO , EtOH
O
O
O
O
HO
Topic: Reaction Products
98. What is the product of the following reaction?
O
OH-
O
O
Ans:
?
OH-
O
O
661
HO
HO
I
major product
O
II
III
Chapter 17
Topic: Reaction Products
99. What is the product of the following reaction?
O
i. LDA, THF
ii.
O
O
Ans:
?
O
i. LDA, THF
ii.
O
OH
Topic: Multistep Reaction Sequence
100. What is the product of the following reaction sequence? Give structural details of all
significant intermediates.
O
O
Ans:
i. OHii.
heat
O
?
OH-
O
heat
O
O
662
Chapter 18
MULTIPLE CHOICE QUESTIONS
Topic: Nomenclature
1. Which of the following is the best name for the following compound?
O
O
A)
B)
C)
Isobutyl ethanoate
Ethyl isopropanoate
3-methylbutyl ethanoate
D) Ethoxy isobutyl ketone
E) Ethyl 3-methylbutanoate
Ans: C
Topic: Nomenclature
O
2.
O
A correct name for
A) 2-Methylbutyl 2-methylbutanoate
B) 2-Methylbutyl 3-methylbutanoate
C) 3-Methylbutyl isovalerate
D) Isopentyl isovalerate
E) Isopentyl isobutyrate
Ans: B
is:
Topic: Nomenclature
Cl
3.
Cl
What is the IUPAC name for
A)
B)
C)
D)
E)
Ans:
O
∝-Chlorovaleryl chloride
2-Chloropentanoyl chloride
1-Chloropentanoyl chloride
1,2-Dichloropentanal
1-Chloro-1-butanecarbonyl chloride
B
663
Chapter 18
Topic: Nomenclature
4.
O
O
What is the IUPAC name for
2,3−dimethylbutyl acetate
2,3−dimethyl-4-oxoethanal
2,3−dimethylbutyl methanoate
2,3−dimethylbutyl methylate
2,3−dimethylbutyl formylate
C
A)
B)
C)
D)
E)
Ans:
Topic: Nomenclature
5. Which of the following structures is 3,4-dimethylpentyl chloroformate?
H
Cl
O
O
Cl
O
Cl
I
O
III
Cl
Cl
IV
O
O
II
O
A)
B)
C)
D)
E)
O
O
O
V
I
II
III
IV
V
Ans: E
664
Chapter 18
Topic: Nomenclature
6. Which of the following structures is N-benzyl-N-propyl-2,3-dimethylbutanamide?
H
N
N
N
O
O
O
I
II
III
N
N
O
O
IV
A)
B)
C)
D)
E)
Ans:
V
I
II
III
IV
V
E
Topic: Acidity
7. In which of the following sequences are
acidity?
A) CH3COOH > H2O > CH3CH2OH
B) CH3CH2OH > CH3COOH > H2O
C) CH3COOH > CH3CH2OH > H2O
D) H2O > CH3COOH > CH3CH2OH
E) CH3CH2OH > H2O > CH3COOH
Ans: A
665
the compounds listed in order of decreasing
>
>
>
>
>
HC≡CH > NH3
HC≡CH > NH3
NH3 > HC≡CH
HC≡CH > NH3
HC≡CH > NH3
Chapter 18
Topic: Acidity
8. Which compound would be the strongest acid?
A) CHCl2CH2CH2CO2H
B) ClCH2CHClCH2CO2H
C) CH3CCl2CH2CO2H
D) CH3CHClCHClCO2H
E) CH3CH2CCl2CO2H
Ans: E
Topic: Acidity
9. Which of the following would be the strongest acid?
Cl
CO2H
CO2H
CO2H
Cl
I
Cl
II
III
CO2H
Cl
IV
A)
B)
C)
D)
E)
Ans:
Cl
CO2H
Cl
Cl
V
I
II
III
IV
V
C
Topic: Acidity
10. Which of the following acids would have the smallest value for pKa?
A) BrCH2CH2CH2COOH
B) ClCH2CH2CH2COOH
C) Cl2CHCH2CH2COOH
D) ICHBrCH2CH2COOH
E) BrCCl2CH2CH2COOH
Ans: E
666
Chapter 18
Topic: Acidity
11. Which compound would be most acidic?
Acetone
A) Acetic acid
B) Ethanol
C)
D)
E)
Ans:
Phenol
Acetone
Water
A
Topic: Acidity
12. In which of the following are the compounds listed in order of decreasing acidity?
A) CH3CO2H > CH3CH2OH > C6H5OH > H2O
B) C6H5OH > CH3CO2H > H2O > CH3CH2OH
C) CH3CO2H > H2O > C6H5OH > CH3CH2OH
D) H2O > CH3CO2H > C6H5OH > CH3CH2OH
E) None of the above
Ans: E
Topic: Acidity
13. Which of the following would be the strongest acid?
A) Benzoic acid
B) 4-Nitrobenzoic acid
C) 4-Methylbenzoic acid
D) 4-Methoxybenzoic acid
E) 4-Iodobenzoic acid
Ans: B
667
Chapter 18
Topic: Acidity
14. Which of the following would be the strongest acid?
A)
B)
C)
2,3-dimethylhexanoic acid
3,3-diiodopentanoic acid
3-iodo-4-bromopentanoic acid
D) 3-chloro-4-bromohexanoic acid
E) 2-fluoro-4-bromopentanoic acid
Ans: E
Topic: Acidity
15. A 0.2505 g sample of an organic acid is titrated to the stoichiometric endpoint with
20.10 mL of 0.0750 M NaOH. Which of these is a possible structure for the acid?
A) C6H5CO2H
B) p-CH3C6H4CO2H
C) m-ClC6H4CO2H
D) o-NO2C6H4CO2H
E) p-BrC6H4CO2H
Ans: D
Topic: Chemical Tests, Relative Reactivities, and Physical Properties
16. Which reagent would serve as the basis for a simple chemical test to distinguish
between benzoic acid and benzamide?
A) Cold dilute NaOH
B) Cold dilute NaHCO3
C) Cold concd H2SO4
D) More than one of these
E) None of these
Ans: D
668
Chapter 18
Topic: Chemical Tests, Relative Reactivities, and Physical Properties
17. Which reagent would best serve as a basis for a simple chemical test to distinguish
between CH3CH2COOH and CH3COOCH3?
A) Concd H2SO4
B) Br2/CCl4
C) CrO3/H2SO4
D) NaHCO3/H2O
E) KMnO4/H2O
Ans: D
Topic: Chemical Tests, Relative Reactivities, and Physical Properties
18. Which reagent would best serve as the basis for a simple chemical test to distinguish
between C6H5CH=CHCOOH and C6H5CH=CHCH3?
A) Concd. H2SO4
B) Br2/CCl4
C) CrO3/H2SO4
D) NaHCO3/H2O
E) KMnO4/H2O
Ans: D
Topic: Chemical Tests, Relative Reactivities, and Physical Properties
19. Which compound would be most reactive toward nucleophilic acyl substitution?
(CH3CO)2O
A) CH3CO2Na
B) CH3COCl
C) (CH3CO)2O
D) (CH3CONH2
E) CH3CO2CH3
Ans: B
669
Chapter 18
Topic: Chemical Tests, Relative Reactivities, and Physical Properties
20. The relative reactivity of acyl compounds toward nucleophilic substitution is:
A)
B)
C)
Amide > ester > acid anhydride > acyl chloride
Acyl chloride > ester > acid anhydride > amide
Ester > acyl chloride > acid anhydride > amide
D) Acyl chloride > acid anhydride > ester > amide
E) Acid anhydride > acyl chloride > ester > amide
Ans: D
Topic: Chemical Tests, Relative Reactivities, and Physical Properties
21. Alkaline hydrolysis of an ester involves initial attack by hydroxide ion on the carbonyl
carbon. The presence of substituents on the aromatic ring of ethyl benzoate may be
expected to increase/decrease the rate of hydrolysis of this ester. In what order should
the five substituents below be arranged to represent the decreasing order of the rates of
hydrolysis when these substituents are present in the para- position of the aromatic ring
in ethyl benzoate?
A) -NO2 > -H > -Cl > -CH3 > -OCH3
B) -NO2 > -Cl > -H > -CH3 > -OCH3
C) -OCH3 > -CH3 > -Cl > -H > -NO2
D) -Cl > -NO2 > -H > -OCH3 > -CH3
E) -H > -Cl > -CH3 > -OCH3 > -NO2
Ans: B
Topic: Chemical Tests, Relative Reactivities, and Physical Properties
22. Which of the following will not undergo hydrolysis, whether acid or base is present?
A) CH3COCl
B) CH3CONH2
C) (CH3CO)2O
D) CH3CO2CH2CH3
E) CH3COCH2CH2CH3
Ans: D
670
Chapter 18
Topic: Chemical Tests, Relative Reactivities, and Physical Properties
23. Which of these compounds could not be formed by nucleophilic attack by an
appropriate reagent on acetyl chloride?
A) CH3CONH2
B) CH3CO2CH2CH3
C) ClCH2COCl
D) CH3CO2H
E) (CH3CO)2O
Ans: C
Topic: Chemical Tests, Relative Reactivities, and Physical Properties
24. In which of these species are all the carbon-oxygen bonds of equal length?
A) Diethyl carbonate
B) Methyl butanoate
C) Lithium acetate
D) Propionic anhydride
E) Pentanoic acid
Ans: C
Topic: Reaction Products
25. What is the expected product, A, of the following reaction sequence?
i. NaCN
Cl
A + NH4+
ii. 70% H2SO4, reflux
A)
B)
C)
D)
E)
Ans:
HCO2CH2C6H5
C6H5CH2COOH
C6H5CH2OSO3H
C6H5CHClCOOH
O=C(CH2C6H5)2
B
671
Chapter 18
Topic: Reaction Products
26. Predict the major organic
heat
+ KMnO4
−
H2O, OH
product
H3O+
of
?
OH
CO2H
OH
I
CHO
II
III
OH
CHO
IV
A)
B)
C)
V
I
II
III
D) IV
E) V
Ans: B
672
the
reaction
sequence
below:
Chapter 18
Topic: Reaction Products
27. What would be the product of the following reaction ?
18
HA
OH
O
?
+
H
O
18O
18O
O
O
18O
I
II
O
18 O
III
O
18
O
O
O
II
III
\
A)
I
B)
C)
D)
E)
Ans:
II
III
IV
V
A
Topic: Reaction Products
28. What would be the final product, F, of the following sequence of reactions?
i. Mg, Et2O
PBr3
i. LAH, Et2O
F
CO2H ii. H O
2
ii. CO2
iii. H3O+
Br
I
A)
B)
C)
D)
E)
Ans:
CO2H
CO2H
Br
O
CO2H
O
II
IV
III
I
II
III
IV
V
D
673
V
Chapter 18
Topic: Reaction Products
29. The product, Z, of the following sequence of reactions is which compound?
−
i. KMnO4, OH , heat
p-Chlorotoluene
ii. H3O+
Z
iii. SOCl2
iv. CH3CH2OH, base
O
O
O
O
Cl
Cl
I
II
III
O
O
Cl
Cl
Cl
IV
A)
B)
I
II
C)
D)
E)
III
IV
V
V
Ans: B
674
Chapter 18
Topic: Reaction Products
30. Predict the major organic product, P, of the following sequence of reactions:
i. OH− (2eq.), H2O heat
Cl
CO2H ii. H O+
3
Cl
H2
cat.
i. SOCl2
ii. CH3OH
O
O
O
CO2H
O
I
O
II
III
Cl
O
O
O
O
IV
A)
B)
C)
D)
E)
Ans:
P
V
I
II
III
IV
V
D
675
Chapter 18
Topic: Reaction Products
31. Predict the major organic product of the reaction sequence,
O
i. NH3, H2O
O
O
M
ii. dilute HCl, cold
O
O
OH
HO
O
NH2
HO
O
O
I
O
II
O
H
N
O
III
HN
IV
A)
B)
C)
D)
E)
Ans:
NH2
H2N
O
O
V
I
II
III
IV
V
B
Topic: Reaction Products
32. (R)-3-hexanol is subjected to the following sequence of reactions: i) acetyl chloride,
pyridine; ii) NaOH(aq), heat. What is(are) the likely final product(s)?
A) (R)-3-hexanol
B) (S)-2-hexanol
C) Equal amounts of A) and B)
D) 2-Hexene (cis and trans)
E) 1-Hexen-2-ol
Ans: A
676
Chapter 18
Topic: Reaction Products
33. What would be the final product?
C6H5CH2CONH2
A)
B)
C)
D)
E)
Ans:
P4H10
heat
i. CH3MgI, Et2O
ii. H3O+
?
C6H5CH2CO2CH3
C6H5CH2CH2NHCH3
C6H5CH2COCH3
C6H5CH2CH(CH3)CN
C6H5CH2CH=NCH3
C
Topic: Reaction Products
34. Predict the final product likely to be obtained when (S)-2-methyl-3-pentanol is
subjected to the following sequence of reactions: i) benzoyl chloride; ii) NaOH (aq),
heat
A) (S)-2-methyl-3-pentanol
B) (R)-2-methyl-3-pentanol
C) (R,S)-2-methyl-3-pentanol
D) 2-methyl-2-pentene
E) (R,S)-2-methyl-3-pentanol and 2-methyl-2-pentene in more or less equal amounts
Ans: A
Topic: Reaction Products
35. Predict the final product likely to be obtained when (S)-2-methyl-3-pentanol is
subjected to the following sequence of reactions: i) Tosyl chloride; ii) NaOH (aq), heat
A) (S)-2-methyl-3-pentanol
B) (R)-2-methyl-3-pentanol
C) (R,S)-2-methyl-3-pentanol
D) 2-methyl-2-pentene
E) (R,S)-2-methyl-3-pentanol and 2-methyl-2-pentene in more or less equal amounts
Ans: B
677
Chapter 18
Topic: Reaction Products
36. The product of the following reaction is:
O
heat
OH
+
O
?
O
(1 mol)
(1 mol)
O
O
O O
O OH
O
CO2H
CO2H
II
I
O
III
O
O
O
Cl
O
Cl
O
O
H
O
IV
A)
B)
C)
D)
E)
Ans:
V
I
II
III
IV
V
A
Topic: Reaction Products
37. What final product, Q, would be obtained via the following reaction sequence?
O
H3C
H
H
OH
C6H5
Cl
OH−/ H2O
pyridine
heat
A)
cis-3-Methylcyclopentanol
B)
C)
D)
E)
Ans:
trans-3-Methylcyclopentanol
Equal amounts of A) and B)
3-Methylcyclopentanone
None of these
B
O
Q +
C6H5
678
O
Chapter 18
Topic: Reaction Products
38. What would be the final organic product of the following reaction?
NaCN
C6H5
Cl
i. excess LAH, Et2O
ii. H2O
?
A)
C6H5CH2CH2CO2H
B) C6H5CH2CH2NH2 C6H5CH2CH(CH3)CN
C) C6H5CH2CH(CH3)CN
D) C6H5CH2CH=NH
E) C6H5CH2NH2
Ans: B
Topic: Reaction Products
39. What would be the final organic product of the following reaction?
i. Mg, Et2O
Br
ii. CO2
i. LAH, Et2O
ii. H2O
iii. H3O+
A)
B)
C)
D)
E)
Ans:
(CH3)3CCO2H
(CH3)3CCOCH3
(CH3)3CCH2OH
(CH3)3COCH3
(CH3)3CCO2CH3
C
679
?
Chapter 18
Topic: Reaction Products
40. Identify the product(s) of the following reaction.
O
heat
CO2H
?
O
O
O
O
OH
I
II
CO2H
+ CO
IV
A)
B)
I
II
C)
D)
E)
Ans:
III
IV
V
C
III
2
CO2H
V
Topic: Reaction Products
41. What would be the final product?
i. PCl5
CO2H
?
ii. NH3
iii. P4H10, heat
A)
B)
C)
D)
E)
Ans:
+ CO2
O
CH3CH2CH2NH2
CH3CH2CONH2
CH3CH2CONHCOCH2CH3
CH3CH2CN
CH3CH2COO-NH4+
D
680
Chapter 18
Topic: Reaction Products
42. What is the final product of this sequence of reactions?
i. Mg, Et2O
Br
i. SOCl2
ii. excess NH3
ii. CO2
?
+
iii. H3O
O
NH2
NH2
CN
O
I
O
II
III
NHCl
NH2
O
IV
A)
B)
C)
V
I
II
III
D) IV
E) V
Ans: C
681
Chapter 18
Topic: Reaction Products
43. What is the product of the reaction of 1-propanol with phenyl isocyanate, C6H5N=C=O?
H
A)
N
O
C6H5
O
CO2H
B)
C6H5
N
CHO
C)
C6H5
D)
C6H5
N
O
H
N
O
O
E)
C6H5
H
N
O
Ans: A
Topic: Reaction Products
44. What would be the final product of this reaction sequence?
CO2H i. PCl3
?
ii. NH3
iii. P4O10
O
CN
NH2
NH2
II
I
CO2 NH4
III
H
N
H
N
O
IV
A)
B)
C)
D)
E)
Ans:
V
I
II
III
IV
V
A
682
Chapter 18
Topic: Reaction Products
45. What
is
the
ultimate
O
Cl
O
O
H
N
N
H
H
N
of
this
sequence
CH3CH2OH
(1 eq.)
CH3NH2
O
O
Cl
N
H
O
I
A)
B)
C)
D)
E)
Ans:
Cl
product
II
O
O
O
V
Topic: Reaction Products
46. What is the product of this reaction?
O
CH3OH
?
O
O
HO
CO2H
O
O
I
II
O
O
III
HO
O
O
O
O
O
IV
A)
B)
C)
D)
E)
Ans:
O
V
I
II
III
IV
V
B
683
Cl
O
I
II
III
IV
V
A
O
reactions?
?
IV
III
of
Chapter 18
Topic: Reaction Products
47. What is the product of the reaction of propanamide with methylmagnesium bromide (1
eq.)?
O
A)
N
H
O
B)
N
O
C)
O
D)
NHMgBr
+ CH4
O
E)
H2N
OMgBr
Ans: D
Topic: Reaction Products
48. What is the final product obtained when toluene is subjected to the following reaction
sequence?
i) Cl2 (large excess), heat, hν; ii) OH−/H2O, heat; iii) H3O+
OH
OH
OH
CO2H
CCl3
HO
I
II
III
Cl
HO
CCl3
OH
Cl
IV
A)
B)
C)
D)
E)
Ans:
Cl
V
I
II
III
IV
V
B
684
Chapter 18
Topic: Structure Identification (Spectroscopy)
49. A compound has the molecular formula, C6H12O2. Its IR spectrum shows a strong
absorption band near 1740 cm-1; its 1H NMR spectrum consists of two singlets, at δ 1.4
and δ 2.0. The most likely structure for this compound is:
O
O
O
O
O
O
I
II
III
O
O
IV
A)
B)
C)
D)
E)
Ans:
O
O
V
I
II
III
IV
V
D
685
H
Chapter 18
Topic: Structure Identification (Spectroscopy)
50. A compound has the molecular formula C8H14O4. Its IR spectrum shows a strong
absorption band near 1740 cm-1. Its 1H NMR spectrum consists of:
δ 1.3
δ 2.6
δ 4.2
triplet,
singlet,
quartet,
The most likely structure for the compound is:
O
O
O
O
O
O
I
O
O
O
O
O
O
II
III
O
HO2C
CO2H
O
O
O
IV
A)
B)
C)
D)
E)
Ans:
V
I
II
III
IV
V
A
686
Chapter 18
Topic: Structure Identification (Spectroscopy)
51. A compound with the molecular formula C5H10O2 gave the following 1H NMR
spectrum:
triplet,
multiplet,
singlet,
triplet,
δ 0.90
δ 1.60
δ 1.95
δ 3.95
The IR spectrum showed a strong absorption band near 1740 cm-1. The most likely
structure for the compound is:
O
A)
O
O
B)
O
O
C)
O
D)
H
O
O
O
E)
HO
Ans: A
687
Chapter 18
Topic: Structure Identification (Spectroscopy)
52. A compound with the molecular formula C18H18O4 has a 1H NMR spectrum that
consists of:
δ 2.7
δ 3.1
δ 7.3
singlet,
singlet,
multiplet,
The IR spectrum shows a strong absorption band near 1750 cm-1. The most likely
structure for the compound is:
O
A)
O
C H
C6H5
O
6 5
O
O
B)
C6H5
O
O
O
C6H5
O
C)
C6H5
O
O
C6H5
O
O
D)
C6H5
O
O
C6H5
O
O
E)
C6H5
O
O
C6H5
O
Ans: C
Topic: Structure Identification (Spectroscopy)
53. The IR spectrum of a compound exhibits a broad absorption band at 2500-3000 cm-1
and a sharp band at 1710 cm-1. Which of these compounds could it be?
A) 1-Butanol
B) Propyl acetate
C) Butanoic acid
D) Acetyl chloride
E) Acetic anhydride
Ans: C
688
Chapter 18
Topic: Synthesis
54. Which of the following reactions would constitute a reasonable synthesis of propyl
acetate?
OH
HA
A)
+
OH
O
B)
C)
OH
+
OH
+
pyridine
Cl
O
O
O
O
D) All of these
E) None of these
Ans: D
Topic: Synthesis
55. Which of the following would serve as a synthesis of 2,2-dimethylpropanoic acid?
A)
Br
i. Mg, Et2O
ii. CO2
B)
C)
iii. H3O+
OH
i. KMnO4, OH−, heat
Br
ii. H3O+
i. CN−
ii. OH−, H2O, heat
iii. H3O+
D) All of these
E) Answers A) and B) only
Ans: E
689
Chapter 18
Topic: Synthesis
56. Which of the reactions listed below would serve as a synthesis of acetophenone,
C6H5COCH3 ?
O
A)
C6H5
+ (CH3)2CuLi
Cl
O
B)
H3C
C)
Cl
O
AlCl3
+ C6H6
+ CH3MgI
OCH3
C6H5
D) Two of these
E) All of these
Ans: D
Topic: Synthesis
57. Which of the following would serve as syntheses of (CH3)3CCO2H?
O
i. Cl2/OH− (excess)
A)
ii. H3O+
−
i. CN
B)
Br
C)
Br
+
ii. H3O (heat)
i. Mg, Et2O
ii. CO2
iii. H3O+
D) Answers A) and B) only
E) Answers A) and C) only
Ans: E
690
Chapter 18
Topic: Synthesis
58. Which of the reactions listed below would serve as a synthesis of benzyl acetate,
CH3CO2CH2C6H5 ?
Benzyl alcohol + acetic anhydride Æ
A) Benzyl alcohol + acetic anhydride Æ
B) Benzyl alcohol + acetic acid + H3O+ Æ
C) Benzyl alcohol + acetyl chloride Æ
D) Answers A) and C) only
E) Answers A), B), and C)
Ans: E
Topic: Synthesis
59. Which reactant is unlikely to produce the indicated product upon strong heating?
A) 2,2-dimethylpropanedioic acid Æ 2-methylpropanoic acid
B) 2-ethylpropanedioic acid Æ Butanoic acid
C) 2-methyl-3-oxo-pentanoic acid Æ 3-Pentanone
D) 2-methyl-4-oxo-pentanoic acid Æ 2-Methyl-3-butanone
E) 4-methyl-3-oxo-heptanoic acid Æ 3-Methyl-2-hexanone
Ans: D
Topic: Synthesis
60. N,N-Dimethylbenzamide can be made from which of the following?
+ N
A) C6H5 O
H
O
B)
Cl
C6H5
+
O
C)
C6H5
D)
C6H5
O
O
O
O
NH2
C6H5
N
H
+
N
H
+ CH3MgCl
E) A), B), and C) only
Ans: E
691
Chapter 18
Topic: Synthesis
61. Choose the reagent(s) that would bring about the following reaction:
CH3CH2CH2COOH ⎯⎯⎯→ CH3CH2CH2CH2OH
A) H2/Ni
B) Li/liq NH3
C) LiAlH[OC(CH3)3]3
D) NaBH4, CH3OH
E) LiAlH4, ether
Ans: E
Topic: Synthesis
62. Choose the reagent(s) that would bring about the following reaction:
CH3CH2CH2CO2CH3 ⎯⎯⎯→ CH3CH2CH2CHO
A) H2/Ni
B) Li/liq NH3
C) LiAlH[OC(CH3)3]3
D) NaBH4, CH3OH
E) LiAlH4, ether
Ans: C
Topic: Synthesis
63. Choose the reagent(s) that would bring about the following reaction:
CH3C≡CCO2CH3 ⎯⎯⎯→ (E)-CH3CH=CHCO2CH3
A) H2/Ni
B) Li/liq NH3
C) LiAlH[OC(CH3)3]3
D) NaBH4, CH3OH
E) LiAlH4, ether
Ans: B
692
Chapter 18
Topic: Synthesis
64. Choose the reagent(s) that would bring about the following reaction:
CH3C≡CCH2CO2CH2CH3 ⎯⎯⎯→ CH3CH≡CHCH2CH2OH
A) H2/Ni
B) Li/liq NH3
C) LiAlH[OC(CH3)3]3
D) NaBH4, CH3OH
E) LiAlH4, ether
Ans: E
Topic: Synthesis
65. Which of the following compounds is capable of forming a δ-lactone?
A) 5-hydroxypentanoic acid
B) Pentanedioic acid
C) 4-hydroxypentanoic acid
D) 3-hydroxypentanoic acid
E) 2-hydroxypentanoic acid
Ans: A
Topic: Synthesis
66. Which of the following reactions could be used to synthesize ethyl acetate?
A)
B)
C)
D)
E)
Ans:
CH3CH2OH + (CH3CO)2O + pyridine ⎯⎯⎯→
CH3CH2OH + (CH3CO)2O ⎯⎯⎯→
CH3CH2OH + CH3CO2H + H3O+ ⎯⎯⎯→
Answers A) and C) only
Answers A), B), and C)
E
693
Chapter 18
Topic: Synthesis
67. Which of the following would yield (S)-2-butanol?
i. CH3CO2−Na+
A)
(R)-2-Bromobutane
−
ii. OH , H2O, heat
−
B)
(R)-2-Bromobutane
C)
(S)-sec-Butyl acetate
OH , H2O, heat
OH−, H2O, heat
D) All of these
E) None of these
Ans: D
Topic: Synthesis
68. Which of the following would serve as a reasonable synthesis of ethyl benzoate?
OH
HA
A) C6H5
OH
+
reflux
O
excess
Cl
C6H5
B)
base
OH
+
O
C)
C6H5
O
C6H5
O
O
+
OH
D) All of the above
E) None of the above
Ans: D
Topic: Synthesis
69. On theoretical grounds, one would predict that the (1:1) reaction of acetic propionic
anhydride with methyl alcohol would form chiefly:
A) Methyl acetate + propionic acid
B) Acetone + propionic acid
C) Methyl propionate + acetic acid
D) Methyl acetate + acetic acid
E) Methyl ethyl ketone + acetic acid
Ans: A
694
Chapter 18
Topic: Synthesis
70. Which of the following combinations of reagents would not produce an ester?
OAg
A) H3C
Br
+
O
B)
ONa
H3C
C)
H3C
D)
H3C
E)
H3C
OH
+
O
OH
O
CH3
O
O
O
HA
OH
+
+
OH
Cl
O
+
OH
Ans: B
Topic: Synthesis
71. Which carboxylic acid would decarboxylate when heated to 100-150°C?
CO2H
HO2C
I
A)
B)
C)
D)
E)
Ans:
O
CO2H
II
CO2H
HO2C
III
I
II
III
More than one of these
None of these
D
Topic: Methods and Miscellaneous
72. An acid chloride is prepared from the related carboxylic acid by reaction with which of
these?
A) HCl
B) Cl2
C) SOCl2
D) HOCl
E) AlCl3
Ans: C
695
Chapter 18
Topic: Methods and Miscellaneous
73. Which of the following is not a method for preparing butanoic acid?
A) CH3CH2CH2Br + NaCN; then H3O+, reflux
B) CH3CH2CH2MgBr + CO2; then H3O+
C) CH3CH2CH2OH + CO
D) CH3CH2CH2CO2Et + OH−/H2O; then H3O+
E) CH3CH2CH2CH2OH + KMnO4/OH−/H2O/heat; then H3O+
Ans: C
Topic: Methods and Miscellaneous
74. Intramolecular dehydration to form a cyclic monoester is most likely to occur when
which of the following is heated with acid?
A) CH3CH2CH2CHOHCO2H
B) CH3CH2CHOHCH2CO2H
C) CH3CH2CH2CH2CO2H
D) CH3CHOHCH2CH2CO2H
E) HO2CCH2CH2CO2H
Ans: D
Topic: Methods and Miscellaneous
75. Which of these combinations will not produce benzoic acid?
A) C6H5CH2OH + KMnO4/OH−/H2O, heat; then H3O+
B) C6H5CH3 + KMnO4/OH−/H2O, heat; then H3O+
C) C6H6 + CO2, high pressure
D) C6H5COCH3 + Cl2/OH−/H2O; then H3O+
E) C6H5COCl + OH−/H2O; then H3O+
Ans: C
696
Chapter 18
Topic: Methods and Miscellaneous
76. Reasoning by analogy, one would predict that the reaction of carbon disulfide with secbutylmagnesium bromide should yield which of the following (after acidification)?
S
S
SH
SH
SH
III
II
I
S
S
SH
IV
A)
B)
C)
D)
E)
Ans:
V
I
II
III
IV
V
A
Topic: Methods and Miscellaneous
77. γ - And δ-hydroxy acids can be esterified intramolecularly to form compounds known as
which of these?
A) Anhydrides
B) Cycloalkenes
C) Lactones
D) Lactams
E) Cyclic ketones
Ans: C
697
Chapter 18
Topic: Methods and Miscellaneous
78. Which of the following statements concerning nitriles is incorrect?
A) Nitriles can be hydrolyzed to carboxylic acids.
B) Nitriles can be formed from (many) alkyl halides by nucleophilic substitution by
cyanide ion.
C) Nitriles can be reduced with excess lithium aluminum hydride to primary amines,
RNH2.
D) Nitriles react with Grignard reagents to form tertiary alcohols.
E) Nitriles can be made by the dehydration of amides.
Ans: D
SHORT ANSWER QUESTIONS
Topic: Nomenclature
79. While the IUPAC name for HCO2H is methanoic acid, it is commonly known as
__________.
Ans: formic acid
Topic: Nomenclature
80. Ethanoic acid (CH3CO2H) is usually called _______________, from the Latin for
“vinegar”.
Ans: acetic acid
Topic: Reaction Mechanisms
81. Acyl compounds tend to react by acyl substitution mechanisms because they all have a
good or reasonably good ________________ attached to the carbonyl carbon.
Ans: leaving group
Topic: Miscellaneous
82. Cyclic esters are called _____________, while cyclic amides are called _________.
Ans: lactones; lactams
698
Chapter 18
Topic: Methods and Miscellaneous
83. An acid-catalyzed esterification (a reaction between a carboxylic acid and an alcohol) is
called a ________________.
Ans: Fischer esterification
Topic: Methods and Miscellaneous
84. Base-promoted hydrolysis of esters is sometimes called ______________.
Ans: saponification
Topic: Methods and Miscellaneous
85. The linkages that join amino acids together to form proteins are primarily ___________
bonds.
Ans: amide
Topic: Methods and Miscellaneous
86. Litmus paper turns ___________ when an aqueous solution of a carboxylic acid is
dropped on it.
Ans: red
Topic: Miscellaneous
87. The only carboxylic acid derivative with two carbonyl groups is the ___________.
Ans: anhydride
Topic: Miscellaneous
88. The presence of electron-withdrawing substituents close to the carboxylic acid
functional group will ___________ the acidity of the molecule by _________.
Ans: increase; induction
Topic: Reactions
89. When a cyclic ketone undergoes the Bayer-Villiger oxidation, the product can be
described as a ______________.
Ans: lactone (cyclic ester)
699
Chapter 18
Topic: Reactions
90. When a gamma-hydroxy acid is treated with dilute acid, cyclization occurs, producing a
______________.
Ans: lactone (cyclic ester)
Topic: Miscellaneous
91. The functional group in a lactam is _________________.
Ans: amide
Topic: Nomenclature
92. Draw the structure for 3-methylbutanoic anhydride.
O
O
Ans:
O
Topic: Nomenclature
93. Draw the structure for 3-ethyl-4,4-dimethylhexanenitrile.
Ans:
CN
Topic: Nomenclature
94. Draw the structure for 3-oxo-5-bromohexanoic acid.
OH
Ans:
Br
O
O
Topic: Nomenclature
95. Draw the structure for 4-methylpentyl 3-ethylpentanoate.
O
Ans:
O
700
Chapter 18
Topic: Reaction Sequence
96. Complete the following reaction sequence, giving details of all significant intermediates.
O
O
i.) 2 equiv CH3Li
ii.) CH3I
O
Ans:
?
OLi
2 equiv CH3Li
CH3I
OCH3
O
Topic: Reaction Sequence
97. Complete the following reaction sequence, giving details of all significant intermediates.
O
i.) SOCl2
?
ii.) CH3CH2CH2NHCH3
OH
Ans:
O
O
SOCl2
OH
O
CH3CH2CH2NHCH3
Cl
N
Topic: Synthetic Strategy
98. Suggest a suitable synthetic strategy for the transformation of 3-methyl-1-pentanol into
propyl 3-methylpentanoate
OH i. KMnO , OH-, H O
O
Ans:
4
2
ii. H3O+
OH
SOCl2
O
OH
O
O
Cl
701
Chapter 18
Topic: Reaction Sequence
99. Complete the following reaction sequence, giving details of all significant intermediates.
i. KMnO4, OH-, H2O
ii. H3O+
?
iii. NaBH4, H2O
+
iv. H3O
Ans:
i. KMnO4, OH-, H2O
ii. H3O+
O
O
OH
NaBH4, H2O
+
O
H3O
OH
O
OH
O
702
Chapter 18
Topic: Reaction Sequence, Spectroscopic Analysis
100. When propylbenzene is subjected to the sequence of reactions outlined below, the major
product is found to exhibit the following 1H NMR spectrum:
d 1.2 ppm, triplet
2.2 ppm, quartet
7.5 ppm, multiplet
Account for the formation of this product, giving details of all significant intermediates.
i. NBS
ii. NaOH
iii. CrO3
iv. RCO3H
?
Ans: Reaction with NBS would lead to benzylic bromination, followed by nucleophilic
substitution to afford the corresponding alcohol. Oxidation with CrO3 would yield
an alkyl aryl ketone, which would undergo Bayer-Villiger oxidation with a peroxy
acid to afford the ester shown below (along with the likely 1H NMR signal
assignment).
Br
NBS
OH
NaOH
CrO3
2.2 ppm
O
O
RCO3H
O
7.5 ppm
1.2 ppm
703
Chapter 18
Topic: Reaction Sequence
101. Complete the following reaction sequence, giving details of all significant intermediates.
OH
i. COCl2 (1 equivalent)
?
ii. CH3CH2CH2CH2NHCH3
Ans:
OH
O
COCl2 (1 equivalent)
Cl
O
H
N
O
N
O
704
Chapter 19
MULTIPLE CHOICE QUESTIONS
Topic: Relative Acidities
1. Which of the compounds listed below is more acidic than 1-butanol?
A) Ethyl acetoacetate
B) 2-butanone
C) Ethyl pentanoate
D) All of these answers
E) Answers A) and B) only
Ans: A
Topic: Relative Acidities
2. Which of the compounds listed below is more acidic than 2-pentanol?
A) Ethyl 3-oxopentanoate
B) 2-pentanone
C) Pentanal
D) All of these answers
E) Answers A) and B) only
Ans: A
Topic: Relative Acidities
3. Which of the following hydrogens is the most acidic?
O
II
I
A)
B)
C)
D)
E)
Ans:
O
I
II
III
IV
V
C
IV
III
O
V
O
Chapter 19
v
Topic: Relative Acidities
4. Which is the most acidic hydrogen in the compound shown?
V
III
II
O
v
v
O
O
v
v
IV I
A)
B)
C)
D)
E)
Ans:
I
II
III
IV
V
C
Topic: Relative Acidities
5. Which of the following compounds is the strongest acid?
OH
O
CN
NO2
NO2
O
II
I
O
III
CN
CN
CO2Et
IV
A)
B)
C)
D)
E)
Ans:
I
II
III
IV
V
E
V
Chapter 19
Topic: Relative Acidities
6. Which of the indicated hydrogens can be replaced by alkylation?
(V)
H
H
H
H
H (IV)
H
S
S
H3C
H3C
H
(I)
A)
B)
C)
D)
E)
Ans:
I
II
III
IV
V
C
H (III)
(II)
Topic: Relative Acidities
7. Which of the indicated hydrogens is most likely to be removed upon treatment with nbutyllithium?
(V)
H
H
H
H
H (IV)
H
S
H3C
H3C
(I)
A)
B)
C)
D)
E)
Ans:
S
H (III)
H
(II)
I
II
III
IV
V
C
Chapter 19
Topic: Reaction Products
8. What would be the major product of the following reaction?
i. NaOC2H5
O
?
+
ii.
H
O
3
O
O
O
O
O
I
O
O
O
O
II
III
O
O
OH O
IV
A)
B)
C)
D)
E)
Ans:
I
II
III
IV
V
C
O
O
O
OH
V
Chapter 19
Topic: Reaction Products
9. What would be the product of the following sequence?
O
O
O
+
O
O
i. NaOC2H5, C2H5OH
?
ii. H3O+
OH O
O
O
O
O
O
II
I
III
O
O
O
O
IV
A)
B)
C)
D)
E)
Ans:
I
II
III
IV
V
A
O
V
O
Chapter 19
Topic: Reaction Products
10. What is the expected product from the following reaction sequence?
O
O O
i. NaOC2H5, C2H5OH
+
?
O
O
ii. H3O+
O
O
O
O
O
O
O
O
O
O
O
II
I
III
O
O
O
IV
A)
B)
C)
D)
E)
Ans:
O
O
V
I
II
III
IV
V
A
Topic: Reaction Products
11. What is the product, Z, of the following reaction sequence?
O
H
N
H3O+
+
O
O
Ph
O
Ph
I
Ph
III
II
Ph
N
O
O
Ph
O
IV
A)
B)
C)
D)
E)
Ans:
I
II
III
IV
V
B
Z
(-H2O)
O
N
H3O+
PhCOCl
V
Chapter 19
Topic: Reaction Products
12. What would be the product, P, of the following reaction sequence?
O
O
i. NaOH
ii.H3O+
i. NaOEt
ii.PhCH2Br
O
O
O
Ph
Ph
I
O
Ph
O
III
II
O
Ph
O
O
IV
A)
B)
C)
D)
E)
Ans:
heat
(-CO2)
I
II
III
IV
V
C
Ph
V
P
Chapter 19
Topic: Reaction Products
13. Predict the product of the following reaction sequence.
i. NaOC2H5
O
O
O
ii. CH3CH2CH2Br
?
iii. NaOH
iv. H3O+, heat
O
OH
O
OH
O
O
O
I
II
III
O
OH
IV
A)
B)
C)
D)
E)
Ans:
I
II
III
IV
V
D
O
O
V
O
Chapter 19
Topic: Reaction Products
14. What is the product, W, of the following reaction sequence?
O
(2 eq)
O
O
O
..
Na
i. NaOH
heat
(-CO2)
+
ii.H3O
(1 eq)
+
CO2H
HO2C
Br
Br
HO2C
CO2Et
EtO2C
CO2H
II
I
III
O
CO2H
O
IV
A)
B)
C)
D)
E)
Ans:
V
I
II
III
IV
V
B
Topic: Reaction Products
15. Predict the product of the following reaction sequence.
i. NaOEt, EtOH
O
O
O
ii. NaOH, heat
+
O O
O
+
iii. H3O
iv. heat
?
O
OH
O
O
O
O
O
I
II
OH
O
O
OH
IV
A)
B)
C)
D)
E)
Ans:
I
II
III
IV
V
B
O
III
OH
HO
O
O
V
W
O
Chapter 19
Topic: Reaction Products
16. The product, L, of the following reaction sequence,
O
i. EtBr
ii. (CH3)3COK
H2N
NH2
EtO2C
CO2Et + NaOEt
−
iii. MeI
EtO
O
O
MeO
Me
EtO2C
OEt
NH2
HN
EtO2C
N
H
O
Et
O
N
H
Me
Et
O
CO2Et
CO2Et
NH
N
H
O
II
I
O
N
H
O
O
H2N
N
H
O
Me Et
O
N
H
NH2
V
IV
A)
B)
C)
D)
E)
Ans:
O
O
N
H n
O
III
O
N
H
N
H
?
I
II
III
IV
V
C
Topic: Reaction Products
17. The product(s) of the reaction of 2 mol of ethyl butanoate with sodium ethoxide is(are):
O
A)
OH + NaOH
O
O
B)
O
C)
O
O
O
D)
O
O
O
E)
O
O
O
Ans: C
Chapter 19
Topic: Reaction Products
18. The reaction of diethyl heptanedioate with sodium ethoxide would give as the product:
O
O
EtO
EtO
I
O
II
O
OEt
III
A)
B)
C)
D)
E)
Ans:
O
O
O
IV
O
V
I
II
III
IV
V
C
Topic: Reaction Products
19. The product of the following reaction is:
O
weak
O
+
CN
base
O
O
OH
CN
CN
O
O
O
CN
O
I
II
III
O
O
OMe
IV
I
II
III
IV
V
D
NH O
OMe
CN
A)
B)
C)
D)
E)
Ans:
?
V
Chapter 19
Topic: Reaction Products
20. Which product can be obtained via the following generalized reaction?
O
R-I +
CN
EtO
O
I
O
R
EtO
R
EtO
N
I
IV
CN
R
II
III
O
A)
B)
C)
D)
E)
Ans:
?
O
CN
EtO
NaOEt
EtOH
O
R
CN
RO
V
I
II
III
IV
V
A
Topic: Reaction Products
21. Malonic ester (diethyl malonate) is treated successively with sodium ethoxide (1 eq.),
ethyl bromide, potassium tert-butoxide, isobutyl chloride, hot aqueous NaOH, HCl, and
heat. What is the final product?
A) 4-Ethyl-2-methylpentanoic acid
B) 6-Methylheptanoic acid
C) 2-Ethyl-3-methylpentanoic acid
D) 2-Ethyl-4-methylpentanoic acid
E) Ethylisobutylmalonic acid
Ans: D
Topic: Reaction Products
22. Malonic ester (diethyl malonate) is treated successively with sodium ethoxide (1 eq.), 1bromopentane, potassium tert-butoxide, iodomethane, hot aqueous NaOH, HCl, and
heat. What is the final product?
A) 2-Methylheptanoic acid
B) 3-Methylhexanoic acid
C) 3-Methylpentanoic acid
D) 2-Methylpentanoic acid
E) Ethyl 2-methylheptanoate
Ans: A
Chapter 19
Topic: Reaction Products
23. What is the product of the Dieckmann condensation of this diester,
O
OEt
EtO
O
O
O
OEt
I
A)
B)
C)
D)
E)
Ans:
O
O
O
OEt
O
II
OEt
III
I
II
III
I and II
I, II, and III
D
Topic: Reaction Products
24. What product(s) is (are) likely to be obtained upon Dieckmann condensation of the
following substance?
O
O
O
O
O
O
O
O
I
A)
B)
C)
D)
E)
Ans:
O
O
O
II
I and II
II and III
III and IV
I and III
II and IV
D
O
O
O
O
O
III
IV
Chapter 19
Topic: Reaction Products
25. The Thorpe reaction of dinitriles is analogous to the Dieckmann condensation of
diesters. What is the product predicted to result from the use of
N≡CCH2CH2CH2CH2C≡N and sodium ethoxide?
NH
NH
HN
CN
NH
CN
II
I
NH
III
CN
NC
NC
CN
CN
IV
A)
B)
C)
D)
E)
Ans:
V
I
II
III
IV
V
A
Topic: Reaction Products
26. What product(s) result from the Claisen condensation carried out with an equimolar
mixture of ethyl acetate and ethyl propanoate?
O
O
O
O
I
I
II
III
IV
All of these
E
O
O
O
A)
B)
C)
D)
E)
Ans:
O
II
O
O
O
III
O
IV
Chapter 19
Topic: Reaction Products
27. What product(s) result from the Claisen condensation carried out with an equimolar
mixture of ethyl 2-methylpropanoate and ethyl propanoate?
O
O
O
O
O
O
O
O
O
O
II
I
A) I and II
B) I and IV
C) I, II and IV
D) II and III
E) All of these
Ans: E
O
III
O
IV
Topic: Reaction Products
28. What product is finally formed when the initial compound formed from cyclohexanone
and pyrrolidine is mixed with allyl chloride and that product is heated and then
hydrolyzed?
O
HO
N
II
I
O
IV
A)
B)
C)
D)
E)
Ans:
I
II
III
IV
V
B
III
O
V
Cl
Chapter 19
Topic: Reaction Products
29. What is the major product of the following reaction?
O
EtO
O
O
O
Q
O
OEt
O
OEt
O
O
OEt
i.
O
ii. H3O+
NaNH2
Et2O
OEt
O
I
II
III
O
O
OEt
O
OEt
O
O
IV
A)
B)
C)
D)
E)
Ans:
V
I
II
III
IV
V
B
Topic: Reaction Products
30. What is the structure for R?
i. C4H9Li, Et2O
ii. C6H5CH2Br
H Ph
HgCl2
MeOH, H2O
S
S
R
Ph
O
Ph
Ph
CHO
CHO
I
II
III
OH
Ph
Ph
CHO
IV
A)
B)
C)
D)
E)
Ans:
I
II
III
IV
V
C
V
O
Chapter 19
Topic: Reaction Products
31. What is the final product from the following reaction sequence?
i. C4H9Li, Et2O
?
S S
Ph ii. C2H5Br
H
iii. HgCl2, CH3OH, H2O
S
S
S
S
H
Ph
Ph
Ph
S
II
I
III
Ph
O
O
Ph
IV
A)
B)
C)
D)
E)
Ans:
SH
V
I
II
III
IV
V
D
Topic: Synthesis
32. Which of the following might be used to synthesize the following substance?
OH
Ph
A)
PhCO2Et
OH
CH3CH2CO2Et
NaOEt, EtOH
H3O+
LiAlH4
Et2O
H2O
Mg
LiAlH4
H3O+
PhCHO + CH3CHBrCH2CO2Et
Et2O
Et2O
Mg
C)
H2O
PhCHO
CH3CHBrCH2CO2Et
Et2O
D) Answers A) and B)
E) Answers A), B), and C)
Ans: A
B)
H2O
Chapter 19
Topic: Synthesis
33. Which of the following might be used to synthesize the following substance?
OH
Ph
OH
O
A)
Ph
i.
O
, NaOEt, EtOH
O
ii. H3O+
O
iii. LAH, Et2O
iv. H2O
O
B)
B. Ph
O
i.
, NaOEt, EtOH
O
+
ii. H3O
O
iii. LAH, Et2O
iv. H2O
O
C)
O
C. Ph
O
i.
ii. H3O
Br
+
iii. LAH, Et2O
iv. H2O
D) Answers A) and C)
E) Answers B), and C)
Ans: B
, Mg, Et2O
Chapter 19
Topic: Synthesis
34. Which of the following could be used to synthesize the following substance in good
yield?
CO2H
O
Cyclohexanone, ClCH2COOH, AlCl3, heat
Cyclohexanone, (CH3CH2)2NH, HA, (-H2O); then BrCH2COOC2H5; then OH-,
H2O, heat; then H3O+
C) Cyclohexylacetic acid, KMnO4, OH-, heat; then H3O+
D) Answers A) and B)
E) Answers A), B), and C)
Ans: B
A)
B)
Topic: Synthesis
35. Which of the following could be used to synthesize the following substance in good
yield?
O
OH
O
Cyclopentanone, ClCH2CH2COOH, AlCl3, heat
Cyclopentanone, (CH3CH2)2NH, HA, (-H2O); then BrCH2CH2COOC2H5; then
OH-, H2O, heat; then H3O+
C) 3-(2-hydroxycyclopentyl)propanal, KMnO4, OH-, heat; then H3O+
D) Answers A) and B)
E) Answers B) and C)
Ans: E
A)
B)
Topic: Synthesis
36. Which of the following would provide the best synthesis of 3,5-dimethyl-2-hexanone?
A) Ethyl acetoacetate + NaOC2H5 + CH3I; then KO-t-Bu + (CH3)3CCH2Br; then
NaOH; then H3O+; then heat
B) Ethyl acetoacetate + NaOC2H5 + (CH3)3CBr; then KO-t-Bu + CH3I; then
NaOH; then H3O+; then heat
C) Ethyl acetoacetate + NaOC2H5 + (CH3)2CHCH2Br; then KO-t-Bu + CH3I;
then NaOH; then H3O+; then heat
D) Ethyl acetoacetate + NaOC2H5 + (CH3)2CHCH2CHBrCH3; then NaOH; then
H3O+; then heat
E) Ethyl acetoacetate + NaOC2H5 + CH3I; then KO-t-Bu + (CH3)2CHBr; then
NaOH; then H3O+; then heat
Ans: C
Chapter 19
Topic: Synthesis
37. Which of the following would provide the best synthesis of 3-ethyl-6-methyl-2heptanone?
A) Ethyl acetoacetate + NaOC2H5 + CH3CH2I; then KO-t-Bu +
(CH3)3CCH2CH2Br; then NaOH; then H3O+; then heat
B) Ethyl acetoacetate + NaOC2H5 + (CH3)3CCH2Br; then KO-t-Bu + CH3CH2I;
then NaOH; then H3O+; then heat
C) Ethyl acetoacetate + NaOC2H5 + (CH3)2CHCH2CH2Br; then KO-t-Bu +
CH3CH2I; then NaOH; then H3O+; then heat
D) Ethyl acetoacetate + NaOC2H5 + (CH3)2CHCH2CH2CHBrCH2CH3; then
NaOH; then H3O+; then heat
E) Ethyl acetoacetate + NaOC2H5 + CH3CH2I; then KO-t-Bu + (CH3)2CHCH2Br;
then NaOH; then H3O+; then heat
Ans: C
Topic: Synthesis
38. Which of the following would afford the best synthesis of diethyl phenylmalonate,
O
O
C2H5O
OC2H5
Ph
A)
O
Ph Br
+ C2H5O
..
Na
B)
O
NaOC2H5
H3O+
+
OC2H5 C2H5O OC2H5 C2H5OH
O
O
NaOC2H5
H3O+
+
OC2H5 C H OH
OC2H5 C2H5O
2 5
O
Ph
C)
Ph
OC2H5
+
O
O
D)
O
O
D. Ph
OC2H5
+
E)
Ph CHO
Ans: B
+ 2
O
H
NaOC2H5
OC2H5 C2H5OH
NaOC2H5
OC2H5 C2H5OH
H3O+
H3O+
Chapter 19
Topic: Synthesis
39. What product is formed during the following reaction?
O
O
i. NaOC2H5, C2H5OH
+
Ph OC2H5
OC2H5 ii. HOAc
O
O
OC2H5
Ph
O
O
Ph
Ph
OC2H5
O
O
III
II
I
O
Ph
OH O
O
O
Ph
IV
A)
B)
C)
D)
E)
Ans:
OC2H5
V
I
II
III
IV
V
B
Topic: Synthesis
40. Which compound may be prepared using a Mannich reaction?
CO2Et
O
CO2H
I
CO2Et
N
III
II
O
O
O
N
IV
A)
B)
C)
D)
E)
Ans:
O
I
II
III
IV
V
C
V
Chapter 19
Topic: Synthesis
41. Which compound could be prepared using a Michael reaction?
O
O
O
HO
OC2H5
O
III
II
I
O
O
O
O
O
OC2H5
IV
A)
B)
C)
D)
E)
Ans:
V
I
II
III
IV
V
C
Topic: Synthesis
42. Which compound could be prepared via Dieckmann condensation?
O
O
O
HO
OC2H5
O
III
II
I
O
O
O
O
O
IV
A)
B)
C)
D)
E)
Ans:
I
II
III
IV
V
D
O
V
Chapter 19
Topic: Synthesis
43. Which reagents would you use to prepare the following substance from ethyl
acetoacetate?
O
O
Ph
A)
i) NaH/DMSO; ii) PhCOCl; iii) OH−/H2O, heat; iv) H3O+; v) heat
B)
i) NaOEt/EtOH; ii) PhCOCH2Br; iii) OH−/H2O, heat; iv) H3O+; v) heat
C)
i) heat; ii) NaOEt/EtOH; iii) PhCOCH2Br
D)
i) NaOEt/EtOH; ii) PhCl
E)
i) NaOEt/EtOH; ii) PhCOCl; iii) OH−/H2O, heat; iv) H3O+; v) heat
Ans: A
Topic: Synthesis
44. Which of the following statements is true about the anion formed from the reaction of
diethyl malonate with sodium ethoxide?
A) It can react with an ∝, β-unsaturated ester by conjugate addition.
B) It can condense with aldehydes and ketones.
C) It can be alkylated with an alkyl halide.
D) It is resonance stabilized.
E) All of the above statements are true.
Ans: E
Chapter 19
Topic: Synthesis
45. Which reagents would be used in a Mannich reaction to synthesize
O
N
O
I
+
O
H
+ (CH3)2NH
O
II
O
+
H
+ (CH3)3N
O
III
+ (CH3)3N
+ base
O
IV
+
H
O
H
+ (CH3)2NH
O
V
A)
B)
C)
D)
E)
Ans:
+ (CH3)2NH
I
II
III
IV
V
D
Chapter 19
Topic: Synthesis
46. What combination of reagents can be used to make the following substance via an
enamine?
O
O
O
I
+
NH
+
O
O
II
+
NH
+
O
O
III
+
Cl
+
O
NH
O
IV
A)
B)
C)
D)
E)
Ans:
+
O
+
NH2
I
II
III
IV
Both reactions II and III
C
Topic: Synthesis
47. Which organic reagents would you need to make 2-ethylpentanoic acid from diethyl
malonate?
A) Bromoethane, 1-bromopropane, sodium ethoxide, and potassium tert-butoxide
B) 3-Bromohexane and sodium ethoxide
C) 3-Bromopentane and sodium ethoxide
D) Bromoethane, 2-bromopropane, sodium ethoxide, and potassium tert-butoxide
E) 2-Bromopentane and sodium ethoxide
Ans: A
Chapter 19
Topic: Synthesis
48. Which organic reagents would you need to make 2-ethylheptanoic acid from diethyl
malonate?
A) Bromoethane, 1-bromopentane, sodium ethoxide, and potassium tert-butoxide
B) 3-Bromooctane and sodium ethoxide
C) 3-Bromoheptane and sodium ethoxide
D) 1-Bromopropane, 2-bromopentane, sodium ethoxide, and potassium tert-butoxide
E) 2-Bromoheptane and potassium tert-butoxide
Ans: A
Topic: Synthesis
49. 2-Methylcyclohexane-1,3-dione can be synthesized from:
O
H3O+
CH3MgBr
I
O
O
KMnO4, OH−
II
O
O
III
O
O
A)
B)
C)
D)
E)
Ans:
I
II
III
IV
V
C
i. NaOEt
ii. H3O+
OEt
i. NaOEt
ii. H3O+
OEt
i. NaOEt
ii. H3O+
O
IV
V
OEt
O
Chapter 19
Topic: Synthesis
50. 2-Heptanone can be synthesized by which reaction sequence?
−
A)
NaOEt
CH3(CH2)3Br i. dil.OH heat
Ethyl acetoacetate
+
EtOH
ii. H O
3
B)
C)
D)
E)
CH3(CH2)3Br
NaOEt
EtOH
−
i. dil.OH heat
NaOEt CH3(CH2)3Br
Ethyl acetoacetate
+
EtOH
ii. H3O
−
i. dil.OH heat
Ethyl acetoacetate CH3(CH2)3Br
+
ii. H3O
−
i. NaOEt, EtOH i. dil.OH heat
Ethyl hexanoate
ii. CH3COCl
ii. H O+
Ethyl acetoacetate heat
3
Ans: A
Topic: Synthesis
51. Consider the synthesis below: What is compound X?
CO2Et
A)
B)
C)
D)
E)
Ans:
+ X
i. NaOEt, EtOH
ii. H3O+
CO2Et
CO2Et
O=C(OEt)2
HCO2Et
EtO-CO-CO-OEt
CH3CO2Et
BrCH2CO2Et
A
Topic: Synthesis
52. Consider the synthesis above. What is compound Y?
CO2Et
CO2Et
1) KOt-Bu
2) "Y"
1) NaOH/H2O, heat
2) H3O+
3) heat (− CO2)
A)
B)
C)
D)
E)
Ans:
BrCH2CO2Et
EtO-CO-CO-OEt
CH3CH2Br
C6H5CH2Br
C6H5Br
D
CO2Et
Chapter 19
Topic: Synthesis
53. Consider the synthesis below. What is compound Z?
CO2Et
CO2Et
1) KOt-Bu
2) "Y"
"Z"
CO2Et
1) NaOH/H2O, heat
2) H3O+
3) heat (− CO2)
CO2Et
EtO2C
I
IV
I
II
III
IV
V
B
EtO2C
II
EtO2C CO2Et
A)
B)
C)
D)
E)
Ans:
CO2Et
CO2Et
III
O CO2Et
V
Chapter 19
Topic: Synthesis
54. The Knoevenagel condensation of p-methoxybenzaldehyde with ethyl acetoacetate in
the presence of diethylamine produces which of these?
CO2Et
O
OH
CO2Et
O
MeO
O
MeO
I
EtO2C
MeO
II
III
O
O
CO2Et
CO2Et
O
MeO
IV
A)
B)
C)
D)
E)
Ans:
O
MeO
V
I
II
III
IV
V
A
Topic: Synthesis
55. What alkylating agent would be used with 2-phenylethanal in the Corey-Seebach
method for the preparation of 4-methyl-1-phenyl-2-pentanone?
A) Isopropyl bromide
B) Butyl bromide
C) sec-Butyl bromide
D) Isobutyl bromide
E) tert-Butyl bromide
Ans: D
Topic: Synthesis
56. What alkylating agent would be used with 2-phenylethanal in the Corey-Seebach
method for the preparation of 6-methyl-1-phenyl-2-heptanone?
A) 1-bromo-4-methylpentane
B) 2-bromo-4-methylpentane
C) 1-bromo-6-methylheptane
D) 1-bromo-5-methylhexane
E) None of the above could be used
Ans: A
Chapter 19
Topic: General Information
57. Which of the following statements is true of the enamine,
N:
A)
B)
The enamine can be made from cyclohexanone + pyrrolidine.
+
N
:
It has another resonance structure :
C) It can be acylated at the ∝-carbon of the original carbonyl compound.
D) It can be alkylated.
E) All of the above are true.
Ans: E
Topic: General Information
58. Which of the following statements is true of the following enamine?
N:
A)
B)
The enamine can be made from cyclopentanone + pyrrolidine.
+
C)
D)
E)
Ans:
It has another resonance structure:
It can be acylated at the ∝-carbon of the original carbonyl compound.
It can be alkylated.
All of the above are true.
E
N
:
Topic: General Information
59. In the synthesis of barbiturates (general structure shown below), the R groups would
originate in which step?
O
HN
O
A)
B)
C)
D)
E)
Ans:
N
H
R'
R
O
Alkylation of the unsubstituted barbituric acid
Alkylation of the urea
Alkylation of the starting diethyl malonate
Alkylation of an enamine
Alkylation via a Grignard reagent
C
Chapter 19
Topic: General Information
60. Which of these is not a reversible process?
A) Base-promoted ester hydrolysis
B) Acid-catalyzed ester hydrolysis
C) Aldol addition
D) Claisen condensation
E) Acetal formation
Ans: A
Topic: General Information
61. The Claisen condensation of which of these esters demands the use of (C6H5)3CNa as
the base (as opposed to sodium ethoxide)?
A) CH3CH2CO2C2H5
B) CH3CO2C2H5
C) C6H5CH2CO2C2H5
D) C6H5CH(CH3)CO2C2H5
E) None of these
Ans: D
Topic: General Information
62. The Claisen condensation of which of these esters demands the use of (C6H5)3CNa as
the base (as opposed to sodium ethoxide)?
A) CH3CH2CH2CO2C2H5
B) CH3CH2CO2C2H5
C) C6H5CH2CH2CO2C2H5
D) (CH3)2CHCH2CO2C2H5
E) None of these
Ans: E
Topic: General Information
63. Which of these halides is predicted to alkylate malonic ester (as the anion) in highest
yield?
A) CH3I
B) C6H5Br
C) (CH3)3CCH2Cl
D) CH3CHClCH3
E) All of these should be equally effective.
Ans: A
Chapter 19
Topic: General Information
64. The Claisen condensation produces which of these?
A) An ∝-keto ester
B) A β-keto ester
C) A β-hydroxy ester
D) A β-hydroxyaldehyde
E) A β-diketone
Ans: B
Topic: General Information
65. Cyclization reactions, such as the Dieckmann condensation, are best carried out using
fairly dilute solutions of the compound to be cyclized. Why is this so?
A) It then is possible to use less base.
B) The reagents generally are expensive.
C) A smaller amount of the compound to be cyclized can be used.
D) Intermolecular condensation is minimized at low concentration.
E) The concentration factor is unimportant.
Ans: D
Topic: General Information
66. Why is CH3ONa not used in the Claisen condensation of ethyl acetate?
A) CH3O- is a weaker base than the CH3CH2O- which is used.
B) CH3O-Na+ is more difficult to prepare than CH3CH2O-Na+.
C) CH3O- would abstract a proton from the ethyl group of the ester.
D) Use of CH3O-Na+ would result in transesterification.
E) CH3O-Na+ can be used as well as CH3CH2O-Na+.
Ans: D
Topic: General Information
67. Which of these combinations is not one which would result in the formation of
essentially one Claisen condensation product when one compound is added slowly to
the mixture of the other and the base employed?
HCO2Et + CH3CH2CO2Et
A) HCO2Et + CH3CH2CO2Et
B) PhCO2Et + CH3CO2Et
C) (CO2Et)2 + PhCH2CO2Et
D) (CH3)3CCO2Et + CH3CO2Et
E) PhCH2CO2Et + CH3CO2Et
Ans: E
Chapter 19
Topic: General Information
68. Which structure represents an ester enolate?
O
O
O
O
O
O
I
II
A) I
B) II
C) III
D) IV
E) Both B) and C)
Ans: E
O
O
III
III
Topic: General Information
69. Which of these amines is/are used with aldehydes and ketones to form enamines?
H
N
I
A)
B)
C)
D)
E)
Ans:
NH
II
N
III
N
III
I
II
III
IV
Both I and II
E
Topic: General Information
70. Which base is employed in the alkylation of ethyl pentanoate with methyl iodide?
A) Sodium methoxide
B) Sodium ethoxide
C) Sodium hydride
D) Potassium tert-butoxide
E) Lithium diisopropylamide
Ans: E
Topic: General Information
71. Which base is employed in the alkylation of methyl hexanoate with ethyl iodide?
A) Sodium methoxide
B) Sodium ethoxide
C) Sodium hydride
D) Potassium tert-butoxide
E) Lithium diisopropylamide
Ans: E
Chapter 19
SHORT ANSWER QUESTIONS
Topic: General Information
72. Compounds having two carbonyl groups separated by an intervening carbon atom are
called ____________.
Ans: β-dicarbonyl compounds
Topic: Active hydrogens
73. The pKa for protons on the carbon between two carbonyls is about 9-11. This greater
acidity (as compared to single carbonyl systems) can be accounted for by
______________.
Ans: resonance delocalization of the negative charge over two carbonyls rather than
just one
Topic: Claisen condensation
74. Esters frequently react in the presence of alkoxides by a reaction called the __________
condensation.
Ans: Claisen
Topic: Dieckmann condensation
75. An intramolecular Claisen condensation is known as a _____________.
Ans: Dieckmann condensation
Topic: General Information
76. When planning a reaction with an ester and an alkoxide ion, it is important to use an
alkoxide that has the same alkyl group as the ester in order to avoid ________________.
Ans: transesterification
Topic: Synthetic Strategy
77. The ethyl acetoacetate anion is the synthetic equivalent of the ___________.
Ans: acetone enolate
Topic: Malonic Ester synthesis
78. A reaction of a particular β-dicarbonyl compound that gives substituted or disubstituted
acetic acids is called the _____________.
Ans: malonic ester synthesis
Topic: Active Hydrogen compounds
79. Compounds with two electron-withdrawing groups attached to the same carbon atom
are known as ____________ or _____________.
Ans: active hydrogen compounds; active methylene compounds
Topic: Synthetic Strategy
80. “Umpolung” is a German term meaning __________.
Ans: polarity reversal
Chapter 19
Topic: Knoevenagel Condensation
81. An aldol-type condensation between an active methylene compound and an aldehyde or
ketone is called a _____________.
Ans: Knoevenagel condensation
Topic: Synthetic Strategy
82. In Chapter 16 we learned an efficient method for the crossed aldol condensation using
lithium diisopropylamide as the base. This was specific for forming the kinetic enolate
in ketones. If we instead wished to access the thermodynamic enolate instead, what
method should we use?
Ans: The Stork enamine synthesis; convert the ketone to the enamine, which will form
the more highly substituted double bond and act as a synthetic equivalent for the
thermodynamic enolate.
Topic: General Information
83. Physiologically speaking, barbiturates are very effective __________.
Ans: soporifics or sleep-inducers
Topic: Claisen Condensation
84. Explain why ethyl 2-methylpropanoate does undergo the usual Claisen condensation.
Ans: In order for Claisen condensation to occur efficiently, there must be at least 2
hydrogen atoms alpha to the ester group. Although only one hydrogen appears to
be substituted during the reaction, a second alpha hydrogen must originally be
present, otherwise the overall reaction has an unfavorable equilibrium.
Topic: Dieckmann Condensation
85. Explain why diethyl pentanedioate does undergo a Dieckmann condensation.
Ans: Dieckmann condensation of diethyl pentanedioate would afford a four-membered
ring. This ring is expected to have a considerable degree of ring strain and can
account for the observed lack of reactivity.
O
O
O
O
O
O
O
Topic: Dieckmann Condensation
86. Predict what is likely to happen when ethyl 6-oxooctanoate is treated with NaOEt,
followed by acid work-up.
Ans: A cyclization similar to Dieckmann condensation is likely to occur, giving 2propanoylcyclopentanone as the major product:
O
O
O
O
O
Chapter 19
Topic: Acylation of Active Hydrogen Compounds
87. Explain why acylation of active methylene compounds must be carried out in aprotic
solvents, typically using NaH to generate the enolate anion: why can sodium
ethoxide/ethanol not be used in these reactions?
Ans: Acylating agents such as acyl halides and acid anhydrides would react rapidly
with ethanol to form esters; also, during this process, the ethoxide ion would be
neutralized. NaH/aprotic solvent bypasses these issues and affords the expected
products in good yields.
Topic: 1,3-Dithiane Reactivity
88. Explain how the reactivity of 1,3-dithianes is different from that of the aldehydes from
which they are typically prepared.
Ans: The carbonyl carbon of an aldehyde is partially positive and thus, reacts with
nucleophiles. By contrast, upon conversion into a dithiane and subsequent
treatment with butyllithium, this same carbon becomes negatively charged, and
reacts with electrophiles. This phenomenon is described as “umpolung”.
Topic: Multistep Reaction Sequence:
89. What is the final product of the following reaction sequence? Give structural details of
all significant intermediates.
CO2Et
i. NaH
?
CO2Et
ii. Br
Br
iii. NaOEt
iv. NaOH, heat
v. H3O+, heat
Br
Ans:
CO2Et NaH
CO2Et Br
CO2Et
Br
+−
Na :
CO2Et
CO2Et
CO2Et
NaOEt
CO2H
i. NaOH, heat
ii. H3O+, heat
CO2Et
CO2Et
Chapter 19
Topic: Multistep Reaction Sequence:
90. What is the final product of the following reaction sequence? Give structural details of
all significant intermediates.
NH , p-TsOH
i.
O
?
O
ii.
O
, EtOH, reflux
iii. H2O
Ans:
O
:
NH
O
N
p-TsOH
Br
N
+
O
EtOH, reflux
O
O
H2O
O
O
O
Topic: Multistep Reaction Sequence:
91. What is the final product of the following reaction sequence? Give structural details of
all significant intermediates.
NH , p-TsOH
i.
O
?
O
ii. Br
O
iii. H2O
Ans:
NH
O
:
O
N
Br
N
+ Br
O
p-TsOH
O
O
H2O
O
O
O
Chapter 19
Topic: Multistep Reaction Sequence:
92. What is the final product of the following reaction sequence? Give structural details of
all significant intermediates.
O
O
i. Zn,
Br
O
?
ii. NH4Cl
iii. PCC
iv. t-BuOK, t-BuOH
v. CH3I
O
Ans:
i. Zn,
O
OH O
Br
O
O
ii. NH4Cl
PCC
O
O
O
i. t-BuOK, t-BuOH
ii. CH3I
O
O
O
Topic: Multistep Reaction Sequence:
93. What is the expected final product when phenylacetonitrile is subjected to the following
reaction sequence?
i. EtO-, EtOH
CN
?
ii. CH3CH2I
Ans:
i. EtO-, EtOH
CN
CN
ii. CH3CH2I
Chapter 19
Topic: Multistep Reaction Sequence:
94. What is the final product of the following reaction sequence? Give structural details of
all significant intermediates.
O
O
i. Cl2, FeCl3
ii.
?
-
O
, EtO , EtOH
O
Ans:
O
O
O
Cl2, FeCl3
O
O
O
O
O
O
-
EtO , EtOH
Cl
Cl
Topic: Synthetic Strategy
95. Suggest a reasonable synthetic strategy to carry out the following transformation.
OH
O
O
O2N
O2N
OH
Ans:
O
O2N
O
NO2
O O
EtO-, EtOH
O2N
O
NO2
i. LAH
ii. H2O
OH
O2N
OH
NO2
Chapter 19
Topic: Synthetic Strategy
96. Suggest a reasonable synthetic strategy to carry out the following transformation.
O
O
Br
O
Br
Br
O O
Ans:
O
EtO-, EtOH
O
Br
O
Br
Br
H3O+, heat
O
O
Br
Br
OH
O
Br
Br
Topic: Multistep Reaction Sequence
97. What is the final product of the following reaction sequence? Give structural details of
all significant intermediates.
O
O
i. EtO-, EtOH
O
ii. H3O+
O
?
iii. NaBH4, H2O
Ans:
O
O
O
O
i. EtO-, EtOH
ii. H3O+
NaBH4
H2O
O
O
O
HO
O
O
Chapter 19
Topic: Multistep Reaction Sequence
98. What is the final product of the following reaction sequence? Give structural details of
all significant intermediates.
i. NaH, DMF
O
O
?
ii. CH3CH2COCl
O
iii. NaOH, H2O
iv. H3O+, heat
Ans:
O
O
O
i. NaH, DMF
O
ii. CH3CH2COCl O
i. NaOH, H2O
OO
ii. H3O+, heat
O
O
O
OH
O
O
Topic: Reaction Products
99. Suggest a reasonable synthetic strategy for the synthesis of 3-heptanone from propanal.
SH
S
Ans:
O HS
+
H3O
S
propanal
i. n-BuLi
ii.
I
O
3-heptanone
HgCl2
CH3OH, H2O
S
S
Chapter 19
Topic: Reaction Products
100. Suggest a reasonable mechanism for the following reaction.
O
+
O
O
O
i.EtONa,
EtOH, 25oC
ii. H3O+
O
O
O
O
O
.. −
EtO : +
..
O
O
EtOH +
H
O
−:
O
O
O
:
O
O
:
Ans:
O
O
O
O
O
−
..
O
O
O
:O :
O
O
O
O
O
O
H3O+
O
O
O
:O :
O
:O
.. :
−
O
O
Chapter 20
MULTIPLE CHOICE QUESTIONS
Topic: Nomenclature
1. Which of these is an acceptable alternative name to "(1-methylbutyl)amine"?
A) 2-Aminopentane
B) 2-Pentanamine
C) Isopentylamine
D) sec-Pentylamine
E) Both A) and B)
Ans: E
Topic: Nomenclature
2. Which is a correct common name for the following substance?
N
A)
B)
C)
D)
E)
Ans:
Ethylethylisobutylamine
Diethylisobutylamine
sec-Butyldiethylamine
Ethylethyl-sec-butylamine
2-Diethylaminobutane
C
Topic: Nomenclature
3. Which of the following is a tertiary amine?
A) CH3CH2CH2CH2NH2
B) CH3CH2NHCH2CH(CH3)2
C) (CH3CH2)2NCH2CH(CH3)2
D) (CH3CH2)4N+ OHE) (CH3CH2)3CNH2
Ans: C
Topic: Nomenclature
4. What type of amine is N-methyl-2-methyl-3-hexanamine?
A) Primary
B) Secondary
C) Tertiary
D) Quaternary
E) None of these
Ans: B
747
Chapter 20
Topic: Nomenclature
5. Which of these is properly termed a "quaternary ammonium salt"?
A) (CH3)3CCH2CH2NH3+ Cl−
B) (CH3CH2CH(CH3)CH2)2NH2+ Cl−
C) (CH3CH2CH2)3NH+ Cl−
D) (CH3CH2CH2)4N+ Cl−
E) None of these
Ans: D
Topic: Chemical Tests and Separations
6. Which reagent would serve as the basis for a simple chemical test that would distinguish
between the pair of compounds listed below?
H
N
NH2
and
A)
B)
C)
D)
E)
Ans:
AgNO3 in H2O
Dilute NaHCO3
Dilute NaOH
C6H5SO2Cl/OH-, then H3O+
Dilute HCl
D
Topic: Chemical Tests and Separations
7. Which reagent would serve as the basis for a simple chemical test that would distinguish
between the pair of compounds listed below?
NH2
and
A)
B)
C)
D)
E)
Ans:
NH2
AgNO3 in H2O
HONO, 0-5°C then β-naphthol
Dilute NaOH
C6H5SO2Cl and OH- in H2O
Dilute HCl
B
Topic: Chemical Tests and Separations
8. Which reagent will distinguish between C6H5NH2 and (C6H5)2NH?
A) HCl (aq)
B) NaOH
C) C6H5SO2Cl/OH-, then H3O+
D) Br2/CCl4
E) KMnO4
Ans: C
748
Chapter 20
Topic: Chemical Tests and Separations, Nomenclature
9. Which reagent will distinguish between 2-amino-2,3-dimethylpentane and 1-amino-3methyl-2-pentene?
A) HONO, 0-5°C
B) C6H5SO2Cl/OH-, then H3O+
C) NaOH
D) HCl
E) Br2/CCl4
Ans: E
Topic: Chemical Tests and Separations, Nomenclature
10. Which reagent could be used to separate a mixture of aniline and toluene?
A) KMnO4 in H2O
B) Dilute NaOH
C) Dilute NaHCO3
D) Ag(NH3)2OH
E) Dilute HCl
Ans: E
749
Chapter 20
Topic: Chemical Tests and Separations
11. Compound W has the molecular formula C11H17N. Treatment of W with
benzenesulfonyl chloride in base gives no reaction. Acidification of the resulting
mixture gives a clear solution. The 1H NMR spectrum of W consists of:
δ 1.0
δ 2.5
δ 3.6 (2H)
δ 7.3 (5H)
triplet,
quartet,
singlet,
multiplet,
The most likely structure for W is:
A)
N
H
B)
N
C)
N
D)
N
E)
N
Ans: B
Topic: Chemical Tests and Separations
12. Which of these could be resolved into separate enantiomers?
A) 4-methyl-1-pentanamine
B) 4-methyl-2-pentanamine
C) N-methyl-1-butanamine
D) N,N-dimethyl-1-propanamine
E) N-Butyltrimethylammonium bromide
Ans: B
750
Chapter 20
Topic: Reaction Products
13. What final product is expected from the following reaction sequence?
i. Fe/HCl
NO2
ii. NaHCO3
?
iii. Br2/H2O
Cl
iv. HONO, 0-5oC
v. H3PO2
Br
Br
Br
OH
NO2
Cl
Br
Cl
I
III
II
Br
Br
PO2H
Br
Cl
IV
A)
B)
C)
D)
E)
Ans:
Cl
NO
HO2P
Cl
V
I
II
III
IV
V
B
751
Chapter 20
Topic: Reaction Products
14. What compound is likely to be obtained via the following reaction sequence?
i. HONO, 0-5oC
NH
2
Br
Br
ii. CuCl
?
iii. Fe/HCl
NO2
iv. NaOH, H2O
o
v. HONO, 0-5 C
vi. H3PO2
Br
Br
Br
Br
Br
Br
A)
B)
C)
D)
E)
Ans:
Cl
NO2
I
II
Cl
Cl
Br
Br
Br
Br
PO2H
III
I
II
III
IV
V
C
752
IV
V
Chapter 20
Topic: Reaction Products
15. What would be the product, L, of the following reaction?
Cl
NH2
i. NaCN
ii. H2, Ni
?
CN
N
H
I
III
II
NH2
IV
A)
B)
C)
D)
E)
Ans:
V
I
II
III
IV
V
E
Topic: Reaction Products
16. What final product is expected when toluene is subjected to the following reaction
sequence?
i) KMnO4, NaOH; ii) H3O+ ( product = C7H6O2); iii) SOCl2;
iv) NH3 (product = C7H7NO); v) Br2, NaOH
A) C6H5CONH2
B) C6H5CH2NH2
C) p-CH3C6H4SO2NH2
D) p-CH3C6H4NH2
E) C6H5NH2
Ans: E
753
Chapter 20
Topic: Reaction Products
17. What would be the product of the following reaction sequence?
Cl
i)
, AlCl3
O
?
ii) (CH3)2NH
iii) LiBH3CN
N
O
O
N
N
II
I
III
OH
N
N
IV
A)
B)
C)
D)
E)
Ans:
V
I
II
III
IV
V
C
754
Chapter 20
Topic: Reaction Products
18. What would be the product of the following reaction sequence?
i. CH3I
?
N
ii. Ag2O, H2O
iii. heat
OH
N
I
A)
B)
C)
D)
E)
Ans:
N
N
N
II
III
IV
I
II
III
IV
Answers A) and C) only
A
Topic: Reaction Products
19. Which product could not be formed during the following reaction?
NH2
HONO
0-5oC
?
OH
OH
I
II
III
HO
IV
A)
B)
C)
D)
E)
Ans:
OH
V
I
II
III
IV
V
E
755
Chapter 20
Topic: Reaction Products
20. What would be the product of the following reaction sequence?
i. Fe/HCl
iv. NaOH, H2O
NO2
?
iii. Br2/H2O
v. HONO, 0-5oC
vi. H3PO2
Cl
Br
Br
OH
II
I
A)
B)
C)
D)
E)
Ans:
Br
Br
PO2H
Br
NO2
III
Br
Br
PO2H
NO
IV
V
I
II
III
IV
V
B
Topic: Reaction Products
21. Identify the compound (Z) formed when aniline is subjected to the following series of
reactions:
i.) NaNO2, H2SO4, dil.H2SO4 0-5oC;
ii) Cu2O, Cu(NO3)2 , H2O
A)
B)
C)
D)
E)
Ans:
C6H5NH2
C6H5Cl
C6H5OH
C6H6
None of these
C
Topic: Reaction Products
22. Which is the predominant product formed when m-dinitrobenzene is treated with
H2S/NH3 ?
A) m -HSC6H4NH2
B) m -C6H4(NH2)2
C) m -HOC6H4NH2
D) m -H2NC6H4NO2
E) C6H5NH2
Ans: D
756
Chapter 20
Topic: Reaction Products
23. Which is the major product of the following reaction?
NH2
i. HONO, 0-5oC
ii.
?
, NaOH
OH
NO2
OH
OH
I
III
II
HO
N
N
N
OH
N
V
IV
2
A)
B)
C)
D)
E)
Ans:
I
II
III
IV
V
E
Topic: Reaction Products
24. What is the final product?
OH
i. Fe/HCl
−
ii. OH
C7H9N i. Br2(excess)
ii. H2O
product
i. HONO, 0-5oC
ii. H2O, Cu2O,
Cu(NO3)2
Final Product
A)
B)
C)
D)
E)
Ans:
2-Bromo-4-methylaniline
2,6-Dibromo-4-methylaniline
2,6-Dibromo-4-methylphenol
2,4-Dibromophenol
3,5-Dibromotoluene
C
757
Chapter 20
Topic: Reaction Products
25. What is the final product, C?
O
NH3
excess
O
+
NH4 + C4H5NO3
−
H3O+
Br2
C4H7NO3
OH−
C
O
HO
O
NH2
O
CO2−
H2N
I
A)
B)
C)
D)
E)
Ans:
−
−
O2C
II
NH2
O
Br
O2C
O
NH2
III
IV
I
II
III
IV
V
C
Topic: Reaction Products
26. What would be the product of the following reaction sequence?
i) KMnO4, NaOH, heat
Br
ii) H3O+
?
iii) PCl5
iv) NaN3
v) heat
vi) H2O
A)
B)
C)
D)
E)
Ans:
p-bromobenzamide
p-bromobenzaldehyde oxime
p-bromobenzenesulfonamide
p-bromoaniline
4-bromo-3-chlorobenzoic acid
D
758
N-Br
O
V
Chapter 20
Topic: Reaction Products
27. What is the final product of the following sequence of reactions?
H2S, NH3
EtOH
O2N
i. HONO, 5oC
C6H6N2O2
ii. KI
Final product
NO2
A)
B)
C)
D)
E)
Ans:
1-Iodo-3-nitrobenzene
3-Nitroaniline
3-Nitrophenol
3-Nitrosoaniline
3-Nitrosophenol
A
Topic: Reaction Products
28. When an equimolar mixture of ammonia and butyl bromide reacts, which of these
products will form?
A) Butylamine
B) Dibutylamine
C) Tributylamine
D) Tetrabutylammonium bromide
E) All of these
Ans: E
Topic: Reaction Products
29. What is the chief alkene product when butylethylmethylpropylammonium hydroxide is
heated to 150°C?
A) CH2=CH2
B) CH3CH=CH2
C) CH3CH2CH=CH2
D) E-CH3CH=CHCH3
E) Z-CH3CH=CHCH3
Ans: A
759
Chapter 20
Topic: Synthesis
30. Which of the following reactions would yield C6H5CH2NH2?
A)
i. NaCN
Br
ii. LiAlH4, ether
iii. H2O
NH3
B)
O
H2, Ni
C)
i. NaN3
Br
ii. LiAlH4, ether
iii. H2O
D) All of these
E) Answers B) and C) only
Ans: E
Topic: Synthesis
31. Which of the following reaction sequences would yield aniline?
A)
i. HNO3, H2SO4
ii. Fe, HCl
iii. NaOH, H2O
Cl
B)
i. NH3
O
C)
Br
−
ii. Br2,OH
i. NH3 (excess)
ii. HONO, 0-5oC, ether
iii. H3PO2
D) All of the above
E) Answers A) and B) only
Ans: E
760
Chapter 20
Topic: Synthesis
32. Which of the following might be used to synthesize m-bromoaniline?
A)
i. HNO3, H2SO4
ii. Fe, HCl
iii. NaOH, H2O
iv. Br2, H2O
Cl
B)
i. Br2, FeBr3
O
ii. HNO3, H2SO4
iii. H2, Pt
OH
C)
i. Br2, FeBr3
O
ii. SOCl2
iii. NH3
iv. Br2, NaOH
D) Answers A) and B)
E) Answers B) and C)
Ans: C
Topic: Synthesis
33. The best synthesis of 3,5-dibromotoluene would be:
A) Toluene, Br2, Fe and heat
B) p-CH3C6H4NH2, Br2, H2O; then HONO, 0-5°C; then H3PO2
C) Toluene, fuming HNO3, H2SO4; then NH3, H2S; then HONO, 0-5°C; then CuBr
D) m-Dibromobenzene, CH3Cl, AlCl3, heat
E) m-Bromotoluene, HNO3, H2SO4; then NH3, H2S; then HONO, 0-5°C; then CuBr
Ans: B
Topic: Synthesis
34. The best synthesis of m-dibromobenzene would be:
A) Benzene, Br2, FeBr3, heat
B) Aniline, Br2, H2O; then HONO, 0-5°C; then CuBr
C) Nitrobenzene, HNO3, H2SO4, heat; then Fe/HCl (excess); then 2 HONO, 0-5°C;
then 2 CuBr
D) Bromobenzene, HNO3, H2SO4; then Fe, HCl, C2H5OH, reflux; then HONO, 05°C; then CuBr
E) Answers C) and D)
Ans: C
761
Chapter 20
Topic: Synthesis
35. The best synthesis of m-fluoronitrobenzene would be:
A) Nitrobenzene, fuming HNO3, H2SO4, heat; then NH3, H2S; then HONO, 0-5°C;
then HBF4; then heat
B) Aniline, F2, heat
C) Fluorobenzene, HNO3, H2SO4, heat
D) o-Nitroacetanilide, NH3, H2S; then Br2, OH-; then HF
E) m-C6H4(NH2)2, HONO, 0-5°C; then CuNO2; then HBF4
Ans: A
Topic: Synthesis
36. Which is the best preparation of benzonitrile, C6H5CN, from benzene?
A) HNO3; then Fe/HCl; then HONO, 0-5°C; then CuCN
B) CH3I, AlCl3; then hot KMnO4; then H3O+; then SOCl2; then NH3; then NaOH +
Cl2
C) CH3I, AlCl3; then Br2, hν; then KCN
D) Br2, Fe; then KCN
E) CH3I, AlCl3; then Br2 (2 eq.), hν; then hot NaOH; then HCN
Ans: A
Topic: Synthesis
37. Which of the following can be used to prepare 2-aminopentane (pure)?
I. (1mol) 2-bromopentane + (1mol) NH3
II. (1mol) 2-bromopentane + (1mol) NaN3
III. Phthalimide
A)
B)
C)
D)
E)
Ans:
KOH
2-bromopentane
I
II
III
I and II
II and III
E
762
LAH
Et2O
NH2NH2
C2H5OH
Chapter 20
Topic: Synthesis
NH2
38.
Which is the best method to prepare
O
A)
NH3(excess)
H2, Ni
Br
B)
NH3
C)
D)
CH3NH2
H2, Ni
O
NO2
NH2
+
E)
?
+ HNO3
HONO
0-5oC
hν
H2, Ni
Ans: A
Topic: Synthesis
H
N
39.
Identify the best method(s) to prepare
NH2
CH3I
A)
(1 mol)
(1 mol)
(CH3)2NH
B)
O
H2, Ni
C)
O
.
CH3NH2
H2, Ni
D) A) and B)
E) B) and C)
Ans: C
Topic: Synthesis, Nomenclature
40. Your task is to convert o-xylene into o-diaminobenzene. Which sequence of reagents
constitutes the best method?
A) NaNH2 and heat
B) NBS/CCl4; then NH3; then Br2/OHC) KMnO4, OH-, heat; then H3O+; then PCl5; then NH3, then Br2/OHD) KMnO4, OH-, heat; then H3O+; then SOCl2; then NH3; then LiAlH4; then Br2/OHE) KMnO4, OH-, heat; then H3O+; then NH3 with H2/Ni
Ans: C
763
Chapter 20
Topic: Synthesis
41. How would one carry out the following transformation: ?
?
H2N
A)
B)
C)
D)
E)
Ans:
Cl
Cl
NaNO2,HCl, 0-5°C; then HNO3
NaNO2,HCl, 0-5°C; then H3PO2
NaNO2,HCl, 0-5°C; then H2,Ni
C6H5SO2Cl, OH-; then HCl
NaH, DMSO
B
Topic: Synthesis
42. How could one carry out this synthesis?
O
OH
NH2
?
Br
A)
B)
C)
D)
E)
Ans:
Br
SOCl2; then NH3; then H3PO2
CH3Li, ether; then NH3, H2, Ni
SOCl2; then NH3; then Br2, NaOH
PCl5; then NH3; then HCl, NaNO2, 0-5°C
PCl5; then CH3NH2; then KMnO4, OH-, heat
C
Topic: Synthesis
43. Consider the synthesis below. What is reagent A?
NO2
NH2
A
A)
B)
C)
D)
E)
Ans:
B
C
Br
Br
Br2, FeBr3
Fe, HCl; then OH−
NH2Cl, AlCl3
H3PO2
LiNH2
B
764
Chapter 20
Topic: Synthesis
44. Consider the synthesis below. What is compound B?
NO2
NH2
A
B
C
Br
Br
NO2
NH2
Br
Br
Br
II
I
III
NO2
NH2
Br
Br
Br
IV
A)
B)
C)
D)
E)
Ans:
V
I
II
III
IV
V
C
Topic: Synthesis
45. Consider the synthesis below. What is reagent C?
NO2
NH2
A
A)
B)
C)
D)
E)
Ans:
B
C
Br
Br
excess Br2, H2O
Fe, HCl; then OHNH2Cl, AlCl3
CuBr
HNO3, H2SO4, Fe
A
765
Chapter 20
Topic: Synthesis
46. Consider the synthesis below. What is reagent A?
NH2
Br
A)
B)
C)
D)
E)
Ans:
CN
Br
i. HCl, NaNO2
ii. "A"
Br
Br
H3PO2
HCN
P4O10
CuCN
CuCl2
D
Topic: Synthesis
47. What reagent can effect the following transformation?
CN
Br
A)
B)
C)
D)
E)
Ans:
Br
Br
?
CO2H
Br
CO2; then H3O+
H3O+, heat
Mg; then CO2; then H3O+
LiAlH4, ether
KMnO4, OH-, heat; then H3O+
B
Topic: Synthesis
48. What reagent can effect the following transformation?
Br
CO2H
Br
Br
?
CO2H
Br
CO2H
A)
B)
C)
D)
E)
Ans:
+
CO2; then H3O
H3O+, heat
Mg; then CO2; then H3O+
H2O2, heat
KMnO4, OH-, heat; then H3O+
E
766
Chapter 20
Topic: Synthesis
49. What reagent can effect the following transformation?
NO2
NO2
?
NO2
A)
B)
C)
D)
E)
Ans:
NH2
-
Fe/HCl; then OH
H2S, NH3, C2H5OH
H2, Ni, high pressure
Zn(Hg)/HCl
Raney Ni
B
Topic: Synthesis
50. Consider the synthesis below. What is reagent “Z”?
NO2
NO2
o
i. HONO, 0-5 C
ii. "Z"
NH2
A)
B)
C)
D)
E)
Ans:
Cl
CuCl
CuCl2
NaCl
KCl
HCl/heat
A
Topic: Synthesis
51. What reagent can effect the following transformation?
NO2
NH2
?
Cl
A)
B)
C)
D)
E)
Ans:
Fe/HCl; then OHNH2Cl
H3PO2
CuCN
HONO, then NH3
A
Cl
767
Chapter 20
Topic: Synthesis
52. Consider the following reaction: what is the expected product?
NH2
Br2, H2O
?
Cl
Cl
NH2
Br
Br
H2N
Cl
Br
Br
NH2
Cl
I
Br
Br
II
III
NH2
NH2
Br
Br
Br
A)
B)
C)
D)
E)
Ans:
Cl
Cl
IV
V
I
II
III
IV
V
B
Topic: Synthesis
53. Consider the synthesis below. What is reagent “Y”?
NH2
Br
Br
i. HONO, 0-5oC
ii. "Y"
Cl
A)
B)
C)
D)
E)
Ans:
Br
Br
Cl
Br2/FeBr3
CuBr
CuBr2
H3PO2
H3PO4
D
768
Chapter 20
Topic: Synthesis
54. What is the chief product of the Hofmann elimination reaction applied to the compound
shown?
N
OH
II
I
IV
A)
B)
C)
D)
E)
Ans:
III
V
I
II
III
IV
V
A
769
Chapter 20
Topic: Synthesis
55. What is the principal product when aniline is treated with sodium nitrite and
hydrochloric acid at 0-5°C and this mixture is added to p-ethylphenol?
OH
OH
OH
N
N
N
I
II
N
III
OH
HO
N
N
IV
A)
B)
C)
D)
E)
Ans:
V
I
II
III
IV
V
A
Topic: Synthesis
56. What is the final product in the Curtius rearrangement of the acyl azide formed from
butanoyl chloride?
NH2
NH2
N
II
I
III
O
O
N
H
IV
A)
B)
C)
D)
E)
Ans
N3
V
I
II
III
IV
V
A
770
C O
Chapter 20
Topic: Synthesis
57.
Which combination of reactants will not produce
O
A)
i. LiAlH4, ether
N
ii. H2O
H
O
B)
H
i. LiAlH4, ether
N
ii. H2O
C)
NaBH3CN
+ H2N
H
N
?
O
D)
O
E)
H
N
+
H2N
NaBH3CN
Br2, NaOH
O
Ans: E
Topic: Synthesis
58. The overall conversion RBr ⎯⎯⎯⎯→ RCH2NH2 can be accomplished by
successive application of which of these sets of reagents?
A) Mg, ether; then NH3
B) NaN3; then LiAlH4, ether
C) NaCN; then LiAlH4, ether
D) H2C=O; then NH3
E) H2NOH; then LiAlH4, ether
Ans: C
Topic: Synthesis
59. Which would be a good method to synthesize m-nitroaniline?
A) 1,3-Dinitrobenzene + H2S and NH3 in EtOH
B) Aniline + HNO3/H2SO4
C) Aniline + CH3COCl; then HNO3/H2SO4
D) Nitrobenzene + NH3 and AlCl3
E) More than one of the above
Ans: A
771
Chapter 20
Topic: Basicity
60. Which of the following compounds would be the strongest base?
O
NH2
NH2
NH2
I
III
II
NH2
NH2
O2 N
IV
A)
B)
C)
D)
E)
Ans:
V
I
II
III
IV
V
B
Topic: Basicity
61. Which of the following compounds would be the strongest base?
N
N
H
NH2
I
III
II
O
A)
B)
C)
D)
E)
Ans:
OH
N
N
IV
V
I
II
III
IV
V
D
Topic: Basicity
62. Which would be the weakest base?
A) p-Methylaniline
B) p-Methoxyaniline
C) Hexylamine
D) p-Nitroaniline
E) Dipropylamine
Ans: D
772
Chapter 20
Topic: Basicity
63. Arrange the following amines in order of decreasing basicity in aqueous solution:
NH3
I
A)
B)
C)
D)
E)
Ans:
CH3CH2CH2NH2
II
(CH3CH2CH2)2NH
III
(CH3CH2CH2)3N
IV
IV > III > II > I
III > II > IV > I
I > IV > II > III
II > III > I > IV
III > I > II > IV
B
Topic: Basicity
64. In aqueous solution, which of the following (as the conjugate acid) possesses the
smallest value for pKa?
A) C6H5NH2
B) NH3
C) (CH3CH2)3N
D) (CH3CH2)2NH
E) CH3CH2CH2NH2
Ans: A
Topic: Basicity
65. Which of these is the strongest acid?
NH3
NH3
NH3
O2N
II
I
III
NH3
NH3
I
IV
A)
B)
C)
D)
E)
Ans:
V
I
II
III
IV
V
B
773
Chapter 20
Topic: Basicity
66. Which of these is the strongest acid?
O
O
O
II
I
O
A)
B)
C)
D)
E)
Ans:
N
N
H
NH2
III
O
N
H
NH3
IV
V
I
II
III
IV
V
D
Topic: Basicity
67. Which of these compounds is soluble in dilute sodium hydroxide solution?
O
O
NH
I
O
S
O
H
N
N
III
II
O
N
H
IV
A)
B)
C)
D)
E)
Ans:
N
H
V
I
II
III
IV
V
A
774
Chapter 20
Topic: Reactive Intermediates and General Considerations
68. Which is not an intermediate in the Hofmann degradation reaction?
KOH
RCONH2
RNH2
Br2
A) RN=C=O
O
B)
R
NHBr
O
C)
..
..
R
NBr
O
D)
HO
N
H
R
O
E)
R
N
Br
Br
Ans: E
Topic: Reactive Intermediates and General Considerations
69. Which of these compounds is expected to possess the lowest boiling point?
A) CH3CH2CH2CH2CH2NH2
B) CH3CH2CH2NHCH2CH3
C) (CH3CH2)2NCH3
D) (CH3CH2)2CHOH
E) (CH3)3CCH2NH3+ ClAns: C
Topic: Reactive Intermediates and General Considerations
70. This type of compound is the only one of these which can be converted by reduction
into a 1°, 2° or 3° amine, according to its particular structure:
A) Nitrile
B) Oxime
C) Azide
D) Amide
E) Nitroalkane
Ans: D
775
Chapter 20
Topic: Reactive Intermediates and General Considerations
71. When the aromatic -NH2 group is to be converted to an -OH group via a diazonium salt,
the nitrous acid used is generated in situ by the reaction of a nitrite salt with which acid?
A) HCl
B) HBr
C) HNO3
D) H2SO4
E) CH3CO2H
Ans: D
Topic: Reactive Intermediates and General Considerations
72. The reaction of which of these compounds with nitrous acid results in a stable N-nitroso
compound?
A) C6H5NH2
B) C6H5N(CH3)2
C) CH3CH2CH2CH2CH2NH2
D) C6H5NHCH3
E) CH3CH2CONH2
Ans: D
Topic: Reactive Intermediates and General Considerations
73. When the process ArNH2 ⎯⎯⎯⎯→ ArY is carried out via an intermediate
diazonium salt, this salt is isolated only in the case in which Y is which of these groups?
A) -F
B) -Cl
C) -Br
D) -I
E) -CN
Ans: A
Topic: Reactive Intermediates and General Considerations
74. What is the basis for the successful resolution of racemic C6H5CHOHCO2H through
use of the chiral amine, C6H5CH(NH2)CH3?
A) One enantiomer is more soluble than the other.
B) The racemic mixture is converted into a single isomer in the basic solvent.
C) The diastereomeric salts formed have different solubilities.
D) The diastereomeric salts have different boiling points.
E) The diastereomeric salts have different melting points.
Ans: C
776
Chapter 20
Topic: Reactive Intermediates and General Considerations
75. Which of these alkyl halides cannot be used effectively in a Gabriel amine synthesis?
A) 1-bromopentane
B) 1-bromo-3-methylbutane
C) 2-bromo-3-methylpentane
D) 1-bromo-2,3-dimethylbutane
E) 2-bromo-2,3-dimethylbutane
Ans: E
SHORT ANSWER QUESTIONS
Topic: Stereochemistry
76. If the three alkyl groups of a tertiary amine are all different, the amine will be chiral. In
practice, however, it is virtually impossible to separate the two enantiomers, because
they interconvert rapidly through a process called _________________.
Ans: nitrogen inversion
Topic: Relative Basicity
77. In solution, two factors affect the relative basicity of amines. These factors are:
__________________.
Ans: electron donation by alkyl substituents and stabilization of the alkylaminium ion
by hydrogen bonding with solvent
Topic: Relative Basicity
78. Arylamines are less basic than alkylamines because of _______________.
Ans: delocalization of the lone pair on nitrogen into the aromatic system
Topic: Enantiomer Resolution
79. Explain how an enantiomerically pure amine can be used to resolve racemic forms of
acidic compounds.
Ans: The amine reacts with the acidic compound to form a pair of diastereomeric salts,
which can be easily separated.
Topic: General
80. ______________ is used in automotive airbags.
Ans: Sodium azide
Topic: General, Synthesis of Amines
81. There are two very versatile synthetic methods that can be used to synthesize primary,
secondary, and tertiary amines. These methods are: __________________.
Ans: reductive amination of aldehydes and ketones, and reduction of amides
777
Chapter 20
Topic: Chemical Analysis
82. While primary amines react with nitrous acid to generate diazonium salts, secondary
amines react with nitrous acid to produce _____________.
Ans: N-nitrosoamines
Topic: UV-Vis Absorption
83. Azo compounds are usually intensely colored because _________________.
Ans: the N=N linkage brings two aromatic rings into conjugation, giving an extended π
system that generally absorbs in the visible region
Topic: General
84. Chemicals that inhibit the growth of microbes are called ______________.
Ans: antimetabolites
Topic: Hofmann Elimination
85. Eliminations that follow the Hofmann Rule tend to give the ______________ alkene as
the major product.
Ans: least-substituted
Topic: Nomenclature
86. Draw the structure corresponding to the following name:
(4Z,3S)-2,4,7-trimethyl-3-amino-4-octene
Ans:
H2N
H
Topic: Nomenclature
87. Draw the structure corresponding to the following name:
(3S,6R)-6-(2,4-dinitrophenyl)-3-amino-1-heptyne
Ans: H2N H
CH
3
H
NO2
NO2
778
Chapter 20
Topic: Exhaustive Methylation, Hofmann Degradation, Nomenclature
88. Before modern spectroscopic methods were developed, structures of many complex
amines were elucidated by examining the products obtained during Hofmann
elimination of their quaternary ammonium hydroxides. The typical procedure involved
the following steps, repeated as necessary, depending on whether the amine was
primary, secondary or tertiary, until the final product was nitrogen-free: i) CH3I
(excess); ii) Ag2O, H2O; iii) heat. What product(s) is (are) likely to be formed if 3,4dimethylpiperidine is subjected to the above procedure?
Ans: The reaction sequence would need to be repeated twice to obtain a nitrogen-free
product, since the starting compound is a secondary amine. The final product
expected is 2,3-dimethyl-1,4-pentadiene along with trimethylamine.
N
H
CH3I
(excess)
N I
Ag2O
H2O
heat
N OH
N
i. CH3I (excess)
ii. Ag2O, H2O
iii. heat
N
+
Topic: Multistep Reaction Sequence
89. What final product is likely when cyclohexene is subjected to the following reaction
sequence? Give structural details of all significant intermediates.
i.) H3O+, H2O; ii) PCC; iii) (CH3)2NH; iv) NaBH3CN
Ans:
OH
O
N
+
H3O
N
(CH3)2NH
PCC
H2O
779
OH
NaBH3CN
Chapter 20
Topic: Multistep Reaction Sequence
90. What final product is likely when p-dinitrobenzene is subjected to the following reaction
sequence? Give structural details of all significant intermediates.
i.) H2S, NH3, EtOH; ii.)CH3CH2COCl; iii.) LiAlH4; iv) H2O
Ans:
NO2
NH2
H2S, NH3
EtOH
HN
O
CH3CH2COCl
NO2
NO2
NO2
i. LiAlH4, Et2O
ii. H2O
HN
NO2
Topic: Multistep Reaction Sequence, Nomenclature
91. What final product is likely when acetophenone is subjected to the following reaction
sequence? Give structural details of all significant intermediates.
i) Br2, FeBr3; ii) CH3CH2CH2NH2 ; ii.) NaBH3CN
Ans:
O
O
Br2
FeBr3
N
CH3CH2CH2NH2
Br
Br
NaBH3CN
NH
Br
780
Chapter 20
Topic: Gabriel Synthesis
92. Outline the steps involved in the Gabriel synthesis of 3-amino-2-methylpentane.
Ans:
O
NH
O
KOH
O
Br
NK
O
N
O
O
NH2NH2
EtOH
reflux
O
NH
NH
+
NH2
O
3-amino-2-methylpentane
Topic: Synthesis, Hofmann Rearrangement
93. Outline the steps involved in the synthesis of 4-bromo-3-nitro-aniline from 4bromobenzaldehyde
Ans: i. HNO3, H2SO4 ; ii) KMnO4, OH-, H2O; iii) H3O+; iv) SOCl2; v) NH3
vi) Br2, OHO
O
O
−
i. KMnO4, OH , H2O
HNO3
H2SO4
Br
OH
NO2
Br
+
NO2
ii. H3O
Br
4-bromobenzaldehyde
SOCl2
O
NH2
NH2
O
Cl
NH3
Br2, NaOH
NO2
NO2
Br
Br
4-bromo-3-nitro-aniline
781
NO2
Br
Chapter 20
Topic: Synthetic Strategy
94. Outline the steps involved in the synthesis of 4-bromo-3-nitro-phenol from 4bromobenzoic acid
Ans: i) HNO3,H2SO4 ; ii) SOCl2; iii) NH3; iv) Br2, OH-; iv) NaNO2, H3O+, 0-5oC
v) Cu2O, Cu(NO3)2, H2O
O
OH
O
OH
O
Cl
O
NH3
SOCl2
HNO3
H2SO4
NO2
Br
NO2
Br
NO2
Br
Br
Br2
NaOH
4-bromobenzoic acid
OH
N2+
NO2
NH2
Cu2O, Cu(NO3)2
H2O
NaNO2, H3O+
0-5oC
NO2
Br
NH2
NO2
Br
Br
4-bromo-3-nitro-phenol
Topic: Synthesis, Nomenclature
95. Outline the steps involved in the synthesis of 3-chloro-4-fluorophenyl acetate from 4aminoacetophenone
Ans: i) Cl2, FeCl3; ii) NaNO2, H3O+, 0-5oC; iii) HBF4; iv) heat ; v) MCPBA
O
O
O
NaNO2, H3O
Cl2
FeCl3
NH2
+
o
0-5 C
Cl
Cl
NH2
N2+
4-aminoacetophenone
i. HBF4
ii. heat
O
O
O
MCPBA
Cl
F
3-chloro-4-fluorophenyl acetate
782
Cl
F
Chapter 20
Topic: Synthesis, Nomenclature
96. What is the principal product when p-toluidine is treated with sodium nitrite and
hydrochloric acid at 0-5°C and this mixture is added to o-ethylphenol?
NaNO2, HCl
Ans:
NH2
N2+
0-5oC
p-toluidine
OH
OH
N
N
Topic: Relative Base Strength
97. Explain why cyclohexylamine is a stronger base than aniline
Ans: There are two major reasons for the observed difference in basicity between the
two substances:
In aniline, the electron pair on N is delocalized over the aromatic ring
i)
through resonance: this has the effect of decreasing the electron
density at the nitrogen atom, relative to cyclohexylamine, in which no
such resonance delocalization can occur, leading to lower base strength
for aniline.
In cyclohexylamine, the N atom is attached to an sp3 carbon, while in
ii)
aniline the N atom is attached to an sp2 carbon: an sp2 carbon typically
exerts a stronger electron-withdrawing effect, effectively decreasing
the electron density on the nitrogen atom, resulting in weaker base
strength.
Topic: Product Analysis by NMR
98. You have just completed an experiment involving the reduction of m-nitrobenzoic acid
with Fe/HCl. In order to determine if your reaction has worked as planned, you have
obtained the 1H NMR of your crude product. What will you scrutinize the spectrum for,
in order to help you make this assessment?
Ans: The appearance of 2 new N-H signals, in the range of about 2-6 ppm will be
indicative of successful reduction of the –NO2 group to the –NH2 group. These
signals are characteristically somewhat broad, and should disappear (or decrease
markedly) if the spectrum is rerun with an added drop of D2O to the original
sample prepared for obtaining the 1H NMR. The rest of the signals are likely to
remain more or less similar, with some changes in chemical shift likely (probably
upfield shifts), since the –NO2 group is electron-withdrawing, while the –NH2
group is electron-donating.
783
Chapter 21
MULTIPLE CHOICE QUESTIONS
Topic: Nomenclature
1. The common name for 4-methylphenol is which of these?
A) Catechol
B) p-Cresol
C) Resorcinol
D) Hydroquinone
E) p-Xylenol
Ans: B
Topic: Nomenclature
2. The common name for 3-methylphenol is which of these?
A) Catechol
B) m -Cresol
C) m-tolylol
D) Hydroquinone
E) p-Xylenol
Ans: B
Topic: Nomenclature
3. The common name for 1,2-benzenediol is which of these?
A) Catechol
B) m -Cresol
C) Resorcinol
D) Hydroquinone
E) o-Xylenol
Ans: C
Topic: Nomenclature
4. What is the systematic (IUPAC) name for
OH
OH
A)
B)
C)
D)
E)
Ans:
m-Hydroxyphenol
m-Dihydroxybenzene
Resorcinol
1,3-dihydroxybenzene
1,3-Benzenediol
E
784
Chapter 21
Topic: Nomenclature
5. What is the systematic (IUPAC) name for
OH
A)
B)
C)
D)
E)
Ans:
p-Hydroxyphenol
p-Dihydroxybenzene
Resorcinol
1,4-dihydroxybenzene
1,4-Benzenediol
E
Topic: Nomenclature
6. What is the systematic (IUPAC) name for
OH
NO2
A)
B)
C)
D)
E)
Ans:
m-Hydroxy-p-nitrotoluene
3-Hydroxy-4-nitrotoluene
5-methyl-2-nitrophenol
3-methyl-2-nitrophenol
2-Hydroxy-4methylnitrobenzene
C
Topic: Nomenclature
7. What is the systematic (IUPAC) name for
OH
Br
O
A)
B)
C)
D)
E)
Ans:
p-Hydroxy-m-bromoacetophenone
m-bromo-p-hydroxypropiophenone
3-bromo-4-hydroxypropiophenone
2-bromo-4-propanoylphenol
2-hydroxy-5-propanoylbromobenzene
D
785
Chapter 21
Topic: Nomenclature
8. Which of the following is 5-bromo-3-methyl-1-naphthol?
OH
OH
OH
Br
Br
Br
I
II
III
OH
HO
Br
Br
IV
A)
B)
C)
D)
E)
Ans:
V
I
II
III
IV
V
B
Topic: Acid/Base
9. Which of the following phenols would have the largest pKa?
F
HO
O2 N
OH
CH3
OH
I
II
III
OH
OH
Cl
IV
A)
B)
C)
D)
E)
Ans:
V
I
II
III
IV
V
C
786
Chapter 21
Topic: Acid/Base
10. Which of the following phenols would have the smallest pKa?
NO2
O
HO
O
OH
OH
I
O
II
III
O
NH2
OH
HO
IV
A)
B)
C)
D)
E)
Ans:
V
I
II
III
IV
V
C
Topic: Acid/Base
11. Which of the following substances would have the smallest pKa?
NO2
O
HO
O
OH
OH
I
O
III
II
NH2
OH
OH
HO
IV
A)
B)
C)
D)
E)
Ans:
O
V
I
II
III
IV
V
E
787
Chapter 21
Topic: Acid/Base
12. Which of the following compounds would you expect to be the strongest acid?
A) CH3OH
B) C6H5CH2OH
C) p-CH3C6H4OH
D) C6H5OH
E) p-NO2C6H4OH
Ans: E
Topic: Acid/Base
13. Which compound would be most acidic?
A) Cyclohexanol
B) 1-Hexanol
C) Phenol
D) 4-Methylphenol
E) 4-Chlorophenol
Ans: E
Topic: Acid/Base
14. Which of the following would be the strongest acid?
HO
HO
CH3
OH
I
III
II
OH
OH
F3C
Br
IV
A)
B)
C)
D)
E)
Ans:
V
I
II
III
IV
V
D
788
Chapter 21
Topic: Acid/Base
15. Which of the following would be the strongest acid?
O
Br
OH
OH
OH
I
O
III
II
SO3H
OH
OH
HO
IV
A)
B)
C)
D)
E)
Ans:
V
I
II
III
IV
V
D
Topic: Acid/Base
16. Which would be soluble in aqueous sodium bicarbonate?
A) Benzoic acid
B) 4-Methylphenol
C) 2,4-Dinitrophenol
D) More than one of these
E) All of these
Ans: D
Topic: Acid/Base
17. Which reagent will distinguish between C6H5OH and C6H5CH2OH?
A) NaHCO3 (aq)
B) NaOH (aq)
C) H2SO4
D) A) and B)
E) B) and C)
Ans: B
789
Chapter 21
Topic: Acid/Base
18. Which reagent will distinguish between C6H5OCH3 and p-CH3C6H4OH?
A) NaOH (aq)
B) NaHCO3 (aq)
C) H2CrO4
D) A) and B)
E) None of these
Ans: A
Topic: Acid/Base
19. Which compound reacts most rapidly with CH3ONa?
Br
Br
NO2
Br
NO2
I
III
II
NO2
Br
Br
O2 N
O2N
IV
A)
B)
C)
D)
E)
Ans:
NO2
V
I
II
III
IV
V
D
790
Chapter 21
Topic: Acid/Base
20. Which of these species is the strongest base?
O
O
O
H3C
I
Cl
II
III
O
O
O2N
I
IV
A)
B)
C)
D)
E)
Ans:
V
I
II
III
IV
V
B
Topic: Reaction Products
21. Indicate the correct product, if any, of the following reaction.
HO
HBr
Br
Br
HO
HO
Br
Br
HO
Br
I
A)
B)
C)
D)
E)
Ans:
II
III
I
II
III
IV
There is no net reaction.
E
791
Br
Br
IV
Chapter 21
Topic: Reaction Products
22. Refluxing anisole, CH3OC6H5, with excess concentrated HBr would yield which of
these product mixtures?
A) C6H5Br + CH3OH
B) C6H5OH + CH4
C) C6H5OH + CH3OH
D) C6H5Br + CH3Br
E) C6H5OH + CH3Br
Ans: E
Topic: Reaction Products
23. What product(s) would you expect from the following reaction?
Cl
F3C
NaNH2
NH3(l), -33oC
NH2
alone
I
A)
B)
C)
D)
E)
Ans:
F3C
NH2
F3C
F3C
alone
NH2
III
II
I
II
III
Substantial amounts of I and II
Substantial amounts of I, II, and III
B
792
alone
Chapter 21
Topic: Reaction Products
24. What product(s) would you expect from the following reaction?
Br
i. NaOH, NaHCO3,
?
H2O, heat
NO2
O2N
ii. H3O
+
Br
HO
NaO
NO2
O2N
I
A)
B)
C)
D)
E)
Ans:
NO2
O2N
HO
NH2
III
II
I
II
III
Substantial amounts of I and II
Substantial amounts of I, II, and III
A
Topic: Reaction Products
25. What would be the major product of the following reaction?
HO
Br2 (excess)
?
H2O
Br
Br
Br
HO
HO
HO
Br
Br
I
A)
B)
C)
D)
E)
Ans:
II
HO
III
I
II
III
IV
A mixture of I and II
D
793
Br
IV
Chapter 21
Topic: Reaction Products
26. What products would you expect from the following reaction?
i. OH−/ H2O, 400oC
?
4-Chlorotoluene
ii. H3O+
A)
B)
C)
D)
E)
Ans:
2-Methylphenol
3-Methylphenol
4-Methylphenol
A) and B)
B) and C)
E
Topic: Reaction Products
27. What is the major product of the following reaction?
i) conc H2SO4,100oC
Phenol
?
ii) HNO3 / H2SO4, heat
iii) H3O+/H2O, heat, steam distill
A)
B)
C)
D)
E)
Ans:
p-Hydroxybenzenesulfonic acid
p-Nitrophenol
o-Nitrophenol
4-Hydroxy-3-nitrobenzenesulfonic acid
2-Hydroxy-4-nitrobenzenesulfonic acid
C
Topic: Reaction Products
28. Which products would be formed in the following reaction?
OCH3
H3C
A)
B)
C)
D)
E)
Ans:
conc HBr (excess)
heat
?
Methoxybenzene + methyl bromide
2-Methylphenol + methyl bromide
2-Bromotoluene + methanol
2-Bromotoluene + methyl bromide
Bromobenzene + methyl bromide
B
794
Chapter 21
Topic: Reaction Products
29. What is the final product?
Phenol t-butyl chloride
AlCl3
i. NaOH
C10H14O
(para-isomer) ii. CH3CH2I
A) 1-tert-Butyl-4-ethoxybenzene
B) 1-tert-Butyl-4-ethylbenzene
C) 1-tert-Butoxy-4-ethoxybenzene
D) tert-Butyl ethyl ether
E) 1-tert-Butoxy-3-ethylbenzene
Ans: A
?
Topic: Reaction Products
30. What products(s) would you expect from the following reaction?
Cl
H3C
NaNH2
NH3(l), -33oC
?
NH2
H3C
NH2
H3C
alone
I
A)
B)
C)
D)
E)
Ans:
alone
NH2
III
II
I
II
III
Products I and II
All of the above
D
Topic: Reaction Products
31. What is the product of the following synthesis?
1. H2SO4, 100oC
phenol
?
2. 2 equiv Br2, H2O
3. dil. H2SO4, heat
A)
B)
C)
D)
E)
Ans:
H3C
2,3-Dibromophenol
2,4-Dibromophenol
2,6-Dibromophenol
2-Hydroxy-3,5-dibromobenzenesulfonic acid
2,4,6-Tribromophenol
C
795
alone
Chapter 21
Topic: Reaction Products
32. What is the IUPAC name of the final product obtained via the following reaction
sequence?
1. 2-bromopropane, AlCl3
2-bromophenol
?
2. NaOH
3. (CH3CO)2O
A) 1-isopropyl-3-bromo-4-acetylbenzene
B) 2-bromo-4-isopropyl -acetophenone
C) 3-bromo-1-isopropyl-4-phenyl acetate
D) (2-bromo-4-isopropyl)phenyl acetate
E) None of the above correctly describes the product
Ans: D
Topic: Reaction Products
33. What is the IUPAC name of the final product obtained via the following reaction
sequence?
2-bromophenol 1. CH3I, base
?
2. Butanoyl chloride, AlCl3
3. Zn(Hg), HCl
4. conc HBr, heat
A) 1-butanoyl-3-bromo-4-methoxybenzene
B) 2-bromo-4-butylphenol
C) 3-bromo-1-butyl-4-methoxybenzene
D) 3,4-dibromo-1- butanoylbenzene
E) None of the above correctly describes the product
Ans: B
Topic: Reaction Products
34. What is the IUPAC name of the final product obtained via the following reaction
sequence?
1. H2, Ni
2-nitrophenol
?
2. NaNO2, HCl, 0-5oC
3. KI
4. Br2, 5oC, CS2
A) 4-bromo-2-iodophenol
B) 2-bromo-4-iodophenol
C) 2,4-dibromophenol
D) 2,4-dibromo-6- iodophenol
E) 1-bromo-3-iodobenzene
Ans: A
796
Chapter 21
Topic: Reaction Products
35. What is the final product obtained via the following reaction sequence?
1. NBS
1-ethyl-4-methoxybenzene
?
2. Mg, Et2O
3. (CH3)2C=O
4. H3O+
5. conc HBr, heat
Br
OH
H3CO
HO
OCH3
OCH3
I
II
III
Br
OH
HO
IV
A)
B)
C)
D)
E)
Ans:
V
I
II
III
IV
V
E
797
Chapter 21
Topic: Reaction Products
36. What product should be obtained if benzyne is generated in the presence of 1,3butadiene?
I
II
III
IV
A)
B)
C)
D)
E)
Ans:
V
I
II
III
IV
V
A
Topic: Reaction Products
37. Which product is likely to be obtained during the following reaction of the isotopelabeled allyl ether (14C-isotopic site marked with an asterisk)?
O
200oC
*
O
OH
*
OH
*
I
III
II
OH
OH
*
*
IV
A)
B)
C)
D)
E)
Ans:
?
V
I
II
III
IV
V
B
798
*
Chapter 21
Topic: Reaction Products
38. When sodium phenoxide is heated to 125°C with carbon dioxide under pressure and the
product mixture acidified, which of these is produced?
O
OH
O
O
OH O
HO
OH
I
II
O
III
OH O
OH
OH
HO
IV
A)
B)
C)
D)
E)
Ans:
OH
V
I
II
III
IV
V
B
799
Chapter 21
Topic: Reaction Products
39. What product is likely to be obtained by the action of Ag+ or Fe+3 on the following
substance?
OH
CH3
Ag+ or Fe3+
?
OH
O
OH O
CH3
O
O
O
O
OH
O
I
II
III
O
OH
CH3
HO H
IV
A)
B)
C)
D)
E)
Ans:
O
CH3
O
V
I
II
III
IV
V
A
800
Chapter 21
Topic: Reaction Products
40. What is the product of the reaction of 1 mol of p-benzoquinone with 1 mol of isoprene
(2-methyl-1,3-butadiene)?
O
O
O
O
I
O
III
II
O
O
O
O
IV
A)
B)
C)
D)
E)
Ans:
O
V
I
II
III
IV
V
A
801
Chapter 21
Topic: Reaction Products
41. What are the products of the reaction of phenol with propanoic anhydride in the
presence of base?
O
O
O
O
O
+
+
CH3CH2CO2
−
O
+
CH3CH2CO2−
I
CH3CH2CO2−
II
III
O
O
+
CH3CH2CO2−
IV
A)
B)
C)
D)
E)
Ans:
+
CH3CH2CH2OH
V
I
II
III
IV
V
C
802
Chapter 21
Topic: Reaction Products
42. What is the product of the reaction of phenol and chloroacetic acid in basic solution,
followed by acidification?
O
Cl
O
OH O
OH
Cl
O
O
I
II
OH
O
III
OH
OH
O
O
IV
A)
B)
C)
D)
E)
Ans:
V
I
II
III
IV
V
D
803
Chapter 21
Topic: Reaction Products
43. Predict the product of this Cope rearrangement:
o
C6H5
180 C
?
C6H5
C6H5
C6H5
I
II
C6H5
III
C6H5
IV
A)
B)
C)
D)
E)
Ans:
V
I
II
III
IV
V
C
Topic: Reaction Products
44. What product is likely to be obtained when 1-bromo-2-nitrobenzene is heated with
aqueous NaHCO3?
A) 2-bromophenol
B) 2-nitrophenol
C) 2-bromophenyl carbonate
D) 2-nitrophenyl carbonate
E) Sodium 2-nitrophenoxide
Ans: E
Topic: Reaction Products
45. What product is likely to be obtained when 1-chloro-3-nitrobenzene is heated with
aqueous NaHCO3?
A) 3-chlorophenol
B) 3-nitrophenol
C) 3-chlorophenyl carbonate
D) Sodium 2-nitrophenoxide
E) None of the above: no reaction is likely to occur
Ans: E
804
Chapter 21
Topic: Reaction Products
46. What product is likely to be obtained when 1-fluoro-2,4-dinitrobenzene is treated with
sodium ethoxide?
A) 2,4-dinitrophenol
B) 1-ethoxy-2,4-dinitrobenzene
C) 2,4-diethoxy-1-fluorobenzene
D) 2,4-dinitrophenylsodium
E) None of the above: no reaction is likely to occur
Ans: E
Topic: Reaction Products
47. What product is likely to be obtained when 1-chloro-2-trifluoromethylbenzene is treated
with sodium amide in liquid ammonia?
A) 2-(trifluoromethyl)aniline
B) 3-(trifluoromethyl)aniline
C) 4-(trifluoromethyl)aniline
D) 2-chloroaniline
E) None of the above: no reaction is likely to occur
Ans: B
Topic: Synthesis
48. Which of the following would provide a synthesis of aspirin,
o-CH3COOC6H4COOH ?
A) C6H5COOH, CH3COOH, AlCl3, heat; then H2O
B) CH3COOC6H5, CO2, heat; then H3O+
C) CH3COOC6H5, HCOOC2H5, C2H5O-; then H3O+; then OHD) C6H5OH, CO2, H3O+; separate isomers; then CH3COOH, AlCl3
E) C6H5OH, OH-, CO2, heat, pressure; then H3O+; then (CH3CO)2O
Ans: E
Topic: Synthesis
49. Identify the commercial preparation(s) of phenol.
A) Chlorobenzene + NaOH at 350°C; then H3O+
B) Benzene + O2
C) C6H5CH(CH3)2 + O2; then H3O+, heat
D) A) and B)
E) A) and C)
Ans: E
805
Chapter 21
Topic: Synthesis
50. Which method could be used for preparing methoxybenzene from phenol?
A) NaOH, then CH3I
B) NaOH, then CH3OSO2CH3
C) NaOH, then CH3OCH3
D) More than one of these
E) All of these
Ans: D
Topic: Synthesis
51. Which of these is an acceptable synthesis of phenetole (ethyl phenyl ether)?
A)
CH3CH2ONa
Chlorobenzene
100oC
B)
C)
Sodium phenoxide
Sodium phenoxide
CH3CH2I
70oC
(CH3CH2O)2SO2
o
100 C
D)
Phenol
CH3CH2OH, H2SO4
70oC
E) More than one of these
Ans: E
Topic: Synthesis
52. Which of the following reactions would yield p-tert-butylphenol?
A)
H3O+
Phenol +
Phenol +
OH
H3O+
Phenol +
Cl
AlCl3
B)
C)
D) All of these
E) None of these
Ans: D
806
Chapter 21
Topic: Synthesis
53. Which of these is a satisfactory method for the preparation of m-nitrophenol from
benzene?
A) i) HNO3 / H2SO4; ii) Sn / HCl; iii) NaNO2, H2SO4, H2O, 0-5oC;
iv) Cu2O, Cu2+, H2O; v) HNO3
iv) H3O+
B)
i) Cl2, Fe; ii) HNO3 / H2SO4; iii) NaOH, H2O;
C)
i) HNO3/H2SO4; ii) Cl2, Fe; iii) NaOH, H2O;
D)
i) 2 HNO3 / 2 H2SO4; ii) NH3, H2S; iii) NaNO2, H2SO4, H2O, 0-5oC;
iv) Cu2O, Cu2+, H2O
E)
i) HNO3 / H2SO4; ii) NaOH / fusion; iii) H3O+
Ans: D
Topic: Synthesis
54. Which of these reactions does not produce phenol?
A)
i. NaOH, H2O
O
O
ii. H3O+
H3O+
B)
ONa
C)
O
HI
D)
Cl
H2O
NH2
100oC
i. NaNO2, H2SO4, H2O, 0-5oC
E)
ii. Cu2O, Cu2+, H2O
Ans: D
807
iv) H3O+
Chapter 21
Topic: Relative Reactivities and General Considerations
55. Which of the following would be most likely to undergo a nucleophilic substitution
reaction with aqueous sodium hydroxide by an addition-elimination mechanism?
Br
Br
Br
O
O
I
II
III
Br
NO2
Br
NO2
IV
A)
B)
C)
D)
E)
Ans:
O2N
NO2
V
I
II
III
IV
V
D
Topic: Relative Reactivities and General Considerations
56. In terms of reactivity towards nucleophiles, bromobenzene is most similar to which of
these?
A) Allyl bromide
B) Vinyl bromide
C) tert-Butyl bromide
D) Propyl bromide
E) Methyl bromide
Ans: B
Topic: Relative Reactivities and General Considerations
57. In nucleophilic aromatic substitution, the attacking species (the nucleophile) necessarily
is:
A) A Lewis base
B) Neutral
C) Positively charged
D) Negatively charged
E) Electron deficient
Ans: A
808
Chapter 21
Topic: Relative Reactivities and General Considerations
58. Which of the following substances readily undergoes ring bromination with Br2, even in
the absence of a Lewis acid catalyst?
A) 2,4-Dinitrotoluene
B) 2,4-Dimethylbenzoic acid
C) 3-Nitroaniline
D) 4-Bromophenyl acetate
E) 1,3-benzenediol
Ans: D
Topic: Relative Reactivities and General Considerations
59. Which is the leaving group when the following substance reacts with sodium cyanide in
DMSO solution?
I
Br
VH
Br
II
H
III
NO2
A)
B)
C)
D)
E)
Ans:
IV
I
II
III
IV
V
A
Topic: Relative Reactivities and General Considerations
60. Which position is predicted to be the chief site of substitution when the following
substance reacts with bromine in carbon disulfide at 10°C ?
II
HO
A)
B)
C)
D)
E)
Ans:
III
IV
V
I
CH3
I
II
III
IV
V
A
809
Chapter 21
Topic: Relative Reactivities and General Considerations
61. Which of these resonance structures makes the greatest contribution to the hybrid for the
intermediate in the SNAr reaction of o-chloronitrobenzene with methoxide ion?
H3CO
Cl
O
H3CO
Cl
O
H3CO
O
O
II
H3CO
Cl
O
N
O
I
III
O
H3CO
Cl
N
O
N
O
O
IV
A)
B)
C)
D)
E)
Ans:
Cl
N
N
V
I
II
III
IV
V
E
Topic: Relative Reactivities and General Considerations
62. The formation of equal amounts of m-toluidine (m-aminophenol) and p-toluidine in the
reaction of p-bromotoluene with sodium amide in liquid ammonia at -33°C suggests this
species as the reaction intermediate:
Br
NH2
Br
NH2
II
I
IV
A)
B)
C)
D)
E)
Ans:
III
V
I
II
III
IV
V
D
810
Chapter 21
Topic: Relative Reactivities and General Considerations
63. Phenol is less reactive than expected in Friedel-Crafts acylations for which of these
reasons?
A) The acid chloride or anhydride hydrolyzes under the reaction conditions.
B) The electronegativity of the -OH group reduces the reactivity of phenol.
C) Interaction of the -OH group with AlCl3 forms a species less reactive than phenol.
D) The reaction gives O-acylation only, resulting in an ester.
E) The aromatic ring complexes with AlCl3 with a resulting decrease in phenol
reactivity.
Ans: C
Topic: Spectroscopic Analysis
64. Thymol, isolated from thyme, has the following structure:
OH
Which of the following types of signal is NOT expected to be observed in its NMR
spectrum?
A) singlet
B) doublet
C) triplet
D) septet
E) all of the above types of signals are expected to be observed
Ans: C
Topic: Spectroscopic Analysis
65. Methyl salicylate, commonly found in topical muscle relaxants, has the following
structure:
OH
O
O
Which of the following signals is NOT expected to be observed in its NMR spectrum?
A) Singlet at ~4 ppm (3H)
B) Doublet at ~7 ppm (1H)
C) Quartet at about 7 ppm (4H)
D) Singlet at ~ 6 ppm (1H)
E) Two of the above types of signals are not expected to be observed
Ans: C
811
Chapter 21
Topic: Spectroscopic Analysis
66. Eugenol, isolated from cloves, has the following structure:
OH
O
Which of the following signals is NOT expected to be observed in its NMR spectrum?
A) Singlet at ~4 ppm (3H)
B) Doublet at ~2.5 ppm (2H)
C) Singlet at ~ 7 ppm (1H)
D) Singlet at ~ 6 ppm (1H)
E) All of the above types of signals are expected to be observed
Ans: E
Topic: Structure Identification
67. Compound L has the molecular formula C11H16O. L is insoluble in water but dissolves
in aqueous NaOH. The infrared spectrum of L shows a broad absorption band in the
3200-3600 cm-1 region; its 1H NMR spectrum consists of:
triplet,
singlet,
quartet,
singlet,
multiplet,
δ 0.80
δ 1.2
δ 1.5
δ 4.5
δ 7.0
(6H)
(1H)
(4H)
The most likely structure for compound L is:
HO
HO
II
I
O
III
HO
IV
A)
B)
C)
D)
E)
Ans:
O
V
I
II
III
IV
V
E
812
Chapter 21
SHORT ANSWER QUESTIONS
Topic: Synthesis
68. The most important laboratory synthesis of phenols is by _______________.
Ans: hydrolysis of arenediazonium salts
Topic: Claisen, Cope Rearrangement
69. The Claisen and Cope rearrangements are two examples of a general class of reactions
called ___________.
Ans: pericyclic reactions
Topic: General
70. The carbon-halogen bonds in aryl and vinyl halides are both __________ and
_____________ than the corresponding bonds in alkyl halides.
Ans: shorter; stronger
Topic: General, Nucleophilic Aromatic Substitution
71. Nucleophilic aromatic substitution reactions (SNAr) take place via a delocalized
carbanion intermediate called a ______________.
Ans: Meisenheimer complex
Topic: General, Nucleophilic Aromatic Substitution
72. Nucleophilic aromatic substitution reactions (SNAr) require three components: a good
leaving group on the aromatic ring, a strong nucleophile, and ________________.
Ans: an electron-withdrawing substituent ortho or para to the leaving group
Topic: General, Nucleophilic Aromatic Substitution
73. In general, there are two types of nucleophilic aromatic substitution mechanisms. These
are: ______________.
Ans: addition-elimination or SNAr; elimination-addition or benzyne
Topic: Benzyne Intermediate
74. Benzyne is a very unstable intermediate, because _____________.
Ans: the two carbons involved in the extra π bond are sp hybridized, causing a severe
amount of bond angle distortion and strain
Topic: Nomenclature
75. The commonly used name for hydroxybenzene is __________.
Ans: phenol
Topic: Industrial Synthesis
76. Most of the worldwide industrial preparation of phenol is now based on the
___________.
Ans: cumene hydroperoxide method
813
Chapter 21
Topic: Industrial Synthesis
77. The cumene hydroperoxide synthesis of phenol is industrially very satisfying, because it
takes two inexpensive starting materials, ___________ and ___________, and converts
them into two valuable products, phenol and __________.
Ans: benzene; propene; acetone
Topic: Nomenclature
78. Draw the structure corresponding to the following IUPAC name:
(2E,4S)-4-(3-hydroxyphenyl)-2-pentenoic acid
Ans: H3C H
OH
O
OH
Topic: Nomenclature
79. Draw the structure corresponding to the following IUPAC name:
(R)-2-Bromo-5-(3-methylpentyl)phenol
H CH3
Ans:
Br
OH
Topic: Nomenclature
80. Draw the structure corresponding to the following IUPAC name:
4-benzyl-3,5-diiodophenol
I
Ans:
HO
I
Topic: Nomenclature
81. Give the IUPAC name of the following substance:
H3C
H
O
HO
O
Ans: (R)-methyl 6-(4-hydroxyphenyl)-4-methylhexanoate
814
Chapter 21
Topic: Relative Acidity
82. Draw the structure of the ionic species obtained when the following substance is treated
with NaOH:
NH2
OH
HO
Ans:
−
NH2
O Na
+
HO
Topic: Chemical Analysis, Relative Acidity
83. Suggest a simple chemical test to distinguish between 4-methylphenol and 2,4,6trinitrophenol, briefly explaining your rationale.
Ans: Both substances will dissolve in aq NaOH via an acid-base reaction. However,
only 2,4,6-trinitrophenol, being significantly more strongly acidic, is likely to
react with aq NaHCO3, releasing gaseous CO2 (easily observed as effervescence).
The differences in reactivity can be attributed to the fact that nitro groups are
strongly electron-withdrawing, while the methyl group is weakly electrondonating.
NO2
OH
OH
O2N
NO2
stronger acid
gives CO2 with NaHCO3(aq)
weaker acid
no reaction with NaHCO3(aq)
Topic: Reaction Product
84. Draw the structure of the product obtained when benzoquinone reacts with 2 molar
equivalents of 1,3-butadiene via a Diels Alder reaction.
O
Ans: O
Diels Alder
+
Reaction
O
1 mol
O
2 mol
815
Chapter 21
Topic: Reaction Product
85. Draw the structure of the product obtained when the following substance is heated
strongly:
O
O
Ans:
OH
heat
(Claisen Rearrangement)
Topic: Reaction Product
86. Draw the structure of the final product obtained when 2-nitrophenol reacts with acetic
anhydride in presence of base.
O O
Ans:
NO2
NO2
OH
O
O
O
base
Topic: Reaction Product
87. Draw the structure of the final product obtained when 1-chloro-2,4-dinitrobenzene is
heated with aqueous NaHCO3 at 100oC, followed by neutralization with H3O+ .
NO2
NO2
Ans:
OH
Cl i. NaHCO3 (aq), 100oC
+
ii. H3O
O N
O N
2
2
Topic: Reaction Product
88. Draw the structure of the final product obtained when 3-methylphenol is subjected to
the following reaction sequence:
i) NaOH; ii) CH3CH2CH2OSO2OCF3
Ans:
OH
O
i) NaOH
ii) CH3CH2CH2OSO2OCF3
Topic: Reaction Sequence
89. Draw the structure of the final product obtained when 4-cyclopentylphenol is subjected
to the following reaction sequence:
i) NaOH; ii) CH3I; iii) NBS; iv) CH3ONa/CH3OH, heat
OH
O
Ans:
i) NaOH
ii) CH3I
iii) NBS
iv) CH3ONa, CH3OH, heat
816
Chapter 21
Topic: Synthesis
90. The insecticide Sevin is synthesized from 1-naphthol and methyl isocyanate
(CH3-N=C=O). Draw the structure of this insecticide.
O
Ans:
OH
N
H
O
+ CH3-N=C=O
Sevin
Topic: Synthesis
91. Orange II is an azo dye made from 2-naphthol and the diazonium salt of paminobenzenesulfonic acid. Indicate the synthetic steps involved in this synthesis and
draw the structure of Orange II.
NaNO2
Ans:
SO3H
0-5oC
H2N
SO3H
H2SO4
−
Cl+N2
OH
N
OH
N
SO3H
OrangeII
Topic: Synthesis
92. Draw the structure of the product likely to be obtained from the reaction between
acetoacetic ester, bromobenzene and 2 molar equivalents of sodium amide (in liquid
ammonia)? Briefly explain your rationale.
Ans: This is likely to be a benzyne mediated (elimination-addition mechanism)
nucleophilic substitution reaction of bromobenzene. 1 equivalent of sodium
amide is likely to promote elimination to form benzyne, while the 2nd equivalent
of base would generate the anion of acetoacetic ester, which can then react with
the benzyne to give the following product.
O
O
O
817
Chapter 21
Topic: Synthesis
93. Outline the steps involved in the synthesis of 3-bromo-4-methylphenol from 4nitrotoluene
Ans: i. Br2, FeBr3 ; ii) Fe, HCl; iii) NaNO2, H2SO4, 0-5oC; iv) Cu2O, Cu2+, H2O
Br
NO2
OH
3-bromo-4-methylphenol
4-nitrotoluene
Br
NO2
Br
Br2
FeBr3
NO2
Fe
HCl
NH2
4-nitrotoluene
i. NaNO2, H2SO4
0-5oC
ii. Cu2O, Cu2+,
H2O
Br
OH
3-bromo-4-methylphenol
818
Chapter 21
Topic: Industrial Synthesis
94. A key step in the industrial synthesis of phenol from cumene is the oxidation of cumene
to cumene hydroperoxide with oxygen. Outline the mechanistic steps that lead to this
oxidation product.
O2
O OH
95-135oC
cumene hydroperoxide
cumene
Ans: This is a radical chain reaction. A radical initiator is required to initiate the
process, which begins with abstraction of the benzylic hydrogen to produce a 3o
radical. The ensuing chain reaction leads to the formation of cumene
hydroperoxide.
Initiation
Step 1
H + R.
.
+ R−H
cumene
Propagation
Step 2
+ . O-O .
.
O
O.
Step 3
O
O. +
H
O
OH +
cumene hydroperoxide
819
.
Chapter 21
Topic: Industrial Synthesis
95. The industrial synthesis of phenol from cumene involves the acid hydrolysis of the key
intermediate, cumene hydroperoxide (formed from cumene via oxidation). Outline the
mechanistic steps that lead to this oxidation product. Which other industrially
important substance is produced as a by-product in this reaction?
H3O+
O OH
OH
50-90oC
cumene hydroperoxide
phenol
Ans: This is a hydrolytic rearrangement reaction. Protonation of the –OH group of
cumene hydroperoxide, leads to a 1,2-phenyl migration with concomitant loss of
water. Nucleophilic addition to the resultant carbocation ultimately leads to the
formation of phenol. Another industrially important substance, acetone, is
formed as a by-product during this reaction, making this a doubly attractive
synthetic strategy.
H3O+
O OH
OH + O
50-90oC
cumene hydroperoxide
O OH
phenol
H3O+
O OH2
+
- H2O
acetone
+
cumene hydroperoxide
O
- H3O+
+ HO
acetone
O
H2O
HO
H
O
+
phenol
820
+
H2O
O
Chapter 21
Topic: Synthesis
96. Suggest a reasonable synthetic strategy for the synthesis of 3-bromophenol from
acetophenone
Ans: i) Br2, FeBr3; ii) MCPBA; iii) H3O+
Br
OH
O
acetophenone
3-bromophenol
Br
O
Br2
FeBr3
Br
O
MCPBA
O
O
H3O+
heat
acetophenone
Br
OH
3-bromophenol
821
Chapter 21
Topic: Synthesis, Separation of (o-, p-) Products
97. When phenol is nitrated with dilute nitric acid, a mixture of o- and p- substituted
products is obtained. These products can then be separated by the technique of steam
distillation. However, analogous products from the nitration of methoxybenzene cannot
be similarly separated by steam distillation. Explain.
Ans: In order for efficient separation of substances (from a mixture) by steam
distillation, there must be relatively large differences in volatility. The more
volatile substance passes over with the steam, leaving the less volatile substance
in the distillation flask.
Differences in the volatility of the o- and p- nitrophenols may be predicted by
considering hydrogen bonding effects: o-nitrophenol can exhibit intramolecular
hydrogen bonding, while p-nitrophenol can exhibit intermolecular hydrogen
bonding. Consequently, o-nitrophenol is likely to be significantly more volatile
than p-nitrophenol, leading to the possibility of efficient separation by steam
distillation.
By contrast, since no hydrogen bonding is possible in the products obtained by
nitration of methoxybenzene, there is no predictable difference in volatility.
Hence, these products cannot be separated by steam distillation.
O
O
N
H
O
O
O
N
O
N
H
O
O
o-nitrophenol
intramolecular hydrogen bonding
more volatile
OCH3
methoxybenzene
O
H
p-nitrophenol
intermolecular hydrogen bonding
less volatile
OCH3
+
O2N
OCH3
NO2
no hydrogen bonding in either product:
volatility differences not likely to be significant
822
Chapter 21
Topic: Synthesis
98. Outline the steps involved in the synthesis of 3-nitrophenol from 1,3-dinitrobenzene
Ans: i) NH3, H2S, EtOH; ii) NaNO2, H2SO4, 0-5oC; iii) Cu2O, Cu2+, H2O
O2N
O2N
NO2
OH
1,3-dinitrobenzene
O2 N
3-nitrophenol
O2N
NO2
o
NH3
H 2S
EtOH
NH2
O2N
i) NaNO2, H2SO4, 0-5 C
OH
ii) Cu2O, Cu2+, H2O
Topic: Synthesis, Hofmann/Curtius Rearrangement
99. Outline the steps involved in the synthesis of 3-nitrophenol from benzoic acid
Ans: i) HNO3, H2SO4 ; ii) SOCl2; iii) NH3; iv) Br2, NaOH; v) NaNO2, H2SO4,
0-5oC; vi) Cu2O, Cu2+, H2O
or
i) HNO3, H2SO4 ; ii) SOCl2; iii) NaN3; iv) heat; v) H2O; vi) NaNO2, H2SO4,
0-5oC; vii) Cu2O, Cu2+, H2O
O2N
CO2H
OH
3-nitrophenol
benzoic acid
O
O
OH
HNO3
H2SO4
O
OH
O
Cl
SOCl2
NO2
NH2
NH3
NO2
NO2
i) NaN3
ii) heat
iii) H2O
Br2
NaOH
OH
NH2
o
i) NaNO2, H2SO4, 0-5 C
NO2
ii) Cu2O, Cu2+, H2O
3-nitrophenol
823
NO2
Chapter 21
Topic: Synthesis, Grignard Synthesis
100. Outline the steps involved in the synthesis of p-methoxybenzoic acid from phenol.
Ans: i) NaOH ; ii) CH3I; iii) Br2, FeBr3 ; iv) Mg, Et2O; v) CO2; vi) H3O+
OH
OH
O
O
p-methoxybenzoic acid
phenol
OH
O
i) NaOH
ii) CH3I
O
O
i) Mg, Et2O
Br2
FeBr3
ii) CO2
Br
iii) H3O+
(+ o- isomer)
824
O
OH
Chapter 22
MULTIPLE CHOICE QUESTIONS
Topic: Nomenclature, Definitions and Types of Carbohydrates
1. An aldaric acid is represented by:
HO
HO
H
H
CHO
H
H
OH
OH
CH2OH
HO
HO
H
H
I
A)
B)
C)
D)
E)
Ans:
CO2H
H
H
OH
OH
CH2OH
HO
HO
H
H
II
CO2H
H
H
OH
OH
CO2H
HO
HO
H
H
III
CH2OH
H
H
OH
OH
CH2OH
HO
HO
H
H
IV
CHO
H
H
OH
OH
CO2H
V
I
II
III
IV
V
C
Topic: Nomenclature, Definitions and Types of Carbohydrates
2. Epimers are represented by:
CH2OH
O
HO
H
H
H
OH
OH
CH2OH
H
HO
H
H
I
A)
B)
C)
D)
E)
Ans:
CHO
OH
H
OH
OH
CH2OH
II
HO
HO
H
H
CHO
H
H
OH
OH
CH2OH
III
I and II
II and III
I, II, and III
III and IV
I, II, and V
B
825
H
H
HO
HO
CHO
OH
OH
H
H
CH2OH
IV
HO
HO
HO
HO
CHO
H
H
H
H
CH2OH
V
Chapter 22
Topic: Nomenclature, Definitions and Types of Carbohydrates
3. Which is a ketohexose?
A) D-Glucose
B) D-Fructose
C) D-Mannose
D) D-Ribose
E) (+)-Sucrose
Ans: B
Topic: Nomenclature, Definitions and Types of Carbohydrates
4. Which of these compounds, I, II, III, IV, is a reducing disaccharide?
H
OH
CH2OH
O
H
OH H
H
CH2OH
H H
O
OH
O
H
OH H
H
H
H
OH
OH
OH
CH2OH
O
H
OH H
H
H
OH
H
H H
O
OH
H
OH
II
CH2OH
O
H
H
OH H
OCH3
H
H
OH
OH
CH2OH
O
H
OH H
H
III
A)
B)
C)
D)
E)
Ans:
O
OH
I
CH2OH
O
H
OH H
H H
CH2OH
O
H
H
OH H
OH
OH
O
OH
826
H
OH H
H
IV
I alone
II alone
III alone
IV alone
I, II, III, and IV
B
O
H
OH
CH2OH
Chapter 22
Topic: Nomenclature, Definitions and Types of Carbohydrates
5. Sugars that undergo mutarotation in neutral aqueous solution are:
CH2OH
O
H
H
H
OH H
OH
OH
H
CH2OH
O OH
H
H
OH H
H
OH
OH
H
I
H
OH
OH
H
II
CH2OH
O
H
OH H
H
CH2OH
O
H
H
H
OH H
OCH3
OH
CH2OH
H H
O
O
H
OH H
OH
H
H
H
OH
OH
OH
CH2OH
O
H
OH H
H
H H
O
CH2OH
O
H
H
OH H
OH
OH
IV
A)
B)
C)
D)
E)
Ans:
OH
III
H
OH
V
I and III
III and IV
II, III, and IV
I and IV
I, II, and V
E
Topic: Nomenclature, Definitions and Types of Carbohydrates
6. Which of the following is an L-aldotetrose?
CHO
H
OH
H
OH
CH2OH
I
A)
B)
C)
D)
E)
Ans:
CHO
HO
H
H
OH
CH2OH
CHO
H
OH
HO
H
CH2OH
CO2H
H
OH
HO
H
CH2OH
CH2OH
H
OH
HO
H
CO2H
II
III
IV
V
I
II
III
IV
V
C
827
Chapter 22
Topic: Nomenclature, Definitions and Types of Carbohydrates
7. Which of the following is a D-aldotetrose?
H
H
CHO
OH
OH
CH2OH
I
HO
H
CO2H
H
OH
CH2OH
II
A)
B)
C)
D)
E)
Ans:
H
HO
CHO
OH
H
CH2OH
III
H
HO
CH2OH
OH
H
CH2OH
IV
HO
HO
CHO
H
H
CH2OH
V
I
II
III
IV
V
A
Topic: Nomenclature, Definitions and Types of Carbohydrates
8. An aldonic acid is represented by:
HO
HO
H
H
CO2H
H
H
OH
OH
CH2OH
I
A)
B)
C)
D)
E)
Ans:
HO
HO
H
H
CH2OH
H
H
OH
OH
CH2OH
II
HO
HO
H
H
CO2H
H
H
OH
OH
CO2H
III
I
II
III
IV
V
A
828
HO
HO
H
H
CHO
H
H
OH
OH
CO2H
IV
HO
HO
H
H
CO2H
H
H
OH
OH
CHO
V
Chapter 22
Topic: Nomenclature, Definitions and Types of Carbohydrates
9. Which of the following is an L-aldopentose?
CHO
H
OH
H
OH
CH2OH
H
H
HO
I
CHO
OH
OH
H
CH2OH
CHO
H
OH
HO
H
CH2OH
II
A)
B)
C)
D)
E)
Ans:
HO
HO
H
III
CH2OH
O C
HO
H
HO
H
CH2OH
CHO
H
H
OH
CH2OH
IV
V
I
II
III
IV
V
B
Topic: Nomenclature, Definitions and Types of Carbohydrates
10. Which compound is D-galactose?
CH2OH
O
OH
OH
H
H
H
H
H
CH2OH
O H
H
H
OH H
OH
OH
OH OH
H
I
OH
H
II
CH2OH
O
H
H
H
H
H
OH
OH
OH
III
CH2OH
O H
OH
H
OH OH
OH
H
OH OH
H
IV
A)
B)
C)
D)
E)
Ans:
CH2OH
O
H
OH
H
OH H
H
OH
H
V
I
II
III
IV
V
C
829
Chapter 22
Topic: Nomenclature, Definitions and Types of Carbohydrates
11. Which of these is ∝-D-glucopyranose?
CH2OH
O
OH
OH
CH2OH
O
CH2OH
O
OH
OH
OH
OH
OH
OH
I
OH
II
CH2OH
O
OH
OH
OCH3
III
OH
CH2OH
O
OH
OH
OH
OH
IV
A)
B)
C)
D)
E)
Ans:
OH
OH
V
I
II
III
IV
V
A
830
OH
OH
Chapter 22
Topic: Nomenclature, Definitions and Types of Carbohydrates
12. Which of these is a non-reducing monosaccharide?
CH2OH
O
OH
OH
CH2OH
O
CH2OH
O
OH
OH
OH
OH
OH
OH
OH
I
OH
II
CH2OH
O
OH
OCH3
III
HOH2C
O
HO
OH
OH
OH
IV
A)
B)
C)
D)
E)
Ans:
OH
V
I
II
III
IV
V
D
831
OH
CH2OH
OH
Chapter 22
Topic: Nomenclature, Definitions and Types of Carbohydrates
13. Which of these is a glycoside?
CH2OH
O
OH
OH
CH2OH
O
OH
OH
OH
OH
CH2OH
O
OH
OH
OH
I
OH
II
CH2OH
O
OH
OCH3
III
OH
CH2OCH3
O
OH
OH
OH OH
OH
IV
A)
B)
C)
D)
E)
Ans:
OH
V
I
II
III
IV
V
D
832
OH
Chapter 22
Topic: Nomenclature, Definitions and Types of Carbohydrates
14. Which are the anomers?
CH2OH
O
OH
OH
CH2OH
O
OH
OH
OH
OH
OH
OH
I
II
CH2OH
O
CH2OH
O
OH
OH
OH
OH
III
A)
B)
C)
D)
E)
Ans:
OH
OH
OH OH
IV
I and II
I and III
II and III
II and IV
III and IV
B
833
Chapter 22
Topic: Nomenclature, Definitions and Types of Carbohydrates
15. Consider the structures shown below. Which structure represents β-D-glucopyranose?
OH
HO OH
O
HO
HO
O
HO
OH
OH
OH
OH
I
II
OH
HO OH
O
HO
HO
OH
O
OH
OH
OH
III
IV
OH
O
HO
HO
OH
OCH3
V
A)
B)
C)
D)
E)
Ans:
I
II
III
IV
V
C
834
OH
Chapter 22
Topic: Nomenclature, Definitions and Types of Carbohydrates
16. Consider the structures shown below. Which compound is not a reducing sugar?
OH
HO OH
O
HO
HO
O
HO
OH
OH
OH
OH
I
II
OH
HO OH
O
HO
HO
O
OH
OH
OH
OH
OH
III
IV
OH
O
HO
HO
OH
OCH3
V
A)
B)
C)
D)
E)
Ans:
I
II
III
IV
V
E
Topic: Nomenclature, Definitions and Types of Carbohydrates
17. Which of the following is the structure of D-galacturonic acid?
H
HO
HO
H
CH2OH
OH
H
H
OH
CO2H
I
A)
B)
C)
D)
E)
Ans:
H
HO
HO
H
CO2H
OH
H
H
OH
CH2OH
II
H
HO
HO
H
CHO
OH
H
H
OH
CO2H
III
I
II
III
IV
V
C
835
H
HO
HO
H
CO2H
OH
H
H
OH
CO2H
IV
HO
H
H
HO
CHO
H
OH
OH
H
CO2H
V
Chapter 22
Topic: Nomenclature, Definitions and Types of Carbohydrates, Analysis
18. Which is not likely to give a brick-red precipitate with Benedict’s solution?
A) Maltose
B) Cellobiose
C) Sucrose
D) Lactose
E) None of these
Ans: C
Topic: Nomenclature, Definitions and Types of Carbohydrates
19. A glycoside is a compound which contains the structural features of these classes of
organic compounds:
A) Aldehydes and alcohols
B) Acetals and alcohols
C) Hemiacetals and alcohols
D) Ketones and alcohols
E) Alcohols and carboxylic acids
Ans: B
Topic: Nomenclature, Definitions and Types of Carbohydrates
20. Which of these is an example of a glucan?
A) Maltose
B) Sucrose
C) Lactose
D) Cellobiose
E) Amylose
Ans: E
Topic: Nomenclature, Definitions and Types of Carbohydrates
21. Which is a reducing sugar with an ∝-glycosidic linkage?
A) Sucrose
B) Maltose
C) Lactose
D) Cellobiose
E) None of these
Ans: B
836
Chapter 22
Topic: Nomenclature, Definitions and Types of Carbohydrates
22. Which compound will not reduce Ag(NH3)2+ ?
OH
OH
O
HO
HO
O
HO
HO
OH
OCH3
OH
HOH2C
OH
CH2OH
OH
OH
OH OH
OH
III
O
O
HO
CH2OH
A)
B)
C)
D)
E)
Ans:
OH
OH
II
CHO
OH
O
HO
I
H
O OH
OH
IV
V
I
II
III
IV
V
A
Topic: Nomenclature, Definitions and Types of Carbohydrates
23. What is the correct description of this disaccharide?
CH2OH
O
O
A)
B)
C)
D)
E)
Ans:
CH2OH
O
The ∝-anomer of two D-glucose units joined by an ∝(1→4) linkage
The ∝-anomer of two D-galactose units joined by an ∝(1→4) linkage
The β-anomer of two D-galactose units joined by a β(1→4) linkage
The β-anomer of two D-glucose units joined by a β(1→4) linkage
The ∝-anomer of two D-galactose units joined by a β(1→4) linkage
E
837
Chapter 22
Topic: Nomenclature, Definitions and Types of Carbohydrates
24. What is a correct general description of the monosaccharide shown here?
HOH2C
O
OH
CH2OH
OH OH
A)
B)
C)
D)
E)
Ans:
The ∝-anomer of the pyranose form of an aldohexose
The β-anomer of the pyranose form of an aldohexose
The ∝-anomer of the furanose form of a ketohexose
The β-anomer of the furanose form of a ketohexose
The β-anomer of the furanose form of an aldohexose
D
Topic: Nomenclature, Definitions and Types of Carbohydrates
25. What can be said, correctly, about a monosaccharide, the name of which is preceded
only by (+) ?
A) The compound is the ∝-anomer.
B) The compound exists in the pyranose form.
C) The compound is dextrorotatory.
D) The compound has the same stereochemistry at the penultimate carbon as D-(+)glucose.
E) The compound exists only in open-chain form.
Ans: C
Topic: Nomenclature, Definitions and Types of Carbohydrates
26. How many stereoisomers of the L series would exist for the following pentose?
O=CHCHOHCHOHCHOHCH2OH
A) 2
B) 3
C) 4
D) 5
E) 8
Ans: C
838
Chapter 22
Topic: Reactions
27. Reaction of the following substance with sodium borohydride (NaBH4) would yield:
HO
HO
H
H
CHO
H
H
OH
OH
CH2OH
HO
HO
H
H
CO2H
H
H
OH
OH
CH2OH
I
A)
B)
C)
D)
E)
Ans:
NaBH4
HO
HO
H
H
CO2H
H
H
OH
OH
CO2H
II
?
HO
HO
H
H
CH2OH
H
H
OH
OH
CH2OH
III
I
II
III
IV
V
C
839
HO
HO
H
H
CHO
H
H
OH
OH
CO2H
IV
HO
HO
H
H
CH3
H
H
OH
OH
CH2OH
V
Chapter 22
Topic: Reactions
28. Reaction of the following substance with bromine water would yield:
HO
HO
H
H
CHO
H
H
OH
OH
CH2OH
Br2
H2O
HO
HO
H
H
CO2H
H
H
OH
OH
CH2OH
HO
HO
H
H
I
A)
B)
C)
D)
E)
Ans:
?
CO2H
H
H
OH
OH
CO2H
II
HO
HO
H
H
CH2OH
H
H
OH
OH
CH2OH
III
I
II
III
IV
V
A
840
HO
HO
H
H
CHO
H
H
OH
OH
CO2H
IV
HO
HO
H
H
CHO
H
H
OH
OH
CHO
V
Chapter 22
Topic: Reactions
29. If J (below) were treated with dilute aqueous hydrochloric acid and the solution allowed
to stand, what compounds (other than methanol) would be formed in the solution?
CH2OH
O H
dil HCl
H
H 2O
OH H
OCH3
HO
H OH
H
CHO
H
H
OH
OH
CH2OH
HO
HO
H
H
A)
B)
C)
D)
E)
Ans:
HO
HO
H
H
I
I and II
I and III
II and III
I, III, and IV
II, III, and IV
D
CHO
H
H
OH
OH
CH2OH
?
CH2OH
O H
H
H
OH H
HO
OH
H OH
II
CH2OH
O OH
H
OH H
H
HO
H OH
H
III
IV
Topic: Reactions
30. A compound X reacts with 3 mol of HIO4 to yield 2 mol of HCO2H and 2 mol of
HCHO. What is the structure of X?
CH2OH
CHO
CHO
CH2OH
CHOH
CHOH
CHOH
C
O
C
O
CHOH
CHOH
CHOH
C
O
C
O
CH2OH
CH2OH
CHO
CH2OH
CHO
I
A)
B)
C)
D)
E)
Ans:
II
III
IV
V
I
II
III
IV
V
A
841
CHO
Chapter 22
Topic: Reactions
31. A D-aldohexose, X, is subjected to a Ruff degradation. The degradation product is
treated with nitric acid to yield an optically inactive aldaric acid. A possible structure
for X is:
HO
HO
H
H
CHO
H
H
OH
OH
CH2OH
H
H
H
H
I
A)
B)
C)
D)
E)
Ans:
CHO
OH
OH
OH
OH
CH2OH
HO
H
H
HO
II
CHO
H
OH
OH
H
CH2OH
H
HO
HO
H
III
CHO
OH
H
H
OH
CH2OH
H
HO
HO
HO
IV
CHO
OH
H
H
H
CH2OH
V
I
II
III
IV
V
B
Topic: Reactions
32. Which of the following would yield D-glucose and D-mannose when subjected to a
Kiliani-Fischer synthesis?
HO
H
H
CHO
H
OH
OH
CH2OH
I
A)
B)
C)
D)
E)
Ans:
H
H
H
CHO
OH
OH
OH
CH2OH
II
H
H
HO
CHO
OH
OH
H
CH2OH
III
I
II
III
IV
V
A
842
H
HO
H
CHO
OH
H
OH
CH2OH
IV
HO
HO
H
CHO
H
H
OH
CH2OH
V
Chapter 22
Topic: Reactions
33. Reaction of the following substance with nitric acid would yield:
HO
HO
H
H
CHO
H
H
OH
OH
CH2OH
HO
HO
H
H
CO2H
H
H
OH
OH
CH2OH
HNO3
HO
HO
H
H
I
A)
B)
C)
D)
E)
Ans:
CO2H
H
H
OH
OH
CO2H
?
HO
HO
H
H
II
CH2OH
H
H
OH
OH
CH2OH
HO
HO
H
H
III
CHO
H
H
OH
OH
CO2H
HO
HO
H
H
IV
CO2H
H
H
OH
OH
CHO
V
I
II
III
IV
V
B
Topic: Reactions
34. Which aldohexose would yield an optically active aldaric acid when treated with nitric
acid?
HO
HO
H
H
CHO
H
H
OH
OH
CH2OH
I
A)
B)
C)
D)
E)
Ans:
H
H
H
H
CHO
OH
OH
OH
OH
CH2OH
II
HO
H
H
HO
CHO
H
OH
OH
H
CH2OH
III
I
II
III
IV
V
A
843
H
HO
HO
H
CHO
OH
H
H
OH
CH2OH
IV
HO
HO
HO
HO
CHO
H
H
H
H
CH2OH
V
Chapter 22
Topic: Reactions
35. Which compound or compounds would be formed when D-glucose is dissolved in
methanol and then treated with anhydrous acid?
OCH3
OH
O
HO
HO
OCH3
OH
O
H3CO
H3CO
OCH3
I
II
OH
OH
O
HO
HO
OCH3
OH
O
HO
HO
III
A)
B)
C)
D)
E)
Ans:
OCH3
IV
OH
OCH3
I
II
III
IV
I and IV
E
Topic: Reactions
36. A D-aldohexose, Z, is subjected to a Ruff degradation. The degradation product is
treated with nitric acid to yield an optically active aldaric acid. A possible structure for
Z is:
HO
HO
H
H
CHO
H
H
OH
OH
CH2OH
I
A)
B)
C)
D)
E)
Ans:
H
H
H
H
CHO
OH
OH
OH
OH
CH2OH
HO
H
H
HO
II
CHO
H
OH
OH
H
CH2OH
III
I
II
III
IV
More than one of the above
E
844
H
HO
HO
H
CHO
OH
H
H
OH
CH2OH
IV
Chapter 22
Topic: Reactions
37. Which of the following substances will afford an optically inactive product upon
reaction with nitric acid?
HO
HO
H
H
CHO
H
H
OH
OH
CH2OH
H
H
H
H
I
A)
B)
C)
D)
E)
Ans:
CHO
OH
OH
OH
OH
CH2OH
HO
H
H
HO
II
CHO
H
OH
OH
H
CH2OH
H
HO
HO
H
III
CHO
OH
H
H
OH
CH2OH
IV
I
II
III and IV
II, III and IV
All of the above
D
Topic: Reactions
38. Which of the following substances will afford a meso- product upon reaction with
Br2/H2O ?
HO
HO
H
H
CHO
H
H
OH
OH
CH2OH
I
A)
B)
C)
D)
E)
Ans:
H
H
H
H
CHO
OH
OH
OH
OH
CH2OH
II
HO
H
H
HO
CHO
H
OH
OH
H
CH2OH
III
I
II
III and IV
II, III and IV
None of the above
E
845
H
HO
HO
H
CHO
OH
H
H
OH
CH2OH
IV
Chapter 22
Topic: Reactions
39. Which of these reacts with dilute HCl to produce methanol?
CH2OH
O OH
CH2OH
O
CH2OCH3
O
OH
OCH3
OH
OH
OH
I
HOH2C
II
O
OCH3
CH2OH
OH OH
IV
A)
B)
C)
D)
E)
Ans:
OH
OH
OH
OCH3
H3COH2C
OH
OH
III
CH2OH
O
OH
OH
OH
V
I
II
III
IV
V
D
Topic: Reactions
40. Which monosaccharide is recovered from the hydrolysis of glycogen?
A) D-Galactose
B) D-Glucose
C) D-Gulose
D) Cellobiose
E) Maltose
Ans: B
Topic: Reactions
41. Which monosaccharide is recovered from the hydrolysis of starch?
A) D-Galactose
B) D-Glucose
C) D-Gulose
D) Cellobiose
E) Maltose
Ans: B
846
Chapter 22
Topic: Reagents
42. Sucrose reacts with which of these reagents?
A) C6H5NHNH2
B) Cu2+
C) Br2/H2O
D) H3O+
E) Ag(NH3)2+
Ans: D
Topic: Reagents
43. Which reagent would cause the following conversion to take place?
OH
OCH3
A)
B)
C)
D)
E)
Ans:
?
O
HO
HO
OCH3
OH
O
H3CO
H3CO
OCH3
OCH3
Excess CH3OH and KOH
Excess CH3OH and HCl
Excess (CH3)2SO4 and OHExcess CH3I and H3O+
Excess (CH3CO)2O
C
Topic: Reagents
44. Select the reagent(s) needed to perform the following transformation.
HO
HO
O
HO
HO
?
O
HO
HO
OH
HO
HO
A)
B)
CH3I, KOH
C)
D)
E)
(CH3)2SO4, NaOH
CH3OH, HCl
O
O
O
O
OCH3
Ans: D
847
OCH3
Chapter 22
Topic: Reagents
45. Which reagent will distinguish between the members of the following pair?
H
HO
H
H
CHO
OH
H
OH
OH
CH2OH
H
HO
H
H
I
CH2OH
OH
H
OH
OH
CH2OH
II
Ag(NH3)2+
AgNO3/C2H5OH
Br2/CCl4
HCl
Hot KMnO4
A
A)
B)
C)
D)
E)
Ans:
Topic: Reagents
46. Which reagent will distinguish between the members of the following pair?
H
HO
H
H
CHO
OH
H
OH
OH
CH2OH
I
A)
B)
C)
D)
E)
Ans:
H
HO
H
H
CO2H
OH
H
OH
OH
CH2OH
II
Ag(NH3)2+
AgNO3/C2H5OH
Br2/CCl4
HCl
Hot KMnO4
A
848
Chapter 22
Topic: Reagents
47. Which reagent will distinguish between the members of the following pair?
H
HO
H
H
CHO
OH
H
OH
OH
CH2OH
H
HO
H
H
I
A)
B)
C)
D)
E)
Ans:
CHO
OCH3
H
OCH3
OCH3
CH2OH
II
Ag(NH3)2+
AgNO3/C2H5OH
Br2/CCl4
KMnO4
HIO4
E
Topic: Reagents
48. Which of the following would give a positive test with Benedict's or Fehling's solution?
OH
OH
O
HO
HO
OH
OCH3
O
HO
HO
I
OH
OCH3
II
OH
HOH2C
O
HO
HO
OCH3
OH
O
HO
CH2OH
OCH3
OH
III
A)
B)
C)
D)
E)
Ans:
IV
I
II
III
IV
All of these
C
849
Chapter 22
Topic: Product Ratios
49. What is the ratio of products formed by the reaction of periodic acid with the following
compound?
H
H
H
H
CHO
OH
OH
OH
OH
CH2OH
I
II
III
IV
V
A)
B)
C)
D)
E)
Ans:
H2C=O
HCO2H
CO2
5
3
1
1
0
1
3
5
4
4
0
0
0
1
2
I
II
III
IV
V
C
850
Chapter 22
Topic: Product Ratios
50. What is the ratio of products formed by the reaction of periodic acid with the following
compound?
CH2OH
C O
H
OH
HO
H
H
OH
CH2OH
H2C=O
5
3
1
2
0
I
II
III
IV
V
A)
B)
C)
D)
E)
Ans:
HCO2H
1
3
5
3
4
CO2
0
0
0
1
2
I
II
III
IV
V
D
Topic: Product Ratios
51. Which of these is a component of the mixture formed when D-galactose is placed in
aqueous base (de Bruyn - van Ekenstein transformation)?
HO
H
H
HO
CHO
H
OH
OH
H
CH2OH
I
A)
B)
C)
D)
E)
Ans:
H
H
HO
H
CHO
OH
OH
H
OH
CH2OH
II
H
HO
H
H
CHO
OH
H
OH
OH
CH2OH
III
I
II
III
IV
V
E
851
HO
HO
H
H
CHO
H
H
OH
OH
CH2OH
IV
CH2OH
HO
HO
H
C O
H
H
OH
CH2OH
V
Chapter 22
Topic: Named Reactions and Structure Identification
52. The Kiliani-Fisher Synthesis is the reaction of an aldose with:
A) Br2/H2O; then HCN; then H3O+; then Na-Hg, H2O
B) HCN; then Ba(OH)2; then H3O+; then Na-Hg, H2O
C) HCN; then H3O+; then Ba(OH)2; then Na-Hg, H2O
D) Br2/H2O; then H2O2, Fe2(SO4)3
E) Br2/H2O
Ans: B
Topic: Named Reactions and Structure Identification
53. The Ruff degradation is the reaction of an aldose with:
A) Br2/H2O; then HCN; then H3O+; then Na-Hg, H2O
B) HCN; then Ba(OH)2; then H3O+; then Na-Hg, H2O
C) HCN; then H3O+; then Ba(OH)2; then Na-Hg, H2O
D) Br2/H2O; then H2O2, Fe2(SO4)3
E) Br2/H2O
Ans: D
Topic: Named Reactions and Structure Identification
54. An aldopentose, X, is subjected to a Kiliani-Fischer synthesis to produce two
aldohexoses, Y and Z. Both Y and Z, when oxidized with nitric acid, yield optically
active aldaric acids. Which structure represents X?
HO
HO
H
CHO
H
H
OH
CH2OH
I
A)
B)
C)
D)
E)
Ans:
HO
H
H
CHO
H
OH
OH
CH2OH
II
H
H
H
CHO
OH
OH
OH
CH2OH
III
I
II
III
IV
V
B
852
HO
HO
HO
CHO
H
H
H
CH2OH
IV
H
H
HO
CHO
OH
OH
H
CH2OH
V
Chapter 22
Topic: Named Reactions and Structure Identification
55. Compound X is a reducing sugar which, on hydrolysis, affords two molar equivalents of
D-glucose. This hydrolysis is catalyzed by an enzyme specific for glucosides of this
type:
CH2OH
O
OH
OH
OR
OH
What is the identity of X?
A) Sucrose
B) Lactose
C) Maltose
D) Cellobiose
E) None of these
Ans: C
Topic: Named Reactions and Structure Identification
56. A polysaccharide, Y, undergoes hydrolysis on catalysis by an enzyme which catalyzes
the hydrolysis of cellobiose, but not maltose. Complete methylation and acid hydrolysis
of the permethylated Y yields better than 95% of 2,4,6-tri-0-methyl-D-glucose.
Which is a plausible structure for the repeating unit of Y?
CH2OH
O OR
CH2OR
OH
OR
O
O
OR
OH
OR
OH
OR
OH
OH
II
I
CH2OH
OR
OH
O
OH
OH
IV
OR
OH
III
CH2OH
O OR
OH
A)
B)
C)
D)
E)
Ans:
CH2OH
OR
OR
V
I
II
III
IV
V
D
853
Chapter 22
Topic: Named Reactions and Structure Identification
57. If the methyl glycoside of an aldohexose is treated with HIO4, one molar equivalent of
HCHO is formed but no HCOOH. What size ring is present in the glycoside?
A) Three-membered
B) Four-membered
C) Five-membered
D) Six-membered
E) Seven-membered
Ans: C
Topic: Named Reactions and Structure Identification
58. The pyranose form of an aldohexose which can react 1:2 with acetone in the presence of
acid is which of these?
HO OH
OH
HO OH OH
O
OH
OH
O
HO
HO
OH
OH
O
I
II
OH
HO
HO
III
HO OH
O
O
OH
OH
OH
OH
OH
IV
A)
B)
C)
D)
E)
Ans:
OH
OH
OH
V
I
II
III
IV
V
C
854
Chapter 22
Topic: Named Reactions and Structure Identification
59. Sugars that would yield the same phenylosazone are:
CH2OH
C O
HO
H
H
OH
H
OH
CH2OH
H
HO
H
H
I
A)
B)
C)
D)
E)
Ans:
CHO
OH
H
OH
OH
CH2OH
II
HO
HO
H
H
CHO
H
H
OH
OH
CH2OH
III
H
HO
H
HO
CHO
OH
H
OH
H
CH2OH
H
H
HO
H
IV
CHO
OH
OH
H
OH
CH2OH
V
I and II
II and III
I, II, and III
III and IV
I, II, and V
C
Topic: Named Reactions and Structure Identification
60. Which is not an intermediate monosaccharide in the Kiliani-Fischer synthesis of Dmannose from D-glyceraldehyde?
A) D-Ribose
B) D-Threose
C) D-Arabinose
D) D-Erythrose
E) More than one of these
Ans: E
Topic: Stereochemical Considerations
61. Reaction of D-ribose with bromine water would yield an optically:
A) active aldonic acid.
B) inactive aldonic acid.
C) active aldaric acid.
D) inactive aldaric acid.
E) active uronic acid.
Ans: A
Topic: Stereochemical Considerations
62. Reaction of D-ribose with HNO3 would yield an optically:
A) active aldonic acid.
B) inactive aldonic acid.
C) active aldaric acid.
D) inactive aldaric acid.
E) inactive uronic acid.
Ans: D
855
Chapter 22
Topic: Stereochemical Considerations
63. Which would undergo mutarotation in neutral aqueous solution?
CH2OH
CH2OH
O
O
OH
OH
HO
OH
OH
A)
B)
C)
D)
E)
Ans:
OH
OH
OH
CH2OH
O OH
CH2OH
O OCH3
OH
OH
OH
OH
OH
OH
III
II
I
I
II
III
IV
More than one of these
E
IV
Topic: Stereochemical Considerations
64. Which of the following structures represent enantiomers?
H
HO
H
H
CHO
OH
H
OH
OH
CH2OH
I
A)
B)
C)
D)
E)
Ans:
HO
HO
H
H
CHO
H
H
OH
OH
CH2OH
II
H
H
H
H
CHO
OH
OH
OH
OH
CH2OH
III
I and II
II and III
III and IV
III and V
IV and V
D
856
H
H
HO
H
CHO
OH
OH
H
OH
CH2OH
IV
HO
HO
HO
HO
CHO
H
H
H
H
CH2OH
V
Chapter 22
Topic: Stereochemical Considerations
65. Which of the following structures represent epimers?
H
HO
H
H
CHO
OH
H
OH
OH
CH2OH
HO
HO
H
H
I
A)
B)
C)
D)
E)
Ans:
CHO
H
H
OH
OH
CH2OH
H
H
H
H
II
CHO
OH
OH
OH
OH
CH2OH
H
H
HO
H
III
CHO
OH
OH
H
OH
CH2OH
HO
HO
HO
HO
IV
CHO
H
H
H
H
CH2OH
V
I and II
II and III
III and IV
III and V
IV and V
A
Topic: Stereochemical Considerations
66. Refer to the following structures. Which D-aldohexose would react with NaBH4 to
yield an optically inactive alditol?
H
HO
H
H
CHO
OH
H
OH
OH
CH2OH
I
A)
B)
C)
D)
E)
Ans:
HO
HO
H
H
CHO
H
H
OH
OH
CH2OH
II
H
H
H
H
CHO
OH
OH
OH
OH
CH2OH
III
I
II
III
IV
V
C
857
H
H
HO
H
CHO
OH
OH
H
OH
CH2OH
IV
HO
HO
HO
HO
CHO
H
H
H
H
CH2OH
V
Chapter 22
Topic: Stereochemical Considerations
67. Refer to the following structures. Which aldohexose when subjected to Fischer's endgroup interchange would be converted to a compound identical with itself?
H
HO
H
H
CHO
OH
H
OH
OH
CH2OH
HO
HO
H
H
I
A)
B)
C)
D)
E)
Ans:
CHO
H
H
OH
OH
CH2OH
H
H
H
H
II
CHO
OH
OH
OH
OH
CH2OH
H
H
HO
H
III
CHO
OH
OH
H
OH
CH2OH
HO
HO
HO
HO
IV
CHO
H
H
H
H
CH2OH
V
I
II
III
IV
V
B
Topic: Stereochemical Considerations
68. Which is an L-monosaccharide that would yield an optically active aldaric acid on
oxidation by nitric acid?
CH2OH
C O
HO
H
H
OH
H
OH
CH2OH
I
A)
B)
C)
D)
E)
Ans:
H
HO
H
H
CHO
OH
H
OH
OH
CH2OH
II
HO
H
H
HO
CHO
H
OH
OH
H
CH2OH
III
I
II
III
IV
V
D
858
H
H
HO
HO
CHO
OH
OH
H
H
CH2OH
IV
HO
HO
HO
HO
CHO
H
H
H
H
CH2OH
V
Chapter 22
Topic: Stereochemical Considerations
69. Which is the only one of these classes of carbohydrates which can include optically
inactive members?
A) Uronic acids
B) Alditols
C) Ketoses
D) Glycosides
E) Aldonic acids
Ans: B
Topic: Stereochemical Considerations
70. What are the correct designations for the stereogenic centers in this aldose:
H
HO
HO
CHO
OH
H
H
CH2OH
A)
B)
C)
D)
E)
Ans:
2R,3S,4R
2R,3S,4S
2S,3R,4R
2S,3S,4R
2R,3R,4S
B
859
Chapter 22
Topic: Combination Problems
71. Consider the structures of D-allose below:
H
H
H
H
CHO
OH
OH
OH
OH
CH2OH
D-Allose
Which of the following structures represents L-allose?
HO
H
H
HO
CHO
H
OH
OH
H
CH2OH
H
H
H
HO
I
A)
B)
C)
D)
E)
Ans:
CHO
OH
OH
OH
H
CH2OH
HO
H
HO
H
II
CHO
H
OH
H
OH
CH2OH
III
HO
HO
HO
HO
CHO
H
H
H
H
CH2OH
IV
H
H
HO
HO
CHO
OH
OH
H
H
CH2OH
V
I
II
III
IV
V
B
Topic: Stereochemistry
72. Consider the structures below: what term(s) describe(s) the relationship between them?
HO
H
H
HO
CHO
H
OH
OH
H
CH2OH
A)
B)
C)
D)
E)
Ans:
H
H
H
HO
CHO
OH
OH
OH
H
CH2OH
Enantiomers
Epimers
Diastereomers
Anomers
More than one of the above
E
860
Chapter 22
Topic: Combination Problems
73. Consider the structures below. Which of the structures II-VI are epimers of I ?
H
H
H
H
CHO
OH
OH
OH
OH
CH2OH
I
HO
H
H
HO
CHO
H
OH
OH
H
CH2OH
II
A)
B)
C)
D)
E)
Ans:
H
H
H
HO
CHO
OH
OH
OH
H
CH2OH
III
HO
H
HO
H
CHO
H
OH
H
OH
CH2OH
IV
HO
HO
HO
HO
CHO
H
H
H
H
CH2OH
V
H
H
HO
HO
CHO
OH
OH
H
H
CH2OH
VI
II
III
IV
V
VI
B
Topic: Combination Problems
74. Consider the structures below. An L-aldohexose, X, is treated with nitric acid to yield
an optically inactive aldaric acid. The same L-aldohexose, X, is subjected to a Ruff
degradation and the degradation product is oxidized with nitric acid to produce an
optically inactive aldaric acid. Which is a possible structure for X?
HO
H
H
HO
CHO
H
OH
OH
H
CH2OH
I
A) I
B) II
C) III
D) IV
E) V
Ans: D
H
H
H
HO
CHO
OH
OH
OH
H
CH2OH
II
HO
H
HO
H
CHO
H
OH
H
OH
CH2OH
III
861
HO
HO
HO
HO
CHO
H
H
H
H
CH2OH
IV
H
H
HO
HO
CHO
OH
OH
H
H
CH2OH
V
Chapter 22
Topic: Combination Problems
75. Consider the structures below. Which monosaccharides would yield an optically active
aldonic acid when oxidized with bromine water?
H
H
H
H
CHO
OH
OH
OH
OH
CH2OH
I
A)
B)
C)
D)
E)
Ans:
HO
H
H
HO
CHO
H
OH
OH
H
CH2OH
H
H
H
HO
II
I, II, and III
I, II, and V
III, IV, and VI
II, III, and IV
All of these
E
CHO
OH
OH
OH
H
CH2OH
HO
H
HO
H
III
CHO
H
OH
H
OH
CH2OH
HO
HO
HO
HO
IV
CHO
H
H
H
H
CH2OH
H
H
HO
HO
V
CHO
OH
OH
H
H
CH2OH
VI
Topic: Combination Problems
76. Consider the structures above. Which monosaccharides would yield the same
phenylosazone when treated with excess phenylhydrazine?
H
H
H
H
CHO
OH
OH
OH
OH
CH2OH
I
A)
B)
C)
D)
E)
Ans:
HO
H
H
HO
CHO
H
OH
OH
H
CH2OH
II
H
H
H
HO
CHO
OH
OH
OH
H
CH2OH
III
HO
H
HO
H
CHO
H
OH
H
OH
CH2OH
IV
I and V
I and III
II and III
III and VI
IV and V
C
862
HO
HO
HO
HO
CHO
H
H
H
H
CH2OH
V
H
H
HO
HO
CHO
OH
OH
H
H
CH2OH
VI
Chapter 22
Topic: Combination Problems
77. Refer to the structures below. Which sugar(s) would yield an optically active aldaric
acid on oxidation with nitric acid?
H
H
H
CHO
OH
OH
OH
CH2OH
I
A)
B)
C)
D)
E)
Ans:
HO
H
H
CHO
H
OH
OH
CH2OH
II
I and III
I, II, III, and V
II
III and IV
I and V
C
H
HO
H
CHO
OH
H
OH
CH2OH
HO
H
HO
III
CHO
H
OH
H
CH2OH
HO
HO
HO
CHO
H
H
H
CH2OH
IV
V
Topic: Combination Problems
78. Refer to the structures below. Which are L-sugars?
CHO
H
OH
H
OH
H
OH
CH2OH
I
A)
B)
C)
D)
E)
Ans:
CHO
HO
H
H
OH
H
OH
CH2OH
CHO
H
OH
HO
H
H
OH
CH2OH
CHO
HO
H
H
OH
HO
H
CH2OH
CHO
HO
H
HO
H
HO
H
CH2OH
II
III
IV
V
II and IV
I, II, and III
I and V
III, IV, and V
IV and V
E
863
Chapter 22
Topic: Combination Problems
79. Refer to the structures below. Which sugars would react with phenylhydrazine to yield
the same phenylosazone?
H
H
H
CHO
OH
OH
OH
CH2OH
I
A)
B)
C)
D)
E)
Ans:
HO
H
H
CHO
H
OH
OH
CH2OH
II
H
HO
H
CHO
OH
H
OH
CH2OH
III
HO
H
HO
CHO
H
OH
H
CH2OH
IV
HO
HO
HO
CHO
H
H
H
CH2OH
V
I and II
III and IV
I and V
II and III
III and V
A
Topic: General Information
80. The D-glucose unit at the branching point of amylopectin has free hydroxyl groups at
which positions?
A) C2, C3, and C6
B) C2 and C3
C) C3 and C4
D) C3, C4, and C6
E) C4 and C6
Ans: B
Topic: General Information
81. Cellulose differs from chitin in which way?
A) Cellulose has β-glycosidic linkages; chitin has ∝-glycosidic linkages.
B) Cellulose contains only D-glucose units; chitin contains only N-acetyl-Dglucosamine units.
C) Cellulose cannot be hydrolyzed; chitin can be hydrolyzed.
D) Cellulose has a linear structure; chitin has a helical structure.
E) Cellulose chains are branched; chitin chains are unbranched.
Ans: B
Topic: General Information
82. Cellulose lacks nutritive value for humans because:
A) the products of its digestion are excreted without utilization.
B) its conformation prevents attack by digestive enzymes.
C) we lack the enzymes which can catalyze the hydrolysis of the glycosidic linkages.
D) it passes through the digestive tract so rapidly.
E) the molecules possess such a high molecular weight.
Ans: C
864
Chapter 22
SHORT ANSWER QUESTIONS
Topic: General Information
83 Sugars that give positive tests with Benedict's or Tollens' solution are called
_______________.
Ans: reducing sugars
Topic: General Information
84. Bromine water is a reagent that converts an aldose to an ___________.
Ans: aldonic acid
Topic: General Information
85. The simplest carbohydrates (those that cannot be hydrolyzed to simpler carbohydrates)
are called ______________.
Ans: monosaccharides
Topic: General Information
86. Carbohydrates that hydrolyze to give a large number of molecules of monosaccharides
are called _____________.
Ans: polysaccharides
Topic: General Information
87. Carbohydrates are synthesized in green plants by _____________.
Ans: photosynthesis
Topic: General Information
88. A monosaccharide containing six carbon atoms and an aldehyde functionality is called
an _____________.
Ans: aldohexose
Topic: General Information
89. Many carbohydrates exist in equilibrium with cyclic hemiacetals. Those carbohydrates
that form a five-membered cyclic hemiacetal are called _____________.
Ans: furanoses
Topic: General Information
90. Ordinary table sugar is a disaccharide called __________.
Ans: sucrose
Topic: General Information
91. Milk contains a disaccharide called __________.
Ans: lactose
865
Chapter 22
Topic: General Information
92. The method of lengthening the carbon chain of an aldose through the addition of HCN
followed by hydrolysis and reduction is called the ______________.
Ans: Kiliani-Fischer synthesis
Topic: General Information
93. A polysaccharide found in the shells of lobsters, crabs, and in the exoskeletons of
insects and spiders is called _________.
Ans: chitin
Topic: Aldaric Acids
94. Draw the structure(s) of the γ-lactone(s) that can be formed from D-Glucaric acid.
CO2H
H
HO
OH
H
H
OH
H
OH
CO2H
D-glucaric acid
Ans:
CO2H
H
HO
OH
H
O
CO2H
-H2O
H
OH
O
H
H
OH
H
OH
H
OH
H
OH
CO2H
C
H
and
γ−lactones
866
OH
HO
H
H
O
H
OH
CO2H
O
D-glucaric acid
C
Chapter 22
Topic: Aldaric Acids
95. Consider the structure of D-erythrose, shown below. Draw the structure of the product
obtained upon treatment of D-erythrose with nitric acid. Give its IUPAC name and
comment on its stereochemistry.
O
H
H
OH
H
OH
CH2OH
D-erythrose
Ans: The product, 2,3-dihydroxybutanedioic acid, is expected to be optically inactive,
due to a meso- configuration: (2R,3S) or (2S,3R).
O
H
H
OH
H
OH
CH2OH
D-erythrose
CO2H
HNO3
H
OH
H
OH
CO2H
2,3dihydroxybutanedioic acid
(meso)
Topic: Periodate Oxidations
96. What products are expected from the reaction of 3-hydroxy-2,4-pentanedione with 2
molar equivalents of periodic acid?
O
O
O
O
Ans:
2 eq HIO4
2
+
OH
OH
867
H
OH
Chapter 22
Topic: Periodate Oxidations
97. Suggest a simple chemical test that you might use to distinguish between 2,3pentanediol and 2,4-pentanediol?
Ans: Periodic acid test: 2,3- pentanediol is expected to undergo oxidative cleavage with
periodic acid to afford acetic acid and propanoic acid. By contrast, no reaction is
expected in an analogous treatment of 2,4-pentanediol, since the hydroxyl groups
are separated by a methylene group.
OH
OH
OH
OH
HIO4
OH
+
HIO4
O
O
no reaction
OH
Topic: Kiliani Fischer Synthesis
98. What products are expected to be formed upon subjecting L-erythrose (shown below) to
the following reaction sequence? (Draw Fischer projection formulas)
i) HCN; ii) Ba(OH)2; iii) H3O+; iv) Na(Hg), H2O, pH 3-5
Ans: Kiliani Fischer Synthesis:
O
O
H
H
i) HCN
O
H
HO
H
HO
H
CH2OH
L-erythrose
ii) Ba(OH)2
HO
iii) H3O+
HO
H
HO
H
iv) Na(Hg), H2O,
pH 3-5
868
H
H
CH2OH
+
OH
HO
H
HO
H
CH2OH
Chapter 22
Topic: Ruff Degradation
99. What products are expected to be formed upon subjecting D-galactose (shown below) to
the following reaction sequence? (Draw Fischer projection formulas)
i) Br2, H2O; ii) H2O2, Fe2(SO4)3
CHO
H
OH
HO
H
HO
H
H
OH
CH2OH
D-galactose
Ans: Ruff degradation
CHO
H
OH
HO
H
HO
H
H
OH
CHO
i. Br2, H2O
ii. H2O2, Fe2(SO4)3
HO
H
HO
H
H
OH
CH2OH
CH2OH
D-galactose
Topic: Epimers
100. Draw structures for 3 different epimers of D-glucose.
CHO
H
HO
OH
H
H
OH
H
OH
CH2OH
D-Glucose
CHO
Ans:
CHO
CHO
HO
H
H
OH
H
HO
H
H
OH
HO
H
H
OH
H
OH
HO
H
H
OH
H
OH
H
CH2OH
CH2OH
OH
OH
CH2OH
869
Chapter 23
MULTIPLE CHOICE QUESTIONS
Topic: General Lipid Types and Information
1. Which of the following statements regarding lipids is not true?
A) Lipids are soluble in non-polar organic solvents.
B) All lipids have the same functional groups.
C) Lipids include waxes, steroids, and triacylglycerols.
D) Lipids have little in common except their solubility.
E) Many lipids have biological roles.
Ans: B
Topic: General Lipid Types and Information
2. Which of the following statements regarding triacylglycerols is not true?
A) They undergo alkaline hydrolysis to yield soaps.
B) They are liquid if they have alkene bonds.
C) They are solid if they do not have alkene bonds.
D) Some can be hydrogenated.
E) They are soluble in water.
Ans: E
Topic: General Lipid Types and Information
3. Which of the following statements regarding triacylglycerols is true?
A) They form micelles when mixed with water.
B) They are liquid if they are unsaturated.
C) They are solid if they have alkyne bonds.
D) They can be used to wash dirty dishes
E) They are soluble in water.
Ans: B
Topic: General Lipid Types and Information
4. Which of the following statements regarding triacylglycerols is not true?
A) They are synthesized in the body by adipocytes.
B) They have an odd number of carbon atoms.
C) They are solid if they do not have alkene bonds.
D) Those which can be hydrogenated, undergo a significant change in melting point
upon hydrogenation.
E) They can be hydrolyzed to give glycerol and fatty acids.
Ans: B
870
Chapter 23
Topic: General Lipid Types and Information
5. Which of the following statements regarding triacylglycerols is not true?
A) Some undergo autooxidation with oxygen from air.
B) They are solid if they do not have alkene bonds.
C) They form lipid bilayers.
D) They form micelles in water.
E) Two of the above are false statements.
Ans: E
Topic: General Lipid Types and Information
6. Which of the following is a phosphatidic acid?
CH2OOCR
CH2OH
CH2OPO3H2
CHOOCR'
CHOOCR
CHOH
CH2OOCR"
CH2OOCR'
CH2OH
I
II
III
CH2OOCR
CH2OPO3PO3H2
CHOOCR'
CHOOCR
CH2OPO3H2
CH2OOCR'
IV
A)
B)
C)
D)
E)
Ans:
V
I
II
III
IV
V
D
Topic: General Lipid Types and Information
7. Which type of lipid gives these products on saponification:
HOCH2CHOHCH2OH
A) Fat
B) Wax
C) Lecithin
D) Cephalin
E) Plasmalogen
Ans: D
RCO2¯
R'CO2¯
871
PO43¯
HOCH2CH2NH2 ?
Chapter 23
Topic: General Lipid Types and Information
8. Which type of lipid gives the following products on saponification?
HOCH2CHOHCH2OH
A) Fat
B) Wax
C) Lecithin
D) Cephalin
E) Plasmalogen
Ans: C
RCO2¯
R'CO2¯
PO43¯
[HOCH2CH2N(CH3)3]+OH−
Topic: General Lipid Types and Information
9. Which type of lipid gives these products on saponification:
HOCH2CHOHCH2OH
A) Fat
B) Wax
C) Lecithin
D) Cephalin
E) Plasmalogen
Ans: E
RCO2¯
RCH2CHO
PO43¯
Topic: General Lipid Types and Information
10. Which of these lipids does not yield glycerol upon hydrolysis?
A) A lecithin
B) A sphingolipid
C) A cephalin
D) A triacylglycerol
E) A plasmalogen
Ans: B
Topic: General Lipid Types and Information
11. Which of these lipids yields glycerol upon hydrolysis?
A) A lecithin
B) A phosphatidylserine
C) A cephalin
D) A triacylglycerol
E) All of the above
Ans: E
872
HOCH2CH2NH2 ?
Chapter 23
Topic: General Lipid Types and Information
12. Which of these lipids yields choline upon hydrolysis?
A) A lecithin
B) A sphingomyelin
C) A cephalin
D) A cerebroside
E) Two of the above
Ans: E
Topic: General Lipid Types and Information
13. Choline cannot be found as a product of hydrolysis of any representative of this class of
lipids.
A) Sphingomyelins
B) Lecithins
C) Plasmalogens
D) Waxes
E) Both C) and D)
Ans: D
Topic: General Lipid Types and Information
14. Choline cannot be found as a product of hydrolysis of any representative of this class of
lipids.
A) Cerebrosides
B) Phosphatidylserine
C) Plasmalogens
D) Waxes
E) None of the above yield choline upon hydrolysis
Ans: E
Topic: General Lipid Types and Information
15. D-galactose is an example of a class of compounds that can be formed during the
hydrolysis of
A) Sphingomyelins
B) Cerebrosides
C) Plasmalogens
D) Waxes
E) Both C) and D)
Ans: B
873
Chapter 23
Topic: General Lipid Types and Information
16. Which of these is a wax?
H
A)
CH3(CH2)20C=O
O
B)
CH3(CH2)24COCH2(CH2)28CH3
H H
C)
O
CH3(CH2)7C=C(CH2)7COCH2CH3
D)
CH2OOC(CH2)16CH3
CHOOC(CH2)14CH3
CH2OOC(CH2)14CH3
E) CH3(CH2)24CH2OH
Ans: B
Topic: General Lipid Types and Information
17. Which of these is most likely to have properties resembling natural waxes?
O
I
O
II
O
O
III
O
O
O
IV
O
O
V
A)
B)
C)
D)
E)
Ans:
N
H
O
I
II
III
IV
V
D
874
O
Chapter 23
Topic: General Lipid Types and Information
18.
OCH2CH2N(CH3)3 is a structural unit of which type of phosphatide?
A) Cephalins
B) Phosphatidyl serines
C) Plasmalogens
D) Lecithins
E) Both C) and D)
Ans: E
Topic: General Lipid Types and Information
19. OCH2CH2NH2 is a structural unit of which type of phosphatide?
A) Cephalins
B) Phosphatidyl serines
C) Plasmalogens
D) Lecithins
E) Both A) and C)
Ans: E
Topic: Fats and Oils
20. Which of the following would serve as the basis for a simple chemical test that would
distinguish between stearic acid and oleic acid?
A) NaOH/H2O
B) NaHCO3/H2O
C) HCl/H2O
D) Ag(NH3)2+
E) Br2/CCl4
Ans: E
Topic: Fats and Oils
21. Which of the following would not be helpful in distinguishing between oleic acid,
linoleic acid and linolenic acid?
A) Examine the stoichiometry of the reaction with Br2, CCl4
B) Examine the stoichiometry of complete catalytic hydrogenation.
C) Examine products obtained after subjecting the sample to catalytic hydrogenation
D) Examine products obtained after subjecting the sample to : i) O3, CH2Cl2; ii) Zn,
CH3CO2H
E) All of the above would be helpful.
Ans: C
875
Chapter 23
Topic: Fats and Oils
22. Which of the following might be helpful in distinguishing between oleic acid and
linoleic acid?
A) Examine the stoichiometry of the reaction with NaOH/H2O
B) Examine the stoichiometry of complete catalytic hydrogenation.
C) Examine products obtained after subjecting the sample to catalytic hydrogenation
D) Examine products obtained after subjecting the sample to : i) O3, CH2Cl2; ii) Zn,
CH3CO2H
E) Two of the above
Ans: E
Topic: Fats and Oils
23. Which fatty acid is not likely to occur commonly in natural sources?
A) CH3(CH2)12COOH
B)
CH3(CH2)14COOH
C)
CH3(CH2)5
(CH2)7CO2H
H
D)
CH3(CH2)4
CH2
H
E)
H
H
H
(CH2)7CO2H
H
CH3(CH2)12CHCH2CO2H
CH3
Ans: E
24. Which fatty acid is not likely to occur commonly in natural sources?
A) Tetradecanoic acid
B) Pentadecanoic acid
C) Hexadecanoic acid
D) (9Z,12Z)-octadeca-9,12-dienoic acid
E) (Z)-hexadeca-9-enoic acid
Ans: B
876
Chapter 23
25. Which fatty acid is not likely to occur commonly in natural sources?
A) (Z)- 11-Tetradecenoic acid
B) (Z)- 9-Pentadecenoic acid
C) Hexadecanoic acid
D) (9Z,12Z)- 9,12-octadecadienoic acid
E) (Z)-9-hexadecenoic acid
Ans: B
26. Which fatty acid is likely to occur commonly in natural sources?
OH
I
O
O
II
OH
OH
III
O
OH
IV
O
O
V
A)
B)
C)
D)
E)
Ans:
OH
II
I and IV
II and III
II, III and IV
V
E
Topic: Fats and Oils
27. Which of the following could be used to prepare myristic acid, CH3(CH2)12COOH?
A) CH3(CH2)11CH2Br, CN-, heat; then H3O+, heat
B) CH3(CH2)11CH2Br, Mg, (C2H5)2O; then CO2; then H3O+
C) CH3(CH2)12CHO, Ag2O, OH-; then H3O+
D) Answers A) and B)
E) Answers A), B), and C)
Ans: E
877
Chapter 23
Topic: Fats and Oils
28. Which of the following could be used to prepare myristic acid, CH3(CH2)12COOH?
A) CH3(CH2)11CH2Br, CN-, heat; then H3O+, heat
B) CH3(CH2)11CH2Br, Mg, (C2H5)2O; then CO2; then H3O+
C) CH3(CH2)13CHO, Ag2O, OH-; then H3O+
D) Answers A) and B)
E) Answers A), B), and C)
Ans: D
Topic: Fats and Oils
29. Which of the following could be used to prepare stearic acid, CH3(CH2)16COOH?
A) CH3(CH2)15CH2Br, CN-, heat; then H3O+, heat
B) CH3(CH2)15CH2Br, Mg, (C2H5)2O; then HCHO; then H3O+
C) CH3(CH2)16CH2Br, NaOH; then KMnO4,OH-, H2O; then H3O+
D) Answers A) and B)
E) Answers A) and C)
Ans: E
Topic: Fats and Oils
30. How could you synthesize stearolic acid, CH3(CH2)7C≡C(CH2)7COOH from oleic acid,
CH3(CH2)7CH=CH(CH2)7COOH?
A) Br2, CCl4; then 3 NaNH2, heat; then H3O+
B) Li, liq. NH3; then H3O+
C) H2, Pd
D) Peracid; then H3O+; then HA, H2O, heat
E) excess HCl; then KOH, C2H5OH, heat
Ans: A
Topic: Fats and Oils
31. Which of these reagents would not react with oleic acid?
A) H2, Ni
B) PBr3
C) CH3MgI
D) NH3/H2O
E) NaBH4
Ans: E
878
Chapter 23
Topic: Fats and Oils
32. Which of these reagents would not react with oleic acid?
A) H2, Ni
B) DIBAL-H
C) O3/CH2Cl2
D) NH3/H2O
E) LAH
Ans: B
Topic: Fats and Oils
33. Which of these reagents would not react with stearic acid?
A) H2, Ni
B) SOCl2
C) CH3MgI
D) NH3/H2O
E) LAH
Ans: A
Topic: Fats and Oils
34. What would be the product, X, of the following reaction sequence?
i. SOCl2
X
CH3(CH2)12COOH
ii. NH3
iii. Br2, NaOH
CH3(CH2)12CHCOOH
A)
Br
B)
CH3(CH2)11CHCONH2
Br
C)
CH3(CH2)11CH2NH2
D)
CH3(CH2)12CONH2
E)
CH3(CH2)12COBr
Ans: C
879
Chapter 23
Topic: Fats and Oils
35. The final product of the following reaction sequence would be:,
OH
O
i. PBr3
ii. LiAlH[OC(CH3)3]
iii. NaCN, HCl
+
iv. H3O , heat
?
OH
O
OH
O
I
II
O
O
OH
III
IV
O
OH
V
A)
B)
C)
D)
E)
Ans:
I
II
III
IV
V
D
Topic: Fats and Oils
36. What would be the product of the following reaction sequence?
i. H2, Ni
?
ii. SOCl2
iii. CH3(CH2)14CH2OH, HA
A) CH3(CH2)5CH=CH(CH2)14CH3
B) CH3(CH2)5COOCH2(CH2)14CH3 CH3(CH2)5CH=CH(CH2)7COOCH2(CH2)14CH3
C) CH3(CH2)14COOCH2(CH2)14CH3
D) CH3(CH2)13CHClCOOCH2(CH2)14CH3
E) CH3(CH2)5CH=CH(CH2)7COOCH2(CH2)14CH3
Ans: C
CH3(CH2)5CH=CH(CH2)7CO2H
880
Chapter 23
Topic: Fats and Oils
37. The product of the following reaction sequence may be described as?
i. H2, Ni
CH3(CH2)5CH=CH(CH2)7CO2H
?
ii. SOCl2
iii. CH3(CH2)14CH2OH, HA
A) Alkoxyalkane
B) Alkyl alkanoate
C) Alkyl alkenoate
D) Acyl glycerol
E) Acyl Alkane
Ans: B
881
Chapter 23
Topic: Fats and Oils
38. The product of the following reaction sequence is nervonic acid. What is the structure
of nervonic acid?
HC CNa
NaNH2
C10H17Na
C10H18
1-bromooctane
NH3(l)
ICH2(CH2)11CH2Cl
C24H43O2K
KOH
heat
C24H43N
NaCN
C23H43Cl
H3O+
H2, P-2
C24H44O2
A)
CH3(CH2)7
H
B)
CH3(CH2)7
H
C)
CH3(CH2)9
H
D)
E)
CH3(CH2)13
H
(CH2)13CO2H
(CH2)13CO2H
H
(CH2)11CO2H
H
H
H
(CH2)7CO2H
CH3(CH2)13
(CH2)7CO2H
H
C24H46O2 nervonic acid
H
Ans: B
882
Chapter 23
Topic: Fats and Oils
39. Which fatty acid is responsible for the putrid odor of rancid butter?
A) Valeric acid
B) Myristic acid
C) Stearic acid
D) Oleic acid
E) Butyric acid
Ans: E
Topic: Fats and Oils
40. How could you convert an unsaturated fatty acid into a saturated fatty acid?
A) KMnO4, OH-, heat
B) OH-, H2O, heat; then H3O+
C) H2, Ni, pressure
D) H3O+, H2O, heat
E) O3; then Zn, HOAc
Ans: C
Topic: Fats and Oils
41. Which is an untrue statement concerning the fatty acid moieties of naturally-occurring
triacylglycerols?
A) Generally, they possess an even number of carbon atoms.
B) Most have unbranched carbon chains.
C) The double bonds, when present, all are in the cis configuration.
D) Where two or three double bonds are present in the same fatty acid moiety, they
comprise a conjugated system.
E) The fatty acid moieties in a particular triacylglycerol usually are different.
Ans: D
Topic: Fats and Oils
42. Which is not a correct statement concerning naturally-occurring triacylglycerols?
A) The greater the degree of unsaturation, the higher the melting point.
B) Saponification yields glycerol and a mixture of carboxylic acid salts.
C) Solid examples are termed "fats."
D) Regardless of the exact nature of the R groups, such compounds are waterinsoluble.
E) Such compounds frequently, but less correctly, are called "triglycerides."
Ans: A
883
Chapter 23
Topic: Fats and Oils
43. Of the saturated fatty acids found in fats and oils, this one normally is the most
abundant:
A) Capric acid
B) Lauric acid
C) Myristic acid
D) Palmitic acid
E) Stearic acid
Ans: D
Topic: Fats and Oils
44. The biosynthesis of one series of prostaglandins begins with which of these fatty acids?
A) Palmitic acid
B) Stearic acid
C) Oleic acid
D) Linoleic acid
E) Arachidonic acid
Ans: E
Topic: Fats and Oils
45. The ozonolysis of a fatty acid produces these fragments:
O
O
OH
O
O
O
What is the identity of the fatty acid?
A) Stearic acid
B) Palmitoleic acid
C) Oleic acid
D) Linoleic acid
E) Linolenic acid
Ans: D
Topic: Fats and Oils
46. How many isomers, including stereoisomers, exist for the triacylglycerol which, on
saponification, gives 2 molar equivalents of palmitate and 1 molar equivalent of
stearate?
A) 1
B) 2
C) 3
D) 4
E) 6
Ans: C
884
Chapter 23
Topic: Soaps, Detergents, and Micelles
47. Consider a micelle composed of phosphatidyl choline, shown below. Which part(s) of
the molecule would form the hydrophilic surface of the micelle?
CH2OOC15H31
1
CHOOC17H35
2
CH2O
O
A)
B)
C)
D)
E)
Ans:
OCH2CH2N(CH3)3
P
3
O
3
2
1
1 and 2
2 and 3
A
Topic: Soaps, Detergents, and Micelles
48. Which of these is a detergent?
A) CH3(CH2)16COO-Na+
B) [CH3(CH2)14COO-]2Ca+2
C) CH3(CH2)10CH2SO3-Na+
D) HOCH2CHOHCH2OH
E) CH3(CH2)14CH2SH
Ans: C
Topic: Terpenes
49. Which compound is a sesquiterpene?
OH
I
III
A)
B)
C)
D)
E)
Ans:
II
IV
V
I
II
III
IV
V
D
885
Chapter 23
Topic: Terpenes
50. Which structure represents a terpene likely to be found in nature?
O
OH
II
I
III
A)
B)
C)
D)
E)
Ans:
IV
V
I
II
III
IV
V
C
Topic: Terpenes
51. To which class of terpenes does the terpene shown below, bisabolene, belong?
A)
B)
C)
D)
E)
Ans:
Monoterpenes
Sesquiterpenes
Diterpenes
Triterpenes
Tetraterpenes
B
Topic: Terpenes
52. How many isoprene units are in vitamin A?
OH
A)
B)
C)
D)
E)
Ans:
1
2
3
4
More than 4
D
886
Chapter 23
Topic: Terpenes
53. In β-carotene, how many tail-to-tail links of isoprene units are there?
A)
B)
C)
D)
E)
Ans:
1
2
3
4
More than 4
A
Topic: Terpenes
54. Which compound below does not obey the isoprene rule?
OH
OH
I
II
HO
OH
III
A)
B)
C)
D)
E)
Ans:
IV
V
I
II
III
IV
V
D
887
Chapter 23
Topic: Terpenes
55. Which of the following compounds would be most likely to be found in nature? (Hint:
recall the isoprene rule.)
OH
OH
I
II
OH
OH
III
IV
OH
V
A)
B)
C)
D)
E)
Ans:
I
II
III
IV
V
D
Topic: Rubber
56. Which is the proper representation of three successive isoprene units in natural rubber?
H
H
H
H
H
H
I
H
II
H
H
H
H
H
IV
III
H
H
H
V
A)
B)
C)
D)
E)
Ans:
I
II
III
IV
V
B
888
Chapter 23
Topic: Rubber
57. In the vulcanization of rubber,
A) natural rubber is heated with sulfur.
B) reaction occurs at allylic positions.
C) cross-linking results in a hardening of the rubber.
D) disulfide bridges are formed.
E) All of the above
Ans: E
Topic: Rubber
58. Which is the repeating unit of natural rubber?
I
A)
B)
C)
D)
E)
Ans:
II
III
IV
V
I
II
III
IV
V
A
Topic: Steroids
59. What product would be obtained by catalytic hydrogenation of 5-cholesten-3β-ol?
A) 5β-Cholestan-3β-ol
B) 5∝-Cholestan-3β-ol
C) 5∝-Cholestan-3∝-ol
D) 5β-Cholestan-3∝-ol
E) 5-Cholesten-3β,6∝-diol
Ans: B
Topic: Steroids
60. Which reagent might be used to convert 5∝-cholest-1-en-3-ol into 5∝-cholestan-3-ol?
A) CrO3/pyridine
B) KMnO4/H2O
C) CH3MgI
D) H2/Pt
E) Li/C2H5NH2
Ans: D
889
Chapter 23
Topic: Steroids
61. Which reagent might serve as the basis for a simple chemical test that would distinguish
between 5∝-cholest-1-en-3-one and 5∝-cholestan-3-one?
A) Ag(NH3)2+
B) CrO3/H2SO4
C) Br2/CCl4
D) NaOH/H2O
E) C6H5NHNH2
Ans: C
Topic: Steroids
62. The synthesis of cortisone required placing a ketone function at the 11-position of a
steroid. Where is position 11?
III
IV
II
V
I
A)
B)
C)
D)
E)
Ans:
I
II
III
IV
V
C
Topic: Steroids
63. How many stereogenic centers are there in cholesterol?
HO
A)
B)
C)
D)
E)
Ans:
2
4
6
8
16
D
890
Chapter 23
Topic: Steroids
64. Which is an untrue statement concerning cholesterol?
A) Cholesterol decolorizes a solution of Br2 in CCl4.
B) Cholesterol reacts with 2,4-dinitrophenylhydrazine.
C) Cholesterol is optically active.
D) Cholesterol is water-insoluble.
E) All of the above are true.
Ans: B
Topic: Steroids
65. The reaction of cholesterol with dilute aqueous KMnO4 at 0–5°C produces which of
these compounds (A and B rings only shown)?
HO
HO
OH
HO
OH
I
II
HO
HO OH
IV
A)
B)
C)
D)
E)
Ans:
HO
HO
III
O
HO
V
I
II
III
IV
V
D
891
O
OH
Chapter 23
Topic: Steroids
66. In the biosynthesis of vitamin D2, which alkane bond of ergosterol is cleaved?
II
I
IV
III
HO
V
A)
B)
C)
D)
E)
Ans:
I
II
III
IV
V
D
Topic: Steroids
67. Which of the following is a female sex hormone?
A) Ergosterol
B) Estradiol
C) Cortisone
D) Androsterone
E) Cholic acid
Ans: B
Topic: Steroids
68. Which of these is a correct systematic name for progesterone?
O
O
Progesterone
A) 2-Estrene-4,20-dione
B) 5-Androstene-4,19-dione
C) 4-Pregnene-3,20-dione
D) 5-Cholestene-5,19-dione
E) 4-Cholene-3,20-dione
Ans: C
892
Chapter 23
Topic: Steroids
69. Shown below is the formula for the antiinflammatory drug called prednisone. What is a
correct systematic name for prednisone?
O
O
OH
OH
O
A)
B)
C)
D)
E)
Ans:
Prednisone
17∝,21-Dihydroxypregna-1,4-diene-3,11,20-trione
17β,21-Dihydroxypregna-1,4-diene-3,11,20-trione
17∝,19-Dihydroxypregna-1,4-diene-3,11,20-trione
17β,19-Dihydroxypregna-1,4-diene-3,11,20-trione
None of the above
A
Topic: Steroids
70. What product would you expect when progesterone is treated with one molar equivalent
of hydrogen in the presence of a platinum catalyst?
O
O
A)
B)
C)
D)
E)
Ans:
Progesterone
5∝-Pregnane-3,20-dione
5β-Pregnane-3,20-dione
5∝-Estrane-3,20-dione
5β-Estrane-3,20-dione
5∝-Androstane-3,20-dione
A
893
Chapter 23
Topic: Steroids
71. Which is the correct systematic name for the steroid shown below?
CH3
H
H3C
H
H
HO
H
A)
B)
C)
D)
E)
Ans:
5∝-Androstan-3∝-ol
5β-Androstan-3β-ol
5β-Androstan-3∝-ol
5∝-Androstan-3β-ol
5β-Estan-3β-ol
D
Topic: Steroids
72. Which of these is a male sex hormone?
A) Estrone
B) Testosterone
C) Cholic acid
D) Cortisone
E) Estradiol
Ans: B
Topic: Prostaglandins
73. Which of the following characteristics are found in the class of C20 carboxylic acids
called prostaglandins?
A) a five membered ring
B) One or more double bonds
C) Several oxygen containing groups
D) Two of the above
E) All of the above
Ans: E
SHORT ANSWER QUESTIONS
Topic: Lipids
74. Unlike carbohydrates and proteins, which are defined in terms of their structures, lipids
are defined in terms of _____________.
Ans: the physical operation used to isolate them
894
Chapter 23
Topic: Fats and Oils
75. Saturated fatty acids pack efficiently. Therefore the corresponding triacylglycerols tend
to be __________ at room temperature and are called __________.
Ans: solid; fats
Topic: Fats and Oils
76. Unsaturated fatty acids have a bend in the carbon chain that interferes with packing.
Therefore the corresponding triacylglycerols are generally __________ at room
temperature and are called _______.
Ans: liquid; oils
Topic: Fats and Oils
77. The primary function of triacylglycerols (triglycerides) in animals is _____________.
Ans: energy storage
Topic: Fats and Oils
78. Salts of long-chain fatty acids are known as ___________.
Ans: soaps
Topic: Terpenes
79. Terpenes are built up from two or more five-carbon units known as __________.
Ans: isoprene units
Topic: Terpenes
80. When natural rubber is heated with sulfur, a reaction takes place that produces
_____________ between the polymer chains. This process is known as _____________.
Ans: cross-links; vulcanization
Topic: Steroid Hormones
81. There are three major classes of sex hormones:
a. female sex hormones [or ____________]
b. male sex hormones [or ____________]
c. pregnancy hormones [or ______________]
Ans: a. estrogens; b. androgens; c. progestins
Topic: Fats and Oils
82. Sphingolipids, proteins, and polysaccharides make up ___________, the protective
coating around nerve fibers.
Ans: myelin
Topic: Fats and Oils
83. Explain how soap removes dirt from clothes.
Ans: The hydrophobic end dissolves in the dirt and oil, forming a micelle with an ionic
outer layer. The outer layer is hydrophilic and makes it easy for the micelle to
wash away, carrying the dirt with it.
895
Chapter 23
Topic: Fats and Oils
84. Myelin, the protective coating around nerve fibers, is made up of ___________.
Ans: sphingolipids, proteins, and polysaccharides
Topic: Fats and Oils
85. The characteristic odor of rancid butter is attributed to ___________.
Ans: the release of butyric acid during decomposition.
Topic: Fats and Oils
86. In naturally occurring polyunsaturated fatty acids, the relative position of double bonds
is rarely ___________.
Ans: conjugated
Topic: Fats and Oils
87. There is growing evidence that “trans” fats are associated with an increased risk of
___________, and the FDA now requires “trans fat” information to be provided on
nutrition labels.
Ans: cardiovascular disease
Topic: Fats and Oils
88. In naturally occurring polyunsaturated fatty acids, the stereochemistry of the double
bonds is rarely ___________.
Ans: trans-
Topic: Fats and Oils
89. Prolonged heating of naturally occurring polyunsaturated fats may cause the
stereochemistry of the double bonds to change from ___________.
Ans: cis- to trans-
Topic: Fats and Oils
90. Most waxes, commonly found as protective coatings on skin, fur, hair, leaves, fruits,
etc., are ____________.
Ans: Esters of long chain fatty acids and long chain alcohols.
896
Chapter 23
Topic: Fats and Oils
91. Olestra, a synthetic fat substitute, contributes nothing to the caloric value of foods
prepared with it, because it passes through the body without being digested. Why are
the lipases, responsible for facilitating normal fat hydrolysis, unable to facilitate the
hydrolysis of olestra?
Ans: In order for a lipase to catalyze hydrolysis, the fat molecule must be accepted into
the active site of the enzyme. Although olestra contains 6-8 ester groups, the
molecule is extremely bulky: this steric bulk prevents lipases from accepting the
molecule into their active sites. Consequently, the olestra is not hydrolyzed,
hence not absorbed, so that it adds nothing to the caloric value of foods prepared
with it.
Topic: Fats and Oils
92. How could you synthesize stearolic acid, CH3(CH2)7C≡C(CH2)7COOH from oleic acid,
CH3(CH2)7CH=CH(CH2)7COOH? Specify stoichiometric details, providing a brief
explanation of your rationale.
Ans:
Topic: Fats and Oils
93. How are soaps and phosphatides similar in their interaction with water?
Ans: Both have polar and non-polar groups, so that they appear to “dissolve” when
mixed with water. In actuality, they tend to form spherical micelles: the
molecules aggregate in such a way that the polar groups of all the molecules in the
micelle face the water, while the non-polar groups face inward, toward the center
of the micelle, thus maximizing both hydrophilic as well as hydrophobic
interactions.
Topic: Fats and Oils
94. Suggest a reasonable synthetic strategy for the synthesis of N,N-dimethylstearamide ,
CH3(CH2)16CON(CH3)2 from oleic acid, cis 9-octadecenoic acid
Ans: i) H2,Ni; ii) PBr3; iii) (CH3)2NH
H2
CH3(CH2)16CO2H
CH3(CH2)7CH=CH(CH2)7CO2H
Ni
PBr3
CH3(CH2)16CON(CH3)2
(CH3)2NH
897
CH3(CH2)16COBr
Chapter 23
Topic: Synthesis
95. One of the intermediates in a prostaglandin synthesis is shown below.
NO2
CN
O
O
CHO
Which reaction might be employed to make this molecule from 1,1-dimethoxy-3nitropropane and an appropriate α,β-unsaturated aldehyde? Show the structural details
of the reagents as well as any catalysts that might be needed.
Ans: Deprotonation (with base) of the active hydrogen alpha to the –NO2 group would
afford a nucleophilic species. Conjugate addition (Michael reaction) of this
species to either isomer (cis/trans) of the α,β-unsaturated aldehyde drawn below
should afford the desired compound in acceptable yields.
CN
NO2
NO2
O
CN
base
+
O
or
O
O
O
CN
898
O
CHO
Chapter 23
Topic: Soaps and Synthetic Detergents
96. Draw the structures of the following substances, in which the alkyl portion is
CH3(CH2)12−:
sodium carboxylate,
sodium alkanesulfonate and
sodium alkyl sulfate
Explain why these substances may be useful as “dirt removers”.
Ans:
−
O Na
sodium carboxylate
+
O
O
−
sodium alkanesulfonate
S O Na
+
O
O
−
sodium alkyl sulfate
O
+
S O Na
O
Each of these
substances has a polar “head” and a non-polar “tail”. This allows them to form
micelles when mixed with water. These micelle clusters are dispersed throughout
the aqueous phase, with the polar heads on the outside and the non-polar ends on
the inside of the spherical clusters. When these micelles come into contact with
the non-polar, oily surface of “dirt”, the dirt gets attracted to the non-polar center
of the micelle, where it is then “trapped”. Upon rinsing, the micelles, along with
the trapped dirt, get washed away.
Topic: Terpenes
97. What products would be obtained from the ozonolysis of zingiberine, found in the oil of
ginger?
i) O3
ii) Zn, CH3CO2H
Zingiberine
Ans:
?
O
O
O
O
+
+ O
O
899
Chapter 23
Topic: Terpenes
98. Identify the bonds that link the isoprene units in α-farnesene, shown below. How are
these units linked: head to head, tail to head or tail to tail ?
α-farnesene
Ans:
head
tail
head
tail
head
tail
α-farnesene
900
Chapter 23
Topic: Analysis and Structure Elucidation
99. A compound “Z” (C10H16), has λmax at 184 nm in its UV spectrum. Catalytic
hydrogenation of Z affords a mixture of diasteromers, which upon separation are
identified to be cis-1-isopropyl-4-methylcyclohexane and trans-1-isopropyl-4methylcyclohexane. Upon ozonolysis, Z yields formaldehyde and 3-acetyl-6oxoheptanal. Suggest a structure for Z that is consistent with all of the above data,
clearly explaining your rationale.
Ans:
*
excess H2,
Ni
λmax=184 nm
cis- and trans-
O
ozonolysis
O
H2C=O +
*
O
- IHD for Z: (C10H22 - C10H16)/2 = 3
- Hydrogenation affords 1-isopropyl-4-methylcyclohexane: so, Z must have the
same carbon skeleton, along with 2 pi bonds, i.e., 1 ring and 2 pi bonds (so that
IHD =3)
- Examination of the above carbon skeleton suggests that the pi bonds cannot
constitute an alkyne, so that the pi bonds must be present as 2 double bonds.
- λmax at 184 nm in its UV spectrum suggests that these pi bonds are not
conjugated.
- The stereochemistry of the hydrogenation products suggests that there must be a
pi bond at C-4 and a chiral center at C-1: hydrogenation eliminates the
asymmetry (and therefore chirality) at C-4. Hydrogenation from both faces of
the pi bond at C-4 would lead to cis- and trans- orientation of the methyl and
isopropyl groups on the cyclohexane ring.
- The ozonolysis products suggest the position of the 2 double bonds: since there
are only 2 products, one bond must be within the ring, the other outside the ring.
901
Chapter 24
MULTIPLE CHOICE QUESTIONS
Topic: Isoelectric Points and pH
1. Which amino acid would have its isoelectric point near pH 10?
CH2COO
H3NCH2CH2CH2CH2CHCOO
NH3
H3NCHCOO
NH3
NH2
I
II
HO
III
HOOCCH2CH2CHCOO
CH2CHCOO
NH3
NH3
IV
A)
B)
C)
D)
E)
Ans:
V
I
II
III
IV
V
B
Topic: Isoelectric Points and pH
2. Which amino acid would have its isoelectric point near pH 3?
H3CCHCOO
CH2CHCOO
NH3
NH3
I
HOOCCH2CH2CHCOO
NH3
II
III
NH2
H3NCH2CH2CH2CH2CHCOO
H2NCCH2CH2CH2CHCOO
NH2
NH2
IV
A)
B)
C)
D)
E)
Ans:
V
I
II
III
IV
V
C
902
Chapter 24
Topic: Isoelectric Points and pH
3. Which amino acid would have its isoelectric point near pH 10?
CO2
CO2
CO2
H2N
H3N
H
H
H
CH2CH2CH2NH3
CH2CH2CO2H
I
II
CO2
H
CH2OH
IV
A)
B)
C)
D)
E)
Ans:
CH2SH
III
CO2
H3N
NH3
H3N
H
CH2C6H5
V
I
II
III
IV
V
A
Topic: Isoelectric Points and pH
4. Which amino acid would have its isoelectric point near pH 10?
A) Glycine
B) Tryptophan
C) Serine
D) Proline
E) Lysine
Ans: E
Topic: Isoelectric Points and pH
5. Which amino acid would not have its isoelectric point in the pH range 5-7?
A) Leucine
B) Threonine
C) Methionine
D) Arginine
E) Cystine
Ans: D
903
Chapter 24
Topic: Isoelectric Points and pH
6. Which amino acid would not have its isoelectric point in the pH range 5-7?
A) Glycine
B) Proline
C) Cysteine
D) Glutamine
E) All of these amino acids have isoelectric point in the pH range 5-7
Ans: E
Topic: Isoelectric Points and pH
7. What might be concluded upon determining that an unknown amino acid has its
isoelectric point near pH 10?
A) It must have a hydrophobic side chain
B) It must have a hydrophilic side chain
C) Its side chain must contain a basic group
D) Its side chain must contain an acidic group
E) None of the above is a valid conclusion
Ans: C
Topic: Isoelectric Points and pH
8. What might be concluded upon determining that an unknown amino acid has its
isoelectric point near pH 3?
A) It must have a hydrophobic side chain
B) It must have a hydrophilic side chain
C) Its side chain must contain a basic group
D) Its side chain must contain an acidic group
E) None of the above is a valid conclusion
Ans: D
Topic: Isoelectric Points and pH
9. The pH at which the concentration of the dipolar ion (zwitterion) form of an amino acid
is at a maximum and the cationic and anionic forms are at equal concentrations is
termed the
A) end point.
B) equivalence point.
C) neutral point.
D) isoelectric point.
E) dipolar point.
Ans: D
904
Chapter 24
Topic: Isoelectric Points and pH
10. What is the pI of the following amino acid?
HO2C
(CH2)n
CO2H
NH2
pKa1= 2.4
pKa2= 10.0
α-CO2H
A) 1.6
B) 3.2
C) 5.5
D) 6.2
E) 7.0
Ans: B
α-NH3+
pKa3= 4.0
R group
Topic: Isoelectric Points and pH
11 What is the pI of the following amino acid?
HO2C
(CH2)n
CO2H
NH2
pKa1= 2.2
pKa2= 9.7
α-CO2H
α-NH3+
A)
B)
C)
D)
E)
Ans:
pKa3= 4.3
R group
2.1
3.2
5.9
6.5
7.0
B
905
Chapter 24
Topic: Isoelectric Points and pH
12. What is the pI of the following amino acid?
HO2C
(CH2)n
NH2
NH2
pKa1= 2.2
pKa2= 9.0
+
α-NH3
α-CO2H
A) 1.5
B) 6.3
C) 5.6
D) 9.8
E) 6.8
Ans: D
pKa3= 10.5
R group
Topic: Isoelectric Points and pH
13. What is the pI of the following amino acid?
HO2C
(CH2)n
CH3
NH2
pKa1= 2.4
pKa2= 9.6
α-CO2H
A) 12
B) 3.6
C) 4.8
D) 6.0
E) 7.2
Ans: D
α-NH3
+
906
Chapter 24
Topic: Isoelectric Points and pH
14. Consider the following tripeptide:
H2N-leu-lys-phe-CO2H
NH2
What is the best estimate of the pI of this compound?
pKa
leu
lys
phe
A)
B)
C)
D)
E)
Ans:
pKa
COOH
2.4
2.2
1.8
6.2
7.1
9.2
10.1
11.3
D
pKa
NH3+
9.6
9.0
9.1
pI
R
6.0
9.8
5.5
10.5
Topic: Isoelectric Points and pH
15. What would be the predominant form of lysine in water at pH 14?
H3N
CO2H
H3N
CO2
H3N
NH3
NH3
I
NH2
II
H2N
CO2
III
H2N
CO2
NH3
NH2
IV
A)
B)
C)
D)
E)
Ans:
CO2
V
I
II
III
IV
V
E
907
Chapter 24
Topic: Isoelectric Points and pH
16. The predominant form of aspartic acid in water at pH 1 would be:
CO2H
HO2C
CO2H
O2 C
NH3
NH3
NH3
I
II
CO2H
O2C
III
CO2
O2C
NH2
NH2
IV
A)
B)
C)
D)
E)
Ans:
CO2
HO2C
V
I
II
III
IV
V
A
Topic: Isoelectric Points and pH
17. For the accompanying fully-protonated amino acid, what is the arrangement of pKa
values in order of increasing magnitude?
HO2C
I
CO2H
NH3
III
II
A) I < II < III
B) II < I < III
C) III < I < II
D) III < II < I
E) II < III < I
Ans: C
Topic: Amino Acid Structure
18. Which of these amino acids contains a hydrophobic side chain?
A) Lysine
B) Serine
C) Methionine
D) Arginine
E) Cysteine
Ans: C
908
Chapter 24
Topic: Amino Acid Structure
19. Which of these natural amino acids contains an amide function?
A) Asparagine
B) Proline
C) Arginine
D) Histidine
E) None of these
Ans: A
Topic: Amino Acid Structure
20. Which of these natural amino acids contains a heterocyclic ring?
A) Asparagine
B) Proline
C) Arginine
D) Histidine
E) Two of these
Ans: E
Topic: Amino Acid Structure
21. Which of these natural amino acids contains an amide function?
A) Asparagine
B) Methionine
C) Cysteine
D) Glutamine
E) Two of these
Ans: E
Topic: Amino Acid Structure
22. Which of these natural amino acids contains two carboxylic acid groups?
A) Cystine
B) Cysteine
C) Glutamic acid
D) A and B
E) A and C
Ans: A
909
Chapter 24
Topic: Amino Acid Structure
23. Which of these natural amino acids contains a phenolic group?
A) Phenylalanine
B) Tyrosine
C) Tryptophan
D) 4-Hydroxyproline
E) Serine
Ans: B
Topic: Amino Acid Structure
24. Which of these natural amino acids contains a pyrrolidine ring?
A) Phenylalanine
B) Tyrosine
C) Tryptophan
D) 4-Hydroxyproline
E) Serine
Ans: D
Topic: Amino Acid Structure
25. Which of these natural amino acids contains an indole ring?
A) Phenylalanine
B) Tyrosine
C) Tryptophan
D) 4-Hydroxyproline
E) Asparigine
Ans: C
Topic: Amino Acid Structure
26. Which of these natural amino acids contains an imidazole ring?
A) Histidine
B) Lysine
C) Tryptophan
D) 4-Hydroxyproline
E) Two of the above
Ans: A
910
Chapter 24
Topic: Amino Acid Structure
27. Which of these natural amino acids contains an –OH group?
A) Serine
B) Threonine
C) Tyrosine
D) Two of these
E) All of these
Ans: E
Topic: Amino Acid Structure
28. Which of these natural amino acids, when present in a polypeptide, is likely to exhibit
significant hydrogen bonding through its side chain?
A) Serine
B) Threonine
C) Tyrosine
D) Two of these
E) All of these
Ans: E
Topic: Amino Acid Structure
29. Which of these natural amino acids, when present in a polypeptide, is not likely to
exhibit significant hydrogen bonding through its side chain?
A) Leucine
B) Threonine
C) Tyrosine
D) Serine
E) All of these are likely to exhibit significant hydrogen bonding through the side
chain
Ans: A
911
Chapter 24
Topic: Amino Acid Structure
30. Which amino acid is least likely to be found in a natural protein?
H
CO2H
H
CO2
H3N
CO2
H
H3N
H
NH2
CH3
CH2OH
I
II
III
CO2
CO2
H
NH3
H3N
CH3
CH2C6H5
IV
A)
B)
C)
D)
E)
Ans:
H
V
I
II
III
IV
V
D
Topic: Amino Acid Structure
31. Which amino acid is unlikely to be found in a natural protein?
CO2
H3N
H3N
H
CO2
H3N
H
CH3
CH2OH
CH2C6H5
I
II
III
CO2
CO2
H
NH3
CH2SH
IV
A)
B)
C)
D)
E)
Ans:
CO2
H
H3N
H
H
V
I
II
III
IV
V
D
912
Chapter 24
Topic: Amino Acid Structure
32. Which of these amino acids is formed from a precursor amino acid only after the latter
has been incorporated into a polypeptide chain?
A) Serine
B) Arginine
C) Isoleucine
D) Tryptophan
E) Hydroxyproline
Ans: E
Topic: Amino Acid Structure
33. Which of these amino acids is described as an “essential” amino acid?
A) Methionine
B) Phenylalanine
C) Isoleucine
D) Tryptophan
E) All of these are “essential” amino acids
Ans: E
Topic: Amino Acid Structure
34. Which of these amino acids is described as an “essential” amino acid?
A) Threonine
B) Glycine
C) Tyrosine
D) Serine
E) All of these are “essential” amino acids
Ans: A
Topic: Amino Acid Structure
35. Pipecolic acid logically would be substituted for which natural amino acid in the
synthesis of peptide analogs?
NH
Pipecolic acid
CO2H
A)
B)
C)
D)
E)
Ans:
Histidine
Proline
Tryptophan
Phenylalanine
Tyrosine
B
913
Chapter 24
Topic: Amino Acid Structure
36. Logically, the following “unnatural” amino acid would be substituted for which natural
amino acid in the synthesis of peptide analogs?
HO
CO2H
NH2
A)
B)
C)
D)
E)
Ans:
Tyrosine
Proline
Tryptophan
Phenylalanine
Tyrosine
A
Topic: Amino Acid Structure
37. Logically, the following “unnatural” amino acid would be substituted for which natural
amino acid in the synthesis of peptide analogs?
CO2H
HS
A)
B)
C)
D)
E)
Ans:
NH2
Methionine
Cysteine
Cystine
Tyrosine
It could be substituted for all of these amino acids
D
Topic: Amino Acid Structure
38. Logically, the following “unnatural” amino acid would be substituted for which natural
amino acid in the synthesis of peptide analogs?
HO2C
CO2H
NH2
A)
B)
C)
D)
E)
Ans:
Aspartic acid
Glutamine
Lysine
Asparagine
Glutamic acid
E
914
Chapter 24
Topic: Amino Acid Structure
39. Logically, the following “unnatural” amino acid would be substituted for which natural
amino acid in the synthesis of peptide analogs?
CO2H
H2N
NH2
A)
B)
C)
D)
E)
Ans:
Leucine
Lysine
Arginine
Alanine
Valine
B
Topic: Synthesis and Reactions of Amino Acids
40. What product would be obtained upon treating alanine with the following reagent ?
O
O
Cl
O
O
CO2
O
NH3
O
NH3
I
II
III
O
N
H
CO2H
O
NH2
IV
A)
B)
C)
D)
E)
Ans:
N
H
V
I
II
III
IV
V
C
915
CO2H
Chapter 24
Topic: Synthesis and Reactions of Amino Acids
41. Which of the following would provide a synthesis of alanine?
A) CH2=CHCH2OH, HBr, CrO3/H2SO4/H2O; then xs NH3
B) Potassium phthalimide, ClCH2CO2C2H5; then KOH/H2O; then HCl
C) Potassium phthalimide, C6H5CH2Br; then KOH/H2O; then CO2,H3O+
D) CH3CH2COOH, (C6H5)3CNa; then NH3
E) Answers A) and B)
Ans: A
Topic: Synthesis and Reactions of Amino Acids
42. Which of the following would provide a synthesis of valine?
A) (CH3)2C=CHCH2OH, HBr/peroxides; CrO3/H2SO4/H2O; then xs NH3
B) Potassium phthalimide, (CH3)2CHCHClCO2C2H5; then KOH/H2O; then HCl
C) Potassium phthalimide, (CH3)2CHCH2Br; then KOH/H2O; then CO2,H3O+
D) CH3CH2COOH, (C6H5)3CNa; then NH3
E) Answers A) and B)
Ans: A
Topic: Synthesis and Reactions of Amino Acids
43. Which of the following would provide a synthesis of leucine?
A) (CH3)2C=CHCH2OH, HBr/peroxides, CrO3/H2SO4/H2O; then excess NH3
B) Potassium phthalimide, BrCH(CO2C2H5)2; (CH3)2CHCHClCO2C2H5; then
KOH/H2O; then HCl (85%), heat
C) Potassium phthalimide, BrCH(CO2C2H5)2; (CH3)2CHCH2Br; then KOH/H2O;
then HCl (85%), heat
D) (CH3)2CHCOOH, PCl5; then NH3
E) Answers A) and C)
Ans: C
Topic: Synthesis and Reactions of Amino Acids
44. Which of the following would provide a synthesis of phenylalanine?
A) Phenylacetaldehyde, NH3, HCN; H3O+, heat
B) Potassium phthalimide, BrCH(CO2C2H5)2; C6H5CH2Br; then KOH/H2O; then HCl
(85%), heat
C) Potassium phthalimide, (C6H5)CH2CH2Br; then KOH/H2O; then HCl (85%), heat
D) C6H5CH2COOH, SOCl2; then NH3
E) Answers A) and B)
Ans: E
916
Chapter 24
Topic: Synthesis and Reactions of Amino Acids
45. What product(s) would you expect from the following reaction?
Tyrosine
+
Br2 (excess)
H2O
?
Br
CO2
HO
Br
CO2
Br
NH3
CO2
NH3
HO
NH3
Br
Br
I
II
III
Br
O
CO2
Br
NH3
HO
NH3
HO
IV
A)
B)
C)
D)
E)
Ans:
V
I
II
III
IV
V
B
Topic: Synthesis and Reactions of Amino Acids
46. Why is this sequence, CH2=CHCH2OH + HBr, then CrO3/H2SO4/H2O, finally xs NH3,
not a good method for the preparation of L-alanine?
A) NH3 is not sufficiently nucleophilic to perform the final step.
B) HBr does not add to substituted alkenes.
C) 1° alcohols are not oxidized by CrO3 in acidic solution.
D) Initial HBr addition produces a racemic intermediate which leads to racemic
product.
E) Steric hindrance precludes nucleophilic substitution at a 2° carbon atom.
Ans: D
917
Chapter 24
Topic: Synthesis and Reactions of Amino Acids
47. What products would you expect from the following reaction?
O
O
H2
Pd
N
H
A)
OH
B)
O
+
?
N
H
+ CO2 + H2N
C)
O
O
D)
O
N
H
E)
OH
+ H2N
+ CO2 + H2N
Ans: B
Topic: Synthesis and Reactions of Amino Acids
48. Which is an isolable intermediate in the Strecker synthesis of an amino acid?
X
R
CO2H
I
R
R
CN
II
NH2
R
CHO
IV
A)
B)
C)
D)
E)
Ans:
NH2
NH2
OH
III
CO2
NH
CO2
R
CO2
O
V
I
II
III
IV
V
B
918
Chapter 24
Topic: Protein Structure
49. The primary structure of a protein refers to its:
A) sequence of amino acid residues.
B) disulfide bonds.
C) helical structure.
D) hydrogen bonding.
E) All of these
Ans: A
Topic: Protein Structure
50. The secondary structure of proteins is derived from:
A) peptide linkages.
B) disulfide linkages.
C) hydrogen bond formation.
D) hydrophobic interactions.
E) acid-base interactions.
Ans: C
Topic: Protein Structure
51. Which attractive force is responsible for maintaining the tertiary structure of a protein?
A) Disulfide linkages
B) Hydrogen bonds
C) van der Waals forces
D) Hydrophobic interactions
E) All of these
Ans: E
Topic: Protein Structure
52. The occurrence of this amino acid in a polypeptide chain disrupts an ∝-helix:
A) Proline
B) Alanine
C) Methionine
D) Histidine
E) Tyrosine
Ans: A
Topic: Protein Structure
53. Disulfide bonds in proteins:
A) result from an oxidation of thiols.
B) help to maintain the shape of proteins.
C) can be broken by reduction.
D) can link two cysteine amino acid residues.
E) All of the above
Ans: E
919
Chapter 24
Topic: Protein Structure
54. Which of these amino acid residues is expected to prefer the interior of a protein to the
exterior when the protein is in aqueous medium?
A) Threonine
B) Valine
C) Serine
D) Aspartic acid
E) Lysine
Ans: B
Topic: Protein Structure
55. A "conjugated protein" is one which:
A) possesses catalytic properties.
B) is a digestive enzyme.
C) exists largely as an ∝-helix.
D) contains unsaturated amino acids.
E) contains a nonprotein group as part of the molecule.
Ans: E
Topic: Peptide Sequencing, Labeling, and Synthesis
56. A pentapeptide has the molecular formula: Asp, Glu, His, Phe, Val. Partial hydrolysis
of the pentapeptide gives: Val·Asp, Glu·His, Phe·Val, and Asp·Glu. What is the amino
acid sequence of the pentapeptide?
A) Phe·Val·Asp·Glu·His
B) His·Glu·Asp·Val·Phe
C) Asp·Glu·His·Phe·Val
D) Phe·Val·Glu·His·Asp
E) Glu·His·Phe·Val·Asp
Ans: A
Topic: Peptide Sequencing, Labeling, and Synthesis
57. A heptapeptide Ala2, Glu, Phe, Pro, Tyr, Val gives labeled alanine when heated with
DNFB followed by hydrolysis. On partial hydrolysis the unlabeled heptapeptide gives
the following:
Ala·Glu, Pro·Tyr, Ala·Val, Tyr·Ala, Val·Phe·Pro.
What is the amino acid sequence of the heptapeptide?
A) Ala·Phe·Pro·Tyr·Ala·Glu·Val
B) Ala·Val·Phe·Pro·Tyr·Ala·Glu
C) Ala·Val·Phe·Pro·Tyr·Glu·Ala
D) Ala·Val·Phe·Tyr·Pro·Ala·Glu
E) Val·Ala·Phe·Tyr·Pro·Ala·Glu
Ans: B
920
Chapter 24
Topic: Peptide Sequencing, Labeling, and Synthesis
58. When the pentapeptide below is heated first with 2,4-dinitrofluorobenzene (and base)
and then subjected to acidic hydrolysis, which amino acid will bear the dinitrophenyl
group?
Leu·Val·Gly·Phe·Ile
A) Leucine
B) Valine
C) Glycine
D) Phenylalanine
E) Isoleucine
Ans: A
Topic: Peptide Sequencing, Labeling, and Synthesis
59. The Edman degradation uses this reagent to identify the N-terminal amino acid of a
peptide or protein.
A) C6H5NHNH2
B) C6H5NH2
C) C6H5N=C=S
D) C6H5N=C=O
E) Aminopeptidase
Ans: C
Topic: Peptide Sequencing, Labeling, and Synthesis
60. Which amino acid of a polypeptide would become labeled when the polypeptide is
treated with 2,4-dinitrofluorobenzene in base, even though the amino acid is not a
terminal amino acid?
A) Lysine
B) Glycine
C) Alanine
D) Phenylalanine
E) Leucine
Ans: A
Topic: Peptide Sequencing, Labeling, and Synthesis
61. The purple color of the anion formed in the ninhydrin test for ∝-amino acids is due to:
A) the attraction of the anion to a metal in a pi-complex.
B) intermolecular hydrogen bonding.
C) molecular vibrations.
D) the highly conjugated nature of the anion.
E) the color of the ninhydrin.
Ans: D
921
Chapter 24
Topic: Peptide Sequencing, Labeling, and Synthesis
62. Which one of these amino acids does not give the usual purple color with ninhydrin?
A) Histidine
B) Proline
C) Tryptophan
D) Leucine
E) Aspartic acid
Ans: B
Topic: Peptide Sequencing, Labeling, and Synthesis
63. This reagent is used to "protect" the amino group of an amino acid which is to be joined
to a second amino acid by a peptide bond.
O
A)
Cl
O
B)
C6H5
Cl
O
C)
C6H5
O
O
D)
C6H5
Cl
O
O
C6H5
O
E)
O
Cl
Ans: C
Topic: Peptide Sequencing, Labeling, and Synthesis
64. Which of these is used to convert a protein into smaller, more manageable fragments for
subsequent structural studies?
A) Insulin
B) Aminopeptidase
C) Carboxypeptidase
D) Trypsin
E) 2,4-Dinitrofluorobenzene
Ans: D
Topic: Peptide Sequencing, Labeling, and Synthesis
65. How many different tripeptides can exist, each containing one residue of glycine, one of
L-threonine, and one of L-arginine?
A) 2
B) 3
C) 6
D) 8
E) 9
Ans: C
922
Chapter 24
Topic: Peptide Sequencing, Labeling, and Synthesis
66. What use is made of dicyclohexylcarbodiimide (DCC) in peptide synthesis?
A) DCC "protects" the amino group of the intended N-terminal amino acid.
B) DCC activates the carboxyl group of one amino acid so that this amino acid reacts
more readily with a second amino acid.
C) DCC cleaves the blocking groups from the final peptide.
D) DCC is the resin used in the automated synthesis of peptides.
E) DCC removes the peptide from the resin at the conclusion of the synthesis.
Ans: B
Topic: Peptide Sequencing, Labeling, and Synthesis
67. Which is an incorrect statement concerning the tetrapeptide L-arginine-L-leucine-Lcysteine-L-phenylalanine?
A) This peptide would have an isoelectric point greater than 6.0.
B) The peptide will be modified by mild oxidizing agents.
C) Trypsin will catalyze preferentially the hydrolysis of the arginine-leucine peptide
bond.
D) Hydrolysis in dilute aqueous NaOH would lead to racemization of all four amino
acids.
E) All peptide bonds will be cleaved by refluxing with 6 M HCl for 24 hours.
Ans: D
Topic: Stereochemistry
68. Which of these amino acids has the R configuration at the stereogenic center but,
nonetheless, is an L amino acid?
CO2
H3N
H3N
H
CO2
H3N
H
CH2SH
CH2OH
CH2C6H5
I
II
III
CO2
H3N
CO2
H
CH2CH2SCH3
IV
A)
B)
C)
D)
E)
Ans:
CO2
H
H3N
H
CH2CO2H
V
I
II
III
IV
V
A
923
Chapter 24
Topic: Stereochemistry
69. Which of these amino acids cannot be described as an L amino acid?
H2N
H
CO2H
H
HO2C
I
CO2H
H2N
H
H
NH2
II
III
H
NH2
HO2C
H
IV
A)
B)
C)
D)
E)
Ans:
NH2
HO2C
OH
V
I
II, IV and V
I and III
II and IV
III and V
E
Topic: Stereochemistry
70. Which of these amino acids is a D amino acid?
H2N
CO2H
H
H
HO2C
I
CO2H
H2N
H
H
NH2
II
III
H
NH2
HO2C
OH
IV
A)
B)
C)
D)
E)
Ans:
NH2
HO2C
H
V
I
II
III
IV
V
E
924
Chapter 24
Topic: Stereochemistry
71. Which of these amino acids is an R amino acid?
CO2H
H
H2N
I
H
HO2C
C6H5
CO2H
H2N
H
H
NH2
II
III
O
H
NH2
HO2C
HS
NH
IV
A)
B)
C)
D)
E)
Ans:
H
OH
V
II and IV
IV and V
I and III
V
All of these are R amino acids
B
Topic: Stereochemistry
72. Which of the following amino acids is theoretically capable of existing in
diastereomeric forms?
CO2
CO2
NH3
NH3
Glycine
Alanine
I
II
NH3
Threonine
III
CO2
CO2
NH3
NH2
Leucine
Proline
IV
A)
B)
C)
D)
E)
Ans:
CO2
HO
V
I
II
III
IV
V
C
925
Chapter 24
Topic: Stereochemistry
73. Which amino acid is achiral?
CO2
CO2
NH3
NH3
Glycine
Alanine
I
II
NH3
Threonine
III
CO2
CO2
NH3
NH2
Leucine
Proline
IV
A)
B)
C)
D)
E)
Ans:
CO2
HO
V
I
II
III
IV
V
A
SHORT ANSWER QUESTIONS
Topic: Protein Structure
74. The exact sequence of the different α-amino acids along the protein chain is called the
________________ of the protein.
Ans: primary structure
Topic: Henderson-Hasselbach Equation
75. The Henderson-Hasselbach equation shows that the ________________ of an acid is
the ________________ at which the acid is half-neutralized.
Ans: pKa , pH
Topic: Protein Structure
76. The folding of the polyamide chain gives rise to higher levels of complexity called the
_______________ and ______________ of the protein.
Ans: secondary; tertiary structures
Topic: Amino Acid Structure
77. The 22 α-amino acids can be subdivided into three different types on the basis of the
structures of their side chains. These three types are: ____________.
Ans: neutral, acidic, basic
926
Chapter 24
Topic: Isoelectric Point
78. The pH at which the concentration of the zwitterionic form (dipolar form) of an amino
acid is at its highest and the concentrations of the cationic and anionic forms are equal is
called the _____________.
Ans: isoelectric point
Topic: Strecker Synthesis
79. Amino acids can be prepared from aldehydes by treatment with ammonia and HCN
followed by hydrolysis. This method is known as the ___________.
Ans: Strecker synthesis
Topic: Analysis
80. A reagent that reacts with most amino acids to give an intense purple color is called
___________.
Ans: ninhydrin
Topic: Analysis
81. The most widely used method for identifying the N-terminal amino acid in a peptide
chain is called the ____________.
Ans: Edman degradation
Topic: Analysis
82. C-terminal amino acids can be identified through the use of _____________.
Ans: digestive enzymes or carboxypeptidases
Topic: Glucose Metabolism
83. The hormone that regulates glucose metabolism is called ______________. Deficiency
of this hormone is the major cause of __________.
Ans: insulin; diabetes
Topic: Peptide Synthesis
84. Peptide synthesis has four basic steps. These are:
Ans: protect the N-terminus, activate the carboxylic acid, react with a different amine
to form the peptide bond, deprotect the N-terminus
Topic: Protein Structure
85. The secondary structure of proteins has three basic types of folding patterns. These are:
Ans: α-helices, β-pleated sheets, turns
Topic: Enzymes
86. All reactions that occur in living cells are mediated by biological catalysts called
___________. The prevailing hypothesis for enzyme catalysis is called the
____________ hypothesis.
Ans: enzymes; lock-and-key
927
Chapter 24
Topic: Analysis
87. One common method of analysis used to measure the approximate molecular weight of
a protein is called _____________.
Ans: gel electrophoresis
Topic: Proteomics
88. The study of all proteins that are expressed in a cell at a given time is called
___________.
Ans: proteomics
Topic: Anticancer Protein
89. A naturally occurring protein called ___________ acts as a tumor suppressant by
coordinating a complex set of responses to changes in DNA, halting abnormal growth in
normal cells.
Ans: p53
Topic: Dipolar Ions
90. Draw structures to represent what happens when valine is dissolved in a strongly acidic
solution (pH=0).
O
O
O
Ans:
H3O+
OH
OH
O
(pH = 0)
NH2
NH3
NH3
Dipolar ion
Valine
Cationic form
Topic: Dipolar Ions
91. Draw structures to represent what happens when valine is dissolved in a strongly basic
solution (pH=14).
O
O
O
Ans:
OH−
OH
NH2
Valine
O
NH3
Dipolar ion
928
(pH = 14)
O
NH2
Anionic form
Chapter 24
Topic: Dipolar Ions
92. Draw structures to represent what happens when lysine is dissolved in a strongly acidic
solution (pH=0).
O
O
Ans:
H2N
H3N
OH
NH2
O
NH2
Lysine
Dipolar ion
H3O+
(pH = 0)
O
H3N
OH
NH3
Cationic form
Topic: Dipolar Ions
93. Draw structures to represent what happens when lysine is dissolved in a strongly basic
solution (pH=14).
O
O
Ans:
H2N
H3N
OH
NH2
O
NH2
Lysine
Dipolar ion
OH−
(pH = 14)
O
H2N
O
NH2
Anionic form
929
Chapter 24
Topic: Synthesis and Reactions
94. What is the final product formed when potassium phthalimide is subjected to the
following reaction sequence? Give structural details of all significant intermediates,
including stereochemistry, as applicable.
i. BrCH(CO2H5)2
O
ii. NaOEt, C6H5CH2Br
N K
?
iii. NaOH
O
iv. HCl, heat
O
O
Ans:
CO2Et
BrCH(CO2H5)2
N K
N
O
O
CO2Et
NaOEt
C6H5CH2Br
CO2
O
C6H5
N
H
CO2
CO2
O
NaOH
N
O
CO2Et
CH2C6H5
CO2Et
HCl, heat
CO2H
CO2H
+ CO2 +
CO2H
H2N
C6H5
DL-Phenylalanine
Topic: Strecker Synthesis
95. Give the structure of the aldehyde which, upon treatment with HCN and ammonia,
followed by heating with aqueous acid, would afford racemic tryptophan. What is this
strategy for the synthesis of α-amino acids called?
Ans: This is called the Strecker synthesis
O
N
H
930
Chapter 24
Topic: Amino Acid Synthesis
96. What is the final product formed via the following reaction sequence? Give structural
details of all significant intermediates.
CO2Et i. CH =CHCN, NaOEt, EtOH
2
HN
?
CO2Et ii. conc HCl, reflux
O
Ans:
HN
O
CO2Et
CO2Et
CO2Et
CH2=CHCN
NaOEt, EtOH
HN
O
CH2CH2CN
CO2Et
conc HCl
reflux
CO2H
CH3CO2H + CO2 +
H2N
CO2H
DL-Glutamic acid
931
Chapter 24
Topic: Peptide Synthesis
97. Suggest a reasonable strategy for the synthesis of the dipeptide Leu-Phe, using
established protocol for peptide synthesis.
O
Ans:
O
H3N
O
Cl
O
O
O
OH , 25oC
N
H
O
O
Leu
i. Et3N
ii. ClCO2Et
O
H3N
O
O
N
H
H
N
O
O
O
OH
O
O
(Phe)
H2,Pd
H
N
H3N
O
O
+ CO2
+
O
Leu-Phe
932
N
H
O
O
OEt
O
Chapter 24
Topic: Peptide Structure
98. Draw the structure of the following tripeptide (starting with the N-terminal residue and
ending with the C-terminal residue), showing stereochemical details:
Gly-Phe-Met
Ans:
S
H
H
O
H
N
H3N
O
H
N
H
H
O
O
Gly-Phe-Met
Topic: Peptide Structure
99. Draw the structure of the following tripeptide (starting with the N-terminal residue and
ending with the C-terminal residue), showing stereochemical details:
Ala-Ser-Leu
Ans:
H3C
H
O
H
N
H3N
O
H
H
N
H
O
O
HO
Ala-Ser-Leu
933
Chapter 24
Topic: Peptide Structure
100. Draw the structure of the following tripeptide (starting with the N-terminal residue and
ending with the C-terminal residue), showing stereochemical details:
Val-Trp-Asn
O
Ans:
NH2
H
H3N
O
O
H
N
H
N
H
H
O
O
N
H
Val-Trp-Asn
934
Chapter 25
MULTIPLE CHOICE QUESTIONS
Topic: The Double Helix
1. What is the secondary structure of DNA?
A) An alpha-helix
B) A pleated sheet
C) A flat sheet
D) A double helix
E) A random coil
Ans: D
Topic: The Double Helix
2. Which is an incorrect statement concerning the DNA double helix?
A) The sugar-phosphate backbone is on the outside of the helix and the base pairs are
on the inside.
B) The two strands are identical but proceed in opposite directions.
C) Hydrogen bonding holds together the two strands.
D) Only purine-pyrimidine base pairs can be accommodated.
E) The sugar-phosphate backbone is completely regular.
Ans: B
Topic: The Double Helix
3. Which is an incorrect statement concerning the DNA double helix?
A) The sugar-phosphate backbone is on the inside of the helix and the base pairs are
on the outside.
B) The two strands are identical but proceed in opposite directions.
C) The two strands are linked by glycosidic bonds.
D) Two of the above statements are incorrect.
E) All of the above statements are incorrect.
Ans: E
Topic: DNA Structure
4. Which is an incorrect statement concerning DNA structure?
A) The base and the phosphate units are linked by phosphodiester bonds.
B) The sugar and the phosphate units are linked by hydrogen bonds.
C) The base and the sugar units are linked by β-N-glycosidic bonds.
D) Two of the above statements are incorrect.
E) All of the above statements are incorrect.
Ans: D
1
Chapter 25
Topic: DNA Structure
5. Which is a correct statement concerning DNA structure?
A) The base and the phosphate units are linked by hydrogen bonds.
B) The sugar and the phosphate units are linked by phosphodiester bonds.
C) The base and the sugar units are linked by β-N-glycosidic bonds.
D) Two of the above statements are correct.
E) None of the above statements are correct.
Ans: D
Topic: DNA Structure
6. The analytical data for DNA samples which led to generalizations such as (%G + %A) ≈
(%C + %T) came from the research of what individual?
A) James Watson
B) Francis Crick
C) Edwin Chargaff
D) Arthur Kornberg
E) Maurice Wilkins
Ans: C
Topic: DNA Structure
7. Which bases pair in DNA by hydrogen bonding?
A) Cytosine and thymine
B) Cytosine and uracil
C) Adenine and guanine
D) Adenine and thymine
E) Adenine and uracil
Ans: D
Topic: DNA Structure
8. In DNA, thymine can pair via hydrogen bonding to _________?
A) Cytosine
B) Uracil
C) Adenine
D) Guanine
E) Deoxyribose
Ans: C
2
Chapter 25
Topic: DNA Structure
9. In DNA, cytosine can pair via hydrogen bonding to _________?
A) Deoxyribose
B) Uracil
C) Adenine
D) Guanine
E) All of the above
Ans: D
Topic: DNA Structure
10. In DNA, adenine can be linked to _________ .
A) Deoxyribose via β-N-glycosidic bonds
B) Uracil via hydrogen bonds
C) Phosphate via phosphodiester bonds
D) Guanine via amide bonds
E) All of the above
Ans: A
Topic: RNA Structure
11. In RNA, adenine can be linked to _________ .
A) Deoxyribose via β-N-glycosidic bonds
B) Uracil via hydrogen bonds
C) Phosphate via phosphodiester bonds
D) Guanine via amide bonds
E) All of the above
Ans: B
Topic: The Double Helix
12. In DNA, the lactim form of guanine will pair with which base?
A) Cytosine
B) Adenine
C) Uracil
D) Thymine
E) No pairing is possible.
Ans: D
3
Chapter 25
Topic: The Double Helix
13. Consideration of tautomerism in guanine suggests that it can exist in lactam and lactim
forms. The form that is typically found in DNA is __________, typically pairs with
_____________, while the other form is likely to pair with __________, leading to the
possible spontaneous mutations.
A) Lactam, cytosine, thymine
B) Lactam, cytosine, adenine
C) Lactim, cytosine, thymine
D) Lactam, uracil, cytosine
E) Lactim, cytosine, thymine
Ans: A
Topic: The Double Helix
14. Consider the effect of nitrous acid on the adenine residue of a DNA strand. The
product now resembles __________, and is likely to pair with __________, instead of
with ___________, leading to the possible mutations.
A) guanine, cytosine, thymine
B) thymine, cytosine, guanine
C) thymine, guanine, cytosine
D) uracil, cytosine, guanine
E) cytosine, guanine, thymine
Ans: A
Topic: The Double Helix
15. Which is the predominant tautomeric form of cytosine when it is present in DNA?
NH
NH2
I
A)
B)
C)
D)
E)
Ans:
NH
N
N
N
H
O
N
II
N
O
N
O
III
I
II
III
None of these
I, II and III are present in approximately equal amounts.
A
4
Chapter 25
Topic: The Double Helix
16. The following structure represents the less common (in DNA, at least) tautomeric form
of __________.
OH
H3C
N
N
A)
B)
C)
D)
E)
Ans:
O
Cytosine
Thymine
Adenine
Guanine
Uracil
B
Topic: DNA Structure
17. Which is the predominant tautomeric form of guanine when it is present in DNA?
O
N
N
H
OH
N
NH
N
N
H
NH2
I
N
NH2
O
N
N
II
H
N
OH
N
N
N
NH2
NH
III
O
N
IV
A)
B)
C)
D)
E)
Ans:
N
H
N
N
H
N
N
H
NH2
V
I
II
III
IV
V
A
5
NH2
Chapter 25
Topic: DNA Structure
18. Which is the predominant tautomeric form of adenine when it is present in DNA?
NH2
NH2
N
N
H
N
N
N
H
N
H
CH3
I
N
NH2
N
N
II
H
N
NH2
N
N
N
IV
A)
B)
C)
D)
E)
Ans:
N
H
N
N
O
III
NH
N
N
H
NH
N
V
I
II
III
IV
V
B
Topic: Sequencing
19. In DNA sequencing, what is used to convert DNA molecules into smaller, more
manageable fragments?
A) DNA polymerase
B) Dilute HCl
C) Trypsin
D) Chymotrypsin
E) Restriction endonucleases
Ans: E
Topic: Sequencing
20. Separation of the fragments produced in the chemical sequencing of a DNA segment is
achieved by the use of which of these techniques?
A) Column chromatography
B) Fractional distillation
C) Thin layer chromatography
D) Gas chromatography
E) Gel electrophoresis
Ans: E
6
Chapter 25
Topic: Sequencing
21. If the base sequence along a segment of DNA were G—C—C—A—T, what would be
the base sequence of messenger RNA synthesized from this sequence?
A) G—C—C—A—T
B) C—G—G—T—A
C) C—G—G—U—A
D) G—C—C—A—U
E) A—T—T—G—C
Ans: C
Topic: Sequencing
22. The antisense nucleotide G—A—C—T—C, could be synthesized via a __________
template on ________?
A) T—C—A—G—A , RNA
B) C—T—G—A—G, DNA
C) C—T—G—A—G, RNA
D) G—A—C—T—U, DNA
E) A—G—T—C—T, RNA
Ans: B
Topic: Sequencing
23. If the base sequence along a segment of DNA were T—C—G—T—A, what would be
the antisense oligonucleotide synthesized from this sequence?
A) G—C—C—A—T
B) C—G—G—T—A
C) U—G—C—U—T
D) A—G—C—A—U
E) A—G—C—A—T
Ans: E
Topic: Sequencing
24. If the base sequence along a segment of DNA were G—C—C—A—T, what would be
the base sequence of messenger RNA synthesized from this sequence?
A) C—U—U—T—A
B) C—G—G—T—A
C) C—G—G—U—A
D) G—C—C—A—U
E) A—T—T—G—C
Ans: C
7
Chapter 25
Topic: Sequencing
25. During translation, the segment on the mRNA that corresponds to the base sequence
C—G—A along a gene segment of DNA, would bind to the anticodon __________ on a
tRNA.
A) A—U—C
B) G—C—U
C) U—T—C
D) G—C—T
E) C—G—A
Ans: E
Topic: Assorted Reactions
26. Complete hydrolysis of adenylic acid would yield which of these?
A) Adenosine and a phosphate ion
B) Adenine, D-ribose, and a phosphate ion
C) Adenine and D-ribose
D) Adenine and 2-deoxy-D-ribose
E) A pyrimidine, a pentose, and a phosphate ion
Ans: B
Topic: Nucleotide Synthesis
27. Supply the missing reagent in the following synthesis.
HO
O
HO
base
+ ?
H3O
OH OH
O
H3C
A)
B)
C)
D)
E)
Ans:
base
O
+
(CH3)2C=O
HOCH2CH2OH
CH3CHOHCH3
CH3CCl2CH3
CH3CHClCH3
A
8
O
CH3
Chapter 25
Topic: Nucleotide Synthesis
28. Supply the missing reagent(s) in the synthesis below.
O
HO
O
base
O
A)
B)
C)
pyridine
O
H3C
base
O
+ ?
O
P(OCH2C6H5)2
O
CH3
H3C
O
CH3
PO4-3 and C6H5CH2OH
PO4-3 and C6H5CH2Cl
O
P(OCH2C6H5)2
Cl
D) C6H5CH3 + POCl3
E) C6H5CH2OH + H3PO4
Ans: C
Topic: Assorted Reactions
29. The synthesis of adenosine by the reaction shown below likely occurs by what reaction
mechanism?
NH2
Cl
N
N
R
N
N
NH3
N
N
R
N
N
( R = ribofuranosyl )
A) An SN2 reaction
B) An SNAr reaction
C) An elimination-addition reaction
D) An SN1 reaction
E) An E2 reaction
Ans: B
9
Chapter 25
Topic: Assorted Reactions
30. The action of nitrous acid on 5-methylcytosine produces which nitrogen base?
NH2
H3C
N
N
H
O
5-methylcytosine
A) Adenine
B) Guanine
C) Cytosine
D) Thymine
E) Uracil
Ans: D
Topic: Assorted Reactions
31. An RNA nucleoside will undergo hydrolysis in dilute acid to yield which of the
following?
A) 2-Deoxy-D-ribose and a heterocyclic base
B) 2-Deoxy-D-ribose, a heterocyclic base, and phosphate ion
C) D-Ribose and a heterocyclic base
D) D-Ribose, a heterocyclic base, and phosphate ion
E) D-Ribose and phosphate ion
Ans: C
Topic: Assorted Reactions
32. Which are the products of hydrolysis of a RNA nucleotide?
A) D-Ribose, adenine
B) D-Ribose, guanine, phosphate
C) D-Ribose, thymine, phosphate
D) 2-Deoxy-D-ribose, cytosine, phosphate
E) 2-Deoxy-D-ribose, adenine
Ans: B
Topic: Assorted Reactions
33. Hydrolysis of DNA from various species gives which of the following results?
A) The mole percentage of adenine is approximately equal to that of thymine.
B) The mole percentage of cytosine is approximately equal to that of guanine.
C) The total mole percentage of purines is approximately equal to that of
pyrimidines.
D) All of the above
E) None of the above
Ans: D
10
Chapter 25
Topic: Codons and Anticodons
34. What is the transfer RNA anticodon for the messenger RNA codon, G—C—A?
A) T—A—T
B) G—U—T
C) G—C—A
D) A—C—G
E) C—G—U
Ans: E
Topic: Codons and Anticodons
35. Where is the sequence of bases termed an "anticodon" found?
A) mRNA
B) tRNA
C) rRNA
D) DNA
E) Polysomes
Ans: B
Topic: Codons and Anticodons
36. If a tRNA anticodon is GUC, what was the original base sequence in DNA?
A) GUC
B) GTC
C) CAG
D) GAC
E) CTG
Ans: B
Topic: Codons and Anticodons
37. Which is the messenger codon which calls for the initiation of protein synthesis?
A) AUG
B) GUA
C) UAG
D) AUC
E) CAC
Ans: A
Topic: Codons and Anticodons
38. Concerning the genetic code, which of the following is an incorrect statement?
A) Not all codons specify amino acids.
B) There are a total of 64 different triplets.
C) For each amino acid there are the same number of codons.
D) The triplet codons are incorporated in messenger RNA.
E) The triplet AUG is a "start" codon.
Ans: C
11
Chapter 25
Topic: Nucleic Acid Structure
39. The monomeric units of nucleic acids are which of these?
A) D-ribose or 2-deoxy-D-ribose
B) Phosphate ions
C) Purines
D) Nucleosides
E) Nucleotides
Ans: E
Topic: Assorted Definitions
40. The primary function of nucleic acids is:
A) the catalysis of biochemical reactions.
B) the regulation of reactions that occur in the body.
C) the preservation, transcription and translation of information.
D) the acid-catalyzed hydrolysis of nucleotides.
E) the neutralization of nucleic bases.
Ans: C
Topic: Nucleic Acid Structure
41. A nucleotide unit is composed of:
A) a five carbon monosaccharide.
B) a phosphate group.
C) a heterocyclic base.
D) an amino acid.
E) A), B) and C)
Ans: E
Topic: Nucleic Acid Structure
42. A purine:
A) contains four nitrogens in the ring system.
B) is bicyclic.
C) can participate in hydrogen bonding.
D) is a heterocyclic base.
E) All of the above
Ans: E
Topic: Nucleic Acid Structure
43. In a nucleotide unit, the components are sequentially linked:
A) monosaccharide--phosphate--heterocyclic base.
B) amino acid--monosaccharide--phosphate.
C) phosphate--monosaccharide--heterocyclic base.
D) monosaccharide--amino acid--phosphate.
E) heterocyclic base--phosphate--monosaccharide.
Ans: C
12
Chapter 25
Topic: Nucleic Acid Structure
44. Which of the following is not a pyrimidine?
A) Cytosine
B) Thymine
C) Guanine
D) Uracil
E) All of the above are pyrimidines.
Ans: C
Topic: Nucleic Acid Structure
45. The hydrogen bonding for the base pairs of DNA is between:
A) amide carbonyl and -NH2.
B) amide N-H and cyclic amine nitrogens.
C) alcohols and carbonyls.
D) All of these
E) Only A) and B)
Ans: E
Topic: Nucleic Acid Structure
46. Which heterocyclic base found in RNA does not occur in DNA?
A) Guanine
B) Thymine
C) Cytosine
D) Uracil
E) Adenine
Ans: D
Topic: Assorted Definitions
47. The formation of a new DNA molecule which is an exact copy of a pre-existing one is
designated by what term?
A) Duplication
B) Transcription
C) Translation
D) Replication
E) Reproduction
Ans: D
Topic: DNA Mutation
48. Nitrous acid is a suspected mutagen because of its reaction with:
A) amine groups.
B) monosaccharides.
C) phosphates.
D) alcohols.
E) ketones.
Ans: A
13
Chapter 25
Topic: Protein Synthesis
49. The synthesis of proteins according to "the central dogma of molecular genetics" would
require:
A) replication then translation.
B) translation then replication.
C) replication then transcription.
D) transcription then translation.
E) Any of these
Ans: D
Topic: Protein Synthesis
50. Which is the initial N-terminal amino acid in the developing polypeptide in bacteria?
A) Cysteine
B) Methionine
C) N-Formylmethionine
D) Glycine
E) Alanine
Ans: C
Topic: Miscellaneous
51. Which of these has the smallest molecular weight?
A) rRNA
B) mRNA
C) tRNA
D) DNA
Ans: C
Topic: Miscellaneous
52. Which statement concerning the monosaccharide portion of nucleotides is true in every
case?
A) The monosaccharide is in the pyranose form.
B) The linkage to phosphate is at carbon #3.
C) The heterocyclic base is attached to carbon #1 of the monosaccharide.
D) The monosaccharide - nitrogen base linkage is ∝.
E) The monosaccharide is 2-deoxy-D-ribose.
Ans: C
14
Chapter 25
Topic: Miscellaneous
53. Relatively short synthetic strands of DNA complementary to certain portions of a gene
are known as:
A) antisense nucleotides.
B) templates.
C) palindromes.
D) endonucleases.
E) polymerases.
Ans: A
Topic: Transcription
54. The anticodon is part of
A) mRNA
B) rRNA
C) tRNA
D) DNA
E) None of these
Ans: C
Topic: Transcription
55. The codon is part of
A) mRNA
B) rRNA
C) tRNA
D) DNA
E) None of these
Ans: A
Topic: Transcription
56. The codon consists of
A) A sequence of three bases
B) A sequence of three amino acids
C) A sequence of three purines
D) A sequence of three pyrimidines
E) None of these
Ans: A
15
Chapter 25
Topic: Transcription
57. Specific amino acids attach to the arm of a tRNA that ends in __________, via a(n)
_________ bond.
A) CCG, ester
B) ACG, ether
C) CCA, glycosidic
D) ACA, amide
E) CCA, ester
Ans: E
Topic: Transcription
58. The time required to synthesize a protein depends on the number of amino acid residues
it contains. A rough estimate is that each ribosome can cause _________ peptide bonds
to be formed per minute.
A) 30
B) 50
C) 100
D) 150
E) 200
Ans: D
Topic: Polymerase Chain Reaction
59. Which of the following statements about the polymerase chain reaction are true?
A) It generates a large number of copies of DNA
B) It polymerizes the short oligonucleotide strands formed during biosynthesis
C) It catalyzes the formation of mRNA
D) It needs a minuscule sample of DNA
E) Two of the above
Ans: E
Topic: Polymerase Chain Reaction
60. Which of the following is not a valid application of the PCR (polymerase chain
reaction) ?
A) Semen analysis in forensic investigations
B) Evolutionary biology
C) Detection of cytomegalovirus
D) Prenatal diagnosis of sickle cell anemia
E) All of these are valid applications of the PCR
Ans: E
16
Chapter 25
SHORT ANSWER QUESTIONS
Topic: Protein Synthesis
61. The molecular archive of instructions for protein synthesis is called ______. The
molecules that transcribe and translate these instructions are called ______.
Ans: DNA; RNA
Topic: DNA Structure
62. The four bases that make up the DNA “ladder” are: ____________.
Ans: adenine, cytosine, guanine, and thymine
Topic: RNA Structure
63. The four bases found in RNA are: _________.
Ans: adenine, cytosine, guanine, and uracil
Topic: Miscellaneous
64. Each section of DNA that codes for a particular protein is called a _______.
Ans: gene
Topic: Miscellaneous
65. Amino acids bear the same relation to proteins as __________ do to nucleic acids.
Ans: nucleotides
Topic: Miscellaneous
66. While proteins are made up of amino acids connected by amide bonds, nucleic acids are
made up of nucleotides connected by __________.
Ans: phosphate ester linkages
Topic: DNA Structure
67. The secondary structure of DNA was proposed by Watson and Crick to be a
___________________.
Ans: double helix
Topic: DNA Structure
68. In the DNA double helix, two strands of nucleic acids are held together by
_____________.
Ans: hydrogen bonds
Topic: DNA Structure
69. In the DNA base pairs, adenine always bonds with _________ and cytosine with
__________.
Ans: thymine; guanine
17
Chapter 25
Topic: DNA Structure
70. The backbone of DNA is made up of __________.
Ans: sugars and phosphates
Topic: Miscellaneous
71. A chemical that causes mutations in DNA is called a _________.
Ans: Mutagen
Topic: Protein Synthesis
72. Protein synthesis requires that two major processes take place. The first takes place in
the cytoplasm involving messenger RNA (mRNA) and is called ___________. The
second involves two other types of RNA: __________ and ____________.
Ans: transcription; ribosomal RNA [rRNA]; transfer RNA [tRNA]
Topic: Protein Synthesis
73. The anticodon is a specific sequence of ____________, found on the ___________,
which allows it to bind to the ____________, of _____________.
Ans: three bases, tRNA, codon, mRNA
Topic: RNA
74. The molecular weight of tRNA is _________, compared with mRNA or rRNA;
consequently, the solubility of tRNA is __________.
Ans: low, high
Topic: RNA
75. The function of tRNA is ___________.
Ans: to transport amino acids to specific areas on the mRNA bound to the ribosome
Topic: Triplet Code
76. Explain why 3 bases (triplet code) are needed in the genetic code to unambiguously
identify each amino acid.
Ans: There are only four bases and 20 amino acids; thus, each base could uniquely
code for only 4 amino acids. A set of two bases would uniquely code for (42) ,
i.e., 16 amino acids, still not quite enough. However, if each amino acid were to
be coded via a set of three bases, then there could be 64 possible combinations,
allowing for redundancies and certain necessary “punctuation” codes, such as
start/stop codes, etc.
18
Chapter 25
Topic: DNA Structure
77. Draw structures to represent the hydrogen bonding between adenine and thymine.
H
Ans:
..
:O
N H
N
N
N:
H
N
N
N
O
adenine
thymine
Topic: DNA Structure
78. Draw structures to represent the hydrogen bonding between cytosine and guanine.
H
Ans:
..
O:
H N
N
N
:N
N H
N
N
N
guanine
:O
..
H
H
cytosine
Topic: DNA Structure
79. Draw structures to represent the hydrogen bonding between adenine and uracil.
H
Ans:
..
:O
N H
N
N
N:
H
N
N
N
O
uracil
adenine
Topic: DNA Structure
80. Draw structures to represent the bonding between deoxyribose and thymine.
O
Ans:
N
HO
O
N
H
thymine
O
OH H
deoxyribose
19
Chapter 25
Topic: DNA Structure
81. Draw structures to represent the bonding between ribose and uracil.
O
Ans:
H
N
HO
uracil
N
O
O
OH OH
ribose
Topic: DNA Structure
82. Draw the structures to represent what is likely to happen when cyclic AMP (3’,5’-cyclic
adenylic acid) reacts with aqueous sodium hydroxide, briefly explaining your rationale.
NH2
NH2
Ans:
N
5'
O
N
P
3'
O
HO
N
O
NaOH
H2O
O
O
N
N
HO
OH
O
HO
cyclic AMP
O
N
N
N
OH
P
O
3'-adenylic acid
The reaction appears to follow an SN2 mechanism, with nucleophilic attack by
OH− at the primary 5’- position to afford 3’-adenylic acid (rather than at the
secondary 3’- position, which would yield 5’-adenylic acid).
20
Chapter 25
Topic: Nucleotide Synthesis
83. Suggest a reasonable strategy for the specific phosphorylation of the 5’ –OH group of a
nucleoside.
Ans: In order for phosphorylation to occur specifically at the 5’ OH, the 2’- and 3’- OH
groups must first be protected.
HO
HO
base
O
base
O
acetone
H3O+
OH OH
O
O
H3C
CH3
O
P(OCH2C6H5)2
Cl
pyridine
O
O
P(OCH2C6H5)2
O
base
O
P(OCH2C6H5)2
O
base
O
i. H3O+, H2O
ii. H2, Pd
OH OH
O
O
H3C
CH3
Topic: Tautomerism, Mutation
84. Draw the structures of two tautomeric forms of guanine, one a lactam, the other a
lactim. Explain the possible involvement of the lactim form in causing spontaneous
mutation during replication.
Ans: The lactim form resembles adenine and might pair with thymine instead of
cytosine, causing the first error in the new DNA strand being synthesized. This
mistake, if uncorrected, can then be passed on to the next strand being synthesized
from this “mutated” strand, by introducing an adenine residue, where there should
be a guanine residue.
..
O:
OH
N
N
N
H
N H
N
N
N
H
N:
N
H
N
H
H
H
lactam
(likely to pair with cytosine)
lactim
(likely to pair with thymine)
21
Chapter 25
85. What product is likely to be formed from the reaction between an adenine residue on a
nucleotide and nitrous acid? Explain the possible involvement of the product of this
reaction in causing spontaneous mutation during replication.
Ans: The product resembles a guanine nucleotide, thus might pair with cytosine instead
of thymine, causing the first error in the new DNA strand being synthesized. This
mistake, if uncorrected, can then be passed on to the next strand being synthesized
from this “mutated” strand, by introducing a guanine residue, where there should
be an adenine residue.
NH2
N
N
R
N
O
N
HNO2
N
adenine nucleotide
(pairs with thymine)
N
R
NH
N
hypoxanthine nucleotide
(likely to pair with cytosine)
22
Related documents